{"id": 402, "year": 2016, "question_id_specific": 183, "full_question": "Juan, a second year resident, attends Sofia, a 15 year old girl in the emergency room, who apparently fainted at school without losing consciousness. The patient says that she was due to take an exam, which caused her a lot of anxiety. From the interrogation, it appears that she was being bullied by her classmates and that she may have an eating disorder. Vital signs and neurological examination are normal. Juan keeps Sofia under observation while waiting for her parents to come to the service, periodically taking a look at how the patient is doing. After the initial scare, the patient seems to be increasingly animated and is very friendly. On one occasion, Juan finds her actively chatting on her cell phone. Juan tells her that it would be better for her to put the cell phone down and rest and to reassure her, he tells her that he also uses social networks a lot since college. Sofia apologizes for not knowing that she should have her cell phone turned off, and after turning it off, asks if she can make a friend request on Facebook. What do you think is Juan's best response?", "full_answer": "There is no doubt that the doctor-patient relationship today transcends the physical environment of the consultation room. But we must not forget that we must maintain the same ethical and professional codes as in the real environment. The code of ethics Article 26-3. The clinical practice of medicine by means of consultations exclusively by letter, telephone, radio, press or internet, is contrary to the deontological norms. The correct action implies inescapably the personal and direct contact between the doctor and the patient.", "type": "PRIMARY CARE AND SOCIAL NETWORKS", "options": {"1": "Tell him to make the friend request and that he will accept it, as he is sure that there are no inappropriate items on his page for a girl of Sofia's age.", "2": "Because you consider Sofia a vulnerable patient and are concerned that she may misinterpret a rejection, agree to let her make the request but only allow her access to certain content on your page.", "3": "Answer that it is important to maintain certain professional boundaries between patients and physicians and that, unfortunately, if you make the request you will not be able to accept it, so it is better not to do so.", "4": "Tell him to make the request but without any intention of accepting it.", "5": NaN}, "correct_option": 3, "explanations": {"1": {"exist": false, "char_ranges": [], "word_ranges": [], "text": ""}, "2": {"exist": false, "char_ranges": [], "word_ranges": [], "text": ""}, "3": {"exist": true, "char_ranges": [[429, 535]], "word_ranges": [[66, 82]], "text": "The correct action implies inescapably the personal and direct contact between the doctor and the patient."}, "4": {"exist": false, "char_ranges": [], "word_ranges": [], "text": ""}, "5": {"exist": false, "char_ranges": [], "word_ranges": [], "text": ""}}} {"id": 237, "year": 2014, "question_id_specific": 144, "full_question": "After a traffic accident a 38-year-old patient is admitted to the ICU in coma. After several days the patient does not improve neurologically and a CT scan shows hemorrhagic punctate lesions in the corpus callosum and cortico-subcortical junction. What is the diagnosis?", "full_answer": "Diffuse axonal injury produces an early and sustained deterioration of the level of consciousness (as mentioned in the case statement) without a lesion on CT scan to justify the picture. Sometimes, punctate hemorrhages at the level of the corpus callosum, corticosubcortical junction and dorsolateral portion of the brainstem are evidenced in this imaging test.", "type": "NEUROLOGY", "options": {"1": "Acute subdural hematoma.", "2": "Trobocytopenic purpura.", "3": "Cerebral hemorrhagic contusion.", "4": "Severe diffuse axonal injury.", "5": "Hypoxic-ischemic encephalopathy."}, "correct_option": 4, "explanations": {"1": {"exist": false, "char_ranges": [], "word_ranges": [], "text": ""}, "2": {"exist": false, "char_ranges": [], "word_ranges": [], "text": ""}, "3": {"exist": false, "char_ranges": [], "word_ranges": [], "text": ""}, "4": {"exist": true, "char_ranges": [[0, 186]], "word_ranges": [[0, 30]], "text": "Diffuse axonal injury produces an early and sustained deterioration of the level of consciousness (as mentioned in the case statement) without a lesion on CT scan to justify the picture."}, "5": {"exist": false, "char_ranges": [], "word_ranges": [], "text": ""}}} {"id": 620, "year": 2022, "question_id_specific": 121, "full_question": "79-year-old woman admitted for an osteoporotic hip fracture. Regarding secondary prevention of fragility fractures, point out the WRONG answer:", "full_answer": "In osteoporosis, one of the main risks associated with the increased risk of fracture is low adherence to treatment, so answer 1 is correct. Answer 2 is found in the SER guidelines, which confirm that some studies conclude that bone remodeling markers can be useful for early monitoring of adherence and response to treatment. Answer 4 is correct because again in the SER 2019 guidelines they quote: \"The current scientific evidence allows us to affirm that neither increasing dietary calcium nor taking calcium supplements alone protects against the appearance of fractures\". Therefore, the correct answer to this question is option 3. Patients on pharmacological treatment for OP should use calcium and vitamin D supplements because practically all clinical trials that have demonstrated efficacy of antiosteoporotic drugs routinely include calcium supplements and cholecalciferol (vitamin D3), but not in monotherapy.", "type": "TRAUMATOLOGY", "options": {"1": "Low adherence to treatment is associated with an increased risk of fracture.", "2": "Bone remodeling markers may be useful for early monitoring of treatment response.", "3": "Vitamin D monotherapy is effective in reducing these fractures in non-institutionalized elderly people.", "4": "Increasing dietary calcium or taking calcium supplements in isolation does not protect against the appearance of fractures.", "5": NaN}, "correct_option": 3, "explanations": {"1": {"exist": true, "char_ranges": [[0, 140]], "word_ranges": [[0, 24]], "text": "In osteoporosis, one of the main risks associated with the increased risk of fracture is low adherence to treatment, so answer 1 is correct."}, "2": {"exist": true, "char_ranges": [[140, 326]], "word_ranges": [[23, 54]], "text": "correct. Answer 2 is found in the SER guidelines, which confirm that some studies conclude that bone remodeling markers can be useful for early monitoring of adherence and response to treatment."}, "3": {"exist": true, "char_ranges": [[577, 920]], "word_ranges": [[91, 138]], "text": "Therefore, the correct answer to this question is option 3. Patients on pharmacological treatment for OP should use calcium and vitamin D supplements because practically all clinical trials that have demonstrated efficacy of antiosteoporotic drugs routinely include calcium supplements and cholecalciferol (vitamin D3), but not in monotherapy."}, "4": {"exist": true, "char_ranges": [[327, 576]], "word_ranges": [[54, 91]], "text": "Answer 4 is correct because again in the SER 2019 guidelines they quote: \"The current scientific evidence allows us to affirm that neither increasing dietary calcium nor taking calcium supplements alone protects against the appearance of fractures\"."}, "5": {"exist": false, "char_ranges": [], "word_ranges": [], "text": ""}}} {"id": 396, "year": 2016, "question_id_specific": 136, "full_question": "A 41-year-old man comes to the Emergency Department for three days of swelling and pain in the right knee, with functional impotence and fever Two weeks earlier he had had a self-limited diarrhea. On examination there is joint effusion, so we proceed to perform an arthrocentesis and obtain 50 cc of cloudy fluid, with decreased viscosity and the following analytical parameters: leukocytes 40. 000/microL (85% neutrophils), glucose 40 mg/dL, absence of crystals, Gram stain: no microorganisms are observed. Which of the following statements about this patient is 'CORRECT:", "full_answer": "Gram negative does NOT rule out infection. We mark four.", "type": "TRAUMATOLOGY AND ORTHOPEDICS", "options": {"1": "Treatment with cloxacillin and ceftriaxone should be initiated pending the result of the fluid culture.", "2": "It is advisable to perform daily arthrocentesis to relieve symptoms and avoid joint destruction.", "3": "If the culture is negative, it is probably reactive arthritis.", "4": "The negativity of the Gram stain rules out septic arthritis.", "5": NaN}, "correct_option": 4, "explanations": {"1": {"exist": false, "char_ranges": [], "word_ranges": [], "text": ""}, "2": {"exist": false, "char_ranges": [], "word_ranges": [], "text": ""}, "3": {"exist": false, "char_ranges": [], "word_ranges": [], "text": ""}, "4": {"exist": true, "char_ranges": [[0, 42]], "word_ranges": [[0, 7]], "text": "Gram negative does NOT rule out infection."}, "5": {"exist": false, "char_ranges": [], "word_ranges": [], "text": ""}}} {"id": 253, "year": 2014, "question_id_specific": 86, "full_question": "A 37-year-old woman with extensive ulcerative colitis presents a severe flare-up for which treatment with prednisone at a dose of 1 mg/kg is started. After one week of treatment, the patient shows no improvement. What is the next therapeutic measure to be taken?", "full_answer": "When a severe flare does not respond to iv corticosteroid therapy at full doses (1mg/kg body weight), it is necessary to switch to cyclosporine or infliximab. Etanercept is not approved in Europe for inflammatory bowel disease (for the time being).", "type": "DIGESTIVE TRACT", "options": {"1": "Emergency subtotal colectomy and in a second time prostectomy and ileoanal reservoir.", "2": "Associate an immunosuppressant such as azathioprine.", "3": "Associate mesalazine in doses of 4g per day orally and rectal triamcinolone 1 application every 12 hours.", "4": "Intravenous cyclosporine 2mg/kg.", "5": "Treatment with etanercept (an anti-TNFa antibody) should be considered."}, "correct_option": 4, "explanations": {"1": {"exist": false, "char_ranges": [], "word_ranges": [], "text": ""}, "2": {"exist": false, "char_ranges": [], "word_ranges": [], "text": ""}, "3": {"exist": false, "char_ranges": [], "word_ranges": [], "text": ""}, "4": {"exist": true, "char_ranges": [[0, 158]], "word_ranges": [[0, 26]], "text": "When a severe flare does not respond to iv corticosteroid therapy at full doses (1mg/kg body weight), it is necessary to switch to cyclosporine or infliximab."}, "5": {"exist": true, "char_ranges": [[159, 248]], "word_ranges": [[26, 40]], "text": "Etanercept is not approved in Europe for inflammatory bowel disease (for the time being)."}}} {"id": 224, "year": 2014, "question_id_specific": 63, "full_question": "A 52-year-old patient who has been complaining of dyspnea on medium exertion for 6 months. He has not presented angina or syncope. Physical examination and diagnostic tests show a mean aortic transvalvular gradient of 55 mmHg and a calculated area of 0.7 cm². Ejection fraction of 65%. The treatment to be indicated for this patient is:", "full_answer": "Something aortic. They give me a gradient, which without knowing the values seems somewhat high (55 mmHg!) and the area, which looks tiny to the naked eye: this is going to be aortic stenosis... And he has dyspnea. Symptomatic aortic stenosis, will the examiner want me to send him for surgery? With the guidelines in hand, that's right: this is a severe stenosis both in terms of area (<1 cm²) and mean gradient (>40 mmHg), with a preserved ejection fraction, and symptomatic. Surgical indication. If we do 1 we take away life expectancy, because we allow the ventricle to claudicate. 2 is outdated: balloon valvuloplasty? Only if the surgical risk is unaffordable, and in that case we would do 5, TAVI implantation (see [1]). But this is a 52-year-old man, who we assume has good bodywork because he walks to have dyspnea, so to the operating room. So to the operating room. A homograft? That's the Ross technique, I take the pulmonary and put it where the aortic. But it's rare, complex and only done in pediatrics. The correct thing to do is 3, without a doubt: replace with a prosthesis, preferably mechanical unless the patient has a contraindication for anticoagulation.", "type": "CARDIOLOGY", "options": {"1": "Diuretics and more frequent check-ups by a specialist.", "2": "In case of increasing dyspnea on exertion, percutaneous dilatation of the aortic valve with a balloon catheter should be performed.", "3": "Replacement of the aortic valve with a prosthesis/bioprosthesis.", "4": "Replacement of the aortic valve with a homograft.", "5": "Implantation of a percutaneous valve."}, "correct_option": 3, "explanations": {"1": {"exist": true, "char_ranges": [[499, 585]], "word_ranges": [[84, 100]], "text": "If we do 1 we take away life expectancy, because we allow the ventricle to claudicate."}, "2": {"exist": true, "char_ranges": [[586, 666]], "word_ranges": [[100, 112]], "text": "2 is outdated: balloon valvuloplasty? Only if the surgical risk is unaffordable,"}, "3": {"exist": true, "char_ranges": [[338, 498]], "word_ranges": [[58, 84]], "text": "this is a severe stenosis both in terms of area (<1 cm²) and mean gradient (>40 mmHg), with a preserved ejection fraction, and symptomatic. Surgical indication."}, "4": {"exist": true, "char_ranges": [[971, 1018]], "word_ranges": [[169, 177]], "text": "it's rare, complex and only done in pediatrics."}, "5": {"exist": true, "char_ranges": [[732, 824]], "word_ranges": [[125, 142]], "text": "this is a 52-year-old man, who we assume has good bodywork because he walks to have dyspnea,"}}} {"id": 430, "year": 2018, "question_id_specific": 115, "full_question": "A 65-year-old woman who underwent hematopoietic precursor transplantation 30 days ago and has severe neutropenia. She starts with cough and hemoptotic expectoration in addition to fever and moderate exertional dyspnea that does not improve despite five days of treatment with amoxicillin-clavulanic acid (875/125 mg/8 h) and levofloxacin (500 mg/12 h). Chest X-ray shows multiple ill-defined pulmonary nodules, some of them cavitated. Among the following, which is the most probable diagnosis of suspicion?", "full_answer": "Patient on her 30th post-transplant day of hematopoietic precursor transplantation, in severe neutropenia who presents with cough with hemoptotic expectoration. This picture is usually seen in infection by Aspergillus spp. We can rule out infection by Staphylococcus aureus because of previous treatment, although we do not know if it raises the response also a possible MRSA.", "type": "INFECTIOUS DISEASES AND MICROBIOLOGY", "options": {"1": "Septic embolisms.", "2": "Invasive pulmonary aspergillosis.", "3": "Pneumonia due to Staphylococcus aureus.", "4": "Systemic candidiasis.", "5": NaN}, "correct_option": 2, "explanations": {"1": {"exist": false, "char_ranges": [], "word_ranges": [], "text": ""}, "2": {"exist": true, "char_ranges": [[0, 222]], "word_ranges": [[0, 30]], "text": "Patient on her 30th post-transplant day of hematopoietic precursor transplantation, in severe neutropenia who presents with cough with hemoptotic expectoration. This picture is usually seen in infection by Aspergillus spp."}, "3": {"exist": true, "char_ranges": [[223, 304]], "word_ranges": [[30, 42]], "text": "We can rule out infection by Staphylococcus aureus because of previous treatment,"}, "4": {"exist": false, "char_ranges": [], "word_ranges": [], "text": ""}, "5": {"exist": false, "char_ranges": [], "word_ranges": [], "text": ""}}} {"id": 588, "year": 2022, "question_id_specific": 63, "full_question": "A 36-year-old male consults for conjunctival hyperemia and foreign body sensation. Which of the following pathologies does NOT correspond to the above mentioned examination?", "full_answer": "The involvement of the preauricular node is the typical lymphadenopathy of conjunctival infectious conditions. It is true that typical bacterial conjunctivitis does not usually present with lymphadenopathy. They have a more rapid and self-resolving course and do not awaken the lymphoid response as adenoviruses typically do. Or some more atypical bacteria that produce chronic conjunctivitis, such as Chlamydia, or much more infrequent bacteria such as those responsible for Parinaud's oculoglandular syndrome (Bartonella henselae, Francisella tularensis, Sporothrix schenckii, etc). Adenopathy being very typical of adenoviral conjunctivitis, this option is the easiest to rule out. Considering that chlamydiae arouse the lymphoid response in a similar way to adenoviruses (they also produce conjunctival follicles, for example), this would be the second easiest option to rule out. Perhaps the difficult thing is to rule out oculoglandular syndrome of Parinaud, because it is a very rare disease. If we do not realize that it is an infectious disease, we could hesitate. In any case, allergic conjunctivitis is not an infectious problem. So it is not going to have adenopathy. Even if we doubt with some of the previous options (especially with Parinaud), if we stay with the basic concepts (allergic conjunctivitis does not present with lymphadenopathies) we can be right. Option 2 is the correct one.", "type": "OPHTHALMOLOGY", "options": {"1": "Adenoviral conjunctivitis.", "2": "Allergic conjunctivitis.", "3": "Parinaud's oculoglandular syndrome.", "4": "Chlamydia conjunctivitis.", "5": NaN}, "correct_option": 2, "explanations": {"1": {"exist": true, "char_ranges": [[585, 684]], "word_ranges": [[78, 93]], "text": "Adenopathy being very typical of adenoviral conjunctivitis, this option is the easiest to rule out."}, "2": {"exist": true, "char_ranges": [[1180, 1405]], "word_ranges": [[174, 211]], "text": "Even if we doubt with some of the previous options (especially with Parinaud), if we stay with the basic concepts (allergic conjunctivitis does not present with lymphadenopathies) we can be right. Option 2 is the correct one."}, "3": {"exist": true, "char_ranges": [[1074, 1179]], "word_ranges": [[156, 174]], "text": "In any case, allergic conjunctivitis is not an infectious problem. So it is not going to have adenopathy."}, "4": {"exist": true, "char_ranges": [[685, 884]], "word_ranges": [[93, 123]], "text": "Considering that chlamydiae arouse the lymphoid response in a similar way to adenoviruses (they also produce conjunctival follicles, for example), this would be the second easiest option to rule out."}, "5": {"exist": false, "char_ranges": [], "word_ranges": [], "text": ""}}} {"id": 11, "year": 2011, "question_id_specific": 74, "full_question": "Ten days after performing a hemithyroidectomy on a patient with a 1.5 cm thyroid nodule you receive the following anatomopathologic report: \"very cellular follicular pattern without capsular invasion and infiltrating adjacent blood vessels and nerves\". Point out the correct answer:", "full_answer": "I think the correct answer is 3 answers 2,4 and 5 are clearly false. The doubt is between 1 and 2. The statement of 1 is correct in cases of follicular carcinoma without capsular invasion a total thyroidectomy can be performed. But if there are data of poor prognosis and the possibility of metastases - as in this case - it is best to complete the thyroidectomy and perform a radioactive iodine scan to rule out metastases or treat them if they are present.", "type": "SURGERY", "options": {"1": "As there is no capsular invasion, no further surgery is necessary.", "2": "Follicular pattern is an indication for prophylactic cervical lymph node emptying.", "3": "This is a follicular carcinoma and requires completion of thyroidectomy.", "4": "The report allows us to rule out with total certainty a papillary carcinoma in the remaining thyroid.", "5": "Measurement of plasma calcitonin will allow us to distinguish between papillary and follicular carcinoma."}, "correct_option": 3, "explanations": {"1": {"exist": true, "char_ranges": [[100, 458]], "word_ranges": [[21, 84]], "text": "The statement of 1 is correct in cases of follicular carcinoma without capsular invasion a total thyroidectomy can be performed. But if there are data of poor prognosis and the possibility of metastases - as in this case - it is best to complete the thyroidectomy and perform a radioactive iodine scan to rule out metastases or treat them if they are present."}, "2": {"exist": false, "char_ranges": [], "word_ranges": [], "text": ""}, "3": {"exist": true, "char_ranges": [[232, 458]], "word_ranges": [[42, 84]], "text": "if there are data of poor prognosis and the possibility of metastases - as in this case - it is best to complete the thyroidectomy and perform a radioactive iodine scan to rule out metastases or treat them if they are present."}, "4": {"exist": false, "char_ranges": [], "word_ranges": [], "text": ""}, "5": {"exist": false, "char_ranges": [], "word_ranges": [], "text": ""}}} {"id": 535, "year": 2021, "question_id_specific": 121, "full_question": "A 27-year-old black woman consults for the appearance of edema in the lower limbs, decreased diuresis, fever and a malar rash in butterfly wings of 20 days of evolution. Laboratory tests showed creatinine 3 mg/dl, leukopenia, hemoglobin 10.5 g/dl (normochromic normocytic), CRP 9 mg/dl and ESR 60 mm. Regarding the pathology presented by this patient, point out the correct answer:", "full_answer": "Another question about SLE antibodies. Clinical case of SLE with renal involvement, in which positive double-stranded anti-DNA increases the risk of nephritis. As for the rest, anti-Sm are specific to SLE, ANA are detected in more than 90% of cases, and anti-centromere are typical of scleroderma.", "type": "RHEUMATOLOGY", "options": {"1": "The anti-Sm antibodies are not specific for this pathology.", "2": "Anti-centromere antibodies are detected in 90% of cases.", "3": "ANA (antinuclear antibodies) are detected in 50% of cases.", "4": "Anti-two-stranded or native anti-DNA correlates with the risk of nephritis.", "5": NaN}, "correct_option": 4, "explanations": {"1": {"exist": true, "char_ranges": [[177, 205]], "word_ranges": [[26, 31]], "text": "anti-Sm are specific to SLE,"}, "2": {"exist": true, "char_ranges": [[254, 297]], "word_ranges": [[41, 46]], "text": "anti-centromere are typical of scleroderma."}, "3": {"exist": true, "char_ranges": [[206, 249]], "word_ranges": [[31, 40]], "text": "ANA are detected in more than 90% of cases,"}, "4": {"exist": true, "char_ranges": [[39, 159]], "word_ranges": [[5, 22]], "text": "Clinical case of SLE with renal involvement, in which positive double-stranded anti-DNA increases the risk of nephritis."}, "5": {"exist": false, "char_ranges": [], "word_ranges": [], "text": ""}}} {"id": 452, "year": 2018, "question_id_specific": 149, "full_question": "In a woman with an epileptic seizure presenting with the following clinical features: epigastric aura, unpleasant odor, disconnection from the environment, motor automatisms (sucking, swallowing, opening and closing of one hand) and postcritical amnesia, what is your diagnostic suspicion?", "full_answer": "Clearly the answer is 4, with a very characteristic clinic of temporary seizures.", "type": "NEUROLOGY", "options": {"1": "Generalized non-convulsive seizure or typical absence.", "2": "Continuous partial epilepsy.", "3": "Amyotonic crisis.", "4": "Complex partial temporal lobe seizure.", "5": NaN}, "correct_option": 4, "explanations": {"1": {"exist": false, "char_ranges": [], "word_ranges": [], "text": ""}, "2": {"exist": false, "char_ranges": [], "word_ranges": [], "text": ""}, "3": {"exist": false, "char_ranges": [], "word_ranges": [], "text": ""}, "4": {"exist": true, "char_ranges": [[0, 81]], "word_ranges": [[0, 13]], "text": "Clearly the answer is 4, with a very characteristic clinic of temporary seizures."}, "5": {"exist": false, "char_ranges": [], "word_ranges": [], "text": ""}}} {"id": 55, "year": 2011, "question_id_specific": 74, "full_question": "Ten days after performing a hemithyroidectomy on a patient with a 1.5 cm thyroid nodule you receive the following definitive report from the pathologist: \"very cellular follicular pattern without capsular invasion invading adjacent blood vessels and nerves\". Point out the correct answer:", "full_answer": "I find this question a bit more difficult since you have to have clear concepts of pathologic anatomy. In any case, it is possible to exclude by exclusion and by the way the answers are expressed: Answer 2: prophylactic emptying is indicated in the medullary. Answer 4: simply by the way of expressing it, it should already have been ruled out. Answer 5: you already know that calcitonin is used in medullary carcinoma and not in differentiated carcinoma. Answers 1 and 3 are contrary, one recommends completing thyroidectomy and the other does not. The only ones in which complete thyroidectomy is not recommended are microcarcinomas measuring less than 1 cm.", "type": "ENDOCRINOLOGY", "options": {"1": "As there is no capsular invasion it is not convenient to perform further surgery.", "2": "The follicular pattern is an indication for prophylactic cervical lymph node emptying.", "3": "It is a follicular carcinoma and requires completion of thyroidectomy.", "4": "The report allows to rule out papillary carcinoma with total certainty in the remaining thyroid.", "5": "Measurement of plasma calcitonin will allow us to differentiate between papillary and follicular carcinoma."}, "correct_option": 3, "explanations": {"1": {"exist": true, "char_ranges": [[456, 660]], "word_ranges": [[78, 110]], "text": "Answers 1 and 3 are contrary, one recommends completing thyroidectomy and the other does not. The only ones in which complete thyroidectomy is not recommended are microcarcinomas measuring less than 1 cm."}, "2": {"exist": true, "char_ranges": [[207, 259]], "word_ranges": [[38, 45]], "text": "prophylactic emptying is indicated in the medullary."}, "3": {"exist": true, "char_ranges": [[456, 660]], "word_ranges": [[78, 110]], "text": "Answers 1 and 3 are contrary, one recommends completing thyroidectomy and the other does not. The only ones in which complete thyroidectomy is not recommended are microcarcinomas measuring less than 1 cm."}, "4": {"exist": true, "char_ranges": [[270, 344]], "word_ranges": [[47, 61]], "text": "simply by the way of expressing it, it should already have been ruled out."}, "5": {"exist": true, "char_ranges": [[377, 455]], "word_ranges": [[67, 78]], "text": "calcitonin is used in medullary carcinoma and not in differentiated carcinoma."}}} {"id": 281, "year": 2016, "question_id_specific": 33, "full_question": "A 67-year-old woman diagnosed with an infiltrating ductal carcinoma of the breast and no family history of neoplasia. What additional studies should be performed on the tumor for its clinical-therapeutic implications?", "full_answer": "The correct answer would be 2, HORMONE AND HER2 RECEPTOR STUDY, since positive HER2 receptors imply greater aggressiveness of the tumor but treatment with trastuzumab and positive hormone receptors would imply the use of hormone therapy. Answer 1 can be ruled out visu visu, while answers 3 and 4 should be ruled out since, being the first woman in the family with breast carcinoma, the study of the patient's relatives would not be obligatory. Likewise, e-cadherin is important for the differential diagnosis of a lobular breast carcinoma, not a ductal one.", "type": "PATHOLOGICAL ANATOMY", "options": {"1": "Complete phenotypic study by flow cytometry.", "2": "Study of hormone receptors and HER2.", "3": "Study of hormone receptors, e-cadherin and study of first degree relatives.", "4": "Study of BRCA 1-2 and study of first-degree relatives.", "5": NaN}, "correct_option": 2, "explanations": {"1": {"exist": true, "char_ranges": [[238, 274]], "word_ranges": [[36, 44]], "text": "Answer 1 can be ruled out visu visu,"}, "2": {"exist": true, "char_ranges": [[70, 237]], "word_ranges": [[12, 36]], "text": "positive HER2 receptors imply greater aggressiveness of the tumor but treatment with trastuzumab and positive hormone receptors would imply the use of hormone therapy."}, "3": {"exist": true, "char_ranges": [[281, 444]], "word_ranges": [[45, 74]], "text": "answers 3 and 4 should be ruled out since, being the first woman in the family with breast carcinoma, the study of the patient's relatives would not be obligatory."}, "4": {"exist": true, "char_ranges": [[281, 444]], "word_ranges": [[45, 74]], "text": "answers 3 and 4 should be ruled out since, being the first woman in the family with breast carcinoma, the study of the patient's relatives would not be obligatory."}, "5": {"exist": false, "char_ranges": [], "word_ranges": [], "text": ""}}} {"id": 310, "year": 2016, "question_id_specific": 216, "full_question": "A parent comes to the emergency department with his 2-year-old son because he reports that he has hit his right eye while playing. Indeed, a hematoma is observed on the right eyelid, apparently of no importance. The fundus examination showed intraretinal hemorrhages not only in the eye referred by the parents but also in the other eye. It is noteworthy that the child appears drowsy and with little tone. Which of the following statements should you consider regarding this clinical picture?", "full_answer": "Easy question in principle, but with a bit of a catch. If you have the right intuition when you read it and go straight to what you think, you will probably get it right. The problem is option 4, which can confuse us. Let's see: we have a blow to the right eye area, with a hematoma on the eyelid. So far so normal. The problem is that retinal hemorrhages appear not only in that eye (which in itself is rare, because an external blow to the eye in a child rarely causes bleeding in the retina), but also in the other eye. Therefore, something other than the blow has caused hemorrhages in both retinas. Then we are told that the child seems drowsy and with little tone. Here we already have the two keys; we have to suspect what is not mentioned in the answers, but which is what the child has: the shaken baby syndrome (shaken baby syndrome). Characteristic of this syndrome are bilateral retinal hemorrhages, cerebral edema and subdural hematoma. There may also be fracture of long bones and other problems. Numbness and hypotonia allow us to suspect intracranial complications. The cause of shaken baby syndrome is almost always abuse, so the answer is 1. Options 2 and 3 are easy to rule out: nothing indicates perforating trauma, and it does not explain the hemorrhages in the other eye. And the berlin edema is an edema of the macula and appears as a whitish lesion, which is not described in the statement. The catch is option 4. Purstcher's retinopathy can also appear in the context of the child of the shaken child, and therefore occur in the case of child abuse. Seeing these two partly related questions, we may hesitate. However, if we read answer 4 carefully, it does not tell us that if there is child maltreatment Purstcher can occur. It states that the picture described above (intraretinal hemorrhages, without other findings) is characteristic of Purstcher's angiopathy. It is not. In Purstcher's there are hemorrhages, but also cottony exudates and edema. If there are only hemorrhages, it's not Purstcher's. Here's the messy bit, but it's really well worded. If he had a Purstcher it could also be for child abuse, but he doesn't.", "type": "OPHTHALMOLOGY", "options": {"1": "It is a highly suggestive history of child abuse.", "2": "It is the normal evolution of a non-perforating intraocular trauma.", "3": "The most likely diagnosis is Berlin edema.", "4": "It is the characteristic picture of Purstcher's traumatic retinal angiopathy.", "5": NaN}, "correct_option": 1, "explanations": {"1": {"exist": true, "char_ranges": [[1082, 1159]], "word_ranges": [[193, 208]], "text": "The cause of shaken baby syndrome is almost always abuse, so the answer is 1."}, "2": {"exist": true, "char_ranges": [[1160, 1293]], "word_ranges": [[208, 232]], "text": "Options 2 and 3 are easy to rule out: nothing indicates perforating trauma, and it does not explain the hemorrhages in the other eye."}, "3": {"exist": true, "char_ranges": [[1160, 1293]], "word_ranges": [[208, 232]], "text": "Options 2 and 3 are easy to rule out: nothing indicates perforating trauma, and it does not explain the hemorrhages in the other eye."}, "4": {"exist": true, "char_ranges": [[1902, 2029]], "word_ranges": [[334, 353]], "text": "In Purstcher's there are hemorrhages, but also cottony exudates and edema. If there are only hemorrhages, it's not Purstcher's."}, "5": {"exist": false, "char_ranges": [], "word_ranges": [], "text": ""}}} {"id": 293, "year": 2016, "question_id_specific": 214, "full_question": "A patient comes to the emergency department with erythroderma with fever and general malaise. On examination, the nails show the presence of nail pitting and distal yellowish areas in an oil stain. What is the primary disease that caused the condition?", "full_answer": "Any of the 4 options can be the cause of erythroderma, although the description of nail lesions highly suggestive of psoriasis makes this diagnosis the most likely.", "type": "DERMATOLOGY, VENEREOLOGY AND PLASTIC SURGERY", "options": {"1": "Cutaneous lymphoma.", "2": "Atopic dermatitis.", "3": "Psoriasis.", "4": "Ichthyosis.", "5": NaN}, "correct_option": 3, "explanations": {"1": {"exist": false, "char_ranges": [], "word_ranges": [], "text": ""}, "2": {"exist": false, "char_ranges": [], "word_ranges": [], "text": ""}, "3": {"exist": true, "char_ranges": [[64, 164]], "word_ranges": [[12, 27]], "text": "the description of nail lesions highly suggestive of psoriasis makes this diagnosis the most likely."}, "4": {"exist": false, "char_ranges": [], "word_ranges": [], "text": ""}, "5": {"exist": false, "char_ranges": [], "word_ranges": [], "text": ""}}} {"id": 132, "year": 2012, "question_id_specific": 229, "full_question": "A 54-year-old man, with a personal history of DM, hypertension and mild chronic renal insufficiency, presented with somnolence and right hemiparesis. Laboratory tests: creatinine 2.3 mg/dl, Hb 10.3 g/dl and platelets 20000 mm3, with normal coagulation. Cranial CT: ischemic lesions with microhemorrhagic necrosis. Subsequently, he began with progressive deterioration of the level of consciousness and increase of Cr, LDH and unconjugated bilirubin. Direct Coombs' test was negative. Schistocytes in smear. In view of the diagnostic suspicion, the following conduct is to be followed:", "full_answer": "Let's see: a patient with certain history who has: - Schistocytes in smear: indicates microangiopathic anemia, red cell rupture of mechanical cause, which can be seen mainly in PTT, autoimmune hemolytic anemia and DIC. - Bicytopenia: here the causes can be very, very varied. - Elevation of LDH and Bilirubin, which indicates a massive rupture of red blood cells. - Negative direct Coombs' test: essential data, indicating that there is no antibody that sticks to the red blood cells. With these wickers we are going to weave the basket: Answer 1: it cannot be because a hemorrhagic transformation does not give such a brutal elevation of LDH and Cr, in addition to not explaining schistocytosis. Answer 2: urgent plasmapheresis... in case of thrombotic thrombocytopenic purpura, yes; I like this answer because it justifies bicytopenia, since microangiopathic hemolytic anemia is produced by rupture of red blood cells when they are fragmented by ultralong von Willebrand factor multimers in case of deficiency of the ADAMST13 enzyme, resulting in brutal platelet aggregates and the flow cytometer does not count 400 platelets, but a molondron of platelets that form a mass, hence the thrombocytopenia. It also explains the negative direct Coombs, elevated LDH and Cr. Answer 3: a situation as critical as that patient's indicates that something big is going on and I would not sit and wait for glucocorticoids to take effect...and I do not know right now what pathology gives that symptomatology and has that treatment as standard. Answer 4: looking for a trigger for DIC...it would make sense if it were not for the fact that in DIC it is VERY characteristic the consumption of coagulation factors, which is not present in this case. Answer 5: a myeloma does not give schistocytosis nor does it have this symptomatology, they would have to give other data that would support this pathology, such as bone pain and hypercalcemia...", "type": "NEUROLOGY AND NEUROSURGERY", "options": {"1": "New cranial CT in the presence of suspected hemorrhagic transformation of ischemic stroke.", "2": "Initiate urgent plasmapheresis.", "3": "Beginning of treatment with glucocorticoids at a dose of 1 mg/kg/day.", "4": "Search for triggering cause of DIC.", "5": "Suspected multiple myeloma: bone marrow aspirate."}, "correct_option": 2, "explanations": {"1": {"exist": true, "char_ranges": [[538, 696]], "word_ranges": [[89, 115]], "text": "Answer 1: it cannot be because a hemorrhagic transformation does not give such a brutal elevation of LDH and Cr, in addition to not explaining schistocytosis."}, "2": {"exist": true, "char_ranges": [[697, 784]], "word_ranges": [[115, 126]], "text": "Answer 2: urgent plasmapheresis... in case of thrombotic thrombocytopenic purpura, yes;"}, "3": {"exist": true, "char_ranges": [[1270, 1429]], "word_ranges": [[201, 229]], "text": "Answer 3: a situation as critical as that patient's indicates that something big is going on and I would not sit and wait for glucocorticoids to take effect...and"}, "4": {"exist": true, "char_ranges": [[1534, 1736]], "word_ranges": [[246, 283]], "text": "Answer 4: looking for a trigger for DIC...it would make sense if it were not for the fact that in DIC it is VERY characteristic the consumption of coagulation factors, which is not present in this case."}, "5": {"exist": true, "char_ranges": [[1737, 1932]], "word_ranges": [[283, 315]], "text": "Answer 5: a myeloma does not give schistocytosis nor does it have this symptomatology, they would have to give other data that would support this pathology, such as bone pain and hypercalcemia..."}}} {"id": 49, "year": 2011, "question_id_specific": 159, "full_question": "An 8 year old boy with lameness of the right lower extremity, of insidious presentation and one month of evolution. There is no history of trauma or constitutional signs. On examination there is evidence of restriction to passive mobilization of the limb, especially in abduction and internal rotation. Of the following statements all are true EXCEPT one:", "full_answer": "The correct answer is 3. Perthes disease is more common and has a better prognosis in younger children.", "type": "PEDIATRICS", "options": {"1": "It is advisable to perform a hip X-ray.", "2": "In transient synovitis the onset is usually acute.", "3": "In Legg-Calvé-Perthes disease the prognosis is worse the younger the age of onset.", "4": "The cause may be a hypercoagulable state.", "5": "The therapeutic option will vary depending on the degree of involvement."}, "correct_option": 3, "explanations": {"1": {"exist": false, "char_ranges": [], "word_ranges": [], "text": ""}, "2": {"exist": false, "char_ranges": [], "word_ranges": [], "text": ""}, "3": {"exist": true, "char_ranges": [[25, 103]], "word_ranges": [[5, 18]], "text": "Perthes disease is more common and has a better prognosis in younger children."}, "4": {"exist": false, "char_ranges": [], "word_ranges": [], "text": ""}, "5": {"exist": false, "char_ranges": [], "word_ranges": [], "text": ""}}} {"id": 120, "year": 2012, "question_id_specific": 192, "full_question": "A 40-year-old orthopedic surgeon who in a routine serology was found to have anti-HBs levels of 30 IU/l. He refers complete vaccination according to the standard hepatitis B vaccination schedule 4 years ago. It would be advisable:", "full_answer": "As long as the serological levels of anti-HBs are above 10 IU/l it is not necessary to revaccinate.", "type": "PREVENTIVE MEDICINE AND EPIDEMIOLOGY", "options": {"1": "Restart vaccination schedule (0-1-6 months).", "2": "Restart vaccination schedule (0-1-2-12 months).", "3": "Do not revaccinate.", "4": "Give a booster dose.", "5": "Give anti-hepatitis B immunoglobulin and restart vaccination schedule (0-1-2-12 months)."}, "correct_option": 3, "explanations": {"1": {"exist": false, "char_ranges": [], "word_ranges": [], "text": ""}, "2": {"exist": false, "char_ranges": [], "word_ranges": [], "text": ""}, "3": {"exist": true, "char_ranges": [[0, 99]], "word_ranges": [[0, 18]], "text": "As long as the serological levels of anti-HBs are above 10 IU/l it is not necessary to revaccinate."}, "4": {"exist": false, "char_ranges": [], "word_ranges": [], "text": ""}, "5": {"exist": false, "char_ranges": [], "word_ranges": [], "text": ""}}} {"id": 304, "year": 2016, "question_id_specific": 174, "full_question": "87-year-old man with a history of hypertension and gonarthrosis. Baseline situation with full functional and cognitive autonomy that allows him to continue living alone in the community. He is receiving regular treatment with perindopril and thiazide diuretic for blood pressure control and routinely takes ibuprofen 1800 mg/day to control the symptoms derived from his gonarthrosis. After routine control, a persistent blood pressure of 190 and TAD 80 mmHg was observed. What would be the most reasonable therapeutic modification to achieve blood pressure control?", "full_answer": "Before intensifying antihypertensive treatment, the potential causes should be resolved. In this case we are dealing with an elderly patient with chronic treatment with NSAIDs that produces arterial hypertension secondary to renal hypoperfusion. The most reasonable option would be to withdraw ibuprofen and replace it with an analgesic from a group other than NSAIDs (paracetamol, opiates), subsequently monitoring the evolution of blood pressure.", "type": "NEPHROLOGY", "options": {"1": "I would add a calcium channel blocker.", "2": "I would increase the dose of hydrochlorothiazide to 25 mg/day.", "3": "I would change ibuprofen for paracetamol to avoid the possible influence of paracetamol on the effect of hypotensives.", "4": "I would add an alpha blocker because of the high prevalence of prostatic syndrome in males of this age.", "5": NaN}, "correct_option": 3, "explanations": {"1": {"exist": false, "char_ranges": [], "word_ranges": [], "text": ""}, "2": {"exist": false, "char_ranges": [], "word_ranges": [], "text": ""}, "3": {"exist": true, "char_ranges": [[246, 448]], "word_ranges": [[33, 63]], "text": "The most reasonable option would be to withdraw ibuprofen and replace it with an analgesic from a group other than NSAIDs (paracetamol, opiates), subsequently monitoring the evolution of blood pressure."}, "4": {"exist": false, "char_ranges": [], "word_ranges": [], "text": ""}, "5": {"exist": false, "char_ranges": [], "word_ranges": [], "text": ""}}} {"id": 214, "year": 2014, "question_id_specific": 228, "full_question": "A 19-year-old man consults for a 24-hour history of pain, swelling and functional impotence of the right knee accompanied by a fever of 38°C. Physical examination reveals inflammatory signs and joint effusion in the right knee. Laboratory tests showed leukocytosis with neutrophilia and elevated C-reactive protein. The syndromic diagnosis of acute monoarthritis is made. What is the most likely etiologic diagnosis?", "full_answer": "Acute monoarthritis associated with fever, leukocytosis with neutrophilia and increased acute phase reactants does not always have a septic origin. In the absence of further information (more complete anamnesis on the current disease, risk factors, personal and family history, extra-articular symptoms or signs, etc.) it can be said that also 1 and 2 (and very exceptionally 5) could debut with a similar clinical and biological picture. With the data provided and taking into account that this is a young male, the most likely option would be bacterial infectious arthritis (that caused by mycobacteria usually have a chronic course). And above all, because of its implications, the first one to always rule out.", "type": "RHEUMATOLOGY", "options": {"1": "Arthritis due to microcrystals.", "2": "Reactive arthritis.", "3": "Bacterial infectious arthritis.", "4": "Mycobacterial infectious arthritis.", "5": "Rheumatoid arthritis."}, "correct_option": 3, "explanations": {"1": {"exist": true, "char_ranges": [[637, 714]], "word_ranges": [[98, 112]], "text": "And above all, because of its implications, the first one to always rule out."}, "2": {"exist": false, "char_ranges": [], "word_ranges": [], "text": ""}, "3": {"exist": true, "char_ranges": [[466, 575]], "word_ranges": [[71, 89]], "text": "taking into account that this is a young male, the most likely option would be bacterial infectious arthritis"}, "4": {"exist": true, "char_ranges": [[576, 635]], "word_ranges": [[89, 98]], "text": "(that caused by mycobacteria usually have a chronic course)."}, "5": {"exist": false, "char_ranges": [], "word_ranges": [], "text": ""}}} {"id": 249, "year": 2014, "question_id_specific": 120, "full_question": "A 53-year-old woman consults for a fever of 15 days of evolution, without symptoms of infectious focality. On examination, a painful hepatomegaly is detected at 5 cm from the costal margin and the spleen is palpated at 14 cm from the left costal margin. The blood count showed Hb 8.5 g/dL, leukocytes 630/ml (lymphocytes 63%, monocytes 20%, neutrophils 17%) and platelets 35,000/ml. Biochemistry shows moderate elevation of liver biochemistry, LDH is normal and polyclonal hypergammaglobulinemia (3.5 g/dL) is observed in the proteinogram. He has a history of known HIV infection for 10 years and irregular adherence to antiretroviral treatment, with recent determinations of CD4 lymphocytes 350 cells/mL and HIV viral load of 154 copies/mL. For the past 3 months she has been treated for seronegative symmetrical polyarthritis with 10-20 mg/day of prednisone. Which of the following statements is correct?", "full_answer": "Easy and quite typical question. We are told about an HIV patient with irregular follow-up of antiretroviral treatment, presenting fever, hepatosplenomegaly, pancytopenia and polyclonal hypergammaglobulinemia. All these data are suggestive of visceral Leishmaniasis, so an OM biopsy should be performed to look for amastigotes and confirm the diagnosis.", "type": "INFECTIOUS DISEASES", "options": {"1": "I would perform a bone marrow biopsy, since the most probable diagnosis is visceral leishmaniasis.", "2": "Pancytopenia is justified by C virus-associated cirrhosis and I would not perform further testing.", "3": "Probably a medullary toxicity due to prednisone that would be treated with drug withdrawal and filgastrim.", "4": "I would request an ANAs determination to rule out disseminated systemic lupus.", "5": "I would intensify antiretroviral therapy, as it is likely that all manifestations are due to HIV."}, "correct_option": 1, "explanations": {"1": {"exist": true, "char_ranges": [[210, 353]], "word_ranges": [[25, 48]], "text": "All these data are suggestive of visceral Leishmaniasis, so an OM biopsy should be performed to look for amastigotes and confirm the diagnosis."}, "2": {"exist": false, "char_ranges": [], "word_ranges": [], "text": ""}, "3": {"exist": false, "char_ranges": [], "word_ranges": [], "text": ""}, "4": {"exist": false, "char_ranges": [], "word_ranges": [], "text": ""}, "5": {"exist": false, "char_ranges": [], "word_ranges": [], "text": ""}}} {"id": 53, "year": 2011, "question_id_specific": 146, "full_question": "A 31-year-old man with no psychiatric history comes to the emergency room complaining...¨ symptoms of anxiety. Two days ago he experienced another episode of the same symptoms... .... 1 week ago he broke up with his partner... ruled out any organic pathology....:", "full_answer": "Here it is good that they tell you their antecedents (we all have antecedents because we all have a life) and that they rule out organic pathology. That is fundamental. Let us not forget that also the hypochondriacs have body. We first eliminate the most striking \"acute\" personality disorder. Personality accompanies us all our lives. There are no \"acute\" characteristics in our personality. Depressive disorder? There is a time criterion that is not met, it may be sad, but sad is not depression (1). agoraphobia? if it was something like that we would have been told the context post-traumatic stress reaction? That's a good one. The correct term would be post-traumatic stress disorder or acute stress reaction, and according to ICD 10: \"The stressor may be a devastating traumatic experience involving a serious threat to the safety or physical integrity of the patient or loved one(s) (e.g., natural catastrophes, accidents, battles, muggings, rapes) or an abrupt and threatening change in the individual's rank or social environment (e.g., loss of several loved ones, house fire, etc.). )\" or more briefly: \"exceptionally threatening or catastrophic nature\" Breaking up with a partner I understand is hard, but not exceptional. So by elimination 5. Crisis of distress. This is a useful question, common in daily practice, and so we see that we must avoid giving a diagnosis of long evolution for a natural reaction. Diagnoses 1 to 4 would be like calling diabetic the one who has hyperglycemia after eating three muffins. Useful and realistic question.", "type": "PSYCHIATRY", "options": {"1": "Post-traumatic stress reaction.", "2": "Acute\" dependent personality disorder.", "3": "Depressive disorder.", "4": "Agoraphobia.", "5": "Anguish crisis."}, "correct_option": 5, "explanations": {"1": {"exist": true, "char_ranges": [[1166, 1235]], "word_ranges": [[184, 196]], "text": "Breaking up with a partner I understand is hard, but not exceptional."}, "2": {"exist": true, "char_ranges": [[227, 392]], "word_ranges": [[40, 63]], "text": "We first eliminate the most striking \"acute\" personality disorder. Personality accompanies us all our lives. There are no \"acute\" characteristics in our personality."}, "3": {"exist": true, "char_ranges": [[414, 497]], "word_ranges": [[65, 83]], "text": "There is a time criterion that is not met, it may be sad, but sad is not depression"}, "4": {"exist": true, "char_ranges": [[516, 581]], "word_ranges": [[85, 98]], "text": "if it was something like that we would have been told the context"}, "5": {"exist": false, "char_ranges": [], "word_ranges": [], "text": ""}}} {"id": 48, "year": 2011, "question_id_specific": 158, "full_question": "A 10-year-old boy is brought to the Emergency Department because for the last 2 hours he has been involuntarily turning his neck to the right associated with marked cervical pain at each turn. The grandmother subsequently tells us that he has been vomiting since yesterday, so she gave him a syrup; what would be the most appropriate therapeutic approach?", "full_answer": "The correct answer is 1. The anticholinergic can be administered intravenously and also intramuscularly if I am not mistaken. The occurrence of dystonia as a side effect is common in children and is one of the main reasons why pediatricians do not usually prescribe some antiemetics in children.", "type": "PEDIATRICS", "options": {"1": "Inject an intravenous anticholinergic.", "2": "Do not treat until an electroencephalogram is performed in the following days.", "3": "Do not treat until the result of the cerebrospinal fluid culture is known in the next few days.", "4": "Call the on-call psychiatrist.", "5": "Start an intravenous antibiotic after obtaining a pharyngeal swab."}, "correct_option": 1, "explanations": {"1": {"exist": false, "char_ranges": [], "word_ranges": [], "text": ""}, "2": {"exist": false, "char_ranges": [], "word_ranges": [], "text": ""}, "3": {"exist": false, "char_ranges": [], "word_ranges": [], "text": ""}, "4": {"exist": false, "char_ranges": [], "word_ranges": [], "text": ""}, "5": {"exist": false, "char_ranges": [], "word_ranges": [], "text": ""}}} {"id": 490, "year": 2020, "question_id_specific": 106, "full_question": "A 65-year-old woman referred to the emergency department for fever and alterations in the laboratory tests: hemoglobin 11.4 g/dL, leukocytes 0.86 x103/μL,(neutrophils 41.9%, lymphocytes 55.8%),platelets 48.0 x103/μL, fibrinogen 118 mg/dL,D-dimer 20.2 μg/mL. Bone marrow examination was performed and she was diagnosed with acute leukemia with t(15;17) in 60% of the cells. Which of the following answers is correct?", "full_answer": "It is a definitive promyelocytic leukemia with t(15;17), and as it is well known by all of you the treatment is with arsenic trioxide + ATRA. Option 1 leaves the treatment incomplete, there is only ATRA and it must be treated as long as the patient's condition allows it, whether or not there are symptoms. Option 3 3 years ago would have been the correct option but nowadays neither chemotherapy nor heparin is used. And option 4 is false because that fever is due to the leukemia itself and the priority is to start specific treatment for leukemia.", "type": "HEMATOLOGY", "options": {"1": "If asymptomatic, transretinoic acid (ATRA) will be started and day hospital controls will be recommended.", "2": "Start treatment with arsenic trioxide, ATRA and supportive therapy.", "3": "It is a myeloblastic leukemia type M3, so chemotherapy and heparin will be started to control disseminated intravascular coagulation.", "4": "Antibiotic treatment should be started. When the fever disappears, leukemia treatment should be started.", "5": NaN}, "correct_option": 2, "explanations": {"1": {"exist": true, "char_ranges": [[142, 306]], "word_ranges": [[26, 55]], "text": "Option 1 leaves the treatment incomplete, there is only ATRA and it must be treated as long as the patient's condition allows it, whether or not there are symptoms."}, "2": {"exist": true, "char_ranges": [[0, 141]], "word_ranges": [[0, 26]], "text": "It is a definitive promyelocytic leukemia with t(15;17), and as it is well known by all of you the treatment is with arsenic trioxide + ATRA."}, "3": {"exist": true, "char_ranges": [[307, 417]], "word_ranges": [[55, 74]], "text": "Option 3 3 years ago would have been the correct option but nowadays neither chemotherapy nor heparin is used."}, "4": {"exist": true, "char_ranges": [[422, 550]], "word_ranges": [[75, 98]], "text": "option 4 is false because that fever is due to the leukemia itself and the priority is to start specific treatment for leukemia."}, "5": {"exist": false, "char_ranges": [], "word_ranges": [], "text": ""}}} {"id": 416, "year": 2018, "question_id_specific": 76, "full_question": "A 78-year-old woman with dementia and institutionalized is brought in by her caregivers for significant abdominal pain with deterioration of general condition and abdominal distension. CBC shows leukocytosis, elevated hematocrit, renal failure and metabolic acidosis. ECG shows atrial fibrillation. Abdominal tomography shows edematous small bowel loops, with intestinal and portal accumulation. The most probable diagnosis is:", "full_answer": "This is a classic picture and the author of the question has taken care to give us enough clues so that we do not miss it. When reading this statement, multiple alarms go off: - Atrial Fibrillation: at the mir and at the ED door, any elderly person with AF and abdominal pain is emboligenic mesenteric ischemia until proven otherwise. - Acute abdominal pain, SIRS, renal failure, metabolic acidosis (ischemia!), probably an elevated lactic... - CT: The finding of portal gas is also highly indicative of mesenteric ischemia, we could also have been told of intestinal pneumatosis. Why are not the other pictures? - Perforated ulcus: We would have been told of a history of NSAID use, or a history of pain that improves with ingestion, of pneumoperitoneum on chest X-ray. - Biliary ileus: There would be a history of biliary colic or pain in the right hypochondrium, they would also speak of aerobilia in the imaging test and dilatation of the small loops with image (or not) of biliary lithiasis in the terminal ileum. - Neo sigmoidoscopy: In the myocardial imaging they would speak of weight loss, change of stool habit, pneumoperitoneum in Rx or CT and peritonitic abdominal exploration and may even be able to palpate the mass.", "type": "GENERAL SURGERY", "options": {"1": "Perforation of gastric or duodenal ulcus.", "2": "Biliary leak.", "3": "Obstructive neoplasm of the sigma with perforation.", "4": "Mesenteric ischemia.", "5": NaN}, "correct_option": 4, "explanations": {"1": {"exist": true, "char_ranges": [[633, 770]], "word_ranges": [[106, 131]], "text": "We would have been told of a history of NSAID use, or a history of pain that improves with ingestion, of pneumoperitoneum on chest X-ray."}, "2": {"exist": true, "char_ranges": [[788, 1018]], "word_ranges": [[134, 175]], "text": "There would be a history of biliary colic or pain in the right hypochondrium, they would also speak of aerobilia in the imaging test and dilatation of the small loops with image (or not) of biliary lithiasis in the terminal ileum."}, "3": {"exist": true, "char_ranges": [[1040, 1230]], "word_ranges": [[178, 210]], "text": "In the myocardial imaging they would speak of weight loss, change of stool habit, pneumoperitoneum in Rx or CT and peritonitic abdominal exploration and may even be able to palpate the mass."}, "4": {"exist": true, "char_ranges": [[214, 334]], "word_ranges": [[41, 60]], "text": "at the ED door, any elderly person with AF and abdominal pain is emboligenic mesenteric ischemia until proven otherwise."}, "5": {"exist": false, "char_ranges": [], "word_ranges": [], "text": ""}}} {"id": 45, "year": 2011, "question_id_specific": 154, "full_question": "A 9-year-old boy, asymptomatic with an innocent murmur, undergoes an ECG that documents Wolf-Parkinson-White Syndrome. Which of the following statements is FALSE?", "full_answer": "The correct answer is 1. It is also answered with the statement that begins by saying that the child is asymptomatic. The error seems to me to be a misconception because for the study of a murmur an ECG is not usually requested but an echocardiography. Heart failure is a complication of Wolf-Parkinson-White syndrome and rarely a debut form of the disease.", "type": "PEDIATRICS", "options": {"1": "He needs treatment for heart failure.", "2": "She may present with paroxysmal supraventricular tachycardia.", "3": "The 2D-Doppler ultrasound will rule out its association with Ebstein's disease.", "4": "It is convenient to perform a 24-hour ECG (Holter) and ergometry.", "5": "In certain patients, ablation of the accessory pathway with radiofrequency constitutes the therapeutic action."}, "correct_option": 1, "explanations": {"1": {"exist": true, "char_ranges": [[25, 117]], "word_ranges": [[5, 21]], "text": "It is also answered with the statement that begins by saying that the child is asymptomatic."}, "2": {"exist": false, "char_ranges": [], "word_ranges": [], "text": ""}, "3": {"exist": false, "char_ranges": [], "word_ranges": [], "text": ""}, "4": {"exist": false, "char_ranges": [], "word_ranges": [], "text": ""}, "5": {"exist": false, "char_ranges": [], "word_ranges": [], "text": ""}}} {"id": 336, "year": 2016, "question_id_specific": 31, "full_question": "A 20-year-old woman with a 15-cm solid-cystic ovarian tumor detected by ultrasound after presenting with nonspecific abdominal symptoms. In the histopathological study of the corresponding specimen, teeth, hairs, areas of intestinal epithelium, areas of squamous epithelium (15%) and bronchial epithelium, as well as neuroectodermal and embryonal elements are found in several of the histological preparations. In reference to this case, point out the correct diagnosis:", "full_answer": "Mature cystic teratoma is a common benign tumor of the ovary in adult women. The tissues that make up the tumor are well differentiated (mature or adult-type); apart from cutaneous structures many other tissues can be seen, particularly in a thickening or spur that eminences into the cavity, in which there are often teeth, cartilage and bone.", "type": "GYNECOLOGY AND OBSTETRICS", "options": {"1": "Teratocarcinoma.", "2": "Immature teratoma.", "3": "Mature cystic teratoma.", "4": "Dysgerminoma.", "5": NaN}, "correct_option": 3, "explanations": {"1": {"exist": false, "char_ranges": [], "word_ranges": [], "text": ""}, "2": {"exist": false, "char_ranges": [], "word_ranges": [], "text": ""}, "3": {"exist": true, "char_ranges": [[0, 344]], "word_ranges": [[0, 57]], "text": "Mature cystic teratoma is a common benign tumor of the ovary in adult women. The tissues that make up the tumor are well differentiated (mature or adult-type); apart from cutaneous structures many other tissues can be seen, particularly in a thickening or spur that eminences into the cavity, in which there are often teeth, cartilage and bone."}, "4": {"exist": false, "char_ranges": [], "word_ranges": [], "text": ""}, "5": {"exist": false, "char_ranges": [], "word_ranges": [], "text": ""}}} {"id": 609, "year": 2022, "question_id_specific": 115, "full_question": "A 27-year-old male, a regular athlete, refers pain in the right leg after continuous running. He has visited a physiotherapist on several occasions and has been diagnosed with calf overload. Several months have passed, she has not improved and refers intense pain after physical activity that subsides with rest in the hours following exercise. What test can help in the diagnosis?", "full_answer": "Exercise-induced compartment syndrome is an exercise-induced condition of the leg. It is characterized by reversible ischemia of the muscles of a muscle compartment. Diagnosis is made by measuring compartment pressures at rest, during exercise and after exercise (answer 2 correct). Treatment usually consists of fasciotomies of the affected compartments. Although MRI is not very useful in establishing the diagnosis, it can help in the differential diagnosis.", "type": "TRAUMATOLOGY", "options": {"1": "Positron emission tomography with 18 FDG.", "2": "Determination of posterior compartment pressure immediately after activity.", "3": "Doppler ultrasound to rule out a circulatory disorder of the lower extremity.", "4": "Magnetic resonance spectroscopy.", "5": NaN}, "correct_option": 2, "explanations": {"1": {"exist": false, "char_ranges": [], "word_ranges": [], "text": ""}, "2": {"exist": true, "char_ranges": [[0, 282]], "word_ranges": [[0, 40]], "text": "Exercise-induced compartment syndrome is an exercise-induced condition of the leg. It is characterized by reversible ischemia of the muscles of a muscle compartment. Diagnosis is made by measuring compartment pressures at rest, during exercise and after exercise (answer 2 correct)."}, "3": {"exist": false, "char_ranges": [], "word_ranges": [], "text": ""}, "4": {"exist": true, "char_ranges": [[356, 461]], "word_ranges": [[49, 66]], "text": "Although MRI is not very useful in establishing the diagnosis, it can help in the differential diagnosis."}, "5": {"exist": false, "char_ranges": [], "word_ranges": [], "text": ""}}} {"id": 389, "year": 2016, "question_id_specific": 235, "full_question": "A 3-month-old infant correctly vaccinated for his age, who after two weeks of rhinorrhea, sneezing and coughing, is admitted for intensification of coughing spells, with cyanosis at the end of them, ending with deep inspiration or inspiratory rooster, requiring stimulation, aspiration of secretions and oxygen to recover from them. According to your diagnostic suspicion and with respect to the vaccine for this disease, all are true EXCEPT:", "full_answer": "We are being asked about the pertussis vaccine. Option 1 is correct, it is the diphtheria-tetanus-pertussis vaccine. Option 4 is also correct, since they are inactivated microorganisms. Now we are left with the doubt between 2 and 3... Option 3 is the one that \"is not true among all those proposed to us\": immunity lasts about 10 years after the last dose.", "type": "PEDIATRICS", "options": {"1": "The vaccine in Spain is administered in combination with diphtheria and tetanus.", "2": "Adults who will be in contact with infants under 6 months of age should be vaccinated.", "3": "Both natural and vaccinal immunity remains for life.", "4": "Vaccination is prepared from killed microorganisms.", "5": NaN}, "correct_option": 3, "explanations": {"1": {"exist": true, "char_ranges": [[0, 116]], "word_ranges": [[0, 17]], "text": "We are being asked about the pertussis vaccine. Option 1 is correct, it is the diphtheria-tetanus-pertussis vaccine."}, "2": {"exist": false, "char_ranges": [], "word_ranges": [], "text": ""}, "3": {"exist": true, "char_ranges": [[236, 357]], "word_ranges": [[38, 62]], "text": "Option 3 is the one that \"is not true among all those proposed to us\": immunity lasts about 10 years after the last dose."}, "4": {"exist": true, "char_ranges": [[117, 185]], "word_ranges": [[17, 27]], "text": "Option 4 is also correct, since they are inactivated microorganisms."}, "5": {"exist": false, "char_ranges": [], "word_ranges": [], "text": ""}}} {"id": 201, "year": 2013, "question_id_specific": 76, "full_question": "A patient who presents with problems in understanding spoken and written language, inability to name objects and repeat words that are said to him, incomprehensible fluent speech with semantic and phonemic paraphasias. This is a:", "full_answer": "Transcortical have preserved repetition. Broca's aphasia has preserved comprehension and absence of fluent speech. Wernicke's aphasia is characterized by fluent speech (even in excess) with paraphasias and problems with comprehension, nomination and repetition.", "type": "NEUROLOGY", "options": {"1": "Global aphasia.", "2": "Wernicke's aphasia.", "3": "Broca's aphasia.", "4": "Sensitive transcortical aphasia.", "5": "Transcortical motor aphasia."}, "correct_option": 2, "explanations": {"1": {"exist": false, "char_ranges": [], "word_ranges": [], "text": ""}, "2": {"exist": true, "char_ranges": [[115, 261]], "word_ranges": [[14, 33]], "text": "Wernicke's aphasia is characterized by fluent speech (even in excess) with paraphasias and problems with comprehension, nomination and repetition."}, "3": {"exist": true, "char_ranges": [[41, 114]], "word_ranges": [[4, 14]], "text": "Broca's aphasia has preserved comprehension and absence of fluent speech."}, "4": {"exist": true, "char_ranges": [[0, 40]], "word_ranges": [[0, 4]], "text": "Transcortical have preserved repetition."}, "5": {"exist": true, "char_ranges": [[0, 40]], "word_ranges": [[0, 4]], "text": "Transcortical have preserved repetition."}}} {"id": 345, "year": 2016, "question_id_specific": 159, "full_question": "A 69-year-old woman comes to your office reporting genital bleeding of several months' evolution. She denies hormone replacement therapy and anticoagulation. She provides normal cervicovaginal cytology. General and genital physical examination without findings of interest. BMI of 38kg/m2. Indicate the most correct attitude:", "full_answer": "We are being presented with a case of postmenopausal metrorrhagia. In a case like this we must rule out endometrial neoplasia, so an endometrial biopsy would be indicated, either by Cornier cannula if possible, or by hysteroscopy. A cervical biopsy is not of interest in this case since the cytology is normal and hormonal assessment is not going to be useful in the diagnosis of endometrial neoplasia. Finally, prescribing cyclic progesterone is not indicated in a postmenopausal woman.", "type": "GYNECOLOGY AND OBSTETRICS", "options": {"1": "Prescribe cyclic progesterone.", "2": "Endometrial biopsy.", "3": "Random cervical biopsies.", "4": "Hormonal evaluation with FSH, LH and estradiol.", "5": NaN}, "correct_option": 2, "explanations": {"1": {"exist": true, "char_ranges": [[412, 487]], "word_ranges": [[68, 78]], "text": "prescribing cyclic progesterone is not indicated in a postmenopausal woman."}, "2": {"exist": true, "char_ranges": [[0, 171]], "word_ranges": [[0, 28]], "text": "We are being presented with a case of postmenopausal metrorrhagia. In a case like this we must rule out endometrial neoplasia, so an endometrial biopsy would be indicated,"}, "3": {"exist": true, "char_ranges": [[231, 309]], "word_ranges": [[37, 52]], "text": "A cervical biopsy is not of interest in this case since the cytology is normal"}, "4": {"exist": true, "char_ranges": [[314, 402]], "word_ranges": [[53, 67]], "text": "hormonal assessment is not going to be useful in the diagnosis of endometrial neoplasia."}, "5": {"exist": false, "char_ranges": [], "word_ranges": [], "text": ""}}} {"id": 166, "year": 2013, "question_id_specific": 82, "full_question": "A 45-year-old woman presents arterial hypertension (190/120 mmHg) accompanied by K 2.5 mEq/l. An abdominal ultrasound shows stenosis of both renal arteries. Indicate which treatment is contraindicated:", "full_answer": "If the renal arteries are stenosed, blood has difficulty reaching the kidneys. That is, both are poorly perfused and can only create the pressure gradient necessary for filtration by contraction of the efferent arteriole. If we administer an ACEI, we will provoke the relaxation of this arteriole and the fall of the glomerular filtration rate. And if neither kidney filters, because both have arterial stenosis, we have a problem....", "type": "CARDIOLOGY AND CARDIOVASCULAR SURGERY", "options": {"1": "Enalapril.", "2": "Propanolol.", "3": "Amiloride.", "4": "Prazosin.", "5": "Amlodipine."}, "correct_option": 1, "explanations": {"1": {"exist": true, "char_ranges": [[222, 344]], "word_ranges": [[34, 55]], "text": "If we administer an ACEI, we will provoke the relaxation of this arteriole and the fall of the glomerular filtration rate."}, "2": {"exist": false, "char_ranges": [], "word_ranges": [], "text": ""}, "3": {"exist": false, "char_ranges": [], "word_ranges": [], "text": ""}, "4": {"exist": false, "char_ranges": [], "word_ranges": [], "text": ""}, "5": {"exist": false, "char_ranges": [], "word_ranges": [], "text": ""}}} {"id": 481, "year": 2020, "question_id_specific": 171, "full_question": "A 67-year-old woman with a history of dyslipidemia, who comes to the ED with dysuria and pollakiuria followed by fever, chills, and deterioration of general condition. On arrival she appears severe and is tachycardic, tachypneic, with blood pressure 60/40 mmHg and temperature of 39°C. Which of the following measures would NOT be included in the INITIAL management?", "full_answer": "Patient presenting with fever+tachypnea+FC>100 = 3 criteria of SIRS (Systemic Inflammatory Response Syndrome). Reminder: sepsis is a SIRS due to infection, which leads to a decrease in SVR, resulting in a decrease in preload and stroke volume leading to hypotension. The first therapeutic measures are the administration of empirical broad-spectrum antibiotics and fluid replacement (answer 4) to try to compensate for this decrease in preload. Of choice, balanced crystalloids and albumin solutions [1] But when this is insufficient, we must try to correct the decrease in PVR by using vasoactive drugs [2]. But the vasoactive drug of choice in the context of septic shock is noradrenaline ([3], [4]) (answer 1 FALSE) A minimum of two blood cultures is mandatory (answer 3), for the diagnosis of bacteremia and facilitating a targeted antibiotic treatment. [2] On the other hand, it is interesting to determine blood lactic acid levels for two purposes: it is currently considered the best marker of tissue hypoperfusion/hypoxia and it predicts the response to treatment[2] (answer 2).", "type": "CRITICAL CARE", "options": {"1": "Intravenous perfusion of dobutamine.", "2": "Serum lactate measurement.", "3": "Extraction of blood cultures.", "4": "Fluid administration.", "5": NaN}, "correct_option": 1, "explanations": {"1": {"exist": true, "char_ranges": [[613, 718]], "word_ranges": [[93, 111]], "text": "the vasoactive drug of choice in the context of septic shock is noradrenaline ([3], [4]) (answer 1 FALSE)"}, "2": {"exist": true, "char_ranges": [[862, 1086]], "word_ranges": [[133, 168]], "text": "On the other hand, it is interesting to determine blood lactic acid levels for two purposes: it is currently considered the best marker of tissue hypoperfusion/hypoxia and it predicts the response to treatment[2] (answer 2)."}, "3": {"exist": true, "char_ranges": [[719, 857]], "word_ranges": [[111, 132]], "text": "A minimum of two blood cultures is mandatory (answer 3), for the diagnosis of bacteremia and facilitating a targeted antibiotic treatment."}, "4": {"exist": true, "char_ranges": [[121, 444]], "word_ranges": [[14, 65]], "text": "sepsis is a SIRS due to infection, which leads to a decrease in SVR, resulting in a decrease in preload and stroke volume leading to hypotension. The first therapeutic measures are the administration of empirical broad-spectrum antibiotics and fluid replacement (answer 4) to try to compensate for this decrease in preload."}, "5": {"exist": false, "char_ranges": [], "word_ranges": [], "text": ""}}} {"id": 612, "year": 2022, "question_id_specific": 116, "full_question": "95-year-old woman living in a nursing home, independent for her basic activities of daily living, goes out to the garden for walks. She has a history of hypertension, dyslipidemia, osteoporosis and mild cognitive impairment. She suffers a fall when getting up at night to go to the bathroom. X-ray shows a displaced subcapital fracture of the right hip. What is the recommended treatment?", "full_answer": "Correct Answer 1: Given that this is a displaced subcapital hip fracture, the surgical treatment we should consider is hip joint replacement, given that the injury described presents a high probability of necrosis of the femoral head. The options would be total or partial arthroplasty: in elderly patients, partial arthroplasty is preferred because it is a shorter and less aggressive surgery than total hip arthroplasty. Answer 2 incorrect: We would consider this technique in basicervical fractures and trochanteric mass lesions, not in subcapital fractures. Incorrect Answer 3: We would consider this technique in nondisplaced subcapital fractures in young patients. Incorrect answer 4: We would only consider non-surgical treatment in a patient with a very poor baseline general condition: bedridden patients with low life expectancy.", "type": "TRAUMATOLOGY", "options": {"1": "Hip hemiarthroplasty.", "2": "Trochanteric nail fixation.", "3": "Fixation with cannulated screws.", "4": "Conservative: bed-chair life.", "5": NaN}, "correct_option": 1, "explanations": {"1": {"exist": true, "char_ranges": [[0, 422]], "word_ranges": [[0, 65]], "text": "Correct Answer 1: Given that this is a displaced subcapital hip fracture, the surgical treatment we should consider is hip joint replacement, given that the injury described presents a high probability of necrosis of the femoral head. The options would be total or partial arthroplasty: in elderly patients, partial arthroplasty is preferred because it is a shorter and less aggressive surgery than total hip arthroplasty."}, "2": {"exist": true, "char_ranges": [[423, 561]], "word_ranges": [[65, 84]], "text": "Answer 2 incorrect: We would consider this technique in basicervical fractures and trochanteric mass lesions, not in subcapital fractures."}, "3": {"exist": true, "char_ranges": [[562, 670]], "word_ranges": [[84, 99]], "text": "Incorrect Answer 3: We would consider this technique in nondisplaced subcapital fractures in young patients."}, "4": {"exist": true, "char_ranges": [[671, 839]], "word_ranges": [[99, 124]], "text": "Incorrect answer 4: We would only consider non-surgical treatment in a patient with a very poor baseline general condition: bedridden patients with low life expectancy."}, "5": {"exist": false, "char_ranges": [], "word_ranges": [], "text": ""}}} {"id": 585, "year": 2022, "question_id_specific": 74, "full_question": "34-week primigravida with blood pressure of 165/95 and headache of two days of evolution. Laboratory tests: hemoglobin 10.5 g/dL, platelets 98,000/mm3, AST 356 IU/L (0-31), ALT 234 IU/L (0-31), LDH 878 IU/L (125-243). The laboratory is called because of the presence of schistocytes in the peripheral blood smear. She received the second dose of corticosteroids for fetal maturation 24 hours ago. On obstetric ultrasound the estimated fetal weight is in the 1st percentile for gestational age and the umbilical artery Doppler shows absence of end-diastolic flow. Which of the following is the most indicated clinical approach?", "full_answer": "They are describing HELLP syndrome. With these weeks of gestation and with the fetus recently matured (she has already been given two doses of corticosteroids), the attitude is immediate termination.", "type": "OBSTETRICS AND GYNECOLOGY", "options": {"1": "Expectant management with antihypertensive drugs and magnesium sulfate until the maternal platelets improve.", "2": "Expectant attitude with antihypertensive treatment at home and controls every 48 hours.", "3": "Termination of gestation when she completes fetal lung maturation.", "4": "Immediate termination of pregnancy.", "5": NaN}, "correct_option": 4, "explanations": {"1": {"exist": false, "char_ranges": [], "word_ranges": [], "text": ""}, "2": {"exist": false, "char_ranges": [], "word_ranges": [], "text": ""}, "3": {"exist": false, "char_ranges": [], "word_ranges": [], "text": ""}, "4": {"exist": true, "char_ranges": [[0, 199]], "word_ranges": [[0, 30]], "text": "They are describing HELLP syndrome. With these weeks of gestation and with the fetus recently matured (she has already been given two doses of corticosteroids), the attitude is immediate termination."}, "5": {"exist": false, "char_ranges": [], "word_ranges": [], "text": ""}}} {"id": 3, "year": 2011, "question_id_specific": 37, "full_question": "A 76-year-old woman with no history other than hypertension consults for painless jaundice and pruritus with anorexia. Laboratory tests show bilirubin of 12 mg/dl (9.5 direct). Ultrasound shows intrahepatic and extrahepatic biliary dilatation together with a single hepatic nodule smaller than 2 cm located peripherally on the anterior aspect of the left lobe. The CT scan confirms these findings also demonstrating the presence of a pancreatic mass of 3.5 cm. In the pancreatic head. The aspiration puncture of the hepatic nodule is conclusive for adenocarcinoma. Indicate the best therapeutic option:", "full_answer": "This case does not seem so easy to me. It is a pancreatic adenocarcinoma with metastasis, so the treatment should be palliative, so we remove options 1 and 5. In principle she is a patient in moderately good general condition, so the ideal would be to perform a permanent biliary diversion and chemotherapy, option 4 being the one I consider most correct.", "type": "DIGESTIVE", "options": {"1": "Neoadjuvant chemo/radiotherapy, conditioning the option of radical surgery on initial response.", "2": "Percutaneous external biliary drainage with palliative character with eventual reconversion to internal drainage in case of intolerance or complications.", "3": "Palliative surgical biliary bypass with or without prophylactic gastrojejunostomy depending on intraoperative findings.", "4": "Metallic biliary prosthesis by endoscopic retrograde cholangiopancreatography with the option of palliative chemotherapy.", "5": "Cephalic duodenopancreatectomy, with percutaneous ablation by radiofrequency or alcoholization of the hepatic lesion."}, "correct_option": 4, "explanations": {"1": {"exist": true, "char_ranges": [[39, 157]], "word_ranges": [[9, 29]], "text": "It is a pancreatic adenocarcinoma with metastasis, so the treatment should be palliative, so we remove options 1 and 5."}, "2": {"exist": false, "char_ranges": [], "word_ranges": [], "text": ""}, "3": {"exist": false, "char_ranges": [], "word_ranges": [], "text": ""}, "4": {"exist": true, "char_ranges": [[160, 354]], "word_ranges": [[29, 62]], "text": "In principle she is a patient in moderately good general condition, so the ideal would be to perform a permanent biliary diversion and chemotherapy, option 4 being the one I consider most correct."}, "5": {"exist": true, "char_ranges": [[39, 157]], "word_ranges": [[9, 29]], "text": "It is a pancreatic adenocarcinoma with metastasis, so the treatment should be palliative, so we remove options 1 and 5."}}} {"id": 439, "year": 2018, "question_id_specific": 96, "full_question": "A 64-year-old woman. Menopause at 54 years of age. Two pregnancies and eutocic deliveries. She does not take any medication. She comes to the emergency department reporting two days of vaginal bleeding with discomfort in the hypogastrium. The clinical examination performed by the gynecologist does not show any lesions in the external genitalia, vagina or cervix. Vaginal examination is inconclusive. Which of the following seems to you to be the most appropriate first diagnostic approach?", "full_answer": "Since the most frequent cause of vaginal bleeding in postmenopausal women is urogenital atrophy, the existence of an endometrial pathology cannot be ruled out. According to the SEGO: \"Abnormal genital bleeding is the main sign of suspicion that should lead us to rule out an endometrial neoplastic pathology, particularly in postmenopausal patients, or those with associated risk factors (...) Given the sign of suspicion, the performance of a transvaginal ultrasound, or in its absence transrectal, allows us to: rule out organic pathology (fibroids, polyps); measurement of endometrial thickness in a longitudinal cut. It is recommended to use a cut-off point of 3mm for endometrial biopsy in the symptomatic patient\".", "type": "GYNECOLOGY AND OBSTETRICS", "options": {"1": "Transvaginal ultrasound and measurement of endometrial thickness. Depending on this, an endometrial biopsy should be taken or not.", "2": "Endometrial biopsy by aspiration in consultation. With it I can get the diagnosis very reliably and it is inexpensive.", "3": "Hysteroscopy with sedation and directed biopsy if a lesion is seen.", "4": "I would perform a microbleed, since it is the test that would provide me with the definitive diagnosis.", "5": NaN}, "correct_option": 1, "explanations": {"1": {"exist": true, "char_ranges": [[97, 620]], "word_ranges": [[14, 92]], "text": "the existence of an endometrial pathology cannot be ruled out. According to the SEGO: \"Abnormal genital bleeding is the main sign of suspicion that should lead us to rule out an endometrial neoplastic pathology, particularly in postmenopausal patients, or those with associated risk factors (...) Given the sign of suspicion, the performance of a transvaginal ultrasound, or in its absence transrectal, allows us to: rule out organic pathology (fibroids, polyps); measurement of endometrial thickness in a longitudinal cut."}, "2": {"exist": false, "char_ranges": [], "word_ranges": [], "text": ""}, "3": {"exist": false, "char_ranges": [], "word_ranges": [], "text": ""}, "4": {"exist": false, "char_ranges": [], "word_ranges": [], "text": ""}, "5": {"exist": false, "char_ranges": [], "word_ranges": [], "text": ""}}} {"id": 192, "year": 2013, "question_id_specific": 160, "full_question": "The parents of a 5-month-old infant come because their child, who attends day care, has been presenting for 3 days, coinciding with an upper respiratory tract infection, liquid stools numbering 4 per day, occasional alimentary vomiting, and axillary temperature of 38.3°C. He is being fed with mixed breastfeeding. On examination the infant is in good general condition, is well nourished and hydrated and his breathing is eupneic; his weight is 4,730g and has decreased by 70g with respect to the previous week. The abdomen is soft and depressible, without masses or megaliths, and the fontanel is normotensive. Except for the presence of watery rhinorrhea, the rest of the examination by organs and devices is normal. Of the following statements, indicate the answer that you consider CORRECT:", "full_answer": "The baby is not dehydrated and is in good general condition. Try to avoid dehydration by replenishing what is being lost, avoiding drastic changes in feeding habits as much as possible. The rest of the answers touch on many of the \"myths\" and customs surrounding gastroenteritis. There is also no evidence of lactose intolerance (perianal irritation, frothy and acid stools, etc.) that would advise lactose-free milk.", "type": "PEDIATRICS", "options": {"1": "An estimation of losses should be made, recommend a 4-hour fast and rehydrate during this time with oral rehydration solution.", "2": "It is advisable to introduce rice cereals for their astringent effect.", "3": "A stool culture should be performed as soon as possible to exclude a bacterial origin.", "4": "The use of lactose-free formula should be recommended.", "5": "It is advisable to replace losses after each bowel movement with oral rehydration solution and continue with the patient's usual diet."}, "correct_option": 5, "explanations": {"1": {"exist": true, "char_ranges": [[186, 279]], "word_ranges": [[31, 46]], "text": "The rest of the answers touch on many of the \"myths\" and customs surrounding gastroenteritis."}, "2": {"exist": true, "char_ranges": [[186, 279]], "word_ranges": [[31, 46]], "text": "The rest of the answers touch on many of the \"myths\" and customs surrounding gastroenteritis."}, "3": {"exist": true, "char_ranges": [[186, 279]], "word_ranges": [[31, 46]], "text": "The rest of the answers touch on many of the \"myths\" and customs surrounding gastroenteritis."}, "4": {"exist": true, "char_ranges": [[280, 417]], "word_ranges": [[46, 66]], "text": "There is also no evidence of lactose intolerance (perianal irritation, frothy and acid stools, etc.) that would advise lactose-free milk."}, "5": {"exist": true, "char_ranges": [[0, 185]], "word_ranges": [[0, 31]], "text": "The baby is not dehydrated and is in good general condition. Try to avoid dehydration by replenishing what is being lost, avoiding drastic changes in feeding habits as much as possible."}}} {"id": 261, "year": 2014, "question_id_specific": 100, "full_question": "A 63-year-old woman comes to the emergency department reporting severe headache with signs of meningeal irritation, bilateral visual disturbances and ophthalmoplegia. A CT scan showed a 2 cm space-occupying lesion in the sella turcica compatible with pituitary adenoma with signs of intratumoral hemorrhage, with deviation of the pituitary stalk and compression of the glandular tissue. Mark which of the following answers is WRONG:", "full_answer": "Very difficult question of pituitary apoplexy. It is a marginal topic in the MIR and also the answer is complex. Eliminating answers we are left with intervene (answer 4) or not to intervene (answer 2). In patients with hemodynamic instability, decreased level of consciousness, decreased visual acuity and extensive visual field defects, surgical decompression is recommended in the first week after symptom onset. I think the answer is 4, although it is oddly worded. I don't know if the neurosurgeon will support my answer.", "type": "ENDOCRINOLOGY", "options": {"1": "Diagnostic suspicion is pituitary apoplexy.", "2": "Treatment with high-dose corticosteroids should be initiated and the evolution observed, since this treatment could reduce the volume of the lesion and avoid intervention.", "3": "Treatment with glucocorticoids should be considered to avoid secondary adrenal insufficiency that would compromise the patient's vital prognosis.", "4": "The presence of ophthalmoplegia and visual defects are indications for prompt intervention by urgent surgical decompression.", "5": "After resolution of the acute picture, the development of panhypopituitarism is frequent."}, "correct_option": 4, "explanations": {"1": {"exist": false, "char_ranges": [], "word_ranges": [], "text": ""}, "2": {"exist": false, "char_ranges": [], "word_ranges": [], "text": ""}, "3": {"exist": false, "char_ranges": [], "word_ranges": [], "text": ""}, "4": {"exist": true, "char_ranges": [[203, 415]], "word_ranges": [[35, 63]], "text": "In patients with hemodynamic instability, decreased level of consciousness, decreased visual acuity and extensive visual field defects, surgical decompression is recommended in the first week after symptom onset."}, "5": {"exist": false, "char_ranges": [], "word_ranges": [], "text": ""}}} {"id": 289, "year": 2016, "question_id_specific": 65, "full_question": "Point out from among the following the most likely complication presented by a patient operated, 20 years ago, of gastric ulcer by antrectomy and gastrojejunostomy (Billroth II) who comes to your office referring postpradial abdominal pain, abdominal distension, diarrhea and analytical data of malabsorption of fats and vitamin B12:", "full_answer": "The statement has described the clinical features of this syndrome. It is usually a late complication (the patient was operated 20 years ago for a Bilroth II, remember that it is a partial gastrectomy, closure of the gastric stump and anastomosis of the first jejunal loop to the stump), it is accompanied by postprandial pain, abdominal distension. What has occurred is a stenosis of the afferent loop, so that biliary and pancreatic secretions do not mix with the bolus, they accumulate, produce pain that is relieved by vomiting, fat absorption is not possible (lack of pancreatic lipase), so there is diarrhea, and if associated with bacterial overgrowth, there is malabsorption of vitamin B12. If it were gastric adenocarcinoma, they would have spoken of constitutional syndrome, extradigestive manifestations (sister Maria Jose node, Krukenberg tumor, Virchow node). It is not dumping because it does not speak of vegetative symptoms (palpitations, facial flushing, etc...). Biliary reflux gastropathy is an endoscopic diagnosis, moreover it is not associated with diarrhea or malabsorption.", "type": "GENERAL SURGERY", "options": {"1": "Biliary reflux gastropathy.", "2": "Gastric adenocarcinoma.", "3": "Rapid gastric emptying syndrome (dumping).", "4": "Afferent loop syndrome with bacterial overgrowth.", "5": NaN}, "correct_option": 4, "explanations": {"1": {"exist": true, "char_ranges": [[981, 1097]], "word_ranges": [[151, 167]], "text": "Biliary reflux gastropathy is an endoscopic diagnosis, moreover it is not associated with diarrhea or malabsorption."}, "2": {"exist": true, "char_ranges": [[699, 872]], "word_ranges": [[113, 135]], "text": "If it were gastric adenocarcinoma, they would have spoken of constitutional syndrome, extradigestive manifestations (sister Maria Jose node, Krukenberg tumor, Virchow node)."}, "3": {"exist": true, "char_ranges": [[873, 980]], "word_ranges": [[135, 151]], "text": "It is not dumping because it does not speak of vegetative symptoms (palpitations, facial flushing, etc...)."}, "4": {"exist": true, "char_ranges": [[350, 698]], "word_ranges": [[57, 113]], "text": "What has occurred is a stenosis of the afferent loop, so that biliary and pancreatic secretions do not mix with the bolus, they accumulate, produce pain that is relieved by vomiting, fat absorption is not possible (lack of pancreatic lipase), so there is diarrhea, and if associated with bacterial overgrowth, there is malabsorption of vitamin B12."}, "5": {"exist": false, "char_ranges": [], "word_ranges": [], "text": ""}}} {"id": 366, "year": 2016, "question_id_specific": 106, "full_question": "A 32-year-old man from Cameroon consults for fever cough and left hemithorax pain of 1 month of evolution. He took amoxicillin-clavulanic acid for 1 week without improvement of symptoms. A blood test showed a white blood cell count of 8000/microL and a hemoglobin of 12.8 g/dL. Chest X-ray shows a loculated left pleural effusion occupying one third of the hemithorax. A thoracentesis shows a yellowish fluid with the following features: red blood cells 2000/µL, leukocytes 2500/µL with 90% lymphocytes, protein 4.9 g/dL, lactate dehydrogenase 550 VIL, glucose 67 mg/dL, and absence of malignant cells on cytologic study. Which of the following tests would be most useful in diagnosing the cause of the pleural effusion?", "full_answer": "Suspicion is very high for tuberculous pleural effusion, in a young patient from an area with a high rate of TB, a lymphocytic exudate pointing to TB, lymphoma or tumor. Tumor unlikely due to age, negative cytology. Lymphoma could be but no evidence of associated nodal pathology. High adenosine deaminase in LP would point with great certainty to a pleural TB although it would be advisable to confirm the diagnosis with a pleural biopsy since it can also occur in lymphomas and empyemas of other origin.", "type": "PNEUMOLOGY AND THORACIC SURGERY", "options": {"1": "A thoracic computed tomography (CT) scan.", "2": "Pleural fluid pH measurement.", "3": "Measurement of pleural fluid adenosine deaminase.", "4": "Tuberculin test.", "5": NaN}, "correct_option": 3, "explanations": {"1": {"exist": false, "char_ranges": [], "word_ranges": [], "text": ""}, "2": {"exist": false, "char_ranges": [], "word_ranges": [], "text": ""}, "3": {"exist": true, "char_ranges": [[281, 505]], "word_ranges": [[47, 86]], "text": "High adenosine deaminase in LP would point with great certainty to a pleural TB although it would be advisable to confirm the diagnosis with a pleural biopsy since it can also occur in lymphomas and empyemas of other origin."}, "4": {"exist": false, "char_ranges": [], "word_ranges": [], "text": ""}, "5": {"exist": false, "char_ranges": [], "word_ranges": [], "text": ""}}} {"id": 544, "year": 2022, "question_id_specific": 40, "full_question": "16-year-old woman diagnosed with allergic rhinoconjunctivitis due to sensitization to grass pollens since she was 10 years old, who does not fully control the symptomatology during the pollination season with oral antihistamines and nasal topical corticosteroids. Which etiologic therapeutic option would be indicated:", "full_answer": "Currently the only effective treatment capable of modifying the natural course of respiratory allergy is specific allergen immunotherapy for a minimum of 3 consecutive years. In this patient the conventional treatment has not worked, so it is possible to escalate to specific allergen immunotherapy. It is true that it is not necessary to wait 10 years to start it since it is indicated and its use is allowed if there is clinical relevance in patients older than 5 years. Omalizumab is a biological treatment that would only be indicated in cases of severe moderate bronchial asthma secondary to respiratory allergy.", "type": "ALLERGOLOGY", "options": {"1": "Specific immunotherapy by sublingual or subcutaneous route for a minimum period of 3 years.", "2": "Specific immunotherapy by sublingual or subcutaneous route for a maximum period of 1 year.", "3": "Omalizumab subcutaneously every 4 weeks for up to 1 year.", "4": "Omalizumab subcutaneously every 4 weeks for a minimum period of 6 months.", "5": NaN}, "correct_option": 1, "explanations": {"1": {"exist": true, "char_ranges": [[0, 174]], "word_ranges": [[0, 25]], "text": "Currently the only effective treatment capable of modifying the natural course of respiratory allergy is specific allergen immunotherapy for a minimum of 3 consecutive years."}, "2": {"exist": true, "char_ranges": [[0, 174]], "word_ranges": [[0, 25]], "text": "Currently the only effective treatment capable of modifying the natural course of respiratory allergy is specific allergen immunotherapy for a minimum of 3 consecutive years."}, "3": {"exist": true, "char_ranges": [[473, 617]], "word_ranges": [[79, 100]], "text": "Omalizumab is a biological treatment that would only be indicated in cases of severe moderate bronchial asthma secondary to respiratory allergy."}, "4": {"exist": true, "char_ranges": [[473, 617]], "word_ranges": [[79, 100]], "text": "Omalizumab is a biological treatment that would only be indicated in cases of severe moderate bronchial asthma secondary to respiratory allergy."}, "5": {"exist": false, "char_ranges": [], "word_ranges": [], "text": ""}}} {"id": 533, "year": 2021, "question_id_specific": 179, "full_question": "A 32-year-old woman with multiple endocrine neoplasia syndrome type 2A (MEN-2A) and carrier of a mutation in RET. In a neck ultrasound a 6 mm hypoechogenic nodule with calcifications inside is identified. It was decided to perform a total thyroidectomy and cervical lymph node removal. In the macroscopic study a total of three nodules were identified, two in the right lobe of 5 and 6 mm, and one in the left lobe of 4 mm. In the microscopic study the three lesions are constituted by a uniform proliferation of rounded cells arranged in a solid pattern and accompanied by calcifications and amyloid deposits. The nuclei are not clear, nor do they show clefts or pseudoinclusions. Immunohistochemical staining for synaptophysin is positive. In the cervical lymph node clearance metastases are identified. What is the pathologic diagnosis of the lesions identified in the total thyroidectomy?", "full_answer": "Typical medullary thyroid RET (C cells, elevated calcitonin, inherited MEN-2nd, typical MIR question).", "type": "ONCOLOGY", "options": {"1": "Multifocal medullary carcinoma.", "2": "Multifocal papillary carcinoma.", "3": "Follicular carcinoma.", "4": "Hyperplasia of parafollicular cells.", "5": NaN}, "correct_option": 1, "explanations": {"1": {"exist": true, "char_ranges": [[0, 102]], "word_ranges": [[0, 13]], "text": "Typical medullary thyroid RET (C cells, elevated calcitonin, inherited MEN-2nd, typical MIR question)."}, "2": {"exist": false, "char_ranges": [], "word_ranges": [], "text": ""}, "3": {"exist": false, "char_ranges": [], "word_ranges": [], "text": ""}, "4": {"exist": false, "char_ranges": [], "word_ranges": [], "text": ""}, "5": {"exist": false, "char_ranges": [], "word_ranges": [], "text": ""}}} {"id": 80, "year": 2012, "question_id_specific": 37, "full_question": "A 52-year-old man who consults for a second opinion on the need to perform a liver biopsy for the study of hypertransaminasemia detected two years ago in routine company analyses. His family history includes the death of his father of an unidentified etiology of epatic cirrhosis. Asymptomatic and performing social and work life without limits. He denies alcohol consumption. Physical examination shows metallic skin pigmentation and minimal non-painful hepatomegaly. The rest of the physical examination was normal, body mass index 23. Laboratory tests showed the following results: billirubin, albumin, transaminases AST and ALT, normal blood count and prothrombin time: glycemia 150 mg/dl; serum ferritin 950; transferrin saturation >45%, negative for hepatotropic virus. Normal abdominal ultrasound. He has undergone HFE genetic study being homozygous for the C282Y mutation. What would be the best recommendation with the available information?", "full_answer": "This is a patient who already has a diagnosis of hemochromatosis, due to a positive genetic study and data of iron overload (IST >45% and elevated ferritin). Biopsy would be useful if the diagnosis is uncertain or if we would like to assess liver involvement that is not suspected with this analysis. The treatment of choice is phlebotomy.", "type": "DIGESTIVE SYSTEM", "options": {"1": "Perform liver biopsy.", "2": "Perform hepatic MRI.", "3": "Initiate treatment with phlebotomies.", "4": "Start treatment with desferroxamine.", "5": "Start treatment with Vitamin E."}, "correct_option": 3, "explanations": {"1": {"exist": true, "char_ranges": [[158, 300]], "word_ranges": [[27, 52]], "text": "Biopsy would be useful if the diagnosis is uncertain or if we would like to assess liver involvement that is not suspected with this analysis."}, "2": {"exist": false, "char_ranges": [], "word_ranges": [], "text": ""}, "3": {"exist": false, "char_ranges": [], "word_ranges": [], "text": ""}, "4": {"exist": false, "char_ranges": [], "word_ranges": [], "text": ""}, "5": {"exist": false, "char_ranges": [], "word_ranges": [], "text": ""}}} {"id": 394, "year": 2016, "question_id_specific": 225, "full_question": "Mariano is a 53-year-old man who comes to your office reporting that he has been feeling very unwell for several years. He relates his discomfort to a behavior that he finds absurd but that he is unable to avoid. This behavior, which she always does when she comes home from work, consists of turning the doorknob of the bathroom on the upper floor of the house before doing anything else, even before greeting her family. Sometimes he has tried to resist doing this but has only succeeded in distressing himself and delaying the behavior for a few minutes. This behavior that Mariano performs in an incoercible and automatic way is what is called:", "full_answer": "In this question they ask for the name of the symptom, not the disease, so therefore the correct answer is compulsion. According to the DSM-5, a compulsion is a repetitive behavior (e.g., washing hands, tidying up, checking things) or mental act (e.g., praying, counting, repeating words silently) that the subject performs in response to an obsession or in accordance with rules that must be rigidly enforced. The goal of the behaviors or mental acts is to prevent or decrease anxiety or discomfort, or to avoid some feared event or situation; however, these behaviors or mental acts are not realistically connected with those intended to neutralize or prevent or are clearly excessive.", "type": "PSYCHIATRY", "options": {"1": "Obsessive-compulsive disorder.", "2": "Obsession.", "3": "Impulsion.", "4": "Compulsion.", "5": NaN}, "correct_option": 4, "explanations": {"1": {"exist": false, "char_ranges": [], "word_ranges": [], "text": ""}, "2": {"exist": false, "char_ranges": [], "word_ranges": [], "text": ""}, "3": {"exist": false, "char_ranges": [], "word_ranges": [], "text": ""}, "4": {"exist": true, "char_ranges": [[119, 410]], "word_ranges": [[21, 66]], "text": "According to the DSM-5, a compulsion is a repetitive behavior (e.g., washing hands, tidying up, checking things) or mental act (e.g., praying, counting, repeating words silently) that the subject performs in response to an obsession or in accordance with rules that must be rigidly enforced."}, "5": {"exist": false, "char_ranges": [], "word_ranges": [], "text": ""}}} {"id": 229, "year": 2014, "question_id_specific": 178, "full_question": "Previously healthy 10-month-old infant with acute gastroenteritis of one day's evolution and signs of mild dehydration, without blood or mucus in the stool and without oral tolerance. What is the initial treatment of choice in our environment?", "full_answer": "The infant presents with acute GEA without signs of alarm or severe dehydration. The first step would be to try oral tolerance with oral rehydration saline and continue with normal feeding if well tolerated. Answer 1 may give rise to doubts, due to the fact that the statement says that the patient \"does not tolerate the oral route\". In this case, intravenous rehydration could be started... but it would not be advisable to leave the patient on an absolute diet for 8 hours, nor a subsequent astringent diet, so I prefer answer 2. The rest of the treatments do not provide any benefit.", "type": "PEDIATRICS", "options": {"1": "Intravenous rehydration, absolute diet 8 hours and start feeding with astringent diet.", "2": "Rehydration with low osmolality oral rehydration solution (sodium 60-75 mEq/L) and continue with his usual diet.", "3": "Rehydration with low osmolality oral rehydration solution (sodium 60-75 mEq/L) maintain usual feeding and oral amoxicillin 10 days.", "4": "Rehydration with low osmolality oral rehydration solution (sodium 60-75 mEq/L) and start feeding with lactose-free formulas.", "5": "Rehydration with low osmolality oral rehydration solution (sodium 60-75 mEq/L) maintain usual feeding and loperamide 7 days."}, "correct_option": 2, "explanations": {"1": {"exist": true, "char_ranges": [[81, 532]], "word_ranges": [[13, 93]], "text": "The first step would be to try oral tolerance with oral rehydration saline and continue with normal feeding if well tolerated. Answer 1 may give rise to doubts, due to the fact that the statement says that the patient \"does not tolerate the oral route\". In this case, intravenous rehydration could be started... but it would not be advisable to leave the patient on an absolute diet for 8 hours, nor a subsequent astringent diet, so I prefer answer 2."}, "2": {"exist": true, "char_ranges": [[81, 532]], "word_ranges": [[13, 93]], "text": "The first step would be to try oral tolerance with oral rehydration saline and continue with normal feeding if well tolerated. Answer 1 may give rise to doubts, due to the fact that the statement says that the patient \"does not tolerate the oral route\". In this case, intravenous rehydration could be started... but it would not be advisable to leave the patient on an absolute diet for 8 hours, nor a subsequent astringent diet, so I prefer answer 2."}, "3": {"exist": true, "char_ranges": [[533, 587]], "word_ranges": [[93, 103]], "text": "The rest of the treatments do not provide any benefit."}, "4": {"exist": true, "char_ranges": [[533, 587]], "word_ranges": [[93, 103]], "text": "The rest of the treatments do not provide any benefit."}, "5": {"exist": true, "char_ranges": [[533, 587]], "word_ranges": [[93, 103]], "text": "The rest of the treatments do not provide any benefit."}}} {"id": 198, "year": 2013, "question_id_specific": 73, "full_question": "A 77-year-old man with a history of type 2 Diabetes Mellitus was diagnosed 2 years ago with mild cognitive impairment. At that time a Minimental (Folstein) 28/30, abbreviated Yesavage Test 14/15, a CBC with normal TSH and maturation factors along with negative LUES and HIV serologies and a magnetic resonance imaging (MRI) showing diffuse cortical atrophy predominantly in the posterior area. The patient was accompanied by his wife, who reported a slow progressive evolution of memory impairment (e.g., sometimes confused with insulin doses). There are no hallucinations or behavioral disorder. Current Minimental is 24/30 and Yesavage test 14/15, there is no neurological focality, tremor or tone or gait disorders on physical examination. Two months ago he came to the emergency department after a mild traumatic brain injury due to an accidental fall (the only one in 2 years) and a CT scan was performed which did not provide new information compared to the previous MRI. What is the most likely diagnosis in this patient?", "full_answer": "This case describes an Alzheimer's disease with insidious onset of short-term memory problems. Clinically it can be distinguished from other options by the absence of behavioral problems (typical of frontotemporal dementia) and hallucinations (early in Lewy body disease). Vascular dementia is diagnosed by criteria that include deficits in memory and two other cognitive areas with consequent functional impairment, but cerebral vascular disease based on history, examination and/or neuroimaging tests, which are not indicated in the case. Creutzfeldt-Jacob disease is mostly rapidly progressive and usually presents with other clinical, abnormal neurological examination and MRI alterations.", "type": "NEUROLOGY", "options": {"1": "Frontotemporal dementia.", "2": "Alzheimer's disease.", "3": "Vascular dementia.", "4": "Lewy body disease.", "5": "Creutzfeldt-Jakob disease."}, "correct_option": 2, "explanations": {"1": {"exist": true, "char_ranges": [[0, 272]], "word_ranges": [[0, 38]], "text": "This case describes an Alzheimer's disease with insidious onset of short-term memory problems. Clinically it can be distinguished from other options by the absence of behavioral problems (typical of frontotemporal dementia) and hallucinations (early in Lewy body disease)."}, "2": {"exist": true, "char_ranges": [[0, 272]], "word_ranges": [[0, 38]], "text": "This case describes an Alzheimer's disease with insidious onset of short-term memory problems. Clinically it can be distinguished from other options by the absence of behavioral problems (typical of frontotemporal dementia) and hallucinations (early in Lewy body disease)."}, "3": {"exist": true, "char_ranges": [[273, 416]], "word_ranges": [[38, 58]], "text": "Vascular dementia is diagnosed by criteria that include deficits in memory and two other cognitive areas with consequent functional impairment,"}, "4": {"exist": true, "char_ranges": [[0, 272]], "word_ranges": [[0, 38]], "text": "This case describes an Alzheimer's disease with insidious onset of short-term memory problems. Clinically it can be distinguished from other options by the absence of behavioral problems (typical of frontotemporal dementia) and hallucinations (early in Lewy body disease)."}, "5": {"exist": true, "char_ranges": [[541, 693]], "word_ranges": [[76, 94]], "text": "Creutzfeldt-Jacob disease is mostly rapidly progressive and usually presents with other clinical, abnormal neurological examination and MRI alterations."}}} {"id": 351, "year": 2016, "question_id_specific": 161, "full_question": "A 27-year-old woman referred to a gynecology office for evaluation, reporting dyspareunia for about 8 months, along with dyschezia and occasional rectorrhagia coinciding with menstruation for 3-4 months. She also reports dysmenorrhea for years, which she controls well with ibuprofen. She has been trying to get pregnant for 16 months without success. In the gynecological examination she only shows pain when pressing on the posterior vaginal fornix. Which test do you consider would allow you to reach a diagnosis of certainty of her pathology?", "full_answer": "The clinical case presented is a woman of childbearing age (27 years) presenting dyspareunia + dysmenorrhea + infertility, which should lead us to think of endometriosis. The clinical history and ultrasound give a diagnosis of suspicion, but the diagnosis of certainty is given by direct observation (with or without associated biopsy) of ectopic endometrial tissue implants, either by laparotomy or laparoscopy.", "type": "GYNECOLOGY AND OBSTETRICS", "options": {"1": "Transvaginal ultrasound.", "2": "Diagnostic laparoscopy.", "3": "Magnetic resonance imaging.", "4": "Colonoscopy.", "5": NaN}, "correct_option": 2, "explanations": {"1": {"exist": false, "char_ranges": [], "word_ranges": [], "text": ""}, "2": {"exist": true, "char_ranges": [[171, 412]], "word_ranges": [[26, 61]], "text": "The clinical history and ultrasound give a diagnosis of suspicion, but the diagnosis of certainty is given by direct observation (with or without associated biopsy) of ectopic endometrial tissue implants, either by laparotomy or laparoscopy."}, "3": {"exist": false, "char_ranges": [], "word_ranges": [], "text": ""}, "4": {"exist": false, "char_ranges": [], "word_ranges": [], "text": ""}, "5": {"exist": false, "char_ranges": [], "word_ranges": [], "text": ""}}} {"id": 69, "year": 2012, "question_id_specific": 124, "full_question": "The daughter of an 82-year-old woman consults us because she has found her mother more confused than usual. The patient is diagnosed with moderate Alzheimer's disease. She also has AF, depression and osteoarthritis. She has been receiving stable treatment for 3 years, including digoxin, acenocoumarol, fluoxetine for the last 4 months and ibuprofen for the last month for joint pain. On examination, her blood pressure is 130/80, heart rate 48 bpm and respiratory rate 10 rpm. Which of the following is the most likely cause of the increased confusion?", "full_answer": "The most likely cause is a digitalis intoxication, which would produce this slowing of conduction, which is seen in the heart rate. Probably secondary to renal failure due to ibuprofen toxicity (but this is a bit of a stretch).", "type": "ANESTHESIOLOGY, CRITICAL CARE AND EMERGENCY MEDICINE", "options": {"1": "Progression of Alzheimer's disease.", "2": "Worsening depression.", "3": "Digitalis intoxication.", "4": "Dementia due to Lewy bodies.", "5": "Ibuprofen toxicity."}, "correct_option": 3, "explanations": {"1": {"exist": false, "char_ranges": [], "word_ranges": [], "text": ""}, "2": {"exist": false, "char_ranges": [], "word_ranges": [], "text": ""}, "3": {"exist": true, "char_ranges": [[0, 131]], "word_ranges": [[0, 22]], "text": "The most likely cause is a digitalis intoxication, which would produce this slowing of conduction, which is seen in the heart rate."}, "4": {"exist": false, "char_ranges": [], "word_ranges": [], "text": ""}, "5": {"exist": false, "char_ranges": [], "word_ranges": [], "text": ""}}} {"id": 140, "year": 2012, "question_id_specific": 134, "full_question": "A newborn of 37 weeks gestational age, with no pathological findings in the prenatal ultrasound, presents in the examination performed in the delivery room a \"stop\" to the passage of the nasogastric tube. Chest and abdominal X-ray showed an atresic esophageal pouch, with normal gastrointestinal pneumatization. After a diagnostic evaluation that ruled out other anomalies and being in a stable respiratory condition, surgical intervention was decided. On what basis was the priority to intervene in this patient based?", "full_answer": "The correct answer is 3. The fact that there is intestinal aeration points to the existence of a fistula of the lower esophageal segment with the trachea or with a bronchus. In this case, pulmonary complications are frequent due to the passage of gastric contents into the airway.", "type": "PEDIATRICS", "options": {"1": "Due to the impossibility of swallowing saliva.", "2": "Because of frequently associated cardiac malformations.", "3": "Because of the risk of aspiration pneumonitis.", "4": "Due to the impossibility of enteral feeding.", "5": "Associated tracheomalacia."}, "correct_option": 3, "explanations": {"1": {"exist": false, "char_ranges": [], "word_ranges": [], "text": ""}, "2": {"exist": false, "char_ranges": [], "word_ranges": [], "text": ""}, "3": {"exist": true, "char_ranges": [[25, 280]], "word_ranges": [[5, 48]], "text": "The fact that there is intestinal aeration points to the existence of a fistula of the lower esophageal segment with the trachea or with a bronchus. In this case, pulmonary complications are frequent due to the passage of gastric contents into the airway."}, "4": {"exist": false, "char_ranges": [], "word_ranges": [], "text": ""}, "5": {"exist": false, "char_ranges": [], "word_ranges": [], "text": ""}}} {"id": 475, "year": 2020, "question_id_specific": 89, "full_question": "A 46-year-old man with bipolar disorder is brought to the emergency department after an over-ingestion of lithium carbonate. Examination reveals severe tremor, ataxia, dysarthria, myoclonus and fasciculations. Lithemia is 4.1 mEq/L (toxicity > 1.6 mEq/L). Which of the following therapeutic options would be most indicated?", "full_answer": "We are dealing with a case of severe lithium intoxication (lithemia > 3.5 mEq/L is life threatening), therefore, the therapeutic option of choice is to start hemodialysis urgently. Remember that activated charcoal is NOT indicated in lithium intoxication.", "type": "CRITICAL CARE", "options": {"1": "Aminophylline associated with a cathartic.", "2": "Activated charcoal.", "3": "Hemodialysis.", "4": "Forced diuresis.", "5": NaN}, "correct_option": 3, "explanations": {"1": {"exist": false, "char_ranges": [], "word_ranges": [], "text": ""}, "2": {"exist": true, "char_ranges": [[181, 255]], "word_ranges": [[28, 38]], "text": "Remember that activated charcoal is NOT indicated in lithium intoxication."}, "3": {"exist": true, "char_ranges": [[0, 180]], "word_ranges": [[0, 28]], "text": "We are dealing with a case of severe lithium intoxication (lithemia > 3.5 mEq/L is life threatening), therefore, the therapeutic option of choice is to start hemodialysis urgently."}, "4": {"exist": false, "char_ranges": [], "word_ranges": [], "text": ""}, "5": {"exist": false, "char_ranges": [], "word_ranges": [], "text": ""}}} {"id": 85, "year": 2012, "question_id_specific": 50, "full_question": "A patient with a history of febrile illness and chest pain comes to the hospital with dyspnea and tachypnea. On physical examination his blood pressure is low, jugular venous pressure is elevated with a deep descending sinus \"X\" and he has a paradoxical pulse. What pathology should you suspect?", "full_answer": "Paradoxical pulse is a drop in blood pressure > 10 mmHg during inspiration; it represents an exaggeration of the physiological phenomenon consisting of inspiratory lowering of BP (normal up to 10 mmHg). In cardiac tamponade, inspiration, which causes an increase in blood flow to the right chambers, increasing their volume, secondarily causes a displacement of the interventricular septum to the left, so that the left heart lodges and expels less blood during systole and the pulse, therefore, decreases. In a normal heart this exaggerated displacement, caused by the pressure exerted by the tamponade on the RV free wall, does not occur. Sinus X represents the systolic collapse of the venous pulse, i.e., the pressure drop due to atrial relaxation (also partly due to a downward displacement of the RV base during systole). Sinus Y represents the diastolic collapse of the venous pulse, i.e., the pressure drop that occurs from the moment blood enters the tricuspid valve into the ventricle. In cardiac tamponade, the deep sinus X is characteristic. In constrictive pericarditis, the deep Y sinus. For all these reasons, the correct answer is 5.", "type": "CARDIOLOGY AND VASCULAR SURGERY", "options": {"1": "Ischemic heart disease.", "2": "Dilated cardiomyopathy.", "3": "Severe aortic stenosis.", "4": "Constrictive pericarditis.", "5": "Pericardial effusion with cardiac tamponade."}, "correct_option": 5, "explanations": {"1": {"exist": false, "char_ranges": [], "word_ranges": [], "text": ""}, "2": {"exist": false, "char_ranges": [], "word_ranges": [], "text": ""}, "3": {"exist": false, "char_ranges": [], "word_ranges": [], "text": ""}, "4": {"exist": false, "char_ranges": [], "word_ranges": [], "text": ""}, "5": {"exist": true, "char_ranges": [[641, 1101]], "word_ranges": [[101, 175]], "text": "Sinus X represents the systolic collapse of the venous pulse, i.e., the pressure drop due to atrial relaxation (also partly due to a downward displacement of the RV base during systole). Sinus Y represents the diastolic collapse of the venous pulse, i.e., the pressure drop that occurs from the moment blood enters the tricuspid valve into the ventricle. In cardiac tamponade, the deep sinus X is characteristic. In constrictive pericarditis, the deep Y sinus."}}} {"id": 527, "year": 2021, "question_id_specific": 31, "full_question": "A patient's arterial blood gas result is: pH: 7.40, PaCO2 60 mmHg; bicarbonate 36 mM/L. What is the presenting disturbance:", "full_answer": "Blood gases can be misleading since a normal pH may suggest that there is no disturbance of acid-base balance, instead it is masking a double disturbance in opposite directions as in this case. The patient presents with respiratory acidosis (PaCO2 60 mmHg; normal to 50) and compensatory metabolic alkalosis (bicarbonate 36 mmol/l; normal to 28). This is the typical blood gas of the patient with COPD and compensated chronic respiratory failure.", "type": "NEPHROLOGY", "options": {"1": "None, pH is normal.", "2": "Respiratory acidosis.", "3": "Respiratory acidosis and metabolic alkalosis.", "4": "Metabolic alkalosis.", "5": NaN}, "correct_option": 3, "explanations": {"1": {"exist": false, "char_ranges": [], "word_ranges": [], "text": ""}, "2": {"exist": false, "char_ranges": [], "word_ranges": [], "text": ""}, "3": {"exist": true, "char_ranges": [[194, 446]], "word_ranges": [[33, 71]], "text": "The patient presents with respiratory acidosis (PaCO2 60 mmHg; normal to 50) and compensatory metabolic alkalosis (bicarbonate 36 mmol/l; normal to 28). This is the typical blood gas of the patient with COPD and compensated chronic respiratory failure."}, "4": {"exist": false, "char_ranges": [], "word_ranges": [], "text": ""}, "5": {"exist": false, "char_ranges": [], "word_ranges": [], "text": ""}}} {"id": 407, "year": 2016, "question_id_specific": 81, "full_question": "A 30-year-old woman comes to the clinic reporting anxiety, weight loss of about 6 kgs and a feeling of \"nervousness\" in the last three months. Physical examination showed tachycardia, hyperreflexia and absence of goiter. TSH values are < 0.01 microU/mL, T4 is elevated and thyroglobulin levels are decreased. A scan reveals an absence of uptake in the thyroid region. What seems to you the most likely diagnosis?", "full_answer": "Patients with this disorder voluntarily or accidentally ingest excessive amounts of thyroid hormone, resulting in hyperthyroidism in the absence of goiter. In contrast to all other causes of hyperthyroidism, serum thyroglobulin is not elevated and is almost always low or at the lower limit of normal. The fundamental doubt we had with ovarian teratoma (ovarian struma) which is a rare variant of mature monodermal teratoma composed of thyroid tissue described at the beginning of the 20th century. It is so named when the teratoma is predominantly composed of mature thyroid tissue (at least 50%, in order to be classified as struma ovarii). It represents between 2.5 and 5% of ovarian teratomas. In itself, it is a benign tumor, but in up to 5% of cases it can develop a malignant transformation of its elements, classically referred to as malignant struma ovarii. Mostly, the neoplastic tissue developed is a papillary carcinoma, less frequently it corresponds to follicular carcinoma. This malignant transformation arises mostly from classical forms of struma, but is extremely rare in cases of mature cystic teratomas. Its behavior is similar to that of other primary ovarian tumors, tending to metastasize to the peritoneal cavity and hematogenously to bone, liver and brain. The mean age of presentation of thyroid carcinoma on struma ovarii is between 42.9-44 years and between 21 and 63 for the papillary carcinoma variant. Clinically, the predominant form of presentation is as a pelvic mass or abdominal pain (45%), less frequently hyperthyroidism (5-8%) or ascites (17%). It is postulated that the ovarian thyroid tissue in the struma is autonomous in the production of thyroid hormones, so it is essential to measure serum thyroglobulin levels in patients affected by hyperthyroidism, as they point to the extrathyroid production of thyroid hormone.", "type": "ONCOLOGY (ECTOPIC)", "options": {"1": "Factitious thyrotoxicosis.", "2": "Hyperthyroidism due to Graves' disease.", "3": "Ovarian teratoma (ovarian stromal).", "4": "Subacute thyroiditis.", "5": NaN}, "correct_option": 1, "explanations": {"1": {"exist": false, "char_ranges": [], "word_ranges": [], "text": ""}, "2": {"exist": false, "char_ranges": [], "word_ranges": [], "text": ""}, "3": {"exist": false, "char_ranges": [], "word_ranges": [], "text": ""}, "4": {"exist": false, "char_ranges": [], "word_ranges": [], "text": ""}, "5": {"exist": false, "char_ranges": [], "word_ranges": [], "text": ""}}} {"id": 15, "year": 2011, "question_id_specific": 126, "full_question": "For the third time in the last 6 months an 84-year-old man with advanced dementia is admitted to a hospital for aspiration pneumonia. He has lost 9.5 kg in the last 10 months. He has a pressure ulcer in the sacrum. He does not communicate verbally, does not ambulate and is dependent for all activities of daily living. His wife cares for him at home and does not want to place him in a nursing home. A swallowing study indicates that he chokes on all types of food consistencies. The hospital physician suggests tube feeding. The patient's \"Advance Living Will\" states that his wife is the representative decision maker regarding his medical care and that he does not want extraordinary measures to prolong his life including artificial nutrition. Which of the following is the most appropriate recommendation for this patient:", "full_answer": "This is perhaps the least scientific of this year's MIR questions, since its answer is based on the interpretation of a law, which is also of an autonomous nature. Probably the spirit of the examiner wants to reflect this law, in an extreme case. Let's see, starting from article 11 of Law 41 /2002, of November 14 (1), basic law regulating patient autonomy and the rights and obligations regarding information and clinical documentation (B.O.E. No. 274, of November 15) of a national nature, in Castilla-La Mancha there is Law 6/2005, of July 7 (2), of Castilla-La Mancha, on the Declaration of Advance Directives in matters of one's own health. (D.O.C.M. No. 141, of July 15) and Decree 15/2006, of February 21 (3), of the Registry of Advance Directives of Castilla-La Mancha (D.O.C.M. No. 42, of February 24). In the summary published by the JCCM (4) it says: \"The declaration of advance directives (also known as advance directives or living will) is the written manifestation of a capable person who, acting freely, expresses the instructions to be taken into account regarding the health care he/she wishes to receive in situations that prevent him/her from communicating his/her will personally, or regarding the destination of his/her body or organs once death has occurred. In our declaration of advance directives, we can refer to: 1. The expression of personal values, in order to help in the interpretation of the document itself and to guide physicians in making clinical decisions. 2. Instructions about the care and treatment related to our health that we may or may not wish to receive. 3. We may appoint a representative to act as an interlocutor with the responsible physician or health care team to ensure that the instructions contained in the declaration are carried out. 4. We may also include our decision regarding organ donation. In these cases, the authorization of the family will not be required for the removal or use of the donated organs\". It thus seems clear that the intended response is 5.", "type": "PALLIATIVE CARE", "options": {"1": "Place an indwelling SNG and discharge to a skilled nursing home.", "2": "Place a mid-term NGUS and discharge to a nursing home with specialized care until bedsores heal.", "3": "Discharge to a nursing home with specialized care until pressure ulcers heal.", "4": "Place a gastrostomy tube and discharge home.", "5": "Discharge home with palliative care from the health area."}, "correct_option": 5, "explanations": {"1": {"exist": false, "char_ranges": [], "word_ranges": [], "text": ""}, "2": {"exist": false, "char_ranges": [], "word_ranges": [], "text": ""}, "3": {"exist": false, "char_ranges": [], "word_ranges": [], "text": ""}, "4": {"exist": false, "char_ranges": [], "word_ranges": [], "text": ""}, "5": {"exist": false, "char_ranges": [], "word_ranges": [], "text": ""}}} {"id": 322, "year": 2016, "question_id_specific": 85, "full_question": "A 22-year-old boy with hyposmia presents lack of development of secondary sexual characteristics and infertility. Bilateral testicular volume of 4 mL. Analytically, EST-I 1.2 U/L (vn 5-15); LH 0.6 U/L (vn 3-15); testosterone 100 ng/dL (vn 300-1200), prolactin normal. State your proposed treatment to achieve fertility:", "full_answer": "This is a Kallman syndrome (hypogonadotropic hypogonadism + olfactory disturbance). Triptorelin is a GnRH agonist, which by continuous action reduces FSH and LH, and is used in sex hormone-dependent cancers, as well as precocious puberty. Bromocriptine is a dopaminergic agonist used in hyperPRL (he has normal PRL), pituitary tumors... We stay between 1 and 3. To induce testicular maturation, pulsatile GnRH with pump, hCG or FSH+hCG can be used. LH is not used as a treatment, it would be stimulated by intermittent GnRH.", "type": "UROLOGY", "options": {"1": "GnRH infusion pump.", "2": "Monthly intramuscular administration of triptorelin.", "3": "Intramuscular administration of FSH and LH once a week.", "4": "Treatment with bromocriptine.", "5": NaN}, "correct_option": 1, "explanations": {"1": {"exist": true, "char_ranges": [[362, 448]], "word_ranges": [[55, 69]], "text": "To induce testicular maturation, pulsatile GnRH with pump, hCG or FSH+hCG can be used."}, "2": {"exist": true, "char_ranges": [[84, 238]], "word_ranges": [[10, 35]], "text": "Triptorelin is a GnRH agonist, which by continuous action reduces FSH and LH, and is used in sex hormone-dependent cancers, as well as precocious puberty."}, "3": {"exist": true, "char_ranges": [[449, 524]], "word_ranges": [[69, 83]], "text": "LH is not used as a treatment, it would be stimulated by intermittent GnRH."}, "4": {"exist": true, "char_ranges": [[239, 336]], "word_ranges": [[35, 49]], "text": "Bromocriptine is a dopaminergic agonist used in hyperPRL (he has normal PRL), pituitary tumors..."}, "5": {"exist": false, "char_ranges": [], "word_ranges": [], "text": ""}}} {"id": 610, "year": 2022, "question_id_specific": 116, "full_question": "95-year-old woman living in a nursing home, independent for basic activities of daily living, goes out to the garden for walks. She has a history of hypertension, dyslipidemia, osteoporosis and mild cognitive impairment. She suffers a fall when getting up at night to go to the bathroom. X-ray shows a displaced subcapital fracture of the right hip. What is the recommended treatment?", "full_answer": "Displaced hip fracture, Garden III-IV. Surgical treatment. Hip hemiarthroplasty. Option 2 treatment for pertrochanteric fractures. Option 3. Non-displaced fractures. Option 4. Non ambulatory patients with minimal pain and high surgical risk.", "type": "TRAUMATOLOGY", "options": {"1": "Hip hemiarthroplasty.", "2": "Trochanteric nail fixation.", "3": "Fixation with cannulated screws.", "4": "Conservative: bed-chair life.", "5": NaN}, "correct_option": 1, "explanations": {"1": {"exist": true, "char_ranges": [[0, 80]], "word_ranges": [[0, 9]], "text": "Displaced hip fracture, Garden III-IV. Surgical treatment. Hip hemiarthroplasty."}, "2": {"exist": true, "char_ranges": [[81, 130]], "word_ranges": [[9, 15]], "text": "Option 2 treatment for pertrochanteric fractures."}, "3": {"exist": true, "char_ranges": [[131, 165]], "word_ranges": [[15, 19]], "text": "Option 3. Non-displaced fractures."}, "4": {"exist": true, "char_ranges": [[166, 241]], "word_ranges": [[19, 31]], "text": "Option 4. Non ambulatory patients with minimal pain and high surgical risk."}, "5": {"exist": false, "char_ranges": [], "word_ranges": [], "text": ""}}} {"id": 391, "year": 2016, "question_id_specific": 222, "full_question": "A 26-year-old single man is brought in by his family because he has been refusing to leave his home for 3 months. The reason given by the patient is the belief that he has an asymmetrical jaw and a crooked face. According to the patient, this situation is progressive and he looks more and more deformed when he looks in the mirror. He is ashamed of his appearance, so he does not want to go out, he gets very anxious when he sees his image and he cannot stop thinking about his deformity all day long. He has consulted several maxillofacial surgeons but they tell him that he does not have facial asymmetry and refer him to a psychiatrist. The patient's diagnosis is:", "full_answer": "Another simple question with an immediate answer, which offers no doubt. It describes a patient worried about a non-existent physical defect, whose concern distresses him and prevents him from leaving the house. As a psychiatry resident, I wish the MIR questions in my specialty were a bit more thought-provoking and in-depth, although I know that the seconds you will have saved by marking the fourth one directly are very valuable.", "type": "PSYCHIATRY", "options": {"1": "Major depressive disorder with delusional ideas incongruent with mood.", "2": "Obsessive-compulsive disorder.", "3": "Paranoid schizophrenia.", "4": "Body dysmorphic disorder.", "5": NaN}, "correct_option": 4, "explanations": {"1": {"exist": false, "char_ranges": [], "word_ranges": [], "text": ""}, "2": {"exist": false, "char_ranges": [], "word_ranges": [], "text": ""}, "3": {"exist": false, "char_ranges": [], "word_ranges": [], "text": ""}, "4": {"exist": true, "char_ranges": [[73, 211]], "word_ranges": [[11, 32]], "text": "It describes a patient worried about a non-existent physical defect, whose concern distresses him and prevents him from leaving the house."}, "5": {"exist": false, "char_ranges": [], "word_ranges": [], "text": ""}}} {"id": 324, "year": 2016, "question_id_specific": 55, "full_question": "A 70-year-old female patient is admitted to the ICU after suffering anterior AMI treated by coronary angioplasty and stent placement in the anterior descending artery. Four days later, she suddenly presents hypotension that requires vigorous volume support, initiation of vasoactive drugs, orotracheal intubation and connection to mechanical ventilation. Physical examination revealed a murmur not previously present. Suspecting a mechanical complication of the infarction, transthoracic echocardiography showed pericardial effusion. Mark the CORRECT answer:", "full_answer": "Mortality is not so low, the oximetric jump occurs in the case of septum rupture, as well as fremitus (hence the correct answer is 3) and complications can occur up to the first week.", "type": "CRITICAL, PALLIATIVE AND EMERGENCY CARE", "options": {"1": "Mortality with medical treatment is 20%.", "2": "In case of free wall rupture, there is an oxymetric jump in the right ventricle in the Swan-Ganz catheterization.", "3": "In case of free wall rupture, there is no palpable frémito.", "4": "Mechanical complications usually appear on the first post-infarction day.", "5": NaN}, "correct_option": 3, "explanations": {"1": {"exist": false, "char_ranges": [], "word_ranges": [], "text": ""}, "2": {"exist": false, "char_ranges": [], "word_ranges": [], "text": ""}, "3": {"exist": true, "char_ranges": [[0, 133]], "word_ranges": [[0, 25]], "text": "Mortality is not so low, the oximetric jump occurs in the case of septum rupture, as well as fremitus (hence the correct answer is 3)"}, "4": {"exist": false, "char_ranges": [], "word_ranges": [], "text": ""}, "5": {"exist": false, "char_ranges": [], "word_ranges": [], "text": ""}}} {"id": 240, "year": 2014, "question_id_specific": 150, "full_question": "A 76-year-old man consults for cognitive impairment, slowness and clumsiness of movements, of eight months of evolution. The family reports that the patient had visual hallucinations, so his primary care physician prescribed low doses of risperidone, with a significant worsening of motor status. In view of these data, what is the most likely diagnosis?", "full_answer": "The correct answer is 2 because it presents the case with the typical symptoms of dementia due to Lewy bodies, being the guiding symptom visual hallucinations. Progressive cognitive impairment, parkinsonian motor signs (inferred from clumsiness of movements) and hypersensitivity to neuroleptics even in small doses are also mentioned.", "type": "NEUROLOGY", "options": {"1": "Alzheimer's disease.", "2": "Dementia due to Lewy bodies.", "3": "Frontotemporal dementia.", "4": "Vascular dementia.", "5": "Sporadic Creutzfeldt-Jakob disease."}, "correct_option": 2, "explanations": {"1": {"exist": false, "char_ranges": [], "word_ranges": [], "text": ""}, "2": {"exist": true, "char_ranges": [[0, 159]], "word_ranges": [[0, 26]], "text": "The correct answer is 2 because it presents the case with the typical symptoms of dementia due to Lewy bodies, being the guiding symptom visual hallucinations."}, "3": {"exist": false, "char_ranges": [], "word_ranges": [], "text": ""}, "4": {"exist": false, "char_ranges": [], "word_ranges": [], "text": ""}, "5": {"exist": false, "char_ranges": [], "word_ranges": [], "text": ""}}} {"id": 68, "year": 2012, "question_id_specific": 123, "full_question": "A 64-year-old man with a history of hypertension and ischemic heart disease comes to the emergency department for oppressive chest pain while watching television. In the ED triage consultation, T. art values of 155/95 mmHg and a capillary O2 saturation of 95% are detected. Which of the following is the most correct course of action?", "full_answer": "If the ECG shows ischemic changes, we will obviously treat the patient in the emergency room and notify the cardiologist. If not, you will be treated as if it were angina and observed, with serial enzymes and treatment in the observation area.", "type": "ANESTHESIOLOGY, CRITICAL CARE AND EMERGENCY MEDICINE", "options": {"1": "Instruct her to go to the waiting room. You will be notified for tests.", "2": "This patient should be treated in the vital emergency room (shock room).", "3": "We will notify the cardiologist on duty to evaluate the patient.", "4": "From the Triage consultation, the patient will be referred to the Coronary Unit.", "5": "We will do an ECG in less than 10 minutes."}, "correct_option": 5, "explanations": {"1": {"exist": false, "char_ranges": [], "word_ranges": [], "text": ""}, "2": {"exist": false, "char_ranges": [], "word_ranges": [], "text": ""}, "3": {"exist": false, "char_ranges": [], "word_ranges": [], "text": ""}, "4": {"exist": false, "char_ranges": [], "word_ranges": [], "text": ""}, "5": {"exist": false, "char_ranges": [], "word_ranges": [], "text": ""}}} {"id": 206, "year": 2014, "question_id_specific": 125, "full_question": "Faced with a 49-year-old man, asymptomatic, with a family history of a father who died of prostate cancer, who in a routine company control is identified a PSA (Prostate Specific Antigen) of 5.9 ng / ml, with a ratio of free PSA / total PSA of 11% and that a rectal examination shows increased consistency in the right prostatic lobe, what is the following clinical indication?", "full_answer": "The patient has a PSA >4 and a palpation with increased consistency, which implies the need for a transrectal ultrasound-guided biopsy to diagnose whether it is Benign Prostatic Hyperplasia or Prostate Cancer.", "type": "UROLOGY", "options": {"1": "Suggest to the patient the performance of a transrectal ultrasound and prostate biopsy.", "2": "Perform an abdominopelvic CT scan.", "3": "Initiate treatment with 5 alpha Reductase Inhibitors to reduce PSA levels by half.", "4": "Initiate combined treatment of LHRH analogues and antiandrogens.", "5": "Perform a bone scan."}, "correct_option": 1, "explanations": {"1": {"exist": true, "char_ranges": [[0, 209]], "word_ranges": [[0, 32]], "text": "The patient has a PSA >4 and a palpation with increased consistency, which implies the need for a transrectal ultrasound-guided biopsy to diagnose whether it is Benign Prostatic Hyperplasia or Prostate Cancer."}, "2": {"exist": false, "char_ranges": [], "word_ranges": [], "text": ""}, "3": {"exist": false, "char_ranges": [], "word_ranges": [], "text": ""}, "4": {"exist": false, "char_ranges": [], "word_ranges": [], "text": ""}, "5": {"exist": false, "char_ranges": [], "word_ranges": [], "text": ""}}} {"id": 570, "year": 2022, "question_id_specific": 153, "full_question": "A 19-year-old male with no past history of interest comes to the emergency department because after a respiratory infection he presents with asthenia, malaise, oliguria and headache. Physical examination: BP 210/120 mmHg, fundus with hypertensive retinopathy grade III. Laboratory tests: Hb 7.4 g/dl, platelets 85,000/mm3, 2-3 schistocytes in the peripheral blood smear, LDH 950 IU/ml, serum creatinine 8.75 mg/dl. Urine system: proteinuria 300 mg/dl, sediment 15 red blood cells per field. ADAMTS-13 levels are normal. Given these findings, what would be your main diagnostic suspicion?", "full_answer": "Difficult question. The patient presents with thrombotic microangiopathy (TMA) with acute renal failure. This points to HUS, TTP, drug-induced TMA, or TMA mediated by complement, coagulation, or metabolism. Normal ADAMTS-13 levels rule out TTP (incorrect choice 1). The absence of gastrointestinal symptoms rules out typical Shiga toxin HUS (incorrect choice 4). Pneumococcal infections can also cause HUS, although it is rare and more so in adults. The history of respiratory infection (not specified, it could be pneumococcal pneumonia) together with the absence of DIC data (no mention of bleeding or altered coagulation tests) makes me think of atypical HUS (option 2 correct).", "type": "NEPHROLOGY", "options": {"1": "Thrombotic thrombocytopenic purpura.", "2": "Atypical hemolytic uremic syndrome.", "3": "Disseminated intravascular coagulation.", "4": "Typical hemolytic uremic syndrome.", "5": NaN}, "correct_option": 2, "explanations": {"1": {"exist": true, "char_ranges": [[207, 265]], "word_ranges": [[28, 37]], "text": "Normal ADAMTS-13 levels rule out TTP (incorrect choice 1)."}, "2": {"exist": true, "char_ranges": [[450, 681]], "word_ranges": [[66, 102]], "text": "The history of respiratory infection (not specified, it could be pneumococcal pneumonia) together with the absence of DIC data (no mention of bleeding or altered coagulation tests) makes me think of atypical HUS (option 2 correct)."}, "3": {"exist": false, "char_ranges": [], "word_ranges": [], "text": ""}, "4": {"exist": true, "char_ranges": [[266, 362]], "word_ranges": [[37, 51]], "text": "The absence of gastrointestinal symptoms rules out typical Shiga toxin HUS (incorrect choice 4)."}, "5": {"exist": false, "char_ranges": [], "word_ranges": [], "text": ""}}} {"id": 506, "year": 2020, "question_id_specific": 87, "full_question": "Faced with sudden-onset obsessive-compulsive disorder (OCD) in a 9-year-old boy who also presents with tics and chorea as neurological manifestations, infection by:", "full_answer": "There is a large literature on this subject. It never ceases to amaze us the strange ways of biology and that the axiom \"the longest is the right one\" is fulfilled in this case.", "type": "PSYCHIATRY", "options": {"1": "Streptococcus viridans.", "2": "Alpha-hemolytic streptococcus.", "3": "Beta-hemolytic streptococcus, group A.", "4": "Enterococcus.", "5": NaN}, "correct_option": 3, "explanations": {"1": {"exist": false, "char_ranges": [], "word_ranges": [], "text": ""}, "2": {"exist": false, "char_ranges": [], "word_ranges": [], "text": ""}, "3": {"exist": false, "char_ranges": [], "word_ranges": [], "text": ""}, "4": {"exist": false, "char_ranges": [], "word_ranges": [], "text": ""}, "5": {"exist": false, "char_ranges": [], "word_ranges": [], "text": ""}}} {"id": 619, "year": 2022, "question_id_specific": 121, "full_question": "79-year-old woman admitted for an osteoporotic hip fracture. Regarding secondary prevention of fragility fractures, point out the WRONG answer:", "full_answer": "There are doubts regarding the efficacy of calcium or vitamin D administered in monotherapy for the treatment of osteoporosis; they point out that \"administered together they seem to have a certain degree of efficacy in the prevention of non-vertebral fractures, which is clearer in people who are deficient in them, as may be the case of elderly people living in residences. There is practically no evidence of their efficacy in the prevention of vertebral fractures.", "type": "TRAUMATOLOGY", "options": {"1": "Low adherence to treatment is associated with an increased risk of fracture.", "2": "Bone remodeling markers may be useful for early monitoring of response to treatment.", "3": "Vitamin D monotherapy is effective in reducing these fractures in non-institutionalized elderly people.", "4": "Increasing dietary calcium or taking calcium supplements in isolation does not protect against the occurrence of fractures.", "5": NaN}, "correct_option": 3, "explanations": {"1": {"exist": false, "char_ranges": [], "word_ranges": [], "text": ""}, "2": {"exist": false, "char_ranges": [], "word_ranges": [], "text": ""}, "3": {"exist": true, "char_ranges": [[0, 126]], "word_ranges": [[0, 19]], "text": "There are doubts regarding the efficacy of calcium or vitamin D administered in monotherapy for the treatment of osteoporosis;"}, "4": {"exist": false, "char_ranges": [], "word_ranges": [], "text": ""}, "5": {"exist": false, "char_ranges": [], "word_ranges": [], "text": ""}}} {"id": 161, "year": 2013, "question_id_specific": 118, "full_question": "A 60-year-old woman diagnosed with chronic bronchopathy, diabetes mellitus, hypertension and treated for 4 months with Omeprazole, Metformin, Salbutamol, Ipratropium Bromide and Enalapril 20 mg + Hydrochlorothiazide 25 mg comes to your office for fatigue, decreased appetite, with slight dyspnea and occasional cough, variable stools, sometimes soft and without urinary symptoms. Blood tests show leukocytes 10,000/mm³, Ht 35%, MCV 80, Glucose 150 mg/dl, Urea 80 mg/dl, Creatinine 1.6 mg/dl, Sodium 133 mEq/l and Potassium 2.9 mEq/l. What is the most likely cause of hypokalemia?", "full_answer": "The correct answer is: 4. Antihypertensive. In particular, hydrochlorothiazide, as a thiazide, is primarily responsible for this patient's hypokalemia.", "type": "NEPHROLOGY", "options": {"1": "Renal insufficiency.", "2": "Hyponatremia.", "3": "Potassium deficiency.", "4": "Antihypertensive.", "5": "Metformin."}, "correct_option": 4, "explanations": {"1": {"exist": false, "char_ranges": [], "word_ranges": [], "text": ""}, "2": {"exist": false, "char_ranges": [], "word_ranges": [], "text": ""}, "3": {"exist": false, "char_ranges": [], "word_ranges": [], "text": ""}, "4": {"exist": true, "char_ranges": [[26, 151]], "word_ranges": [[5, 19]], "text": "Antihypertensive. In particular, hydrochlorothiazide, as a thiazide, is primarily responsible for this patient's hypokalemia."}, "5": {"exist": false, "char_ranges": [], "word_ranges": [], "text": ""}}} {"id": 269, "year": 2014, "question_id_specific": 141, "full_question": "A 55-year-old asthmatic patient comes to the ED with an exacerbation. One hour after administration of supplemental oxygen and two nebulizations of salbutamol the patient does not improve. On examination, he breathes at 42 rpm with supraclavicular pull v with scattered inspiratory and expiratory wheezing. Peak flow has decreased from 310 to 220 L/min. A blood gas drawn with 28% oxygen shows a p02 of 54 mmHg and a pC02 of 35 mm Hg. Which of the following do you think is LEAST indicated?", "full_answer": "In a severe asthma crisis all options are indicated or may be indicated before more aggressive options. In the case of a bad evolution with acidosis etc., the patient would be tributary to invasive medication.", "type": "PNEUMOLOGY", "options": {"1": "Increase oxygen flow.", "2": "Administer 80 mg of methylprednisolone iv.", "3": "Nebulize ipratropium bromide together with salbutamol every 20 minutes.", "4": "Administer intravenous magnesium.", "5": " Start noninvasive mechanical ventilation."}, "correct_option": 5, "explanations": {"1": {"exist": false, "char_ranges": [], "word_ranges": [], "text": ""}, "2": {"exist": false, "char_ranges": [], "word_ranges": [], "text": ""}, "3": {"exist": false, "char_ranges": [], "word_ranges": [], "text": ""}, "4": {"exist": false, "char_ranges": [], "word_ranges": [], "text": ""}, "5": {"exist": true, "char_ranges": [[104, 209]], "word_ranges": [[17, 35]], "text": "In the case of a bad evolution with acidosis etc., the patient would be tributary to invasive medication."}}} {"id": 18, "year": 2011, "question_id_specific": 138, "full_question": "If a patient reports having for months now enlargement of the tongue, with no other alterations of the tongue causing constant biting, which disease would you think of the following:", "full_answer": "Question shared with ENT. Amyloidosis is a localized or systemic deposition of fibrillar proteins. It can be primary or secondary. The most frequently affected organs are the kidneys and the heart. Sometimes amyloid deposits appear on the tongue which can produce macroglossia.", "type": "DERMATOLOGY", "options": {"1": "Hypoglossal paralysis.", "2": "Hyperthyroidism.", "3": "Mastocytosis.", "4": "Amyloidosis.", "5": "Facial granuloma."}, "correct_option": 4, "explanations": {"1": {"exist": false, "char_ranges": [], "word_ranges": [], "text": ""}, "2": {"exist": false, "char_ranges": [], "word_ranges": [], "text": ""}, "3": {"exist": false, "char_ranges": [], "word_ranges": [], "text": ""}, "4": {"exist": true, "char_ranges": [[26, 277]], "word_ranges": [[4, 42]], "text": "Amyloidosis is a localized or systemic deposition of fibrillar proteins. It can be primary or secondary. The most frequently affected organs are the kidneys and the heart. Sometimes amyloid deposits appear on the tongue which can produce macroglossia."}, "5": {"exist": false, "char_ranges": [], "word_ranges": [], "text": ""}}} {"id": 250, "year": 2014, "question_id_specific": 78, "full_question": "A 55-year-old man, father of a celiac child, with iron-deficiency anemia and a recent increase in the stool rhythm, has been tested negative for HLA-DQ2 and HLA-DQ8 alleles. Which diagnostic test is the most appropriate in this case?", "full_answer": "Middle-aged male with iron deficiency anemia and increased number of stools, a colonoscopy should be done to rule out colorectal cancer!!! If the HLA is negative it is very unlikely to be celiac disease (high negative predictive value).", "type": "DIGESTIVE SYSTEM", "options": {"1": "Determination of anti-transglutaminase IgA antibodies.", "2": "Upper endoscopy with duodenal biopsies.", "3": "D-xylose test.", "4": "Evaluate response to gluten-free diet.", "5": "Colonoscopy."}, "correct_option": 5, "explanations": {"1": {"exist": false, "char_ranges": [], "word_ranges": [], "text": ""}, "2": {"exist": false, "char_ranges": [], "word_ranges": [], "text": ""}, "3": {"exist": false, "char_ranges": [], "word_ranges": [], "text": ""}, "4": {"exist": true, "char_ranges": [[139, 236]], "word_ranges": [[21, 38]], "text": "If the HLA is negative it is very unlikely to be celiac disease (high negative predictive value)."}, "5": {"exist": true, "char_ranges": [[0, 138]], "word_ranges": [[0, 21]], "text": "Middle-aged male with iron deficiency anemia and increased number of stools, a colonoscopy should be done to rule out colorectal cancer!!!"}}} {"id": 5, "year": 2011, "question_id_specific": 44, "full_question": "A 66-year-old man reports daily heartburn since youth, which he treats with alkalis. An upper endoscopy shows mild erosive esophagitis and displaced squamous columnar junction approximately 7 cm. The esophageal biopsies showed mild erosive esophagitis and a columnar squamous junction displaced approximately 7 cm from the most proximal portion of the gastric folds. Biopsies of the distal esophagus reveal that the normal squamous epithelium has been replaced by intestinal-type columnar epithelium with low-grade dysplasia What is the most appropriate management option for this patient?", "full_answer": "This patient presents Barrett's esophagus with low grade dysplasia. I do not like the question, since not being with acid production inhibition the dysplasia may be conditioned by inflammation and not be real. In any case, Barrett's esophagus should follow endoscopic surveillance and lifelong PPI treatment or antireflux surgery to prevent progression. The correct answer is 4.", "type": "DIGESTIVE", "options": {"1": "Given that the esophagitis is mild and the dysplasia is low grade, it is advisable to continue treatment with alkalines.", "2": "Indefinite treatment with high doses of PPI since it has been demonstrated that this prevents the progression of Barrett's esophagus to adenocarcinoma, making endoscopic surveillance unnecessary.", "3": "Periodic surveillance endoscopies and treatment with PPIs for less than 12 weeks, since prolonged treatment is associated with a high risk of gastrinoma development.", "4": "Periodic surveillance endoscopies and indefinite treatment with PPI.", "5": "Esophagectomy."}, "correct_option": 4, "explanations": {"1": {"exist": false, "char_ranges": [], "word_ranges": [], "text": ""}, "2": {"exist": false, "char_ranges": [], "word_ranges": [], "text": ""}, "3": {"exist": false, "char_ranges": [], "word_ranges": [], "text": ""}, "4": {"exist": true, "char_ranges": [[210, 377]], "word_ranges": [[33, 57]], "text": "In any case, Barrett's esophagus should follow endoscopic surveillance and lifelong PPI treatment or antireflux surgery to prevent progression. The correct answer is 4."}, "5": {"exist": false, "char_ranges": [], "word_ranges": [], "text": ""}}} {"id": 359, "year": 2016, "question_id_specific": 91, "full_question": "The determination in a 70-year-old woman of hematocrit 32%, Hb 9.6 g/dL, MCV 85 fL (70-99), Fe 25 ug/dL (37-145) and ferritin 350 ng/mL (15-150), is more suggestive of:", "full_answer": "Polymyalgia rheumatica. Classic example I suppose, and it is seen in the clinic in droves. Older woman with normocitic anemia and clear ferrokinetic pattern of chronic disorder (hyposideremia but hyperferritinemia). The only one that fits, and it fits perfectly, is number 3; they only needed to give us a high ESR. 1: if it is a recent loss, the MCV could remain normal, but the ferritin should be affected, and certainly not elevated. 2: the same. If it is due to Fe malabsorption, ferritin should be low. And if it were due to B12 malabsorption, the MCV would be clearly higher. 4: 70 tachycardia and normal MCV. This one is too much.", "type": "HEMATOLOGY", "options": {"1": "Recent blood loss through the digestive tract.", "2": "Intestinal malabsorption.", "3": "Polymyalgia rheumatica.", "4": "Thalassemia minor.", "5": NaN}, "correct_option": 3, "explanations": {"1": {"exist": true, "char_ranges": [[316, 436]], "word_ranges": [[51, 73]], "text": "1: if it is a recent loss, the MCV could remain normal, but the ferritin should be affected, and certainly not elevated."}, "2": {"exist": true, "char_ranges": [[437, 581]], "word_ranges": [[73, 101]], "text": "2: the same. If it is due to Fe malabsorption, ferritin should be low. And if it were due to B12 malabsorption, the MCV would be clearly higher."}, "3": {"exist": true, "char_ranges": [[91, 275]], "word_ranges": [[15, 42]], "text": "Older woman with normocitic anemia and clear ferrokinetic pattern of chronic disorder (hyposideremia but hyperferritinemia). The only one that fits, and it fits perfectly, is number 3;"}, "4": {"exist": true, "char_ranges": [[582, 637]], "word_ranges": [[101, 112]], "text": "4: 70 tachycardia and normal MCV. This one is too much."}, "5": {"exist": false, "char_ranges": [], "word_ranges": [], "text": ""}}} {"id": 108, "year": 2012, "question_id_specific": 219, "full_question": "A man has a partial deletion in the dystrophin gene (chromosome Xp21) which causes the semiology of Becker muscular dystrophy. He goes to the genetic consultation with his wife, to assess the risks of transmission of the disease. What correct information will be provided in the course of the genetic counseling?", "full_answer": "It is the male who presents the disease, which we are told is X-linked, so there are only two options left: 2 and 3. 2 is not because the males will receive the Y chromosome from the father and the X from the mother, who is healthy, so the daughters will be able to carry the disease (correct option 3).", "type": "GENETICS AND IMMUNOLOGY", "options": {"1": "According to autosomal dominant inheritance, half of her children will inherit the disease and without distinction of sexes.", "2": "Their daughters will not inherit the disease, but all their future sons will be carriers and can transmit the mutation to 50%.", "3": "Their sons will not inherit the disease, but all their future daughters will be carriers and can transmit the mutation to 50%.", "4": "There is no risk: the inheritance of the disease is of the mitochondrial type, never transmitted by males.", "5": "According to autosomal recessive inheritance, 25% of their children will manifest the disease in childhood, regardless of sex."}, "correct_option": 3, "explanations": {"1": {"exist": false, "char_ranges": [], "word_ranges": [], "text": ""}, "2": {"exist": true, "char_ranges": [[0, 303]], "word_ranges": [[0, 60]], "text": "It is the male who presents the disease, which we are told is X-linked, so there are only two options left: 2 and 3. 2 is not because the males will receive the Y chromosome from the father and the X from the mother, who is healthy, so the daughters will be able to carry the disease (correct option 3)."}, "3": {"exist": true, "char_ranges": [[0, 303]], "word_ranges": [[0, 60]], "text": "It is the male who presents the disease, which we are told is X-linked, so there are only two options left: 2 and 3. 2 is not because the males will receive the Y chromosome from the father and the X from the mother, who is healthy, so the daughters will be able to carry the disease (correct option 3)."}, "4": {"exist": false, "char_ranges": [], "word_ranges": [], "text": ""}, "5": {"exist": false, "char_ranges": [], "word_ranges": [], "text": ""}}} {"id": 255, "year": 2014, "question_id_specific": 94, "full_question": "A 65-year-old woman with a history of joint pain and under treatment with anti-inflammatory drugs is referred for anemia. In the complementary studies she presented red blood cells 3,164,000, Hto. 32%, Hb 11g/dl, MCV 69 fl, Leukocytes 7800, Platelets 370,000, CRP 0.29 mg/dl, Fe 20ng/ml, Ferritin 18 ng/ml, Glucose 105 mg/dl, GOT, GPT, GGT, Alkaline F, total Bilirubin, Cholesterol, Creatinine, Calcium and Phosphorus normal. Negative antitransglutaminase and antigliadin antibodies. Gastroscopy: hiatus hernia of 3 cm, the rest without alterations. Colonoscopy: up to cecum, isolated diverticula in sigma. Intestinal transit without alterations. Ultrasound of the abdomen without alterations. Which of the following examinations do you consider most appropriate to complete the study?", "full_answer": "It is a study of iron deficiency anemia with normal gastroscopy and colonoscopy, having ruled out possible celiac disease. We have yet to see jejunum and ileum, so we will have to perform a capsule endoscopy. In iron deficiency anemia, a digestive cause must always be ruled out.", "type": "DIGESTIVE SYSTEM", "options": {"1": "Jejunum biopsy.", "2": "Arteriography.", "3": "Endoscopic capsule.", "4": "Radioisotopes.", "5": "Pelvic MRI."}, "correct_option": 3, "explanations": {"1": {"exist": false, "char_ranges": [], "word_ranges": [], "text": ""}, "2": {"exist": false, "char_ranges": [], "word_ranges": [], "text": ""}, "3": {"exist": true, "char_ranges": [[0, 208]], "word_ranges": [[0, 36]], "text": "It is a study of iron deficiency anemia with normal gastroscopy and colonoscopy, having ruled out possible celiac disease. We have yet to see jejunum and ileum, so we will have to perform a capsule endoscopy."}, "4": {"exist": false, "char_ranges": [], "word_ranges": [], "text": ""}, "5": {"exist": false, "char_ranges": [], "word_ranges": [], "text": ""}}} {"id": 303, "year": 2016, "question_id_specific": 111, "full_question": "In a patient with rheumatoid arthritis on methotrexate, prednisone and indomethacin who acutely presents with edema and increased plasma creatinine with poorly expressive urine sediment and proteinuria less than 100 mg/24 h the most likely cause is:", "full_answer": "Amyloidosis presents with proteinuria usually in the nephrotic range, and has a more chronic course. GMN associated with rheumatoid arthritis characteristically alters the sediment, and renal failure due to NSAIDs has a more chronic course, with papillary necrosis. Thus, given the course and the alterations described, interstitial nephritis due to methotrexate is more likely, which is more directly toxic than allergic, hence the absence of pyuria, eosinophilia, fever or other hypersensitivity data.", "type": "NEPHROLOGY", "options": {"1": "Renal amyloidosis.", "2": "Glomerulonephritis secondary to rheumatoid arthritis.", "3": "Renal failure due to nonsteroidal anti-inflammatory drugs.", "4": "Interstitial nephritis due to methotrexate.", "5": NaN}, "correct_option": 4, "explanations": {"1": {"exist": true, "char_ranges": [[0, 100]], "word_ranges": [[0, 15]], "text": "Amyloidosis presents with proteinuria usually in the nephrotic range, and has a more chronic course."}, "2": {"exist": true, "char_ranges": [[101, 181]], "word_ranges": [[15, 24]], "text": "GMN associated with rheumatoid arthritis characteristically alters the sediment,"}, "3": {"exist": true, "char_ranges": [[186, 265]], "word_ranges": [[25, 38]], "text": "renal failure due to NSAIDs has a more chronic course, with papillary necrosis."}, "4": {"exist": true, "char_ranges": [[272, 503]], "word_ranges": [[39, 72]], "text": "given the course and the alterations described, interstitial nephritis due to methotrexate is more likely, which is more directly toxic than allergic, hence the absence of pyuria, eosinophilia, fever or other hypersensitivity data."}, "5": {"exist": false, "char_ranges": [], "word_ranges": [], "text": ""}}} {"id": 539, "year": 2021, "question_id_specific": 3, "full_question": "A 45-year-old patient who presents with a first episode of low back pain of 3 weeks' evolution that prevents him from carrying out his normal life. She does not refer trauma or other underlying pathology. In the physical examination there is no neurological deficit. Which imaging test would be indicated?", "full_answer": "A first episode of acute low back pain (less than 6 weeks duration), does not require imaging tests in the absence of \"red flags\" that may indicate an underlying process, such as a fracture, infection or malignant tumor.", "type": "TRAUMATOLOGY", "options": {"1": "As this is the first episode of low back pain, only a simple X-ray of the lumbar spine.", "2": "MRI, which provides more information on soft tissue and possible herniations.", "3": "CT scan, to better assess the bone structure and possible fractures.", "4": "No indication for imaging test.", "5": NaN}, "correct_option": 4, "explanations": {"1": {"exist": false, "char_ranges": [], "word_ranges": [], "text": ""}, "2": {"exist": false, "char_ranges": [], "word_ranges": [], "text": ""}, "3": {"exist": false, "char_ranges": [], "word_ranges": [], "text": ""}, "4": {"exist": true, "char_ranges": [[0, 220]], "word_ranges": [[0, 38]], "text": "A first episode of acute low back pain (less than 6 weeks duration), does not require imaging tests in the absence of \"red flags\" that may indicate an underlying process, such as a fracture, infection or malignant tumor."}, "5": {"exist": false, "char_ranges": [], "word_ranges": [], "text": ""}}} {"id": 408, "year": 2018, "question_id_specific": 190, "full_question": "A 45-year-old woman diagnosed with stage IV breast cancer with multiple bone metastases on hormonal treatment and iv bisphosphonates. She comes to your office referring intense bone pain (VAS=7), in several locations without associated neurological symptoms and exarcerbations of pain (VAS=9) several times a day that force her to remain at rest. In the treatment with ibuprofen (600 mg every 8 h) and tramadol (150 mg per day), what treatment would you consider most appropriate in this patient to improve the pain?", "full_answer": "The correct answer is 4 because if the patient has bone metastases and may suffer fractures we can control inflammatory pain with ibuprofen and it will also help us to use the antalgic synergy. We should change the treatment to a major opioid and treat the disrruptive pain vigorously since she is at almost the maximum pain she can bear.", "type": "PRIMARY CARE", "options": {"1": "Suspend ibuprofen and tramadol and start a major opioid such as morphine or oxycodone.", "2": "Suspend tramadol, maintain ibuprofen and initiate a major opioid such as morphine and oxycodone.", "3": "Discontinue ibuprofen and tramadol and start a major opioid such as morphine or oxycodone and at the same time start treatment of breakthrough pain with sublingual fentanyl.", "4": "Suspend tramadol, maintain ibuprofen and initiate a major opioid such as morphine and oxycodone and at the same time treatment for breakthrough pain with sublingual fentanyl.", "5": NaN}, "correct_option": 4, "explanations": {"1": {"exist": false, "char_ranges": [], "word_ranges": [], "text": ""}, "2": {"exist": false, "char_ranges": [], "word_ranges": [], "text": ""}, "3": {"exist": false, "char_ranges": [], "word_ranges": [], "text": ""}, "4": {"exist": true, "char_ranges": [[0, 338]], "word_ranges": [[0, 60]], "text": "The correct answer is 4 because if the patient has bone metastases and may suffer fractures we can control inflammatory pain with ibuprofen and it will also help us to use the antalgic synergy. We should change the treatment to a major opioid and treat the disrruptive pain vigorously since she is at almost the maximum pain she can bear."}, "5": {"exist": false, "char_ranges": [], "word_ranges": [], "text": ""}}} {"id": 29, "year": 2011, "question_id_specific": 60, "full_question": "A 71-year-old patient, smoker of 2 packs of cigarettes/day for 50 years, with good quality of life and no other pathologic history of interest, consults for cough and hemoptotic expectoration. Physical examination revealed no significant pathological findings. Chest X-ray showed a mass of 4x3.4 cm lower right, a finding that was confirmed in the thoracoabdominal CT scan, which located the lesion in the lower lobe and found no adenopathy or metastasis. Bronchoscopy showed an infiltrating lesion in the right basal pyramid, biopsied as a well-differentiated squamous cell carcinoma. As for respiratory function tests, the FVC is 3,300 cc (84%) and the FEV 1 is 2,240 cc (80%). What should be the next step to follow in this patient?", "full_answer": "1 and 5: These are diagnostic tests, they are meaningless since you have already made the diagnosis. 2: Age should not be a limiting factor, especially considering that it is only 71 years old, but especially because the patient's \"quality of life\" is good (and I put \"quality of life\" because it is a term used erroneously, it is supposed to mean that he maintains a good functional level). 3: Presents a very mild respiratory limitation, does not require such a test. 4: Is the correct one.", "type": "NEUROLOGY AND THORACIC SURGERY", "options": {"1": "Perform mediastinoscopy.", "2": "Surgical intervention with curative intent should be contraindicated due to the advanced age of the patient.", "3": "An oxygen consumption test should be performed, given the respiratory limitation he presents.", "4": "Surgical resection (lobectomy of the lower lobe of the right lung) should be indicated directly.", "5": "A trantoracic puncture should be performed."}, "correct_option": 4, "explanations": {"1": {"exist": true, "char_ranges": [[9, 99]], "word_ranges": [[3, 17]], "text": "These are diagnostic tests, they are meaningless since you have already made the diagnosis."}, "2": {"exist": true, "char_ranges": [[104, 391]], "word_ranges": [[18, 69]], "text": "Age should not be a limiting factor, especially considering that it is only 71 years old, but especially because the patient's \"quality of life\" is good (and I put \"quality of life\" because it is a term used erroneously, it is supposed to mean that he maintains a good functional level)."}, "3": {"exist": true, "char_ranges": [[395, 468]], "word_ranges": [[70, 82]], "text": "Presents a very mild respiratory limitation, does not require such a test."}, "4": {"exist": false, "char_ranges": [], "word_ranges": [], "text": ""}, "5": {"exist": true, "char_ranges": [[9, 99]], "word_ranges": [[3, 17]], "text": "These are diagnostic tests, they are meaningless since you have already made the diagnosis."}}} {"id": 10, "year": 2011, "question_id_specific": 37, "full_question": "A 76-year-old woman with no history other than hypertension consults for painless jaundice and pruritus with anorexia. Laboratory tests show a bilirubin level of 12 mg/dl (with 9.5 direct bilirubin). Abdominal ultrasound shows dilatation of the intrahepatic and extrahepatic biliary tract, together with a single 2 cm nodule located peripherally in the anterior aspect of the left lobe. CT scan confirms the findings, also demonstrating the presence of a 3.5 cm mass in the head of the pancreas. Fine needle aspiration confirms the diagnosis of adenocarcinoma. Indicate the best therapeutic option:", "full_answer": "I think the correct answer is 4, although both lesions are resectable, pancreatic adenocarcinoma with synchronous hepatic metastases has no surgical indication because it does not prolong survival and has a high morbidity. Surgical bypass has more complications than placing a self-expanding metallic prosthesis that has a median patency of 6 months which is similar to the median survival of a patient with hepatic metastases of pancreatic adenocarcinoma.", "type": "SURGERY", "options": {"1": "Adjuvant chemo/radiotherapy, conditioning radical surgery to the initial response.", "2": "Percutaneous external drainage is palliative, with eventual conversion to internal drainage in case of intolerance or complications.", "3": "Surgical biliary bypass, with or without prophylactic gastrojejunostomy depending on intraoperative findings.", "4": "Metallic biliary prosthesis by endoscopic retrograde cholangiography, with the option of palliative chemotherapy.", "5": "Cephalic duodenopancreatectomy with radiofrequency ablation or alcoholization of the liver lesion."}, "correct_option": 4, "explanations": {"1": {"exist": false, "char_ranges": [], "word_ranges": [], "text": ""}, "2": {"exist": false, "char_ranges": [], "word_ranges": [], "text": ""}, "3": {"exist": false, "char_ranges": [], "word_ranges": [], "text": ""}, "4": {"exist": true, "char_ranges": [[0, 221]], "word_ranges": [[0, 33]], "text": "I think the correct answer is 4, although both lesions are resectable, pancreatic adenocarcinoma with synchronous hepatic metastases has no surgical indication because it does not prolong survival and has a high morbidity."}, "5": {"exist": false, "char_ranges": [], "word_ranges": [], "text": ""}}} {"id": 284, "year": 2016, "question_id_specific": 57, "full_question": "A 72-year-old woman presents to the hospital with oppressive chest pain of 2 hours' duration. Blood pressure is 68/32 mm Hg, heart rate is 124 beats/min, respiratory rate is 32 breaths/min, oxygen saturation is 91% with oxygen at 50%, bilateral crackles on pulmonary auscultation. An ECG shows a subepicardial lesion in the anterior face, with a specular image in the inferior face. Chest X-ray shows bilateral alveolar pattern. What is your therapeutic decision?", "full_answer": "Perform urgent coronary angiography.", "type": "CARDIOLOGY AND VASCULAR SURGERY", "options": {"1": "Start i.v. nitrate infusion.", "2": "Perform fibrinolysis with tenecteplase (TNK).", "3": "Perform urgent coronary angioplasty.", "4": "Administer furosemide 40 mg i.v.", "5": NaN}, "correct_option": 3, "explanations": {"1": {"exist": false, "char_ranges": [], "word_ranges": [], "text": ""}, "2": {"exist": false, "char_ranges": [], "word_ranges": [], "text": ""}, "3": {"exist": true, "char_ranges": [[0, 36]], "word_ranges": [[0, 4]], "text": "Perform urgent coronary angiography."}, "4": {"exist": false, "char_ranges": [], "word_ranges": [], "text": ""}, "5": {"exist": false, "char_ranges": [], "word_ranges": [], "text": ""}}} {"id": 188, "year": 2013, "question_id_specific": 230, "full_question": "A 57-year-old patient had an acute myocardial infarction and has a blood test with cholesterol 312 mg/dl, LDL 241 mg/dl, HDL 29 and normal TG. He smoked 30 cig/day, was not obese, his blood pressure was 145/90 mm Hg and he did not have diabetes. When asked about family history, he mentioned that an uncle on his mother's side died suddenly at 53 years of age. Which answer do you agree with the most?", "full_answer": "Question on a fashionable topic in recent years: dyslipidemia; I would answer it by rule out. The treatment for hypercholesterolemia is statins so we rule out answers 2 and 4. The statin in a patient with cardiovascular disease (such as an AMI) is a lifelong treatment (we discard 1) since it is not taken until LDL levels are normalized. Between 3 and 5, 5 is more complete as it includes treatment and control of other risk factors. Ruling out secondary dyslipidemia is easy with the patient's clinical history and a blood test with thyroid function.", "type": "ENDOCRINOLOGY", "options": {"1": "Treat with statins until cholesterol is normalized and recommend healthy lifestyle (diet, exercise and no smoking).", "2": "Rule out hypothyroidism or other systemic disease, recommend a healthy lifestyle and treat with fibrates to normalize cholesterol.", "3": "It is not necessary to rule out secondary dyslipidemia because it is infrequent. A family lipid profile should be performed.", "4": "Rule out secondary dyslipidemia, treat with resins (resincholestyramine) and fibrates and re-commend a healthy lifestyle. Prohibit alcohol.", "5": "Rule out secondary dyslipidemia, request a family study and treat with statins to maintain a good lipid profile. Control of other risk factors."}, "correct_option": 5, "explanations": {"1": {"exist": true, "char_ranges": [[176, 338]], "word_ranges": [[30, 59]], "text": "The statin in a patient with cardiovascular disease (such as an AMI) is a lifelong treatment (we discard 1) since it is not taken until LDL levels are normalized."}, "2": {"exist": true, "char_ranges": [[94, 175]], "word_ranges": [[16, 30]], "text": "The treatment for hypercholesterolemia is statins so we rule out answers 2 and 4."}, "3": {"exist": true, "char_ranges": [[339, 434]], "word_ranges": [[59, 77]], "text": "Between 3 and 5, 5 is more complete as it includes treatment and control of other risk factors."}, "4": {"exist": true, "char_ranges": [[94, 175]], "word_ranges": [[16, 30]], "text": "The treatment for hypercholesterolemia is statins so we rule out answers 2 and 4."}, "5": {"exist": true, "char_ranges": [[339, 434]], "word_ranges": [[59, 77]], "text": "Between 3 and 5, 5 is more complete as it includes treatment and control of other risk factors."}}} {"id": 349, "year": 2016, "question_id_specific": 160, "full_question": "A 45-year-old woman, mother of 3 children, attends an early diagnosis consultation. Cervicovaginal cytology is compatible with a high-grade squamous intraepithelial lesion. Which of the following options would you choose?", "full_answer": "The correct answer is 2. According to the 2014 SEGO Oncoguide on cervical cancer prevention, when a cytology shows a high-grade squamous intraepithelial lesion, the action to be taken is a colposcopy, with or without biopsy depending on the findings (regardless of the patient's age and parity).", "type": "GYNECOLOGY AND OBSTETRICS", "options": {"1": "Repeat cytology in 1 month.", "2": "Colposcopy with possible biopsy.", "3": "Hysterectomy with bilateral salpinguectomy and ovarian conservation.", "4": "Fractionated uterine curettage.", "5": NaN}, "correct_option": 2, "explanations": {"1": {"exist": false, "char_ranges": [], "word_ranges": [], "text": ""}, "2": {"exist": true, "char_ranges": [[25, 295]], "word_ranges": [[5, 47]], "text": "According to the 2014 SEGO Oncoguide on cervical cancer prevention, when a cytology shows a high-grade squamous intraepithelial lesion, the action to be taken is a colposcopy, with or without biopsy depending on the findings (regardless of the patient's age and parity)."}, "3": {"exist": false, "char_ranges": [], "word_ranges": [], "text": ""}, "4": {"exist": false, "char_ranges": [], "word_ranges": [], "text": ""}, "5": {"exist": false, "char_ranges": [], "word_ranges": [], "text": ""}}} {"id": 290, "year": 2016, "question_id_specific": 50, "full_question": "A 58-year-old man referred a 3-week history of progressive, mildly painful skin lesions on his left arm. It had started as an erythematous lesion on his left thumb. He had visible reddish striae as connecting lines between the lesions. The patient had no fever or other general symptoms. He had been working in his garden but did not remember any injuries. The etiologic diagnosis was made by culture of a skin biopsy. What is the most likely causative agent of this process?", "full_answer": "The lesions with sporotrichoid or lymphocutaneous pattern described may correspond to sporotrichosis or aquarium granuloma, so the first two would be ruled out. The history of having been working in the garden points more to a sporotrichosis, although the fact that the question asks for the \"causative agent\" and not the name of the disease misleads a little, so that if we consider sporotrichosis in reality the answer should be Sporothrix schenckii Despite this, knowing that this question will be correct for many examinees although it is poorly worded -which cancels it directly- the challenge will not be promoted.", "type": "DERMATOLOGY, VENEREOLOGY AND PLASTIC SURGERY", "options": {"1": "Dermatophytosis due to Microsporum gypseum.", "2": "Cutaneous infection by Staphylococcus aureus.", "3": "Sporotrichosis.", "4": "Cutaneous infection by Mycobacterium marinum.", "5": NaN}, "correct_option": 3, "explanations": {"1": {"exist": true, "char_ranges": [[0, 160]], "word_ranges": [[0, 23]], "text": "The lesions with sporotrichoid or lymphocutaneous pattern described may correspond to sporotrichosis or aquarium granuloma, so the first two would be ruled out."}, "2": {"exist": true, "char_ranges": [[0, 160]], "word_ranges": [[0, 23]], "text": "The lesions with sporotrichoid or lymphocutaneous pattern described may correspond to sporotrichosis or aquarium granuloma, so the first two would be ruled out."}, "3": {"exist": true, "char_ranges": [[161, 242]], "word_ranges": [[23, 37]], "text": "The history of having been working in the garden points more to a sporotrichosis,"}, "4": {"exist": true, "char_ranges": [[161, 242]], "word_ranges": [[23, 37]], "text": "The history of having been working in the garden points more to a sporotrichosis,"}, "5": {"exist": false, "char_ranges": [], "word_ranges": [], "text": ""}}} {"id": 305, "year": 2016, "question_id_specific": 174, "full_question": "An 87-year-old man with a history of hypertension and gonarthrosis. Baseline situation with full functional and cognitive autonomy that allows him to continue living alone in the community. He is receiving regular treatment with perindopril and thiazide diuretic for blood pressure control and routinely takes ibuprofen 1800 mg/day to control the symptoms derived from his gonarthrosis. After routine control, a persistent blood pressure of 190 and TAD 80 mmHg was observed. What would be the most reasonable therapeutic modification to achieve blood pressure control?", "full_answer": "One of the mechanisms that produce the hypotensive effect of ACE inhibitors such as perindopril is the increased action of vasodilator prostaglan-dins on the kidney, which decreases hydrosaline retention and therefore increases diuresis and lowers blood pressure. NSAIDs inhibit prostaglandin synthesis and decrease renal perfusion, thus antagonizing the hypotensive effect of many antihypertensives, including ACE inhibitors. Therefore, the preferable thing to do in this patient, before increasing the medication, is to change the ibuprofen for paracetamol, which does not have this effect, and to re-evaluate it later.", "type": "NEPHROLOGY", "options": {"1": "I would add a calcium channel blocker.", "2": "I would increase the dose of hydrochlorothiazide to 25 mg/day.", "3": "I would change ibuprofen to paracetamol to avoid the possible influence of paracetamol on the effect of hypotensives.", "4": "I would add an alpha-blocker because of the high prevalence of prostatic syndrome in males of this age.", "5": NaN}, "correct_option": 3, "explanations": {"1": {"exist": false, "char_ranges": [], "word_ranges": [], "text": ""}, "2": {"exist": false, "char_ranges": [], "word_ranges": [], "text": ""}, "3": {"exist": true, "char_ranges": [[264, 621]], "word_ranges": [[37, 87]], "text": "NSAIDs inhibit prostaglandin synthesis and decrease renal perfusion, thus antagonizing the hypotensive effect of many antihypertensives, including ACE inhibitors. Therefore, the preferable thing to do in this patient, before increasing the medication, is to change the ibuprofen for paracetamol, which does not have this effect, and to re-evaluate it later."}, "4": {"exist": false, "char_ranges": [], "word_ranges": [], "text": ""}, "5": {"exist": false, "char_ranges": [], "word_ranges": [], "text": ""}}} {"id": 104, "year": 2012, "question_id_specific": 91, "full_question": "A 34-year-old man referred to the clinic for persistent LDL cholesterol levels above 250 mg/dL. The patient is asymptomatic but on examination there are whitish indurated spots on elbows, knees and eyelids. His BP is 135/85 mmHg. Her father died of myocardial infarction at the age of 48 years. The fasting blood test shows a total cholesterol of 346 mg/dl, LDL cholesterol of 278 mg/dl, HDL cholesterol of 42 mg/dl, triglycerides of 130 mg/dl and glucose of 93 mg/dl. Regarding the disease from which this patient suffers, one of the following statements is FALSE. Point out which one:", "full_answer": "This is a heterozygous familial hypercholesterolemia. We must know how to recognize the disease and know the criteria. It seems to me to be of high difficulty.", "type": "ENDOCRINOLOGY", "options": {"1": "It is an autosomal codominant hereditary disease, with a high penetrance, so it is frequent that some of the parents and siblings of the individual also suffer from it.", "2": "The genetic disorder affects the gene encoding the LDL cholesterol receptor and more than 900 mutations have been described.", "3": "The genetic disorder also affects the synthesis of triglycerides, causing in the advanced stages of the disease, an increase in triglycerides, which is usually higher than 500 mg/dl.", "4": "Untreated males have a near 50% chance of a coronary event before age 60, and women with the disease have a higher prevalence of ischemic heart disease than the general female population.", "5": "Statins are effective in the treatment of the heterozygous form of the disease, but often need to be combined with cholesterol absorption inhibitors to achieve therapeutic goals and disease control."}, "correct_option": 3, "explanations": {"1": {"exist": false, "char_ranges": [], "word_ranges": [], "text": ""}, "2": {"exist": false, "char_ranges": [], "word_ranges": [], "text": ""}, "3": {"exist": true, "char_ranges": [[0, 118]], "word_ranges": [[0, 18]], "text": "This is a heterozygous familial hypercholesterolemia. We must know how to recognize the disease and know the criteria."}, "4": {"exist": false, "char_ranges": [], "word_ranges": [], "text": ""}, "5": {"exist": false, "char_ranges": [], "word_ranges": [], "text": ""}}} {"id": 38, "year": 2011, "question_id_specific": 232, "full_question": "A 52-year-old woman consults for having noticed during the previous week a yellowish discoloration of the conjunctivae. She does not refer to risky sexual behavior or epidemiological history of risk of viral hepatitis. She does not consume alcohol or hepatotoxic drugs. She reports a one-year history of generalized pruritus, asthenia, dry mouth and absence of lacrimation with no known cause. Rest of the anamnesis without pathological data. Physical examination showed scratching lesions conjunctival jaundice and non-painful hepatomegaly. The patient brings a blood test carried out in his company with the following pathological results: total bilirubin 3 mg/dl alkaline phosphatase 400 UI sedimentation rate 40 mm at the first hour. Indicate which would be the best recommendation to establish the etiological diagnosis of the patient's condition:", "full_answer": "A middle-aged woman with Sjögren's-like symptoms (absence of lacrimation, dry mouth), with hyperbilirubinemia, jaundice and hepatomegaly, makes us think of primary biliary cirrhosis. To diagnose it we will use antimitochondrial antibodies (AMA IgG). Therefore, it would be answer 1.", "type": "OPHTHALMOLOGY", "options": {"1": "Anti-mitochondrial antibodies.", "2": "Study of iron metabolism.", "3": "Study of copper metabolism.", "4": "Hepatic MRI.", "5": "Serology of virus B and virus C."}, "correct_option": 1, "explanations": {"1": {"exist": true, "char_ranges": [[0, 249]], "word_ranges": [[0, 33]], "text": "A middle-aged woman with Sjögren's-like symptoms (absence of lacrimation, dry mouth), with hyperbilirubinemia, jaundice and hepatomegaly, makes us think of primary biliary cirrhosis. To diagnose it we will use antimitochondrial antibodies (AMA IgG)."}, "2": {"exist": false, "char_ranges": [], "word_ranges": [], "text": ""}, "3": {"exist": false, "char_ranges": [], "word_ranges": [], "text": ""}, "4": {"exist": false, "char_ranges": [], "word_ranges": [], "text": ""}, "5": {"exist": false, "char_ranges": [], "word_ranges": [], "text": ""}}} {"id": 376, "year": 2016, "question_id_specific": 130, "full_question": "A 39-year-old man who in the last 3 months has had two episodes of wide and involuntary jumping movements of the left limbs, which in about two minutes lead to loss of consciousness. In the neuroradiological study a right frontal expansive lesion suggestive of cerebral glioma was found.How would you classify this patient's epilepsy?", "full_answer": "They refer to a partial seizure that is followed by loss of level of consciousness, so strictly speaking it would be a symptomatic complex partial seizure, not a secondarily generalized partial seizure (which occurs when tonic-clonic movements extend to the rest of the extremities). For this reason the most correct option would be two because it does not specify that it is a simple symptomatic partial seizure, but leaves open the possibility that it is a simple or complex symptomatic partial seizure.", "type": "NEUROLOGY", "options": {"1": "Generalized symptomatic.", "2": "Partially symptomatic.", "3": "Partial cryptogenic.", "4": "Partial secondarily generalized symptomatic.", "5": NaN}, "correct_option": 2, "explanations": {"1": {"exist": false, "char_ranges": [], "word_ranges": [], "text": ""}, "2": {"exist": true, "char_ranges": [[300, 505]], "word_ranges": [[47, 82]], "text": "the most correct option would be two because it does not specify that it is a simple symptomatic partial seizure, but leaves open the possibility that it is a simple or complex symptomatic partial seizure."}, "3": {"exist": false, "char_ranges": [], "word_ranges": [], "text": ""}, "4": {"exist": true, "char_ranges": [[0, 283]], "word_ranges": [[0, 44]], "text": "They refer to a partial seizure that is followed by loss of level of consciousness, so strictly speaking it would be a symptomatic complex partial seizure, not a secondarily generalized partial seizure (which occurs when tonic-clonic movements extend to the rest of the extremities)."}, "5": {"exist": false, "char_ranges": [], "word_ranges": [], "text": ""}}} {"id": 163, "year": 2013, "question_id_specific": 122, "full_question": "A 66-year-old man with a history of ankylosing spondylitis of long evolution. He presents proteinuria of 6 grams per day with hypoalbuminemia and edema. In the urinary sediment no hematuria is detected. His plasma creatinine is 1.6 mg/dL and his glomerular filtration rate is 45 mL/min. Glycemia 110 mg/dL. His severe spinal deformity makes it difficult to perform percutaneous renal biopsy. What is the correct initial approach?", "full_answer": "The correct answer is: 3. Subcutaneous fat biopsy. Lupus erythematosus, some vasculitides, Goodpasture's disease and other systemic diseases (ankylosing spondylitis is one of them) affect the kidney and condition its prognosis. The appearance of urinary abnormalities or renal impairment of parenchymal cause are indications for renal biopsy, even with proteinuria ranges lower than those accepted in primary nephropathies. The determination of anti-GBM antibodies and ANCAS are helpful for diagnosis, but they do not replace renal biopsy, since they lack prognostic value and do not help to plan treatment. In this case, in which we are led to think that renal biopsy is not possible, we must relate AE to renal failure. In most cases, the relationship is established by an IgA nephropathy, which presents with hematuria, a symptom that claims not to exist in this case. A non-negligible number of patients with evolved AD is amyloidosis, which also matches the patient's symptomatology. Therefore, and given that amyloidosis can be diagnosed with a subcutaneous fat biopsy, it is answer 3 that should be considered valid.", "type": "NEPHROLOGY", "options": {"1": "To initiate corticoids for suspicion of minimal change glomerular disease.", "2": "Treat with cyclophosphamide for suspected membranous glomerulopathy.", "3": "Subcutaneous fat biopsy.", "4": "Initiate dialysis.", "5": "Glucose overload test to rule out diabetic nephropathy."}, "correct_option": 3, "explanations": {"1": {"exist": false, "char_ranges": [], "word_ranges": [], "text": ""}, "2": {"exist": false, "char_ranges": [], "word_ranges": [], "text": ""}, "3": {"exist": true, "char_ranges": [[1000, 1123]], "word_ranges": [[153, 174]], "text": "and given that amyloidosis can be diagnosed with a subcutaneous fat biopsy, it is answer 3 that should be considered valid."}, "4": {"exist": false, "char_ranges": [], "word_ranges": [], "text": ""}, "5": {"exist": false, "char_ranges": [], "word_ranges": [], "text": ""}}} {"id": 148, "year": 2012, "question_id_specific": 158, "full_question": "A 23-year-old woman comes to the emergency department because she is very nervous after an argument with her partner. Her medical history shows several similar complaints in the previous year, two of them after a self-injurious gesture. Frequent conflicts in the couple's relationship, job changes and family arguments are also noted. She says she feels misunderstood by everyone, including the psychiatrists attending her. The diagnosis is:", "full_answer": "The woman is 23 years old and is pissed off. She needs support and a lot of emotional education to curb her impulsivity... she feels misunderstood... maybe she is right... The issue is that she cannot stand rejection and has self-injurious gestures. The diagnosis is 1. 3, 4 and 5 are not even close, there is no clinical picture described in the question. Then in real life exploring well almost certainly there would be dissociative and depersonalization symptoms, as well as depressive symptoms... but symptom is not disorder. Why not 2? It could also be very punctilious and spinning very fine... here the discussion is served. I put the criteria of each one and we talk about it,... but the intention of the examiner was the 1 and I see it well defended. It cannot be challenged. - The limit: A general pattern of instability in interpersonal relationships, self-image and self-efficacy, and notable impulsivity, beginning in early adulthood and occurring in a variety of contexts, as indicated by five (or more) of the following items: 1. Frantic efforts to avoid real or imagined abandonment. Note: Do not include suicidal or self-mutilating behaviors covered in Criterion 5. 2. A pattern of unstable and intense interpersonal relationships characterized by alternating between extremes of idealization and devaluation. 3. Identity disturbance: marked and persistently unstable self-image or sense of self. Impulsivity in at least two areas, which is potentially self-harming (e.g., spending, sex, substance abuse, reckless driving, binge eating). Note: Do not include suicidal or self-mutilating behaviors listed in Criterion 5. 5. Recurrent suicidal behaviors, attempts or threats, or self-mutilating behavior. Affective instability due to marked mood reactivity (e.g., episodes of intense dysphoria, irritability, or anxiety, usually lasting a few hours and rarely a few days). 7. Chronic feelings of emptiness. 8. Inappropriate and intense anger or difficulties in controlling anger (e.g., frequent displays of temper, constant anger, recurrent physical fights). 9. Transient stress-related paranoid ideation or severe dissociative symptoms. - The histrionic: A general pattern of excessive emotionality and attention-seeking, beginning in early adulthood and occurring in a variety of contexts, as indicated by five (or more) of the following items: 1. Not comfortable in situations where he/she is not the center of attention. Interaction with others is often characterized by sexually seductive or provocative behavior. 3. 3. Displays a superficial and rapidly changing emotional expression. 4. Constantly uses physical appearance to draw attention to him/herself. 5. Has a way of speaking that is excessively subjective and lacking in nuance. 6. Shows self-dramatization, theatricality and exaggerated emotional expression. 7. Is suggestible, e.g., easily influenced by others or by circumstances. 8. considers his relationships more intimate than they really are.", "type": "PSYCHIATRY", "options": {"1": "Borderline personality disorder.", "2": "Histrionic personality disorder.", "3": "Dysthymia.", "4": "Dissociative disorder.", "5": "Depersonalization disorder."}, "correct_option": 1, "explanations": {"1": {"exist": true, "char_ranges": [[0, 269]], "word_ranges": [[0, 46]], "text": "The woman is 23 years old and is pissed off. She needs support and a lot of emotional education to curb her impulsivity... she feels misunderstood... maybe she is right... The issue is that she cannot stand rejection and has self-injurious gestures. The diagnosis is 1."}, "2": {"exist": false, "char_ranges": [], "word_ranges": [], "text": ""}, "3": {"exist": true, "char_ranges": [[270, 356]], "word_ranges": [[46, 63]], "text": "3, 4 and 5 are not even close, there is no clinical picture described in the question."}, "4": {"exist": true, "char_ranges": [[270, 356]], "word_ranges": [[46, 63]], "text": "3, 4 and 5 are not even close, there is no clinical picture described in the question."}, "5": {"exist": true, "char_ranges": [[270, 356]], "word_ranges": [[46, 63]], "text": "3, 4 and 5 are not even close, there is no clinical picture described in the question."}}} {"id": 178, "year": 2013, "question_id_specific": 94, "full_question": "18-month-old boy, with complete vaccination schedule to date, who consults the Emergency Department for swelling of the right knee after playing in the park, without obvious trauma. In the directed anamnesis, the mother refers that an uncle of hers had similar problems. The ultrasound examination is compatible with hemarthrosis and in the analysis performed only an APTT lengthening of 52″ (normal 25-35″) stands out. What is the most likely diagnostic hypothesis?", "full_answer": "In this question they also do not stop dropping clues to throw us headlong into the correct diagnosis. Let's review: a small child who suffers a hemarthrosis without clear trauma. An uncle of his had similar problems, so we suspect hereditary disease. The APTT, which evaluates the intrinsic coagulation pathway (involving factors V, VIII, IX, X, XI and XII) is elongated. Hereditary coagulation problem. We thus discard answers 1, 2 and 5. We are left with von Willebrand's disease and hemophilia A. I consider that the correct answer is 4, hemophilia A, since the inheritance pattern is X-linked and its main clinical manifestation is soft tissue bruising and spontaneous hemarthrosis, without apparent trauma, while von Willebrand disease is characterized by bleeding after surgery or trauma and a prolonged bleeding time with a normal platelet count is characteristic (data not mentioned in the statement).", "type": "HEMATOLOGY", "options": {"1": "Marfan syndrome.", "2": "Von Willebrand's disease.", "3": "Ehlers-Danlos disease.", "4": "Hemophilia A.", "5": "Bemard-Soulier disease."}, "correct_option": 4, "explanations": {"1": {"exist": true, "char_ranges": [[252, 440]], "word_ranges": [[42, 72]], "text": "The APTT, which evaluates the intrinsic coagulation pathway (involving factors V, VIII, IX, X, XI and XII) is elongated. Hereditary coagulation problem. We thus discard answers 1, 2 and 5."}, "2": {"exist": true, "char_ranges": [[252, 440]], "word_ranges": [[42, 72]], "text": "The APTT, which evaluates the intrinsic coagulation pathway (involving factors V, VIII, IX, X, XI and XII) is elongated. Hereditary coagulation problem. We thus discard answers 1, 2 and 5."}, "3": {"exist": true, "char_ranges": [[713, 871]], "word_ranges": [[113, 137]], "text": "while von Willebrand disease is characterized by bleeding after surgery or trauma and a prolonged bleeding time with a normal platelet count is characteristic"}, "4": {"exist": true, "char_ranges": [[501, 712]], "word_ranges": [[82, 113]], "text": "I consider that the correct answer is 4, hemophilia A, since the inheritance pattern is X-linked and its main clinical manifestation is soft tissue bruising and spontaneous hemarthrosis, without apparent trauma,"}, "5": {"exist": true, "char_ranges": [[252, 440]], "word_ranges": [[42, 72]], "text": "The APTT, which evaluates the intrinsic coagulation pathway (involving factors V, VIII, IX, X, XI and XII) is elongated. Hereditary coagulation problem. We thus discard answers 1, 2 and 5."}}} {"id": 100, "year": 2012, "question_id_specific": 85, "full_question": "A 24-year-old man reports progressive asthenia for at least 6 months, hoarse voice, slow speech, drowsiness and swelling of the hands, feet and face. Examination: pulse at 52 beats per minute, abotarada face and dry and pale skin. CBC: mild anemia, cholesterol 385 mg/dL (normal <220), creatinine 1.3 mg/dL (normal: 0.5-1.1), negative proteinuria, TSH 187μIU/mL (normal 0.35-5.5) and Free T4 0.2 ng/dL (normal 0.85-1.86). What strategy do you think is most appropriate?", "full_answer": "This is severe primary hypothyroidism. Treatment is with thyroxine. If there is no nodule, neither echo nor FNA is indicated.", "type": "ENDOCRINOLOGY", "options": {"1": "Perform a thyroid ultrasound before starting treatment.", "2": "Determine free T3 and perform a pituitary MRI.", "3": "Treat with L-triiodothyronine and a statin.", "4": "Treat with L-Tyroxine and determine antithyroid antibodies.", "5": "Perform thyroid cytology prior to treatment."}, "correct_option": 4, "explanations": {"1": {"exist": true, "char_ranges": [[68, 125]], "word_ranges": [[9, 20]], "text": "If there is no nodule, neither echo nor FNA is indicated."}, "2": {"exist": false, "char_ranges": [], "word_ranges": [], "text": ""}, "3": {"exist": false, "char_ranges": [], "word_ranges": [], "text": ""}, "4": {"exist": true, "char_ranges": [[0, 67]], "word_ranges": [[0, 9]], "text": "This is severe primary hypothyroidism. Treatment is with thyroxine."}, "5": {"exist": true, "char_ranges": [[68, 125]], "word_ranges": [[9, 20]], "text": "If there is no nodule, neither echo nor FNA is indicated."}}} {"id": 398, "year": 2016, "question_id_specific": 140, "full_question": "You evaluate a 66-year-old patient with groin pain accentuated by prolonged standing a few days a month. A plain radiograph of the hips shows narrowing of the femoroacetabular joint space, sclerosis and osteophytes. What is your approach?", "full_answer": "We are talking about a 66-year-old patient with pain on some days, pain of an occasional nature. Although the diagnosis is clearly osteoarthritis, it does not lead directly to a surgical indication. The first step is to try conservative treatment (we ruled out the first option). Osteoarthritis is a degenerative process that opioids certainly do not stop, so we rule out option two. Option four can also be quickly discarded: the statement makes the diagnosis and treatment to follow clear: in this case the MRI will not provide us with any information of interest.", "type": "TRAUMATOLOGY AND ORTHOPEDICS", "options": {"1": "I make the diagnosis of coxarthrosis and send to the traumatologist to place a hip prosthesis.", "2": "Start treatment with weak opioids that have shown evidence in stopping the progression of the disease.", "3": "I insist treatment with paracetamol, explain that the evolution is very variable and the surgical indication depends on the functionality and pain control.", "4": "Because of the radiological features described, I need a hip MRI before making a therapeutic decision.", "5": NaN}, "correct_option": 3, "explanations": {"1": {"exist": true, "char_ranges": [[199, 279]], "word_ranges": [[32, 46]], "text": "The first step is to try conservative treatment (we ruled out the first option)."}, "2": {"exist": true, "char_ranges": [[280, 383]], "word_ranges": [[46, 63]], "text": "Osteoarthritis is a degenerative process that opioids certainly do not stop, so we rule out option two."}, "3": {"exist": true, "char_ranges": [[199, 279]], "word_ranges": [[32, 46]], "text": "The first step is to try conservative treatment (we ruled out the first option)."}, "4": {"exist": true, "char_ranges": [[384, 566]], "word_ranges": [[63, 94]], "text": "Option four can also be quickly discarded: the statement makes the diagnosis and treatment to follow clear: in this case the MRI will not provide us with any information of interest."}, "5": {"exist": false, "char_ranges": [], "word_ranges": [], "text": ""}}} {"id": 28, "year": 2011, "question_id_specific": 58, "full_question": "A 30-year-old donkey man undergoes a chest X-ray to formalize an employment contract in a company. The X-ray shows a bilateral interstitial pattern of reticular type, bilateral hilar and mediastinal adenopathies. Bronchofibroscopy with bronchoalveolar lavage is indicated and the cell count shows the following results: lymphocytes 50%, histiocytes 40%, eosinophils 2%, polymorphonuclear 8% and CD4/Cd8 ratio 5 What is the most likely diagnosis?", "full_answer": "You are describing sarcoidosis.", "type": "NEUROLOGY AND THORACIC SURGERY", "options": {"1": "Extrinsic allergic alveolitis.", "2": "Pneumoconiosis.", "3": "Sarcoidosis.", "4": "Carcinomatous lymphangitis.", "5": "Idiopathic pulmonary hemosiderosis."}, "correct_option": 3, "explanations": {"1": {"exist": false, "char_ranges": [], "word_ranges": [], "text": ""}, "2": {"exist": false, "char_ranges": [], "word_ranges": [], "text": ""}, "3": {"exist": true, "char_ranges": [[0, 31]], "word_ranges": [[0, 4]], "text": "You are describing sarcoidosis."}, "4": {"exist": false, "char_ranges": [], "word_ranges": [], "text": ""}, "5": {"exist": false, "char_ranges": [], "word_ranges": [], "text": ""}}} {"id": 587, "year": 2022, "question_id_specific": 76, "full_question": "A 40-year-old woman consults because she has noticed a lump in the superoexternal quadrant of the right breast for the past month. She provides a report of a mammography that describes a BIRADS 3 lesion:", "full_answer": "The rest of the answers are incorrect. With a BIRADS 3 this is the correct attitude to follow.", "type": "OBSTETRICS AND GYNECOLOGY", "options": {"1": "Reassure her, since an imaging test has already been done and malignancy has been ruled out.", "2": "This classification probably implies surgery, given that the probability of cancer is greater than 10%. He explains it to her and refers her preferentially to the Breast Unit.", "3": "This is a probably benign finding, given that there is less than a 2% chance of cancer. He explains that it requires follow-up every 6-12 months until 24 months or a biopsy.", "4": "The findings are of low suspicion of cancer (between 2 and 10%) but a biopsy is necessary.", "5": NaN}, "correct_option": 3, "explanations": {"1": {"exist": false, "char_ranges": [], "word_ranges": [], "text": ""}, "2": {"exist": false, "char_ranges": [], "word_ranges": [], "text": ""}, "3": {"exist": true, "char_ranges": [[39, 94]], "word_ranges": [[7, 18]], "text": "With a BIRADS 3 this is the correct attitude to follow."}, "4": {"exist": false, "char_ranges": [], "word_ranges": [], "text": ""}, "5": {"exist": false, "char_ranges": [], "word_ranges": [], "text": ""}}} {"id": 554, "year": 2022, "question_id_specific": 50, "full_question": "A 58-year-old man comes for a scheduled check-up for type 2 diabetes mellitus diagnosed 6 years ago. He has a personal history of hypertension, dyslipidemia, ischemic heart disease and pancreatitis of biliary origin. His medication includes lisinopril, metoprolol, metformin, ASA and atorvastatin. Physical examination showed blood pressure 151/93 mmHg, BMI 27.1 kg/m2. Glycated hemoglobin is 8.3%. Which of the following is the most appropriate treatment for this patient?", "full_answer": "Patient with obesity, hypertension and increased HbA1C. In recent studies, Empaglifozin, a highly selective inhibitor of SGLT210, significantly reduces AT, HbA1C and BMI. Glipizide is of the sulfonylurea group. Its use should be avoided in patients with heart disease. Acarbose can interact with beta-blockers, such as those taken by our patient. Sitagliptin has been shown to decrease HbA1C but not HT or BMI.", "type": "ENDOCRINOLOGY", "options": {"1": "Empaglifozin.", "2": "Glipizide.", "3": "Acarbose.", "4": "Sitagliptin.", "5": NaN}, "correct_option": 1, "explanations": {"1": {"exist": true, "char_ranges": [[0, 170]], "word_ranges": [[0, 23]], "text": "Patient with obesity, hypertension and increased HbA1C. In recent studies, Empaglifozin, a highly selective inhibitor of SGLT210, significantly reduces AT, HbA1C and BMI."}, "2": {"exist": true, "char_ranges": [[171, 268]], "word_ranges": [[23, 39]], "text": "Glipizide is of the sulfonylurea group. Its use should be avoided in patients with heart disease."}, "3": {"exist": true, "char_ranges": [[269, 346]], "word_ranges": [[39, 51]], "text": "Acarbose can interact with beta-blockers, such as those taken by our patient."}, "4": {"exist": true, "char_ranges": [[347, 410]], "word_ranges": [[51, 63]], "text": "Sitagliptin has been shown to decrease HbA1C but not HT or BMI."}, "5": {"exist": false, "char_ranges": [], "word_ranges": [], "text": ""}}} {"id": 251, "year": 2014, "question_id_specific": 81, "full_question": "A 50-year-old woman comes to the emergency department with asthenia, anorexia, weight loss, jaundice, fever and abdominal pain. She has no recent history of blood transfusion, risky sexual contacts or addiction to parenteral drugs. Habitual consumption of 60 grams of alcohol per day during the last five years, consumption that has increased during the previous month due to family problems. Temperature 38.5ºC, blood pressure 100/60 mmHg. Physical examination showed temporal and spatial disorientation, malnutrition, asterixis, jaundice and painful hepatomegaly. No ascites or evidence of peritoneal irritation. Laboratory tests include leukocytes 15,000/microliter with 90% polymorphonuclear, red blood cells 3 million/mm3, hemoglobin 10g/dl, mean corpuscular volume 115 fl, bilirubin 15 mg/dl direct predominance, AST (GOT) 300 IU/L, ALT (GPT) 120 IU/L, GGT 635 IU/L, prothrombin time prolongation greater than 50%. What is the most likely diagnosis of the patient's condition?", "full_answer": "Patient with harmful alcohol consumption (60g and female), high fever, hepatic encephalopathy (disorientation, asterixis), very high direct bilirubin, jaundice, painful hepatomegaly, coagulopathy (elevated prothrombin time) and mild elevation of transaminases (less than 5 times the upper limit of normal) suggest acute alcohol hepatitis.", "type": "DIGESTIVE SYSTEM", "options": {"1": "Hepatic abscess.", "2": "Acute cholecystitis.", "3": "Acute cholangitis.", "4": "Alcoholic hepatitis.", "5": "Acute pancreatitis."}, "correct_option": 4, "explanations": {"1": {"exist": false, "char_ranges": [], "word_ranges": [], "text": ""}, "2": {"exist": false, "char_ranges": [], "word_ranges": [], "text": ""}, "3": {"exist": false, "char_ranges": [], "word_ranges": [], "text": ""}, "4": {"exist": true, "char_ranges": [[0, 338]], "word_ranges": [[0, 43]], "text": "Patient with harmful alcohol consumption (60g and female), high fever, hepatic encephalopathy (disorientation, asterixis), very high direct bilirubin, jaundice, painful hepatomegaly, coagulopathy (elevated prothrombin time) and mild elevation of transaminases (less than 5 times the upper limit of normal) suggest acute alcohol hepatitis."}, "5": {"exist": false, "char_ranges": [], "word_ranges": [], "text": ""}}} {"id": 128, "year": 2012, "question_id_specific": 79, "full_question": "A 42-year-old married woman with 2 school-age children consults her family physician for a severe headache of about 7 days' evolution. Although she has been having similar episodes for more than a year, in the last 2 months they have worsened considerably. The pain is like a weight that starts in the occipital area, spreads to both temporal regions and is barely relieved by taking 650 mg paracetamol tablets, so she requests a CT scan (she is convinced that \"something must be wrong with her head\"). Which of the following is the most plausible cause of her headache?", "full_answer": "Just as last year they wanted to paint a nurse as bad for waking us up for a patient with myasthenia, this year they entertain us by presenting a patient asking for a CT scan. I interpret that they show a patient stressed by her children with recurrent headaches of occipital location that she previously relieved with a low dose of paracetamol and now she does not. Both the initially occipital location, the 2-year evolution time, and the sensation of weight lead to answer 4, a tension headache. Migraine and vascular disorders, accompanied by other neurological symptoms or nausea, photophobia, etc., are quickly ruled out, as well as the rare temporal arteritis before the age of 50 years. Tumor headache is usually \"in crescendo\", not recurrent as the case suggests, and is accompanied by other neurological symptoms or intracranial hypertension.", "type": "NEUROLOGY AND NEUROSURGERY", "options": {"1": "Migraine.", "2": "Vascular disorders.", "3": "Temporal artery arteritis.", "4": "Tension.", "5": "Oncologic."}, "correct_option": 4, "explanations": {"1": {"exist": true, "char_ranges": [[499, 694]], "word_ranges": [[88, 118]], "text": "Migraine and vascular disorders, accompanied by other neurological symptoms or nausea, photophobia, etc., are quickly ruled out, as well as the rare temporal arteritis before the age of 50 years."}, "2": {"exist": true, "char_ranges": [[499, 694]], "word_ranges": [[88, 118]], "text": "Migraine and vascular disorders, accompanied by other neurological symptoms or nausea, photophobia, etc., are quickly ruled out, as well as the rare temporal arteritis before the age of 50 years."}, "3": {"exist": true, "char_ranges": [[499, 694]], "word_ranges": [[88, 118]], "text": "Migraine and vascular disorders, accompanied by other neurological symptoms or nausea, photophobia, etc., are quickly ruled out, as well as the rare temporal arteritis before the age of 50 years."}, "4": {"exist": true, "char_ranges": [[367, 498]], "word_ranges": [[67, 88]], "text": "Both the initially occipital location, the 2-year evolution time, and the sensation of weight lead to answer 4, a tension headache."}, "5": {"exist": true, "char_ranges": [[695, 852]], "word_ranges": [[118, 140]], "text": "Tumor headache is usually \"in crescendo\", not recurrent as the case suggests, and is accompanied by other neurological symptoms or intracranial hypertension."}}} {"id": 0, "year": 2011, "question_id_specific": 34, "full_question": "A 27-year-old woman under study in the dermatology department for recurrent episodes of oral aphthous ulcers. She was referred to the gastrointestinal department for a blood test showing Hb 11.5 gr/dl Fe 38 AST 52 ALT 64 Ac antitrasglutaminase IgA 177. The patient reported only occasional diffuse abdominal discomfort. She reports no signs of hemorrhage or jaundice. The physical examination is normal. Which of the following tests do you consider to be the most appropriate to make the diagnosis?", "full_answer": "In this question, they give an important piece of information when it comes to suspecting a diagnosis, which is the antitransglutaminase antibodies that must lead you to suspect celiac disease. The rest of the symptoms fall perfectly within the picture and although probably several of the tests that it says will be done, the answer that will give us the diagnosis is 4 (intestinal biopsy).", "type": "DIGESTIVE", "options": {"1": "Capsule endoscopy.", "2": "Colonoscopy.", "3": "Serology for hepatitis b and c virus.", "4": "Intestinal biopsy.", "5": "Abdominal ultrasound."}, "correct_option": 4, "explanations": {"1": {"exist": false, "char_ranges": [], "word_ranges": [], "text": ""}, "2": {"exist": false, "char_ranges": [], "word_ranges": [], "text": ""}, "3": {"exist": false, "char_ranges": [], "word_ranges": [], "text": ""}, "4": {"exist": true, "char_ranges": [[0, 391]], "word_ranges": [[0, 65]], "text": "In this question, they give an important piece of information when it comes to suspecting a diagnosis, which is the antitransglutaminase antibodies that must lead you to suspect celiac disease. The rest of the symptoms fall perfectly within the picture and although probably several of the tests that it says will be done, the answer that will give us the diagnosis is 4 (intestinal biopsy)."}, "5": {"exist": false, "char_ranges": [], "word_ranges": [], "text": ""}}} {"id": 256, "year": 2014, "question_id_specific": 162, "full_question": "An 80-year-old woman consults for presenting, since 1 month ago, a progressive increase in abdominal perimeter. During the previous 4-5 months she noted asthenia, anorexia and unspecified weight loss. Physical examination revealed tension ascites and malleolar edema, with no other relevant signs. An ultrasound and CT scan of the abdomen showed the presence of ascites, of homogeneous density, without peritoneal implants or abdominal or pelvic masses. Liver, pancreas, adrenal, spleen and kidneys without significant findings. Paracentesis is performed through a thick needle, obtaining with difficulty a somewhat yellowish, dense and gelatinous fluid. What is the most probable etiology?", "full_answer": "The characteristics of the fluid described are milky, i.e., chylous ascitic fluid. Taking into account the age of the patient and the constitutional syndrome described, it is most likely to be a non-Hodgkin's lymphoma and not tuberculosis, which also presents with chylous ascitic fluid, since there is no mention of infectious symptoms. The rest of the entities offered do not present with chylous ascitic fluid.", "type": "DIGESTIVE SYSTEM", "options": {"1": "Hydropic decompensation secondary to liver cirrhosis.", "2": "Chylous ascites due to non-Hodgkin's lymphoma.", "3": "Peritoneal tuberculosis.", "4": "Peritoneal metastasis of adenocarcinoma.", "5": "Ascites secondary to constrictive pericarditis."}, "correct_option": 2, "explanations": {"1": {"exist": true, "char_ranges": [[338, 413]], "word_ranges": [[52, 65]], "text": "The rest of the entities offered do not present with chylous ascitic fluid."}, "2": {"exist": true, "char_ranges": [[83, 337]], "word_ranges": [[12, 52]], "text": "Taking into account the age of the patient and the constitutional syndrome described, it is most likely to be a non-Hodgkin's lymphoma and not tuberculosis, which also presents with chylous ascitic fluid, since there is no mention of infectious symptoms."}, "3": {"exist": true, "char_ranges": [[83, 337]], "word_ranges": [[12, 52]], "text": "Taking into account the age of the patient and the constitutional syndrome described, it is most likely to be a non-Hodgkin's lymphoma and not tuberculosis, which also presents with chylous ascitic fluid, since there is no mention of infectious symptoms."}, "4": {"exist": true, "char_ranges": [[338, 413]], "word_ranges": [[52, 65]], "text": "The rest of the entities offered do not present with chylous ascitic fluid."}, "5": {"exist": true, "char_ranges": [[338, 413]], "word_ranges": [[52, 65]], "text": "The rest of the entities offered do not present with chylous ascitic fluid."}}} {"id": 604, "year": 2022, "question_id_specific": 113, "full_question": "A 35-year-old man, a letter carrier, with no history of interest, who comes to the emergency department for acute cervical pain of 24 hours of evolution, without previous trauma, which radiates to the left arm up to the hand and is accompanied by paresthesia in the radial border of the forearm. She presents no objectifiable loss of strength, preserves neck mobility although it is painful and contracture of the paravertebral musculature is appreciated. The first attitude will be:", "full_answer": "Cervicalgia without alarm signs. Conservative treatment with a success rate of 75%-90%.", "type": "TRAUMATOLOGY", "options": {"1": "Conservative treatment with nonsteroidal anti-inflammatory drugs, local heat and relative rest.", "2": "Urgent call to the neurosurgeon for surgical evaluation.", "3": "Preferential request for MRI and electromyogram.", "4": "Preferential referral to outpatient traumatology consultations.", "5": NaN}, "correct_option": 1, "explanations": {"1": {"exist": true, "char_ranges": [[0, 87]], "word_ranges": [[0, 12]], "text": "Cervicalgia without alarm signs. Conservative treatment with a success rate of 75%-90%."}, "2": {"exist": false, "char_ranges": [], "word_ranges": [], "text": ""}, "3": {"exist": false, "char_ranges": [], "word_ranges": [], "text": ""}, "4": {"exist": false, "char_ranges": [], "word_ranges": [], "text": ""}, "5": {"exist": false, "char_ranges": [], "word_ranges": [], "text": ""}}} {"id": 362, "year": 2016, "question_id_specific": 96, "full_question": "A 25-year-old woman presents to the emergency department with ecchymosis and petechiae in the lower extremities and gingivorrhage in the last few days. Her blood count showed the following data: Hb 13 g/dL, leukocytes 8,500/microL with normal leukocyte formula and platelets 9,000/mm3. The determination of coagulation times is normal. Which of the following diagnostic tests should NOT be performed?", "full_answer": "The first thing is to know the diagnosis of the clinical case that we are presented with, it is a primary immune thrombocytopenia (ITP), only the platelet series is altered with normal coagulation. The first test to be performed is a peripheral blood smear to check if these platelets are real and there is no pseudothrombopenia due to platelet aggregates. In the enlargement study, antinuclear antibodies (ANA) are requested to rule out autoimmune pathologies that may be the cause of the thrombopenia and another test that would be indicated although in this case the patient is young and does not present signs of seriousness (leukopenia, anemia, no response to first line treatment...) is the performance of a bone marrow aspirate, to see if there are thromboforming megakaryocytes and thus confirm that the platelet destruction is of peripheral origin. But what is not indicated is the determination of the JAK-2 mutation, which is used in the diagnosis of chronic myeloproliferative syndromes, which is not the case here.", "type": "HEMATOLOGY", "options": {"1": "JAK-2 mutation.", "2": "Bone marrow aspirate.", "3": "Antinuclear antibodies.", "4": "Peripheral blood smear.", "5": NaN}, "correct_option": 1, "explanations": {"1": {"exist": true, "char_ranges": [[862, 1027]], "word_ranges": [[139, 166]], "text": "what is not indicated is the determination of the JAK-2 mutation, which is used in the diagnosis of chronic myeloproliferative syndromes, which is not the case here."}, "2": {"exist": true, "char_ranges": [[507, 857]], "word_ranges": [[83, 138]], "text": "another test that would be indicated although in this case the patient is young and does not present signs of seriousness (leukopenia, anemia, no response to first line treatment...) is the performance of a bone marrow aspirate, to see if there are thromboforming megakaryocytes and thus confirm that the platelet destruction is of peripheral origin."}, "3": {"exist": true, "char_ranges": [[357, 502]], "word_ranges": [[60, 82]], "text": "In the enlargement study, antinuclear antibodies (ANA) are requested to rule out autoimmune pathologies that may be the cause of the thrombopenia"}, "4": {"exist": true, "char_ranges": [[198, 356]], "word_ranges": [[33, 60]], "text": "The first test to be performed is a peripheral blood smear to check if these platelets are real and there is no pseudothrombopenia due to platelet aggregates."}, "5": {"exist": false, "char_ranges": [], "word_ranges": [], "text": ""}}} {"id": 410, "year": 2018, "question_id_specific": 195, "full_question": "A 75 a man is admitted to the hospital because he refuses to eat and has lost 30% of his weight. He has a history of multiple sclerosis which has worsened in recent months. His only family member who was his sister died 3 months ago and since then he has deteriorated. He has become incontinent, has stopped eating and participating in social events. He is suffering from psychotic depression. He is physically able to eat. Psychotropic medication is started and discontinued due to adverse events. If his nutritional status improves it is possible to resume his medications. He refuses intravenous fluids and then accepts them but the IV is started a few hours later. An alternative is to perform a percutaneous endoscopic gastrostomy (PEG) and discharge him to his residence. Which of the following is the most appropriate course of action for this patient?", "full_answer": "The correct answer is 1 since the patient is in a situation of psychotic depression and is not able to decide for his life and it does not seem that the patient has filled in the last will and testament, the most convenient thing to do is to present the case to the ethics committee of the center to make a decision on the matter in accordance with the law.", "type": "PRIMARY CARE", "options": {"1": "Ask the hospital ethics committee to help determine the decision that is in the best interest of the patient.", "2": "Assess whether the residence will take the patient if PEG is not performed.", "3": "Seek a court order to perform PEG.", "4": "Enrolling the patient in terminal palliative care.", "5": NaN}, "correct_option": 1, "explanations": {"1": {"exist": true, "char_ranges": [[30, 357]], "word_ranges": [[6, 70]], "text": "the patient is in a situation of psychotic depression and is not able to decide for his life and it does not seem that the patient has filled in the last will and testament, the most convenient thing to do is to present the case to the ethics committee of the center to make a decision on the matter in accordance with the law."}, "2": {"exist": false, "char_ranges": [], "word_ranges": [], "text": ""}, "3": {"exist": false, "char_ranges": [], "word_ranges": [], "text": ""}, "4": {"exist": false, "char_ranges": [], "word_ranges": [], "text": ""}, "5": {"exist": false, "char_ranges": [], "word_ranges": [], "text": ""}}} {"id": 361, "year": 2016, "question_id_specific": 94, "full_question": "A 51-year-old man is admitted from the emergency department for macrocytic anemia (Hb 6.3 g/dL, MCV 120 fL). Studies ruled out a deficiency origin. Reticulocytes were 24000/microL. The bone marrow study is compatible with myelodysplastic syndrome (MDS). Cytogenetics shows a 5q deletion. Which is the correct statement regarding this patient?", "full_answer": "This is a young patient with no comorbidities and is therefore a candidate for treatment with curative intent, so option 4 is ruled out. Option 3 is also incorrect since we cannot know the patient's IPSS since we do not know the number of blasts and the number of leukocytes and platelets to calculate the risk. Option 2 is also incorrect since the indication for a bone marrow transplant in MDS is those patients with high or intermediate-2 IPSS, and in this case we do not know the risk, the only thing we know about the risk is the cytogenetics which is of good prognosis that would give us 0 points. Therefore the correct one is 1 since it is true that it has a favorable cytogenetics (5q-) and has a specific treatment lenalidomide, also used in other pathologies such as multiple myeloma.", "type": "HEMATOLOGY", "options": {"1": "This deletion (5q-) is an alteration of good prognosis and has a specific treatment (lenalidomide).", "2": "It would be advisable in this patient to perform HLA typing in order to organize an allogeneic transplant.", "3": "This is a patient with a high International Prognostic Index (IPSS).", "4": "Treatment in this case would be transfusion only.", "5": NaN}, "correct_option": 1, "explanations": {"1": {"exist": true, "char_ranges": [[604, 794]], "word_ranges": [[111, 143]], "text": "Therefore the correct one is 1 since it is true that it has a favorable cytogenetics (5q-) and has a specific treatment lenalidomide, also used in other pathologies such as multiple myeloma."}, "2": {"exist": true, "char_ranges": [[312, 603]], "word_ranges": [[56, 111]], "text": "Option 2 is also incorrect since the indication for a bone marrow transplant in MDS is those patients with high or intermediate-2 IPSS, and in this case we do not know the risk, the only thing we know about the risk is the cytogenetics which is of good prognosis that would give us 0 points."}, "3": {"exist": true, "char_ranges": [[137, 311]], "word_ranges": [[24, 56]], "text": "Option 3 is also incorrect since we cannot know the patient's IPSS since we do not know the number of blasts and the number of leukocytes and platelets to calculate the risk."}, "4": {"exist": true, "char_ranges": [[0, 136]], "word_ranges": [[0, 24]], "text": "This is a young patient with no comorbidities and is therefore a candidate for treatment with curative intent, so option 4 is ruled out."}, "5": {"exist": false, "char_ranges": [], "word_ranges": [], "text": ""}}} {"id": 172, "year": 2013, "question_id_specific": 50, "full_question": "A patient with severe COPD comes to the ED with an acute onset and presents an arterial blood gas drawn with Fi02 of 31% at sea level with a Pa02 of 86 mm Hg, PaC02 65 mm Hg, pH 7.13 and Bicarbonate 27 mmol/liter. Which of the following statements is FALSE?", "full_answer": "Blood gas analysis shows an acute respiratory acidosis, due to an increase in PCO2 of short evolution time because the kidney has not yet had time to retain bicarbonates (bicarbonate level at the high limit of normality). The patient is not hyperventilating but hypoventilating because PCO2 is elevated. The alveolar arterial oxygen gradient is altered since the patient has a normal PaO2, but because he has a high FiO2. If he were without oxygen, he would be hypoxemic. Respiratory acidosis below 7.20 is a reason to consider noninvasive mechanical ventilation acutely and in principle temporarily in a patient with severe COPD.", "type": "PNEUMOLOGY", "options": {"1": "The patient is hyperventilating.", "2": "The alveolar arterial oxygen gradient is elevated.", "3": "The patient is in respiratory acidosis.", "4": "Bicarbonate level is normal.", "5": "Initiation of mechanical ventilation should be considered."}, "correct_option": 1, "explanations": {"1": {"exist": true, "char_ranges": [[222, 303]], "word_ranges": [[37, 48]], "text": "The patient is not hyperventilating but hypoventilating because PCO2 is elevated."}, "2": {"exist": true, "char_ranges": [[304, 421]], "word_ranges": [[48, 69]], "text": "The alveolar arterial oxygen gradient is altered since the patient has a normal PaO2, but because he has a high FiO2."}, "3": {"exist": true, "char_ranges": [[0, 221]], "word_ranges": [[0, 37]], "text": "Blood gas analysis shows an acute respiratory acidosis, due to an increase in PCO2 of short evolution time because the kidney has not yet had time to retain bicarbonates (bicarbonate level at the high limit of normality)."}, "4": {"exist": true, "char_ranges": [[115, 221]], "word_ranges": [[19, 37]], "text": "the kidney has not yet had time to retain bicarbonates (bicarbonate level at the high limit of normality)."}, "5": {"exist": true, "char_ranges": [[472, 630]], "word_ranges": [[78, 101]], "text": "Respiratory acidosis below 7.20 is a reason to consider noninvasive mechanical ventilation acutely and in principle temporarily in a patient with severe COPD."}}} {"id": 83, "year": 2012, "question_id_specific": 48, "full_question": "What therapeutic intervention would you consider in a 67-year-old patient with idiopathic dilated cardiomyopathy, left bundle branch block, left ventricular ejection fraction of 26%, mitral insufficiency grade II-III/IV, undergoing treatment with furosemide, spironolactone, enalapril and bisoprolol and who remains in NYHA functional class III?", "full_answer": "We are talking about a patient with heart failure with severe LV dysfunction (EF < 35%), LBBB (wide QRS), who is following optimal medical treatment and who remains in advanced functional class despite this. Coronary revascularization does not appear to be of much benefit in this patient at this time. Nor does mitral valve replacement, firstly because mitral regurgitation is most likely the result of mitral annular dilatation, and secondly because we would need an echocardiographic assessment of the mitral valve to determine whether there is any alteration. Intra-aortic balloon counterpulsation implantation would not be indicated in this patient (it is used in situations of cardiogenic shock, in post-AMI mechanical complications, as a bridge to cardiac Tx...). The ventricular assist device would also not be indicated in this patient. What would really benefit him of all the answers is number 5, the resynchronization device. By achieving simultaneous stimulation of both ventricles, a mechanical synergy is achieved that improves cardiac function. The indications for the application of CRT (cardiac resynchronization therapy) are precisely those mentioned at the beginning: severe ventricular dysfunction, asynchrony demonstrated by the presence of wide QRS in the ECG (usually due to LBBB) and advanced functional class (III-IV) despite optimal medical treatment.", "type": "CARDIOLOGY AND VASCULAR SURGERY", "options": {"1": "Surgical coronary revascularization.", "2": "Mitral valve replacement.", "3": "Implantation of an aortic balloon pump.", "4": "Implantation of a ventricular assist device.", "5": "Implantation of a cardiac resynchronization system."}, "correct_option": 5, "explanations": {"1": {"exist": true, "char_ranges": [[208, 302]], "word_ranges": [[34, 50]], "text": "Coronary revascularization does not appear to be of much benefit in this patient at this time."}, "2": {"exist": true, "char_ranges": [[303, 563]], "word_ranges": [[50, 88]], "text": "Nor does mitral valve replacement, firstly because mitral regurgitation is most likely the result of mitral annular dilatation, and secondly because we would need an echocardiographic assessment of the mitral valve to determine whether there is any alteration."}, "3": {"exist": true, "char_ranges": [[564, 770]], "word_ranges": [[88, 117]], "text": "Intra-aortic balloon counterpulsation implantation would not be indicated in this patient (it is used in situations of cardiogenic shock, in post-AMI mechanical complications, as a bridge to cardiac Tx...)."}, "4": {"exist": true, "char_ranges": [[771, 845]], "word_ranges": [[117, 129]], "text": "The ventricular assist device would also not be indicated in this patient."}, "5": {"exist": true, "char_ranges": [[938, 1060]], "word_ranges": [[144, 160]], "text": "By achieving simultaneous stimulation of both ventricles, a mechanical synergy is achieved that improves cardiac function."}}} {"id": 141, "year": 2012, "question_id_specific": 135, "full_question": "A 2-year-old boy, his personal history includes: 3 episodes of acute otitis media, 1 meningococcal meningitis and 2 pneumonias (one middle lobe and one left upper lobe). She has been admitted on 3 occasions for thrombopenic purpura (on three occasions antiplatelet antibodies were negative and bone marrow showed normal megakaryocytes). Several males of the maternal family had died in childhood due to infectious processes. The examination showed typical lesions of atopic dermatitis. The immunological study showed a slight decrease in T lymphocyte subpopulations; elevated IgA and IgE; decreased IgM and IgG at the lower limit of normal. What is the most likely diagnosis?", "full_answer": "The correct answer is 1. Wiskott-Aldrich syndrome associates immunodeficiency, thrombopenia and atopic dermatitis with the immunological study described in the statement.", "type": "PEDIATRICS", "options": {"1": "Wiskott-Aldrich syndrome.", "2": "Hyper IgE syndrome.", "3": "Transient hypogammaglobulinemia of infancy.", "4": "Severe X-associated combined immunodeficiency.", "5": "Common variable immunodeficiency."}, "correct_option": 1, "explanations": {"1": {"exist": true, "char_ranges": [[25, 170]], "word_ranges": [[5, 21]], "text": "Wiskott-Aldrich syndrome associates immunodeficiency, thrombopenia and atopic dermatitis with the immunological study described in the statement."}, "2": {"exist": false, "char_ranges": [], "word_ranges": [], "text": ""}, "3": {"exist": false, "char_ranges": [], "word_ranges": [], "text": ""}, "4": {"exist": false, "char_ranges": [], "word_ranges": [], "text": ""}, "5": {"exist": false, "char_ranges": [], "word_ranges": [], "text": ""}}} {"id": 317, "year": 2016, "question_id_specific": 139, "full_question": "A 70-year-old woman with a history of anorexia, weight loss, muscle and proximal joint discomfort and pain in the temporomandibular region comes to the emergency department for unilateral vision loss (hand movement), sudden and painless onset (afferent pupillary defect). What test would you order first for diagnostic purposes?", "full_answer": "It is possibly a temporal arteritis, the clinical picture and a CBC with increased reactants are enough to confirm the suspicion. Then an ocular evaluation should be done to rule out AIN, urgent treatment should be instituted (if there is AIN, 3 boluses of methylprednisolone 1 g, if there is not, prednisone mg/kg) and biopsy should be evaluated.", "type": "RHEUMATOLOGY", "options": {"1": "Lumbar puncture.", "2": "C Reactive Protein.", "3": "Magnetic resonance angiography.", "4": "Carotid ultrasound.", "5": NaN}, "correct_option": 2, "explanations": {"1": {"exist": false, "char_ranges": [], "word_ranges": [], "text": ""}, "2": {"exist": true, "char_ranges": [[0, 129]], "word_ranges": [[0, 21]], "text": "It is possibly a temporal arteritis, the clinical picture and a CBC with increased reactants are enough to confirm the suspicion."}, "3": {"exist": false, "char_ranges": [], "word_ranges": [], "text": ""}, "4": {"exist": false, "char_ranges": [], "word_ranges": [], "text": ""}, "5": {"exist": false, "char_ranges": [], "word_ranges": [], "text": ""}}} {"id": 550, "year": 2022, "question_id_specific": 129, "full_question": "65-year-old male presenting to the emergency department for appearance of cyanotic mottled lesions on the toes of both feet. Personal history: smoking, hypertension and dyslipidemia. Chest X-ray: mediastinal widening. Thoracic-abdominal-pelvic CT angiography: descending thoracic aortic aneurysm 7 cm in diameter, distal to the left subclavian artery and with mural thrombus. Of the following, indicate the correct option:", "full_answer": "Thoracic stent implantation is indicated.", "type": "CARDIOLOGY", "options": {"1": "Statin therapy is indicated to stabilize the thrombus.", "2": "Urgent diagnostic aortography is necessary.", "3": "At surgery, the aneurysmal segment is replaced by a tubular prosthesis with reimplantation of the supra-aortic trunks.", "4": "Thoracic stent implantation is indicated.", "5": NaN}, "correct_option": 4, "explanations": {"1": {"exist": false, "char_ranges": [], "word_ranges": [], "text": ""}, "2": {"exist": false, "char_ranges": [], "word_ranges": [], "text": ""}, "3": {"exist": false, "char_ranges": [], "word_ranges": [], "text": ""}, "4": {"exist": true, "char_ranges": [[0, 41]], "word_ranges": [[0, 5]], "text": "Thoracic stent implantation is indicated."}, "5": {"exist": false, "char_ranges": [], "word_ranges": [], "text": ""}}} {"id": 110, "year": 2012, "question_id_specific": 152, "full_question": "Patient 76 years old, parity 3-0-3-1, with menopause at age 52. She reports that for at least 4-5 years she has had vulvar pruritus of variable intensity which has been treated sometimes with self-medication and other times on the advice of her general practitioner with topical preparations (creams and washes). The pruritus has evolved intermittently, but for the last 3-4 months she has also noticed a small lump on the left labium majus of the vulva, which has been leaking serohaematically on rubbing for a few days; she therefore consulted a gynecologist. On questioning, she reports occasional dysuria and her general condition is good. What is the most likely diagnosis in this patient?", "full_answer": "The correct answer is 5. Vulvar carcinoma is characterized by chronic vulvar itching resistant to multiple treatments, appearance of lumpiness or ulceration, dysuria and urinary urgency, and late pain and bleeding.", "type": "GYNECOLOGY AND OBSTETRICS", "options": {"1": "Genital herpes.", "2": "Paget's disease of the vulva.", "3": "Urethral caruncle.", "4": "Chronic granuloma of the vulva.", "5": "Vulvar squamous cell carcinoma."}, "correct_option": 5, "explanations": {"1": {"exist": false, "char_ranges": [], "word_ranges": [], "text": ""}, "2": {"exist": false, "char_ranges": [], "word_ranges": [], "text": ""}, "3": {"exist": false, "char_ranges": [], "word_ranges": [], "text": ""}, "4": {"exist": false, "char_ranges": [], "word_ranges": [], "text": ""}, "5": {"exist": true, "char_ranges": [[25, 214]], "word_ranges": [[5, 31]], "text": "Vulvar carcinoma is characterized by chronic vulvar itching resistant to multiple treatments, appearance of lumpiness or ulceration, dysuria and urinary urgency, and late pain and bleeding."}}} {"id": 419, "year": 2018, "question_id_specific": 77, "full_question": "An 80-year-old woman comes to the emergency department with abdominal pain starting in the epigastrium and radiating later to the left iliac fossa. An abdominal CT scan is performed, showing inflammation of the sigmoid walls and a 2 cm mesenteric abscess. The treatment of choice is:", "full_answer": "In this question we are presented with a clinical picture whose manifestations are compatible with acute diverticulitis, already asked on numerous occasions previously. A case of acute diverticulitis complicated with small mesenteric abscess (< 2-3 cm) is presented. Abscesses should be drained by CT-guided percutaneous puncture, since the intra-abdominal approach can spread the contents into the abdominal cavity. However, in small abscesses with Hinchey grade I divertciulitis, in patients without great deterioration of the general condition as in the proposed case, they can be managed by conservative treatment with intravenous antiobiotic therapy and absolute diet, therefore the correct option is 1. The urgent surgical approach should be reserved for cases that present peritonitis (grade III, IV) by resection of the affected segment and primary anastomosis in stable patients or carry out the Hartamn intervention that includes resection with terminal colostomy and closure of the distal rectal stump performing anastomosis in a second surgical time.", "type": "GENERAL SURGERY", "options": {"1": "Admission to the ward with absolute diet and broad-spectrum antibiotic treatment.", "2": "Discharge colostomy.", "3": "Drainage by laparoscopic surgery.", "4": "Urgent surgery with sigmoidectomy and colorectal anastomosis.", "5": NaN}, "correct_option": 1, "explanations": {"1": {"exist": true, "char_ranges": [[426, 708]], "word_ranges": [[59, 101]], "text": "in small abscesses with Hinchey grade I divertciulitis, in patients without great deterioration of the general condition as in the proposed case, they can be managed by conservative treatment with intravenous antiobiotic therapy and absolute diet, therefore the correct option is 1."}, "2": {"exist": false, "char_ranges": [], "word_ranges": [], "text": ""}, "3": {"exist": false, "char_ranges": [], "word_ranges": [], "text": ""}, "4": {"exist": true, "char_ranges": [[709, 807]], "word_ranges": [[101, 116]], "text": "The urgent surgical approach should be reserved for cases that present peritonitis (grade III, IV)"}, "5": {"exist": false, "char_ranges": [], "word_ranges": [], "text": ""}}} {"id": 37, "year": 2011, "question_id_specific": 227, "full_question": "A 78-year-old woman consults for acute vision loss of the left eye. In the last 3 weeks she has presented with fever, pain in the shoulders, neck and hips, and moderate headache. Funduscopy shows a pale and edematous optic disc. Mobilization of the shoulders and hips produces pain. There are no alterations on palpation of the temporal arteries. Laboratory data: hemoglobin 9.7 g/dL ferritin 450 ng/mL erythrocyte sedimentation rate 115 mm/h. What is the most appropriate immediate action in this patient?", "full_answer": "The clinical presentation is strongly suggestive of giant cell arteritis (GCA), with febrile fever, shoulder girdle and pelvic pain, and headache. Age and sex also point to this possibility. Abrupt unilateral visual loss with pale papillary edema is highly indicative of arteritic ischemic optic neuropathy due to GCA. The ESR is elevated, which further clarifies the diagnosis. We have to quickly start corticosteroid treatment. We can already rule out 1 and 5 because they do not contribute anything. 4 could make us doubt, and it is true that we will have to ask for a temporal artery biopsy, but that is not immediate: the statement makes it very clear that we have to act immediately. The current trend is to use megadoses of intravenous corticosteroids, and although aspirin has traditionally been proposed to reduce ischemic events, the evidence currently indicates that antiplatelet agents do not contribute much. An oral corticosteroid dose of around 1mg/kg would also be acceptable. Therefore, if we rule out 3 because the dose is too low, we would have to stay with 2. With reservations, since aspirin has a rather dubious role, and the corticosteroid dose remains ambiguous (we do not know how much the patient weighs). Of all the ophthalmology questions, this is perhaps the one most likely to be contested.", "type": "OPHTHALMOLOGY", "options": {"1": "Request a Doppler ultrasound of temporal arteries.", "2": "Start treatment with prednisone 60mg per day and aspirin 100 mg per day.", "3": "Start treatment with prednisone 10 mg per day.", "4": "Request a temporal artery biopsy.", "5": "Request a brain magnetic resonance imaging."}, "correct_option": 2, "explanations": {"1": {"exist": false, "char_ranges": [], "word_ranges": [], "text": ""}, "2": {"exist": true, "char_ranges": [[0, 318]], "word_ranges": [[0, 48]], "text": "The clinical presentation is strongly suggestive of giant cell arteritis (GCA), with febrile fever, shoulder girdle and pelvic pain, and headache. Age and sex also point to this possibility. Abrupt unilateral visual loss with pale papillary edema is highly indicative of arteritic ischemic optic neuropathy due to GCA."}, "3": {"exist": true, "char_ranges": [[1007, 1049]], "word_ranges": [[160, 170]], "text": "we rule out 3 because the dose is too low,"}, "4": {"exist": true, "char_ranges": [[503, 921]], "word_ranges": [[78, 147]], "text": "4 could make us doubt, and it is true that we will have to ask for a temporal artery biopsy, but that is not immediate: the statement makes it very clear that we have to act immediately. The current trend is to use megadoses of intravenous corticosteroids, and although aspirin has traditionally been proposed to reduce ischemic events, the evidence currently indicates that antiplatelet agents do not contribute much."}, "5": {"exist": false, "char_ranges": [], "word_ranges": [], "text": ""}}} {"id": 532, "year": 2021, "question_id_specific": 67, "full_question": "A 26-year-old woman, with a history of a pregnancy at the age of 24 years ending in spontaneous abortion in the 12th week that did not require curettage, comes to the consultation presenting daily vaginal bleeding of one month of evolution, without fever or pain. Ultrasound showed an increase in the endometrial lining, with an intracavitary image of polypoid appearance. The endometrial biopsy shows intermediate trophoblast proliferation. Which diagnosis is correct?", "full_answer": "It elevates chorionic gonadotropin. It has curative treatment.", "type": "ONCOLOGY", "options": {"1": "Placental bed tumor.", "2": "Atypical endometrial hyperplasia.", "3": "Endometrial polyp.", "4": "Choriocarcinoma.", "5": NaN}, "correct_option": 4, "explanations": {"1": {"exist": false, "char_ranges": [], "word_ranges": [], "text": ""}, "2": {"exist": false, "char_ranges": [], "word_ranges": [], "text": ""}, "3": {"exist": false, "char_ranges": [], "word_ranges": [], "text": ""}, "4": {"exist": true, "char_ranges": [[0, 35]], "word_ranges": [[0, 4]], "text": "It elevates chorionic gonadotropin."}, "5": {"exist": false, "char_ranges": [], "word_ranges": [], "text": ""}}} {"id": 272, "year": 2016, "question_id_specific": 69, "full_question": "A 60-year-old man with extensive ulcerative colitis of 15 years of evolution and in clinical remission for the last 3 years, comes to our office to be informed about the risk of colorectal cancer and about the possibility of participating in prevention programs. It is correct to inform him that:", "full_answer": "Patients with inflammatory bowel disease (IBD) have an increased risk of colorectal cancer, whether or not they are smokers. It is recommended to start screening 8-10 years after diagnosis of IBD (or earlier if primary sclerosing cholangitis or other events occur during this interval). The recommended techniques are colonoscopy with random biopsies (answer 3) and pancolonic chromoendoscopy with staining, which is currently the gold standard because it detects dysplasia better, although it requires more technical experience.", "type": "DIGESTIVE SYSTEM", "options": {"1": "Ulcerative colitis is only associated with an increased risk of colorectal cancer in smokers.", "2": "In her case, given that she is in long-term remission, the screening advised to the general population is considered appropriate.", "3": "In cases such as yours, it is considered appropriate to undergo periodic colonoscopies with multiple biopsies staggered along the entire colon.", "4": "In cases such as yours, screening with periodic fecal occult blood tests is considered appropriate, but with a higher frequency than that used in screening the general population.", "5": NaN}, "correct_option": 3, "explanations": {"1": {"exist": false, "char_ranges": [], "word_ranges": [], "text": ""}, "2": {"exist": false, "char_ranges": [], "word_ranges": [], "text": ""}, "3": {"exist": true, "char_ranges": [[125, 407]], "word_ranges": [[19, 59]], "text": "It is recommended to start screening 8-10 years after diagnosis of IBD (or earlier if primary sclerosing cholangitis or other events occur during this interval). The recommended techniques are colonoscopy with random biopsies (answer 3) and pancolonic chromoendoscopy with staining,"}, "4": {"exist": false, "char_ranges": [], "word_ranges": [], "text": ""}, "5": {"exist": false, "char_ranges": [], "word_ranges": [], "text": ""}}} {"id": 428, "year": 2018, "question_id_specific": 61, "full_question": "A 62-year-old man comes to the hospital emergency department with a fever of 38.3″C and chills. He is tachycardic and tachypneic. Blood tests: leukocytosis with left shift. Blood pressure is 90/60 mmHg. No murmurs are heard on cardiac auscultation and chest X-ray rules out pneumonia. As history of interest she refers repeated urinary tract infections. A week ago the primary care physician prescribed cefuroxime, a drug he is currently taking. Two blood cultures were taken and urine was sent for sediment examination and microbiological culture. The sediment was pathological, with intense pyuria, but negative nitrites. After 24 hours, the microbiology laboratory reported that both the urine and the blood cultures were positive, and the results showed gram-positive cocci grouped in chains, pending identification and antibiogram. Pending the definitive result, what would you recommend?", "full_answer": "With the urine nitrite test we detect the presence of nitrites in the urine. These appear due to the presence in the urine of a high concentration of bacteria possessing the enzyme nitrate reductase, which is capable of reducing the nitrates present in the urine to nitrites. This enzyme is active in most of the bacteria that most frequently cause urinary tract infection, such as many Gram-negative bacilli. Other microorganisms, relatively frequent causes of urinary tract infection, do not possess this enzyme and do not reduce nitrates, such as enterococci, staphylococci and yeasts. As the Gram stain indicates that the bacteria are grouped in chains, as this is typical of enterococcus, we suggest the need to cover this group of bacteria with antibiotic treatment. Staphylococci are typically seen on Gram stain as Gram-positive cocci forming clusters and yeasts, although with the same staining, have a different typical morphology than Gram-positive cocci.", "type": "INFECTIOUS DISEASES AND MICROBIOLOGY", "options": {"1": "Continue with cefuroxime while awaiting the definitive result, since the patient has undergone multiple treatments and we should not make a mistake in his current treatment.", "2": "Change to ertapenem, considering the possibility of multiresistant microorganisms.", "3": "Switch to a broad-spectrum antimicrobial treatment that covers Enterococcus spp.", "4": "Switch to broad-spectrum antimicrobial therapy containing imipenem, considering the possibility of Staphylococcus aureus, since our hospital has a high incidence of methicillin-resistant S. aureus (MRSA).", "5": NaN}, "correct_option": 3, "explanations": {"1": {"exist": false, "char_ranges": [], "word_ranges": [], "text": ""}, "2": {"exist": false, "char_ranges": [], "word_ranges": [], "text": ""}, "3": {"exist": true, "char_ranges": [[589, 772]], "word_ranges": [[93, 124]], "text": "As the Gram stain indicates that the bacteria are grouped in chains, as this is typical of enterococcus, we suggest the need to cover this group of bacteria with antibiotic treatment."}, "4": {"exist": true, "char_ranges": [[773, 966]], "word_ranges": [[124, 151]], "text": "Staphylococci are typically seen on Gram stain as Gram-positive cocci forming clusters and yeasts, although with the same staining, have a different typical morphology than Gram-positive cocci."}, "5": {"exist": false, "char_ranges": [], "word_ranges": [], "text": ""}}} {"id": 393, "year": 2016, "question_id_specific": 224, "full_question": "A patient presents with a long history of multiple physical symptoms: memory loss, headache, dizziness, vomiting, genital pain, limb pain, abdominal distention, and menstrual irregularities. The various medical examinations have ruled out any medical illness. Which of the following is most likely to be the case?", "full_answer": "According to ICD-10 the criteria for diagnosing Somatization Disorder (F45.0) are: A. History of multiple physical symptoms, which begins before age 30, persists for several years, and compels seeking medical attention or causes significant impairment socially, occupationally, or in other important areas of the individual's activity. B. All of the following criteria must be met, and each symptom can occur at any time during the disturbance: 1. Four painful symptoms: history of pain related to at least four areas of the body or four functions (e.g., head, abdomen, back, joints, extremities, chest, rectum; during menstruation, sexual intercourse, or urination). Two gastrointestinal symptoms: history of at least two gastrointestinal symptoms other than pain (e.g., nausea, bloating, vomiting [not during pregnancy], diarrhea, or intolerance to different foods). 3. A sexual symptom: history of at least one sexual or reproductive symptom other than pain (e.g., sexual indifference, erectile or ejaculatory dysfunction, irregular menses, excessive menstrual leakage, vomiting during pregnancy). 4. A pseudoneurological symptom: History of at least one symptom or deficit suggestive of a neurological disorder not limited to pain (conversion symptoms of the type of impaired psychomotor coordination or balance, localized muscle paralysis or weakness, difficulty swallowing, lump-in-throat sensation, aphonia, urinary retention, hallucinations, loss of tactile and painful sensation, diplopia, blindness, deafness, seizures; dissociative symptoms such as amnesia; or loss of consciousness other than fainting). C. Either of the following two characteristics: 1. Upon adequate examination, none of the symptoms in Criterion B can be explained by the presence of a known medical illness or by the direct effects of a substance (e.g., drugs or medication). 2. If a medical illness is present, the physical symptoms or social or occupational impairment are excessive compared to what would be expected from the history, physical examination, or laboratory findings. D. The symptoms are not intentionally produced and are not simulated (unlike in factitious disorder and simulation). Everything fits with what is described in the statement. Conversive or dissociative disorder is ruled out because these are usually related to psychological factors, associated with the symptom or deficit (not deliberate). The onset or exacerbation of the condition is preceded by conflicts or other triggers, which is not mentioned in the statement. Neither would it be a hypochondriacal disorder because this implies a preoccupation and fear of having, or the conviction of having, a serious illness based on the personal interpretation of somatic symptoms. But it does not involve physical signs or symptoms.", "type": "PSYCHIATRY", "options": {"1": "Conversive disorder.", "2": "Hypochondriac disorder.", "3": "Somatization disorder.", "4": "Dissociative disorder.", "5": NaN}, "correct_option": 3, "explanations": {"1": {"exist": true, "char_ranges": [[2241, 2534]], "word_ranges": [[321, 365]], "text": "Conversive or dissociative disorder is ruled out because these are usually related to psychological factors, associated with the symptom or deficit (not deliberate). The onset or exacerbation of the condition is preceded by conflicts or other triggers, which is not mentioned in the statement."}, "2": {"exist": true, "char_ranges": [[2535, 2795]], "word_ranges": [[365, 406]], "text": "Neither would it be a hypochondriacal disorder because this implies a preoccupation and fear of having, or the conviction of having, a serious illness based on the personal interpretation of somatic symptoms. But it does not involve physical signs or symptoms."}, "3": {"exist": false, "char_ranges": [], "word_ranges": [], "text": ""}, "4": {"exist": true, "char_ranges": [[2241, 2534]], "word_ranges": [[321, 365]], "text": "Conversive or dissociative disorder is ruled out because these are usually related to psychological factors, associated with the symptom or deficit (not deliberate). The onset or exacerbation of the condition is preceded by conflicts or other triggers, which is not mentioned in the statement."}, "5": {"exist": false, "char_ranges": [], "word_ranges": [], "text": ""}}} {"id": 124, "year": 2012, "question_id_specific": 46, "full_question": "A 58-year-old woman comes for a follow-up visit for diabetes mellitus and hypertension. She feels well but states that she has stopped taking verapamil because of constipation. She presents with intolerance to ACE inhibitors due to coughing. On examination, blood pressure is 156/92 mm Hg. CBC includes a creatinine of 1.6 mg/dL, 24-hour urine protein excretion of 1.5 g/day, and a creatinine clearance of 45 ml/min. On this basis, what is the most effective treatment to slow the progression of the patient's type 2 diabetic nephropathy?", "full_answer": "This question is typical and quite easy. The antihypertensive drugs that have been shown to slow the progression of diabetic nephropathy are the ACE inhibitors and ARAIIs. Specifically, angiotensin II receptor blockers have demonstrated their efficacy in this field for type 2 DM, which together with the side effects that IECAS produce in the patient makes an ARAII the drug of choice. This does not seem to me to be an open question (without going into pharmacoeconomic issues, of course...).", "type": "NEPHROLOGY", "options": {"1": "Angiotensin converting enzyme inhibitor.", "2": "Angiotensin receptor blocker.", "3": "Calcium antagonist.", "4": "Alpha-blocker.", "5": "Beta-blocker."}, "correct_option": 2, "explanations": {"1": {"exist": true, "char_ranges": [[186, 386]], "word_ranges": [[28, 62]], "text": "angiotensin II receptor blockers have demonstrated their efficacy in this field for type 2 DM, which together with the side effects that IECAS produce in the patient makes an ARAII the drug of choice."}, "2": {"exist": true, "char_ranges": [[186, 386]], "word_ranges": [[28, 62]], "text": "angiotensin II receptor blockers have demonstrated their efficacy in this field for type 2 DM, which together with the side effects that IECAS produce in the patient makes an ARAII the drug of choice."}, "3": {"exist": false, "char_ranges": [], "word_ranges": [], "text": ""}, "4": {"exist": false, "char_ranges": [], "word_ranges": [], "text": ""}, "5": {"exist": false, "char_ranges": [], "word_ranges": [], "text": ""}}} {"id": 405, "year": 2016, "question_id_specific": 31, "full_question": "A 20-year-old woman with a 15-cm solid-cystic ovarian tumor detected by ultrasound after presenting with nonspecific abdominal symptoms. In the histopathological study of the corresponding specimen, teeth, hairs, areas of intestinal epithelium, areas of squamous epithelium (15%) and bronchial epithelium, as well as neuroectodermal and embryonic elements are found in several of the histological preparations. In reference to this case, point out the correct diagnosis:", "full_answer": "Teratoma is the most frequent germinal tumor. It presents differentiation in elements of the three embryonic layers: endoderm, mesoderm and ectoderm. Macroscopically they can be cystic or solid, and histologically the tissues composing the tumor can be mature (well differentiated, like adult tissues) or immature (like embryonic tissues).Mature cystic teratoma is the most frequent: they represent on average 10% of ovarian tumors (5-25% depending on the casuistry). They occur at any age. They predominate in ectodermal tissues, such as skin, lining a cavity of keratinous content. In the cavity a spur is recognized from which hair or tooth structures often arise. Frequent tissues include: nervous tissue, usually glial and ependymal epithelium, respiratory and digestive epithelia and various mesodermal structures. They are bilateral in about 10% of cases. The mature cystic teratoma is benign, but in 2% of them a malignant tumor may develop from one of the tissue components (spinocellular carcinoma, carcinoid, adenocarcinoma, thyroid tissue carcinoma, sarcoma). In this case at first we thought that the option was mature cystic teratoma but thanks to Ramón, when we read it more carefully, we found that it had neuroectodermal and embryonal elements in several of the histological preparations, so we consider that the correct answer is 2.", "type": "ONCOLOGY (ECTOPIC)", "options": {"1": "Teratocarcinoma.", "2": "Immature teratoma.", "3": "Mature cystic teratoma.", "4": "Dysgerminoma.", "5": NaN}, "correct_option": 2, "explanations": {"1": {"exist": false, "char_ranges": [], "word_ranges": [], "text": ""}, "2": {"exist": true, "char_ranges": [[1201, 1350]], "word_ranges": [[181, 205]], "text": "we found that it had neuroectodermal and embryonal elements in several of the histological preparations, so we consider that the correct answer is 2."}, "3": {"exist": true, "char_ranges": [[1201, 1350]], "word_ranges": [[181, 205]], "text": "we found that it had neuroectodermal and embryonal elements in several of the histological preparations, so we consider that the correct answer is 2."}, "4": {"exist": false, "char_ranges": [], "word_ranges": [], "text": ""}, "5": {"exist": false, "char_ranges": [], "word_ranges": [], "text": ""}}} {"id": 497, "year": 2020, "question_id_specific": 127, "full_question": "A 60-year-old man, smoker of 40 packs/year, who consults for pain in the ulnar border of the left arm and left palpebral ptosis since one month ago. Chest X-ray shows a mass in the left upper lobe and CT scan confirms the lesion with invasion of the second rib. In relation to the suspected diagnosis, the most appropriate treatment is:", "full_answer": "Typical smoker's pancoast, probably an epidermoid, with Horner's sdr. Best would be chemoradio followed by surgical salvage. In the old days it was just RT and surgery. Maybe not far off enter immunotherapy in first line. There are more and more new things.", "type": "MEDICAL ONCOLOGY", "options": {"1": "Neoadjuvant chemotherapy with combined chemoradiotherapy, followed by surgery.", "2": "Neoadjuvant chemotherapy without surgery, followed by radiotherapy.", "3": "Surgery followed by combined chemoradiotherapy.", "4": "Radiotherapy without chemotherapy, followed by surgery.", "5": NaN}, "correct_option": 1, "explanations": {"1": {"exist": true, "char_ranges": [[0, 124]], "word_ranges": [[0, 17]], "text": "Typical smoker's pancoast, probably an epidermoid, with Horner's sdr. Best would be chemoradio followed by surgical salvage."}, "2": {"exist": false, "char_ranges": [], "word_ranges": [], "text": ""}, "3": {"exist": false, "char_ranges": [], "word_ranges": [], "text": ""}, "4": {"exist": true, "char_ranges": [[125, 168]], "word_ranges": [[17, 27]], "text": "In the old days it was just RT and surgery."}, "5": {"exist": false, "char_ranges": [], "word_ranges": [], "text": ""}}} {"id": 307, "year": 2016, "question_id_specific": 207, "full_question": "A 34-year-old patient playing tennis is hit by a ball at the level of the left orbit. On examination he presents significant palpebral hematoma, hyposphagmia, diplopia to superior vision with limitation of the upper version of the eyeball. What would you suspect?", "full_answer": "This question has a small typo (one would say \"tennis\"), but personally I am a little more bothered by the error in the description of the scan. Specifically when they say \"diplopia to superior vision with limitation of the superior version of the eyeball\". Versions are bilateral conjugated movements of both eyes. That is, a version is composed of the sum of two ductions, since duction is the movement of only one eye. When you explore the versions, specifically the superior version or supraversion, both eyes rotate upward. At that time, you may encounter a limitation of the movement of one eye. In this case, when you scan the supraversion, you suspect or you detect a limitation in the movement of the left eye. And we would call this a limitation to the supraversion of that eye. It doesn't make sense to talk about limitation to the version of the eyeball, because the version is always of both eyeballs. Okay, it is understood, but it is an important misconception that should not be made by the person writing the question (who is supposed to be an expert on the subject), but well, as I do not think it is a reason to challenge the question, let's go on. It is about a patient who has received a blow. All four answers are fractures, so we have a pretty definite picture. The palpebral hematoma and hyposphagma (subconjunctival hemorrhage) does not give us the key data. What is important is that the eye that has received the blow does not go up, and therefore there is double vision in the superior gaze. It might be tempting to answer 3, thinking that because the superior rectus is trapped, it does not work and therefore the eye cannot go up. However, muscles trapped in a fracture do not automatically lose their muscular action. What happens is that they become trapped, they are \"hooked\", and cannot stretch. The problem is that the inferior wall has been broken, the inferior rectus has been trapped, and when the eye must look up, this inferior rectus cannot stretch as it should. It is not paralysis, but a restrictive problem. This question could also be answered by discarding. Options 2 and 4 can be ignored because they are too far away from the extraocular muscles to produce diplopia. And as for the walls of the orbit, the ones that break most frequently are the inferior and medial wall. The superior wall ruptures very rarely.", "type": "OPHTHALMOLOGY", "options": {"1": "Fracture of the inferior wall of the floor of the orbit with trapping of the inferior rectus muscle.", "2": "Zygomatic arch fracture.", "3": "Fracture of the superior wall of the orbit with entrapment of the superior rectus muscle.", "4": "Dentoalveolar fracture.", "5": NaN}, "correct_option": 1, "explanations": {"1": {"exist": true, "char_ranges": [[2215, 2319]], "word_ranges": [[389, 409]], "text": "And as for the walls of the orbit, the ones that break most frequently are the inferior and medial wall."}, "2": {"exist": true, "char_ranges": [[2104, 2214]], "word_ranges": [[369, 389]], "text": "Options 2 and 4 can be ignored because they are too far away from the extraocular muscles to produce diplopia."}, "3": {"exist": true, "char_ranges": [[2320, 2359]], "word_ranges": [[409, 415]], "text": "The superior wall ruptures very rarely."}, "4": {"exist": true, "char_ranges": [[2104, 2214]], "word_ranges": [[369, 389]], "text": "Options 2 and 4 can be ignored because they are too far away from the extraocular muscles to produce diplopia."}, "5": {"exist": false, "char_ranges": [], "word_ranges": [], "text": ""}}} {"id": 589, "year": 2022, "question_id_specific": 63, "full_question": "A 36-year-old male who consults for conjunctival hyperemia and foreign body sensation. Which of the following pathologies does NOT correspond to the above mentioned examination?", "full_answer": "We are being presented with a picture of conjunctival inflammation or conjunctivitis and asked which of them is not associated with pre-auricular lymphadenopathy. Adenovirus conjunctivitis can be associated with inflammation of the pre-auricular or ipsilateral sub-mandibular lymph node (option 1 discarded). Allergic conjunctivitis is a hypersensitivity condition that can occur immediately after contact with the triggering stimulus or after 24-72 hours, and is associated with accompanying signs such as eyelid edema and erythema, changes in periocular skin pigmentation, and a key symptom: itching. What it is not associated with is lymphadenopathy (option 2 correct). The oculoglandular syndrome of Parinaud is a picture that by definition is composed of conjunctivitis of infectious cause associated with ipsilateral pre-auricular or latero-cervical lymphadenopathy (one or several), generally produced by bacteria such as Bartonella henselae (cat scratch disease) or Francisella tularensis (tularemia or rabbit fever): option 3 discarded. Chlamydial conjunctivitis can present as 3 very distinct clinical pictures: trachoma, ophthalmia neonatorum or adult inclusion body conjunctivitis. The latter is considered a sexually transmitted disease and is associated with ipsilateral pre-auricular nodes (option 4 discarded).", "type": "OPHTHALMOLOGY", "options": {"1": "Adenoviral conjunctivitis.", "2": "Allergic conjunctivitis.", "3": "Parinaud's oculoglandular syndrome.", "4": "Chlamydia conjunctivitis.", "5": NaN}, "correct_option": 2, "explanations": {"1": {"exist": true, "char_ranges": [[163, 308]], "word_ranges": [[23, 41]], "text": "Adenovirus conjunctivitis can be associated with inflammation of the pre-auricular or ipsilateral sub-mandibular lymph node (option 1 discarded)."}, "2": {"exist": true, "char_ranges": [[309, 672]], "word_ranges": [[41, 94]], "text": "Allergic conjunctivitis is a hypersensitivity condition that can occur immediately after contact with the triggering stimulus or after 24-72 hours, and is associated with accompanying signs such as eyelid edema and erythema, changes in periocular skin pigmentation, and a key symptom: itching. What it is not associated with is lymphadenopathy (option 2 correct)."}, "3": {"exist": true, "char_ranges": [[673, 1045]], "word_ranges": [[94, 143]], "text": "The oculoglandular syndrome of Parinaud is a picture that by definition is composed of conjunctivitis of infectious cause associated with ipsilateral pre-auricular or latero-cervical lymphadenopathy (one or several), generally produced by bacteria such as Bartonella henselae (cat scratch disease) or Francisella tularensis (tularemia or rabbit fever): option 3 discarded."}, "4": {"exist": true, "char_ranges": [[1046, 1326]], "word_ranges": [[143, 179]], "text": "Chlamydial conjunctivitis can present as 3 very distinct clinical pictures: trachoma, ophthalmia neonatorum or adult inclusion body conjunctivitis. The latter is considered a sexually transmitted disease and is associated with ipsilateral pre-auricular nodes (option 4 discarded)."}, "5": {"exist": false, "char_ranges": [], "word_ranges": [], "text": ""}}} {"id": 350, "year": 2016, "question_id_specific": 160, "full_question": "A 45-year-old woman, mother of 3 children, attends an early diagnosis consultation. Cervicovaginal cytology is compatible with a high-grade squamous intraepithelial lesion. Which of the following options would you choose?", "full_answer": "Colposcopy with possible biopsy. H-SIL (High-grade squamous intraepithelial lesion)*: includes changes suggestive of CIN2 and CIN3/CIS. The cytologic diagnosis must be confirmed by biopsy, for which it is necessary to perform a directed biopsy by colposcopy.", "type": "GYNECOLOGY AND OBSTETRICS", "options": {"1": "Repeat cytology in 1 month.", "2": "Colposcopy with possible biopsy.", "3": "Hysterectomy with bilateral salpinguectomy and ovarian conservation.", "4": "Fractionated uterine curettage.", "5": NaN}, "correct_option": 2, "explanations": {"1": {"exist": false, "char_ranges": [], "word_ranges": [], "text": ""}, "2": {"exist": true, "char_ranges": [[136, 258]], "word_ranges": [[16, 36]], "text": "The cytologic diagnosis must be confirmed by biopsy, for which it is necessary to perform a directed biopsy by colposcopy."}, "3": {"exist": false, "char_ranges": [], "word_ranges": [], "text": ""}, "4": {"exist": false, "char_ranges": [], "word_ranges": [], "text": ""}, "5": {"exist": false, "char_ranges": [], "word_ranges": [], "text": ""}}} {"id": 25, "year": 2011, "question_id_specific": 116, "full_question": "A 40-year-old woman is admitted to the Emergency Department with a fever of 38ºC and right lumbar pain. The CBC shows 25000 leukocytes/mm3 with left deviation and an abdominal ultrasound shows a coralliform lithiasis. What microorganism do we expect to find?", "full_answer": "Epidemiology question shared with URO. Coraliform lithiasis are associated with urea-folding germs: Proteus in the first place, Pseudomonas, Klebsiella and some staphylococci.", "type": "INFECTIOUS", "options": {"1": "Escherichia coli.", "2": "Enterococcus faecalis.", "3": "Salmonella typhi.", "4": "Candida albicans.", "5": "Proteus mirabilis."}, "correct_option": 5, "explanations": {"1": {"exist": false, "char_ranges": [], "word_ranges": [], "text": ""}, "2": {"exist": false, "char_ranges": [], "word_ranges": [], "text": ""}, "3": {"exist": false, "char_ranges": [], "word_ranges": [], "text": ""}, "4": {"exist": false, "char_ranges": [], "word_ranges": [], "text": ""}, "5": {"exist": true, "char_ranges": [[39, 175]], "word_ranges": [[5, 22]], "text": "Coraliform lithiasis are associated with urea-folding germs: Proteus in the first place, Pseudomonas, Klebsiella and some staphylococci."}}} {"id": 103, "year": 2012, "question_id_specific": 89, "full_question": "A 55-year-old patient diagnosed with hypertension 2 months ago at a company check-up. Laboratory tests: glucose 129 mg/dL, cholesterol 202 mg/dL, LDLc 160 mg/dL, HDLc 38 mg/dL, triglycerides 171 mg/dL, creatinine 1.1 mg/dL. He consults for poor blood pressure control, malleolar edema and dyspnea that is occasionally nocturnal. Physical examination: BMI 38 kg/m2, abdominal perimeter 110 cm, BP 157/93, HR 70 bpm, HR 14 rpm. Systolic murmur with reinforcement of the second sound. Peripheral pulses with discrete asymmetry in right pedius and right posterior tibial pulses that are weaker with respect to the left extremity. Chest X-ray and ECG without alterations Which of the following complementary tests is the least necessary for the detection of lesions in target organs?", "full_answer": "HbA1c would be useful in this patient for the diagnosis of diabetes but not for the target organ involvement of hypertension.", "type": "ENDOCRINOLOGY", "options": {"1": "Transthoracic echocardiogram.", "2": "Performance of fundus examination.", "3": "Determination of microalbuminuria.", "4": "Ankle-brachial index.", "5": "Determination of glycosylated hemoglobin."}, "correct_option": 5, "explanations": {"1": {"exist": false, "char_ranges": [], "word_ranges": [], "text": ""}, "2": {"exist": false, "char_ranges": [], "word_ranges": [], "text": ""}, "3": {"exist": false, "char_ranges": [], "word_ranges": [], "text": ""}, "4": {"exist": false, "char_ranges": [], "word_ranges": [], "text": ""}, "5": {"exist": true, "char_ranges": [[0, 125]], "word_ranges": [[0, 21]], "text": "HbA1c would be useful in this patient for the diagnosis of diabetes but not for the target organ involvement of hypertension."}}} {"id": 22, "year": 2011, "question_id_specific": 112, "full_question": "A patient comes to the Emergency Department with fever without focus of 24 h of evolution, hypothyroidism and impression of severity. She underwent a splenectomy 2 years ago for staging of Hosgkin's disease. Which microorganism is most frequently implicated?", "full_answer": "Another one on epidemiology. In splenectomized patients, the risk of acquiring serious infections by encapsulated germs is increased, again Pneumococcus being the most frequent.", "type": "INFECTIOUS", "options": {"1": "Streptococcus pneumoniae.", "2": "Pseudomonas aeruginosa.", "3": "Salmonella no typhi.", "4": "Pneumocystis jirovecii.", "5": "Streptococcus viridans."}, "correct_option": 1, "explanations": {"1": {"exist": true, "char_ranges": [[29, 177]], "word_ranges": [[4, 24]], "text": "In splenectomized patients, the risk of acquiring serious infections by encapsulated germs is increased, again Pneumococcus being the most frequent."}, "2": {"exist": false, "char_ranges": [], "word_ranges": [], "text": ""}, "3": {"exist": false, "char_ranges": [], "word_ranges": [], "text": ""}, "4": {"exist": false, "char_ranges": [], "word_ranges": [], "text": ""}, "5": {"exist": false, "char_ranges": [], "word_ranges": [], "text": ""}}} {"id": 488, "year": 2020, "question_id_specific": 105, "full_question": "A 67-year-old patient who in the last 6 months, in two routine analyses, presents progressive lymphocytosis. In the last one, hemoglobin 15.4 g/dL; leukocytes 18.5 x103/μL with 82% of mature lymphocytes that by flow cytometry express CD5/CD19/CD23 antigens and platelets 240 x103/μL. What attitude do you think is correct?", "full_answer": "A question that I think can be complicated for a MIR examiner. We are told of a leukocytosis at the expense of lymphocytes with a B-CLL phenotype. It seems that there are no other alarming data. No clinical data are given (so we assume that the patient is asymptomatic). No cytopenias... Conclusion: FOLLOW UP. The main hesitation with the answer should be with option 1. Why do I consider it incorrect? Because the TP53 mutation, although it establishes prognosis, is not indicated at diagnosis, but when treatment is going to be initiated (first line and prior to successive lines in case they are needed). It would be possible to go too far in this question in which the percentage given does not refer directly to clonal lymphocytes, so that it could even be a monoclonal B lymphocytosis. But this is going too far.", "type": "HEMATOLOGY", "options": {"1": "Study of TP53 mutations to establish prognosis.", "2": "Bone aspirate/biopsy to confirm the diagnosis.", "3": "PET/CT to establish the therapeutic attitude.", "4": "New clinical and analytical control in 6 months.", "5": NaN}, "correct_option": 4, "explanations": {"1": {"exist": true, "char_ranges": [[412, 608]], "word_ranges": [[72, 104]], "text": "the TP53 mutation, although it establishes prognosis, is not indicated at diagnosis, but when treatment is going to be initiated (first line and prior to successive lines in case they are needed)."}, "2": {"exist": false, "char_ranges": [], "word_ranges": [], "text": ""}, "3": {"exist": false, "char_ranges": [], "word_ranges": [], "text": ""}, "4": {"exist": true, "char_ranges": [[63, 310]], "word_ranges": [[12, 54]], "text": "We are told of a leukocytosis at the expense of lymphocytes with a B-CLL phenotype. It seems that there are no other alarming data. No clinical data are given (so we assume that the patient is asymptomatic). No cytopenias... Conclusion: FOLLOW UP."}, "5": {"exist": false, "char_ranges": [], "word_ranges": [], "text": ""}}} {"id": 111, "year": 2012, "question_id_specific": 153, "full_question": "A 59-year-old woman consults a gynecologist for a spontaneous, spontaneous, uniorificial discharge from the right nipple (unilateral) without palpable nodularity. What is the most likely diagnosis?", "full_answer": "The correct answer is 2. In the presence of unilateral and uniorificial hematic telorrhea, the most frequent cause is intraductal papilloma (50%), followed by ductal ectasia and carcinoma. When associated with palpable mass, 60% correspond to carcinoma. Paget's disease is of the eczematous type. In hyperprolactinemia, telorrhea is bilateral and pluriorificial. Mammography often does not show papillomas, being more useful ultrasound and galactography.", "type": "GYNECOLOGY AND OBSTETRICS", "options": {"1": "Given the hematic nature of the discharge, the most likely diagnosis is infiltrating ductal carcinoma.", "2": "Intraductal papilloma.", "3": "Paget's disease of the nipple.", "4": "Tumor hyperprolactinemia.", "5": "Mammography will indicate the diagnosis."}, "correct_option": 2, "explanations": {"1": {"exist": true, "char_ranges": [[189, 253]], "word_ranges": [[28, 37]], "text": "When associated with palpable mass, 60% correspond to carcinoma."}, "2": {"exist": true, "char_ranges": [[25, 146]], "word_ranges": [[5, 22]], "text": "In the presence of unilateral and uniorificial hematic telorrhea, the most frequent cause is intraductal papilloma (50%),"}, "3": {"exist": true, "char_ranges": [[254, 296]], "word_ranges": [[37, 44]], "text": "Paget's disease is of the eczematous type."}, "4": {"exist": true, "char_ranges": [[297, 362]], "word_ranges": [[44, 51]], "text": "In hyperprolactinemia, telorrhea is bilateral and pluriorificial."}, "5": {"exist": true, "char_ranges": [[363, 454]], "word_ranges": [[51, 63]], "text": "Mammography often does not show papillomas, being more useful ultrasound and galactography."}}} {"id": 119, "year": 2012, "question_id_specific": 119, "full_question": "A 17-year-old woman comes to the emergency department with a high fever, pharyngeal pain and cervical lymphadenopathy. She had previously been diagnosed with acute pharyngitis and was treated with amoxicillin, and later presented with a generalized macular skin rash. Laboratory tests showed slight leukocytosis with lymphocytosis and presence of activated lymphocytes, slight thrombopenia and slightly increased transaminases. What would be the most likely diagnosis of this clinical picture?", "full_answer": "Of course it is a typical picture (EBV, CMV, VH6, Toxoplasma), of which you will not see so clearly at the emergency room door when you are on call. Be careful with the bad blood in answer 3, because an acute toxoplasmosis can behave like this and also give a macular exanthema, although the one of this girl is related to the intake of amoxicillin. By the way, I add a note, if instead of a girl of 17 years so clear it were a young man/girl with risky sexual relations, do not forget the HIV primoinfection as a cause of mononucleosis syndrome.", "type": "INFECTOLOGY", "options": {"1": "It is a typical picture of infectious mononucleosis.", "2": "Varicella zoster virus infection.", "3": "Acute toxoplasmosis.", "4": "Lyme disease.", "5": "Infection by herpes virus 8."}, "correct_option": 1, "explanations": {"1": {"exist": false, "char_ranges": [], "word_ranges": [], "text": ""}, "2": {"exist": false, "char_ranges": [], "word_ranges": [], "text": ""}, "3": {"exist": true, "char_ranges": [[200, 349]], "word_ranges": [[39, 65]], "text": "an acute toxoplasmosis can behave like this and also give a macular exanthema, although the one of this girl is related to the intake of amoxicillin."}, "4": {"exist": false, "char_ranges": [], "word_ranges": [], "text": ""}, "5": {"exist": false, "char_ranges": [], "word_ranges": [], "text": ""}}} {"id": 562, "year": 2022, "question_id_specific": 183, "full_question": "A 45-year-old woman with no history of interest consults for dyspneic sensation of about 4 days of evolution. Examination shows rhythmic tachycardia, without murmurs, and pulmonary auscultation is normal. Arterial blood gases showed a pO2 of 70 mmHg and a pCO2 of 32 mmHg. Hemogram, renal and hepatic function are normal. Prothrombin time 90%, activated partial thromboplastin time (APTT) ratio of 2 to control (N <1.2). Which of the following diagnoses is the most likely:", "full_answer": "In this question what they assume is that this is a 45-year-old woman who is diagnosed with pulmonary thromboembolism (PTE). With this diagnosis, options 3 and 4 are directly excluded (also, she is female, so we would not have to think about hemophilia) As an analytical data, it says that there is a lengthening of the aPTT. So, they want you to know, she has positive lupus ac and link it directly to antiphospholipid syndrome (APS). A Leiden's does not alter coagulation, so it is ruled out as well. Anyway, although it is not within the possible answers, lupus positivity can be related to a myriad of clinical situations, syndromes and pathologies. Assuming that a PTE + lupus is a PFS seems to me to be a bit of a stretch, but it is the most likely with the data provided.", "type": "HEMATOLOGY", "options": {"1": "Antiphospholipid syndrome.", "2": "Factor V of Leiden.", "3": "Hemophilia.", "4": "Acute pericarditis.", "5": NaN}, "correct_option": 1, "explanations": {"1": {"exist": true, "char_ranges": [[254, 435]], "word_ranges": [[43, 76]], "text": "As an analytical data, it says that there is a lengthening of the aPTT. So, they want you to know, she has positive lupus ac and link it directly to antiphospholipid syndrome (APS)."}, "2": {"exist": true, "char_ranges": [[436, 502]], "word_ranges": [[76, 89]], "text": "A Leiden's does not alter coagulation, so it is ruled out as well."}, "3": {"exist": true, "char_ranges": [[0, 253]], "word_ranges": [[0, 43]], "text": "In this question what they assume is that this is a 45-year-old woman who is diagnosed with pulmonary thromboembolism (PTE). With this diagnosis, options 3 and 4 are directly excluded (also, she is female, so we would not have to think about hemophilia)"}, "4": {"exist": true, "char_ranges": [[0, 253]], "word_ranges": [[0, 43]], "text": "In this question what they assume is that this is a 45-year-old woman who is diagnosed with pulmonary thromboembolism (PTE). With this diagnosis, options 3 and 4 are directly excluded (also, she is female, so we would not have to think about hemophilia)"}, "5": {"exist": false, "char_ranges": [], "word_ranges": [], "text": ""}}} {"id": 514, "year": 2021, "question_id_specific": 97, "full_question": "After a traffic accident you attend to an injured person. The patient opens his eyes at your call, but only utters words that are inappropriate. On motor examination, the pain is localized in the right extremities, but spreads on stimulation of the left extremities. How would you describe his condition according to the Glasgow Coma Scale?", "full_answer": "The first thing we should know when applying the Glasgow Coma Scale to a patient is that we should always keep the best value in each sphere (ocular response, motor response and verbal response). In this case, we have a patient who has an ocular opening to the call (ocular response 3 out of 4), emits inappropriate words (verbal response 3 out of 5) and localizes pain with right extremities (motor response 5 out of 6). The Glasgow Coma Scale score would be 11 (out of 15). The best value of the examination is taken because in many cases (assessment of a polytraumatized patient, ischemic stroke or spontaneous acute cerebral bleeding), the motor examination may be artifacted by other elements. For example: in a polytraumatized patient (such as the one in question), the response presented with left extremities may be due to localized trauma in that area, and not strictly speaking to an underlying traumatic brain injury. Likewise, sometimes the verbal response may be underestimated in the polytraumatized patient due to airway obstruction (e.g., due to the tongue retreating into the oropharynx). By always assessing the best response, we will have a truer picture of the patient's condition.", "type": "CRITICAL AND EMERGENCY CARE", "options": {"1": "E3V2M4.", "2": "E3V3M5.", "3": "E2V3M4.", "4": "E2V4M5.", "5": NaN}, "correct_option": 2, "explanations": {"1": {"exist": false, "char_ranges": [], "word_ranges": [], "text": ""}, "2": {"exist": false, "char_ranges": [], "word_ranges": [], "text": ""}, "3": {"exist": false, "char_ranges": [], "word_ranges": [], "text": ""}, "4": {"exist": false, "char_ranges": [], "word_ranges": [], "text": ""}, "5": {"exist": false, "char_ranges": [], "word_ranges": [], "text": ""}}} {"id": 101, "year": 2012, "question_id_specific": 86, "full_question": "A 55-year-old woman consults for fatigue and polyuria of two years' evolution. Laboratory tests reveal hypercalcemia, increased PTH and increased bone turnover markers. The radiographic study shows subperiosteal resorption and osteoporosis. Sestamibi scintigraphy revealed a parathyroid adenoma. Parathyroidectomy was performed with minimally invasive surgery. Postoperatively, the patient developed severe hypocalcemia and tetany, with PTH below 5 ng/l. The patient responds well to initial treatment with intravenous calcium and later with oral calcium and vitamin D. What is the most likely diagnosis?", "full_answer": "Question of very high difficulty, I would not worry about having failed it. It is easy to get confused with the hypoparathyroidism answer. Permanent post-surgical hypoparathyroidism after parathyroidectomy of an adenoma with minimally invasive surgery is rare. I write a brief summary of the hungry bone syndrome that I have found in a clinical case in the Annals of Internal Medicine [1]: \"The pathophysiology of the hungry bone syndrome (HBS) is related to an imbalance between bone formation and bone resorption, which is associated with hypocalcemia, hypophosphatemia and hypomagnesemia (1). This syndrome can be observed after surgery in patients with primary hyperparathyroidism (HPT), as well as in tertiary HPT of chronic renal failure, and to a lesser extent after treatment of processes with excess of circulating thyroid hormones (2). In PTH, there is an excess of parathyroid hormone (PTH) which stimulates osteoclastic activity causing demineralization of the bone matrix and releasing calcium into the bloodstream. After parathyroidectomy, serum PTH levels drop dramatically, thus, PTH-induced bone resorption ceases, while osteoblastic activity continues resulting in increased bone uptake of calcium, phosphate and magnesium, thus SHH appears (3).\"", "type": "ENDOCRINOLOGY", "options": {"1": "Permanent surgical hypoparathyroidism.", "2": "Transfusion of citrated blood.", "3": "Vitamin D insufficiency.", "4": "Hungry bone syndrome.", "5": "Osteomalacia."}, "correct_option": 4, "explanations": {"1": {"exist": true, "char_ranges": [[139, 260]], "word_ranges": [[23, 37]], "text": "Permanent post-surgical hypoparathyroidism after parathyroidectomy of an adenoma with minimally invasive surgery is rare."}, "2": {"exist": false, "char_ranges": [], "word_ranges": [], "text": ""}, "3": {"exist": false, "char_ranges": [], "word_ranges": [], "text": ""}, "4": {"exist": true, "char_ranges": [[390, 590]], "word_ranges": [[62, 89]], "text": "\"The pathophysiology of the hungry bone syndrome (HBS) is related to an imbalance between bone formation and bone resorption, which is associated with hypocalcemia, hypophosphatemia and hypomagnesemia"}, "5": {"exist": false, "char_ranges": [], "word_ranges": [], "text": ""}}} {"id": 230, "year": 2014, "question_id_specific": 179, "full_question": "A 4-year-old boy comes to the health center who, 5 minutes before, starts with angioedema in the face, conjunctivitis, nasal congestion and hoarseness, coinciding with the ingestion of a spoonful of yogurt given to him by mistake at school. Among the antecedents, he was diagnosed with cow's milk protein allergy. Examination revealed mild hypotension, heart rate 110 bpm, O2 Sat 93%, pale and slightly sweaty, with scattered wheezing. What is the first treatment of choice?", "full_answer": "We are facing a case of Anaphylaxis so the first measure is to administer Adrenaline 1/1000 intramuscular (0.01mg/kg). Subsequently, systemic corticosteroids such as methylprednisolone will be given, which will take a few hours to take effect. So adrenaline first, as it is faster acting, then the rest (Question very similar to one last year, instead of yogurt, it was omelet).", "type": "PEDIATRICS", "options": {"1": "Provoke vomiting.", "2": "Adrenaline 1/1000 subcutaneous.", "3": "Intramuscular adrenaline 1/1000.", "4": "Intramuscular methylprednisolone.", "5": "Nebulized salbutamol."}, "correct_option": 3, "explanations": {"1": {"exist": false, "char_ranges": [], "word_ranges": [], "text": ""}, "2": {"exist": false, "char_ranges": [], "word_ranges": [], "text": ""}, "3": {"exist": true, "char_ranges": [[247, 288]], "word_ranges": [[37, 44]], "text": "adrenaline first, as it is faster acting,"}, "4": {"exist": true, "char_ranges": [[119, 243]], "word_ranges": [[18, 36]], "text": "Subsequently, systemic corticosteroids such as methylprednisolone will be given, which will take a few hours to take effect."}, "5": {"exist": false, "char_ranges": [], "word_ranges": [], "text": ""}}} {"id": 479, "year": 2020, "question_id_specific": 164, "full_question": "A 72-year-old man presenting since 48 hours with fever of 38.7°C, cough, purulent expectoration and dyspnea. Constants: BP 85/60 mmHg, heart rate 100 bpm, respiratory rate 35 rpm, SatO2 80%. Of note was the presence of confusion and crackles in the left lower lung field. Chest X-ray confirms the existence of pneumonia of the lingula and left lower lobe. What would be the appropriate place of care?", "full_answer": "Applying the CURB-65 severity scale (one of the most commonly used in the ED for stratifying the severity of pneumonia, although in this case simply applying common sense, the patient should be admitted to the ICU), we would have a score of at least 4 points (we would be missing the serum urea data), which gives us severity criteria. Since there are no criteria that contraindicate admission to the ICU, admission to the unit would be indicated.", "type": "CRITICAL CARE", "options": {"1": "Outpatient treatment at home.", "2": "Admission to an emergency observation unit.", "3": "Hospital admission to the ward.", "4": "Admission to the Intensive Care Unit.", "5": NaN}, "correct_option": 4, "explanations": {"1": {"exist": false, "char_ranges": [], "word_ranges": [], "text": ""}, "2": {"exist": false, "char_ranges": [], "word_ranges": [], "text": ""}, "3": {"exist": false, "char_ranges": [], "word_ranges": [], "text": ""}, "4": {"exist": true, "char_ranges": [[0, 215]], "word_ranges": [[0, 36]], "text": "Applying the CURB-65 severity scale (one of the most commonly used in the ED for stratifying the severity of pneumonia, although in this case simply applying common sense, the patient should be admitted to the ICU),"}, "5": {"exist": false, "char_ranges": [], "word_ranges": [], "text": ""}}} {"id": 515, "year": 2021, "question_id_specific": 101, "full_question": "15-year-old male, admitted after being run over on the public road, resulting in being ejected. He was admitted conscious and oriented, with intense pain in the left hemithorax and dyspnea. Physical examination showed blood pressure 90/60 mmHg, heart rate 130 bpm, basal oxygen saturation 90%, respiratory rate 35 rpm. There is complete hypophonesis in the left hemithorax and dullness to percussion. Chest X-ray shows multiple left costal fractures and massive ipsilateral pleural effusion. Placement of a pleural drain is indicated, with 1700 cc of hematic fluid coming out. What is the decision to be made:", "full_answer": "There are two answers that we can immediately rule out if we stick to the initial management of severe chest trauma: 1 (noninvasive ventilation is not indicated because it does not secure the airway) and 3 (observation in these cases, waiting for worsening, only leads to increased morbidity and mortality). Between options 2 and 4 there could be controversy between the literature and the usual clinical practice, in which we usually have to face in a delicate balance the clinical stability of the patient (and what we can \"make up\" this stability with our life support therapies) and the actions that result in better patient care for the definitive treatment of the lesions. We are dealing with a severe polytraumatized patient. With the data provided by the question, a patient who seems to be in shock (tachycardia, hypoxemia, tachypnea... although we do not receive data, for example, on lactacidemia), and in whom we have to our knowledge a left thoracic trauma, which not only implies bone injury, but may mask other injuries of high severity and which are not diagnosed with a chest X-ray: myocardial contusion, pericardial effusion, splenic injury, diaphragmatic injury... We place a left pleural drain on the patient and obtain a debit of 1700cc of hematic fluid. If we follow the ATLS (ATLS-Advanced Trauma Life Support 10th edition) guidelines for the care of severely polytraumatized patients, they tell us that an immediate bleeding of more than 1500cc of blood is an indication for urgent thoracotomy, and that even in patients with a bleeding of less than 1500cc, if the debit persists at a rate of 200mL/h for 2 to 4h, this indication would also exist. Therefore, the person who asked the MIR question wants us to answer option 2. However, in real life, the course of action will depend on the stability of the patient when we have applied the treatments indicated in option 2: adequate analgesia, oxygen therapy and initiation of blood transfusion. If by optimizing the patient we manage to stabilize him and assess that it is safe to transfer him, the performance of a body CT scan will define in much greater detail the lesions he has, and will help to ensure that the surgery he will probably end up undergoing will be a definitive surgery and not just damage control surgery. In this case, the doubt would arise with option 4, although, as I say, everything will depend on the clinical stability of the patient and the time we have before achieving definitive control of the hemorrhagic focus (which is, after all, what we are after in this situation).", "type": "CRITICAL AND EMERGENCY CARE", "options": {"1": "Institute noninvasive mechanical ventilation and request blood transfusion.", "2": "Analgesia, oxygen therapy, request blood transfusion and indicate urgent surgery.", "3": "Orotracheal intubation, request blood transfusion and observation for, in case of worsening, indicate surgical intervention.", "4": "Orotracheal intubation and urgent CT scan for accurate assessment of lesions.", "5": NaN}, "correct_option": 2, "explanations": {"1": {"exist": true, "char_ranges": [[119, 199]], "word_ranges": [[22, 34]], "text": "(noninvasive ventilation is not indicated because it does not secure the airway)"}, "2": {"exist": true, "char_ranges": [[1750, 1968]], "word_ranges": [[292, 327]], "text": "However, in real life, the course of action will depend on the stability of the patient when we have applied the treatments indicated in option 2: adequate analgesia, oxygen therapy and initiation of blood transfusion."}, "3": {"exist": true, "char_ranges": [[206, 307]], "word_ranges": [[36, 50]], "text": "(observation in these cases, waiting for worsening, only leads to increased morbidity and mortality)."}, "4": {"exist": true, "char_ranges": [[1969, 2299]], "word_ranges": [[327, 388]], "text": "If by optimizing the patient we manage to stabilize him and assess that it is safe to transfer him, the performance of a body CT scan will define in much greater detail the lesions he has, and will help to ensure that the surgery he will probably end up undergoing will be a definitive surgery and not just damage control surgery."}, "5": {"exist": false, "char_ranges": [], "word_ranges": [], "text": ""}}} {"id": 234, "year": 2014, "question_id_specific": 105, "full_question": "A 72-year-old man comes to the Emergency Department with a pathological fracture of the left femur. After surgery, a diagnostic study is performed to determine the underlying pathology with the following findings: hemoglobin 9.5 g/dl, total proteins 11 g/dl, (VN: 6-8 g/dl), serum albumin 2 g/dl, (VN 3.5-5.0 g/dl), beta 2 microglobulin 6 mg/l (VN 1.1-2.4 mg/l), serum creatinine 1.8 mg/dl (VN: 0.1-1.4 mg/dl). Indicate which diagnostic tests would be necessary to confirm the most likely diagnosis:", "full_answer": "She is going to have myeloma. For diagnosis, a sample (bone marrow) and electrophoresis to determine the type of monoclonal peak. Then, for extension, prognostic value and to decide treatments, the rest, but once diagnosed.", "type": "HEMATOLOGY", "options": {"1": "Radiological bone series and bone marrow aspirate.", "2": "Serum and urine electrophoresis and renal function tests.", "3": "Bone marrow aspirate and serum calcium concentration.", "4": "Bone marrow aspirate and serum and urine electrophoresis.", "5": "Biopsy of the pathological fracture and radiological bone series."}, "correct_option": 4, "explanations": {"1": {"exist": false, "char_ranges": [], "word_ranges": [], "text": ""}, "2": {"exist": false, "char_ranges": [], "word_ranges": [], "text": ""}, "3": {"exist": false, "char_ranges": [], "word_ranges": [], "text": ""}, "4": {"exist": true, "char_ranges": [[0, 129]], "word_ranges": [[0, 21]], "text": "She is going to have myeloma. For diagnosis, a sample (bone marrow) and electrophoresis to determine the type of monoclonal peak."}, "5": {"exist": false, "char_ranges": [], "word_ranges": [], "text": ""}}} {"id": 174, "year": 2013, "question_id_specific": 52, "full_question": "A 67-year-old man consults for moderate exertional dyspnea of progressive onset in recent years. He has daily whitish expectoration and sometimes breath sounds, especially in winter with respiratory infections. He has been a smoker of about 20 cigarettes a day for the last 45 years. On examination he is eupneic, normocolored, with an oxygen saturation of 94%, and has generalized decreased vesicular murmur on chest auscultation as the only findings of interest. Chest X-ray shows an elongated cardiac silhouette, with signs of hyperinflation or pulmonary air trapping, without other alterations. Spirometry was performed with the following result: FVC 84%, FEV1 58%, FEV1/FVC 61%, unchanged after bronchodilator. Among those indicated, indicate the most appropriate treatment for this patient:", "full_answer": "In a patient with a low symptomatic COPD that could probably be classified as GOLD 2, the starting treatment can be either with inhaled tiotropium or with a long-acting beta two. Therefore there is no doubt in the correct answer.", "type": "PNEUMOLOGY", "options": {"1": "Daily inhaled corticosteroid.", "2": "Home oxygen therapy with portable oxygen source for ambulation.", "3": "Oral leukotriene antagonist.", "4": "Oral corticosteroid for three months.", "5": "Inhaled tiotropium."}, "correct_option": 5, "explanations": {"1": {"exist": false, "char_ranges": [], "word_ranges": [], "text": ""}, "2": {"exist": false, "char_ranges": [], "word_ranges": [], "text": ""}, "3": {"exist": false, "char_ranges": [], "word_ranges": [], "text": ""}, "4": {"exist": false, "char_ranges": [], "word_ranges": [], "text": ""}, "5": {"exist": true, "char_ranges": [[0, 178]], "word_ranges": [[0, 31]], "text": "In a patient with a low symptomatic COPD that could probably be classified as GOLD 2, the starting treatment can be either with inhaled tiotropium or with a long-acting beta two."}}} {"id": 143, "year": 2012, "question_id_specific": 141, "full_question": "Given a 7-month-old child with fever and irritability, bulging fontanel and a cerebrospinal fluid study with 110 cells/mm3 (75% lymphocytes), protein 120 mg/dl and glucose 28 mg/dl (serum glycemia 89 mg/dl), what is the most reasonable diagnostic suspicion?", "full_answer": "The correct answer is 3. It defines a typical cerebrospinal fluid of tuberculosis. In clinical practice things might not be so simple......", "type": "PEDIATRICS", "options": {"1": "Viral meningitis.", "2": "Bacterial meningitis.", "3": "Tuberculous meningitis.", "4": "Mononucleosis syndrome.", "5": "Guillain-Barre syndrome."}, "correct_option": 3, "explanations": {"1": {"exist": false, "char_ranges": [], "word_ranges": [], "text": ""}, "2": {"exist": false, "char_ranges": [], "word_ranges": [], "text": ""}, "3": {"exist": true, "char_ranges": [[25, 82]], "word_ranges": [[5, 13]], "text": "It defines a typical cerebrospinal fluid of tuberculosis."}, "4": {"exist": false, "char_ranges": [], "word_ranges": [], "text": ""}, "5": {"exist": false, "char_ranges": [], "word_ranges": [], "text": ""}}} {"id": 9, "year": 2011, "question_id_specific": 36, "full_question": "An 87-year-old patient with a history of chronic bronchitis and heart failure, has been diagnosed with acute calculous cholecystitis . After four days of treatment with absolute diet, serum therapy and piperacillin/tazobactam, the patient continues with fever, persistent abdominal pain and leukocytosis. The most appropriate attitude at this time would be:", "full_answer": "I think the correct answer is 2 although the treatment for acute cholecystitis is cholecystectomy, for this it is necessary that the patient is a surgical candidate, in this case it is an elderly patient, with previous diseases that increase the surgical risk and would be an ASA IV for an urgent intervention. In these cases cholecystostomy can cure cholecystitis; and after recovery, assess scheduled surgery according to the patient's situation.", "type": "SURGERY", "options": {"1": "Surgical treatment (urgent cholecystectomy).", "2": "Biliary drainage by percutaneous cholecystostomy.", "3": "Substitute metronidazole + cefotaxime for piperacillin/tazobactam.", "4": "Replace piperacillin/tazobactam with amikacin+clindamycin.", "5": "Add gentamicin."}, "correct_option": 2, "explanations": {"1": {"exist": false, "char_ranges": [], "word_ranges": [], "text": ""}, "2": {"exist": true, "char_ranges": [[0, 309]], "word_ranges": [[0, 53]], "text": "I think the correct answer is 2 although the treatment for acute cholecystitis is cholecystectomy, for this it is necessary that the patient is a surgical candidate, in this case it is an elderly patient, with previous diseases that increase the surgical risk and would be an ASA IV for an urgent intervention."}, "3": {"exist": false, "char_ranges": [], "word_ranges": [], "text": ""}, "4": {"exist": false, "char_ranges": [], "word_ranges": [], "text": ""}, "5": {"exist": false, "char_ranges": [], "word_ranges": [], "text": ""}}} {"id": 601, "year": 2022, "question_id_specific": 112, "full_question": "A 61-year-old woman, administrative, with a history of overweight, hypertension, dyslipidemia and metabolic syndrome, who consults for pain in both buttocks, left trochanteric region, lateral aspect of the left thigh up to the knee and left leg up to the middle third. The pain appears when the lower limb is lifted with the knee extended, but is relieved when the knee is flexed. What is the first clinical suspicion?", "full_answer": "Positive Lasegue's sign, reappearance of the symptoms when performing the extension maneuver of the affected limb, compatible with involvement of nerve roots at the lumbosacral spine level.", "type": "TRAUMATOLOGY", "options": {"1": "Gouty arthritis of the left hip.", "2": "Left coxofemoral arthrosis.", "3": "Radiated low back pain / lumbosciatica.", "4": "Claudication due to canal stenosis.", "5": NaN}, "correct_option": 3, "explanations": {"1": {"exist": false, "char_ranges": [], "word_ranges": [], "text": ""}, "2": {"exist": false, "char_ranges": [], "word_ranges": [], "text": ""}, "3": {"exist": true, "char_ranges": [[0, 189]], "word_ranges": [[0, 27]], "text": "Positive Lasegue's sign, reappearance of the symptoms when performing the extension maneuver of the affected limb, compatible with involvement of nerve roots at the lumbosacral spine level."}, "4": {"exist": false, "char_ranges": [], "word_ranges": [], "text": ""}, "5": {"exist": false, "char_ranges": [], "word_ranges": [], "text": ""}}} {"id": 202, "year": 2013, "question_id_specific": 44, "full_question": "Patient with acute pancreatitis. Computerized Axial Tomography (CAT) performed 72 hours after admission showed necrosis of 50% of the pancreas. At the 3rd week of admission the patient began with high fever and leukocytosis. Urgent chest X-ray and urinary sediment are requested, both being normal. Indicate the next step to follow:", "full_answer": "Although the latest recommendations on the management of severe acute pancreatitis advise against FNA because of the risk of infection of sterile necrosis. Current management is governed by stepwise treatment. In the situation described in the question the current treatment is to start antibiotics and assess response.", "type": "GENERAL SURGERY", "options": {"1": "Fine needle aspiration of pancreatic necrosis guided by ultrasound or CT.", "2": "Bronchoalveolar aspirate, urine culture and blood cultures.", "3": "Abdominal MRI.", "4": "Urgent cholecystectomy.", "5": "Echoendoscopy with puncture of the necrosis."}, "correct_option": 1, "explanations": {"1": {"exist": false, "char_ranges": [], "word_ranges": [], "text": ""}, "2": {"exist": false, "char_ranges": [], "word_ranges": [], "text": ""}, "3": {"exist": false, "char_ranges": [], "word_ranges": [], "text": ""}, "4": {"exist": false, "char_ranges": [], "word_ranges": [], "text": ""}, "5": {"exist": false, "char_ranges": [], "word_ranges": [], "text": ""}}} {"id": 259, "year": 2014, "question_id_specific": 97, "full_question": "A 54-year-old man who goes for a check-up at his company. A body mass index of 32.8 kg/m2 and fasting blood glucose of 138 mg/l were detected. One month later, glycemia 13 mg/dl. Which therapeutic recommendation would you make first?", "full_answer": "Behavioral changes: diet and physical exercise. This is the first thing we do when diagnosed with type 2 diabetes. This is a good question to remember that before any drug, we must insist on changing habits. Then we would prescribe Metformin.", "type": "ENDOCRINOLOGY", "options": {"1": "Administer metformin.", "2": "Prescribe a sulfonylurea.", "3": "Behavioral changes. Diet and physical exercise.", "4": "Insulin before each meal.", "5": "Take acarbose at night, before going to bed."}, "correct_option": 3, "explanations": {"1": {"exist": false, "char_ranges": [], "word_ranges": [], "text": ""}, "2": {"exist": false, "char_ranges": [], "word_ranges": [], "text": ""}, "3": {"exist": true, "char_ranges": [[48, 114]], "word_ranges": [[6, 19]], "text": "This is the first thing we do when diagnosed with type 2 diabetes."}, "4": {"exist": false, "char_ranges": [], "word_ranges": [], "text": ""}, "5": {"exist": false, "char_ranges": [], "word_ranges": [], "text": ""}}} {"id": 204, "year": 2014, "question_id_specific": 220, "full_question": "A 43-year-old man consults for diarrheal syndrome, and refers among his history 3 pneumonias in adulthood. Which of the following immunological studies should we request:", "full_answer": "More than 2 lobar pneumonias make it necessary to rule out an immunologic deficit.", "type": "GENETICS AND IMMUNOLOGY", "options": {"1": "Serum immunoglobulin count and antibody production capacity test.", "2": "Phagocytosis test and oxidative metabolism of neutrophils.", "3": "Apoptosis test (programmed cell death) in the patient's circulating lymphocytes.", "4": "Study of the repertoire and clonality of T lymphocytes (alpha/beta).", "5": "In this patient it would not be appropriate to request any immunological study."}, "correct_option": 1, "explanations": {"1": {"exist": true, "char_ranges": [[0, 82]], "word_ranges": [[0, 14]], "text": "More than 2 lobar pneumonias make it necessary to rule out an immunologic deficit."}, "2": {"exist": false, "char_ranges": [], "word_ranges": [], "text": ""}, "3": {"exist": false, "char_ranges": [], "word_ranges": [], "text": ""}, "4": {"exist": false, "char_ranges": [], "word_ranges": [], "text": ""}, "5": {"exist": false, "char_ranges": [], "word_ranges": [], "text": ""}}} {"id": 4, "year": 2011, "question_id_specific": 40, "full_question": "A man presents abruptly with asthenia and jaundice with transaminases above 2000 IU/L. Serological markers show the following pattern: anti-HAV IgM negative, HBsAg negative anti-HBc IgM positive, anti-HCV negative. What is the diagnosis?", "full_answer": "An interesting one that in general needs to be studied. By the form of presentation, it is clearly an acute hepatitis, so option 1 is ruled out. The negative IgM of HAV rules out 5 and the negative HBs Ag rules out 4 (to have delta virus you need surface antigen B). When having IgM anti HBc we speak of an acute hepatitis B (3), since there is a window period of negativization of the antigen.", "type": "DIGESTIVE", "options": {"1": "Chronic hepatitis B.", "2": "Non-viral acute hepatitis.", "3": "Acute hepatitis B.", "4": "D (delta) virus superinfection.", "5": "Acute hepatitis A and B."}, "correct_option": 3, "explanations": {"1": {"exist": true, "char_ranges": [[56, 143]], "word_ranges": [[10, 27]], "text": "By the form of presentation, it is clearly an acute hepatitis, so option 1 is ruled out."}, "2": {"exist": false, "char_ranges": [], "word_ranges": [], "text": ""}, "3": {"exist": true, "char_ranges": [[267, 394]], "word_ranges": [[52, 76]], "text": "When having IgM anti HBc we speak of an acute hepatitis B (3), since there is a window period of negativization of the antigen."}, "4": {"exist": true, "char_ranges": [[186, 265]], "word_ranges": [[36, 52]], "text": "the negative HBs Ag rules out 4 (to have delta virus you need surface antigen B)."}, "5": {"exist": true, "char_ranges": [[145, 180]], "word_ranges": [[27, 35]], "text": "The negative IgM of HAV rules out 5"}}} {"id": 372, "year": 2016, "question_id_specific": 127, "full_question": "55-year-old man who consults for dysphonia. In the anamnesis he refers to a month of asthenia and unquantified weight loss. Chest X-ray shows increased density in the left upper lobe and occupation of the aortopulmonary window. Bronchoscopy shows paralysis of the left vocal cord, without endoscopic image suggestive of neoplasia. What is the most likely diagnosis?", "full_answer": "The most frequent cause of recurrent nerve palsy is bronchopulmonary carcinoma. The fact that it is not accessible to endoscopic vision does not mean that it is not there.", "type": "PNEUMOLOGY AND THORACIC SURGERY", "options": {"1": "Pulmonary neoplasia.", "2": "Sarcoidosis.", "3": "Silicosis.", "4": "Tuberculosis.", "5": NaN}, "correct_option": 1, "explanations": {"1": {"exist": true, "char_ranges": [[0, 79]], "word_ranges": [[0, 11]], "text": "The most frequent cause of recurrent nerve palsy is bronchopulmonary carcinoma."}, "2": {"exist": false, "char_ranges": [], "word_ranges": [], "text": ""}, "3": {"exist": false, "char_ranges": [], "word_ranges": [], "text": ""}, "4": {"exist": false, "char_ranges": [], "word_ranges": [], "text": ""}, "5": {"exist": false, "char_ranges": [], "word_ranges": [], "text": ""}}} {"id": 377, "year": 2016, "question_id_specific": 131, "full_question": "A 65-year-old female patient debuted two years earlier with an apraxia of speech and has developed rigid-akinetic parkinsonism predominantly in the right hemibody with superimposed myoclonias and alien hand or foreign limb phenomenon.What diagnosis do you consider most likely?", "full_answer": "As it is described as an apraxic phenomenon followed by an asymmetric rigid-akinetic parkinsonism and with superimposed myoclonias, it is typical of corticobasal degeneration. Parkinson's disease cannot be because they do not present apraxia as initial feature, Alzheimer's disease does not present an asymmetric parkinsonism and the typical Huntinton's disease is not of so old people and when it debuts with advanced age it behaves like a chorea.", "type": "NEUROLOGY", "options": {"1": "Parkinson's disease.", "2": "Corticobasal degeneration.", "3": "Alzheimer's disease.", "4": "Huntington's disease.", "5": NaN}, "correct_option": 2, "explanations": {"1": {"exist": true, "char_ranges": [[176, 261]], "word_ranges": [[24, 37]], "text": "Parkinson's disease cannot be because they do not present apraxia as initial feature,"}, "2": {"exist": true, "char_ranges": [[0, 175]], "word_ranges": [[0, 24]], "text": "As it is described as an apraxic phenomenon followed by an asymmetric rigid-akinetic parkinsonism and with superimposed myoclonias, it is typical of corticobasal degeneration."}, "3": {"exist": true, "char_ranges": [[262, 325]], "word_ranges": [[37, 45]], "text": "Alzheimer's disease does not present an asymmetric parkinsonism"}, "4": {"exist": true, "char_ranges": [[330, 448]], "word_ranges": [[46, 68]], "text": "the typical Huntinton's disease is not of so old people and when it debuts with advanced age it behaves like a chorea."}, "5": {"exist": false, "char_ranges": [], "word_ranges": [], "text": ""}}} {"id": 145, "year": 2012, "question_id_specific": 143, "full_question": "A 1 month old infant, exclusively breastfed, consults because he has a bowel movement every 5 or 6 days with straining but with a soft consistency, what would our attitude be?", "full_answer": "The correct answer is 4. The normal range of stool output in young infants is very wide. To suspect aganglionic megacolon there is usually a delay in the evacuation of meconium and often other associated symptoms such as abdominal distension, difficulty in gaining weight, etc.", "type": "PEDIATRICS", "options": {"1": "Start laxative treatment.", "2": "Supplement breastfeeding with anti-constipation formula.", "3": "Start daily rectal stimulation.", "4": "It is considered normal intestinal rhythm.", "5": "Refer to Pediatric Digestive to rule out aganglionic megacolon."}, "correct_option": 4, "explanations": {"1": {"exist": false, "char_ranges": [], "word_ranges": [], "text": ""}, "2": {"exist": false, "char_ranges": [], "word_ranges": [], "text": ""}, "3": {"exist": false, "char_ranges": [], "word_ranges": [], "text": ""}, "4": {"exist": true, "char_ranges": [[25, 88]], "word_ranges": [[5, 17]], "text": "The normal range of stool output in young infants is very wide."}, "5": {"exist": true, "char_ranges": [[89, 277]], "word_ranges": [[17, 45]], "text": "To suspect aganglionic megacolon there is usually a delay in the evacuation of meconium and often other associated symptoms such as abdominal distension, difficulty in gaining weight, etc."}}} {"id": 567, "year": 2022, "question_id_specific": 150, "full_question": "A 47-year-old woman consults for voiding syndrome. She is diagnosed with uncomplicated urinary tract infection and treated with ciprofloxacin for 5 days. A week later, she consulted again for general malaise, arthralgias and the appearance of a skin rash. In the analytical analysis, the glomerular filtration rate was 45 ml/min (CKD-EPI), whereas 6 months ago it was 100 ml/min. In the sediment leukocytes are identified, being negative for nitrites. Of the following, which is the most probable cause of his renal failure?", "full_answer": "General malaise, arthralgias, skin rash, acute renal failure and active sediment with sterile leukocyturia following the intake of a drug is highly suggestive of AIN (option 4 correct). Among drugs, antibiotics are the most frequent cause, and ciprofloxacin is one of the most common. Direct nephrotoxicity is very rare and is characterized by crystallization in the renal tubules (incorrect option 2). Postinfectious glomerulonephritis is more typical of children and adults older than 60 years, and usually manifests with hematuria after streptococcal infections (option 1 incorrect). Acute pyelonephritis would have been presented with high fever and renal fossa pain (option 3 incorrect).", "type": "NEPHROLOGY", "options": {"1": "Postinfectious glomerulonephritis.", "2": "Ciprofloxacin nephrotoxicity.", "3": "Acute pyelonephritis.", "4": "Acute interstitial nephritis.", "5": NaN}, "correct_option": 4, "explanations": {"1": {"exist": true, "char_ranges": [[403, 586]], "word_ranges": [[61, 85]], "text": "Postinfectious glomerulonephritis is more typical of children and adults older than 60 years, and usually manifests with hematuria after streptococcal infections (option 1 incorrect)."}, "2": {"exist": true, "char_ranges": [[285, 402]], "word_ranges": [[44, 61]], "text": "Direct nephrotoxicity is very rare and is characterized by crystallization in the renal tubules (incorrect option 2)."}, "3": {"exist": true, "char_ranges": [[587, 692]], "word_ranges": [[85, 101]], "text": "Acute pyelonephritis would have been presented with high fever and renal fossa pain (option 3 incorrect)."}, "4": {"exist": true, "char_ranges": [[0, 185]], "word_ranges": [[0, 28]], "text": "General malaise, arthralgias, skin rash, acute renal failure and active sediment with sterile leukocyturia following the intake of a drug is highly suggestive of AIN (option 4 correct)."}, "5": {"exist": false, "char_ranges": [], "word_ranges": [], "text": ""}}} {"id": 177, "year": 2013, "question_id_specific": 93, "full_question": "A 30-year-old woman, asymptomatic, is found to be anemic on routine examination. Physical examination reveals conjunctival jaundice and splenomegaly. The patient refers to a family history of biliary lithiasis at an early age. All this suggests the most probable diagnosis of:", "full_answer": "You have probably looked at answer 5, hereditary spherocytosis, associated with biliary lithiasis at an early age. Exactly, that is the correct one. In the statement they have not stopped giving clues: conjunctival jaundice, splenomegaly and family history of biliary lithiasis at an early age. He has reached 30 years of age without suffering a hemolytic crisis, without suffering any symptoms. There was no mention of microcytosis or macrocytosis to suggest thalassemia minor or B12 and/or folic acid deficiency. Nor of hemolytic crisis associated with the consumption of drugs, infections or ingestion of beans or peas. They have not given any clues to stop to think about other possibilities.", "type": "HEMATOLOGY", "options": {"1": "Glucose 6-phosphate dehydrogenase deficiency.", "2": "Thalassemia minor.", "3": "Vitamin Bl2 and/or folic acid deficiency.", "4": "Familial pyruvate kinase deficiency.", "5": "Hereditary spherocytosis."}, "correct_option": 5, "explanations": {"1": {"exist": false, "char_ranges": [], "word_ranges": [], "text": ""}, "2": {"exist": false, "char_ranges": [], "word_ranges": [], "text": ""}, "3": {"exist": false, "char_ranges": [], "word_ranges": [], "text": ""}, "4": {"exist": false, "char_ranges": [], "word_ranges": [], "text": ""}, "5": {"exist": true, "char_ranges": [[202, 622]], "word_ranges": [[32, 96]], "text": "conjunctival jaundice, splenomegaly and family history of biliary lithiasis at an early age. He has reached 30 years of age without suffering a hemolytic crisis, without suffering any symptoms. There was no mention of microcytosis or macrocytosis to suggest thalassemia minor or B12 and/or folic acid deficiency. Nor of hemolytic crisis associated with the consumption of drugs, infections or ingestion of beans or peas."}}} {"id": 125, "year": 2012, "question_id_specific": 105, "full_question": "19-year-old woman, weight 60 kg, with acute dehydration due to long exposure to the sun. Blood pressure lying down 100/60 mmHg. Standing 70/50 mmHg with dizziness. Serum sodium levels 155 mmol/L. What is the most correct treatment, in the first 24 hours, taking into account the data available to us?", "full_answer": "This question is not an easy one. The available literature is somewhat controversial (some guidelines advise using glucose saline, others absolutely forbid it...) so I have decided to rely on the treatment recommended by Harrison, which is the manual that MIR examiners usually rely on. This girl presents with hypernatremia due to extrarenal free water losses. The goal is to replace the lost water and reduce the natremia at a rate of no more than 12 mmol/day. To calculate fluid therapy, the water deficit must first be calculated. Water def. Water = (Natremia-140/140)xTotal body water. TBW = Weight in kg x 0.4 in females/0.5 in males. In this case, the water deficit is: ([[tel:155-140/140|155-140/140]])x60x0.4 = 2.57 liters. There are only two answers that come close to this figure, but 2 seems more correct, as it is the only one that completely covers the water deficit and uses a hyposodium solution.", "type": "NEPHROLOGY", "options": {"1": "Hypertonic saline (3%), 500 ml + 500 ml of 5% glucose.", "2": "Hyposaline saline (0.45%), 3000 ml.", "3": "Serum glucose 5% 1000 ml.", "4": "Oral hydration with 1 liter of water.", "5": "Isotonic saline (0.9%), 2000 ml."}, "correct_option": 2, "explanations": {"1": {"exist": false, "char_ranges": [], "word_ranges": [], "text": ""}, "2": {"exist": true, "char_ranges": [[641, 912]], "word_ranges": [[106, 150]], "text": "In this case, the water deficit is: ([[tel:155-140/140|155-140/140]])x60x0.4 = 2.57 liters. There are only two answers that come close to this figure, but 2 seems more correct, as it is the only one that completely covers the water deficit and uses a hyposodium solution."}, "3": {"exist": false, "char_ranges": [], "word_ranges": [], "text": ""}, "4": {"exist": false, "char_ranges": [], "word_ranges": [], "text": ""}, "5": {"exist": true, "char_ranges": [[641, 912]], "word_ranges": [[106, 150]], "text": "In this case, the water deficit is: ([[tel:155-140/140|155-140/140]])x60x0.4 = 2.57 liters. There are only two answers that come close to this figure, but 2 seems more correct, as it is the only one that completely covers the water deficit and uses a hyposodium solution."}}} {"id": 510, "year": 2021, "question_id_specific": 151, "full_question": "A 43-year-old male with no past history of interest consults for a ten-day history of jaundice of the skin and mucous membranes, choluria and acholia. He does not report abdominal pain, weight loss or other symptoms, except generalized pruritus. She denies alcohol consumption. She does not take any medication except occasional ibuprofen for muscle pain after sports activities. The CBC shows increased bilirubin at the expense of direct bilirubin. An urgent ultrasound shows a normal liver, vesicular cholesterosis and absence of bile duct dilatation, with no other alterations of interest. What is the most likely diagnosis?", "full_answer": "We believe that this question is open to challenge. Ibuprofen hepatitis is a rare adverse effect with a prevalence of about 3.7/100,000 users and 1.1/100,000 prescriptions. It appears that it may be related to dose and duration of treatment, but there is little evidence on this. Among the NSAIDs with the highest risk are diclofenac and sulindac. In terms of frequency, the prevalence of cholangiocarcinoma is 2/100,000 patients, a very comparable frequency. The absence of bile duct dilatation in cholangiocarcinomas can be seen in cases of intrahepatic cholangiocarcinomas or in those with mixed histology with hepatocarcinoma. It is true that the acute course and the absence of constitutional syndrome may make us think against this option and tip the balance towards ibuprofen hepatitis. Regarding the other options: acute cholangitis is characterized by the triad of jaundice, abdominal pain and fever; Gilbert's syndrome is an increase in bilirubin at the expense of indirectness.", "type": "DIGESTIVE", "options": {"1": "Acute cholangitis due to biliary mud.", "2": "Toxic hepatitis due to ibuprofen.", "3": "Cholangiocarcinoma.", "4": "Gilbert's syndrome.", "5": NaN}, "correct_option": 2, "explanations": {"1": {"exist": true, "char_ranges": [[823, 909]], "word_ranges": [[127, 140]], "text": "acute cholangitis is characterized by the triad of jaundice, abdominal pain and fever;"}, "2": {"exist": true, "char_ranges": [[631, 793]], "word_ranges": [[96, 123]], "text": "It is true that the acute course and the absence of constitutional syndrome may make us think against this option and tip the balance towards ibuprofen hepatitis."}, "3": {"exist": true, "char_ranges": [[631, 793]], "word_ranges": [[96, 123]], "text": "It is true that the acute course and the absence of constitutional syndrome may make us think against this option and tip the balance towards ibuprofen hepatitis."}, "4": {"exist": true, "char_ranges": [[910, 988]], "word_ranges": [[140, 152]], "text": "Gilbert's syndrome is an increase in bilirubin at the expense of indirectness."}, "5": {"exist": false, "char_ranges": [], "word_ranges": [], "text": ""}}} {"id": 268, "year": 2014, "question_id_specific": 140, "full_question": "A 50-year-old patient presenting with a pleural effusion with the following features: straw-like appearance, pH 7.3, pleural protein/serum ratio 0.8, pleural LDH/serum ratio 0.9, Gram and Ziehl's negative, total lipids, cholesterol and triglycerides normal, mesothelial cells <5%, intense lymphocytosis without atypia, ADA 64 U/L. What diagnosis would you suggest?", "full_answer": "It is a lymphocytic exudate whose main diagnoses are tumor and tuberculous pleuritis. They do not give the value of glucose which should be lowered, but if the cytology shows absence of malignant cells so we are left with tuberculous pleuritis. Other possible diagnoses such as lymphoma are not mentioned in the answers.", "type": "PULMONOLOGY", "options": {"1": "Pleural empyema.", "2": "Pleural effusion due to heart failure (transudate).", "3": "Pleural mesothelioma.", "4": "Tuberculous pleural effusion.", "5": "Effusion secondary to pulmonary infarction."}, "correct_option": 4, "explanations": {"1": {"exist": false, "char_ranges": [], "word_ranges": [], "text": ""}, "2": {"exist": false, "char_ranges": [], "word_ranges": [], "text": ""}, "3": {"exist": false, "char_ranges": [], "word_ranges": [], "text": ""}, "4": {"exist": true, "char_ranges": [[0, 244]], "word_ranges": [[0, 41]], "text": "It is a lymphocytic exudate whose main diagnoses are tumor and tuberculous pleuritis. They do not give the value of glucose which should be lowered, but if the cytology shows absence of malignant cells so we are left with tuberculous pleuritis."}, "5": {"exist": false, "char_ranges": [], "word_ranges": [], "text": ""}}} {"id": 465, "year": 2020, "question_id_specific": 116, "full_question": "In a subject over 65 years of age a tuberculin test has shown an induration of 3 mm. The induration on a second test, performed 10 days later, is 13 mm. Point out the correct answer:", "full_answer": "Let's see, if that subject was not in any risk group, the positive is in the 15mm, therefore both tests are negative, and as only one answer refers to negative, that's it. But both could be false negatives.", "type": "BIOSTATISTICS", "options": {"1": "The first reaction is a false positive.", "2": "The second reaction is a true positive.", "3": "The first reaction is a true negative.", "4": "The second reaction is a false positive.", "5": NaN}, "correct_option": 3, "explanations": {"1": {"exist": false, "char_ranges": [], "word_ranges": [], "text": ""}, "2": {"exist": false, "char_ranges": [], "word_ranges": [], "text": ""}, "3": {"exist": true, "char_ranges": [[0, 171]], "word_ranges": [[0, 32]], "text": "Let's see, if that subject was not in any risk group, the positive is in the 15mm, therefore both tests are negative, and as only one answer refers to negative, that's it."}, "4": {"exist": false, "char_ranges": [], "word_ranges": [], "text": ""}, "5": {"exist": false, "char_ranges": [], "word_ranges": [], "text": ""}}} {"id": 340, "year": 2016, "question_id_specific": 33, "full_question": "A 67-year-old woman diagnosed with infiltrating ductal carcinoma of the breast with no family history of neoplasia. What additional studies should be performed on the tumor because of its clinical and therapeutic implications?", "full_answer": "The answer is 2, study of hormone receptors and HER2. This is because no study of first-degree relatives is appropriate, as it does not appear to be hereditary breast cancer since there is no family history of neoplasia. On the other hand, there is no indication for BRCA 1-2 study.", "type": "GYNECOLOGY AND OBSTETRICS", "options": {"1": "Complete phenotypic study by flow cytometry.", "2": "Study of hormone receptors and HER2.", "3": "Study of hormone receptors, e-cadherin and study of first degree relatives.", "4": "Study of BRCA 1-2 and study of first-degree relatives.", "5": NaN}, "correct_option": 2, "explanations": {"1": {"exist": false, "char_ranges": [], "word_ranges": [], "text": ""}, "2": {"exist": false, "char_ranges": [], "word_ranges": [], "text": ""}, "3": {"exist": true, "char_ranges": [[70, 220]], "word_ranges": [[13, 38]], "text": "no study of first-degree relatives is appropriate, as it does not appear to be hereditary breast cancer since there is no family history of neoplasia."}, "4": {"exist": true, "char_ranges": [[70, 220]], "word_ranges": [[13, 38]], "text": "no study of first-degree relatives is appropriate, as it does not appear to be hereditary breast cancer since there is no family history of neoplasia."}, "5": {"exist": false, "char_ranges": [], "word_ranges": [], "text": ""}}} {"id": 561, "year": 2022, "question_id_specific": 180, "full_question": "A 26-year-old woman diagnosed with systemic lupus erythematosus, on treatment with hydroxychloroquine, consults for a feeling of generalized weakness that has progressively developed in the last 15 days. The physical examination reveals cutaneous pallor and the CBC shows Hb 7.4 g/dL, Hct 31%, MCV 108. Which of the following tests will be most useful in deciding the course of action?", "full_answer": "It tells us about a patient with a diagnosis of lupus and macrocytic anemia. In the end, all the data are usually given for a reason. The onset of the picture seems relatively rapid and it is also associated with an autoimmune disease, so it seems that they want you to associate it with an autoimmune hemolytic anemia. The low haptoglobin would tell us it is \"hemolytic\", the coombs test would tell us it is \"autoimmune\". Vitamin B12, probably the MCV would be higher than 108 and the onset of the clinical picture would be slower. The ANA would not give us anything in relation to the anemia, since it already has a diagnosis. Therefore, correct answer 2. As a note, autoimmune hemolytic anemias are usually slightly macrocytic due to reticulocytosis. Remember to ask for reticulocytes.", "type": "HEMATOLOGY", "options": {"1": "Haptoglobin.", "2": "Coombs test.", "3": "Vitamin B12.", "4": "Antinuclear antibodies.", "5": NaN}, "correct_option": 2, "explanations": {"1": {"exist": false, "char_ranges": [], "word_ranges": [], "text": ""}, "2": {"exist": true, "char_ranges": [[134, 422]], "word_ranges": [[26, 76]], "text": "The onset of the picture seems relatively rapid and it is also associated with an autoimmune disease, so it seems that they want you to associate it with an autoimmune hemolytic anemia. The low haptoglobin would tell us it is \"hemolytic\", the coombs test would tell us it is \"autoimmune\"."}, "3": {"exist": true, "char_ranges": [[423, 532]], "word_ranges": [[76, 96]], "text": "Vitamin B12, probably the MCV would be higher than 108 and the onset of the clinical picture would be slower."}, "4": {"exist": true, "char_ranges": [[533, 628]], "word_ranges": [[96, 114]], "text": "The ANA would not give us anything in relation to the anemia, since it already has a diagnosis."}, "5": {"exist": false, "char_ranges": [], "word_ranges": [], "text": ""}}} {"id": 451, "year": 2018, "question_id_specific": 175, "full_question": "A 36-year-old man with no PA of interest comes to the emergency department with sudden onset of severe pain in the posterior aspect of the right lower limb extending to the foot. On examination he presented: Positive right Lasège at 10º, decreased strength in plantar flexion of the right foot, hypoesthesia in the external border of the right foot and absence of right Achilles reflex. The simple X-ray of the lumbar spine does not show significant alterations. Which of the following is the most probable diagnosis?", "full_answer": "Acute sciatica probably due to disc extrusion. Both the sensory distribution, the paresis for plantar flexion (tiptoe) and the absence of the Achilles reflex are typical of S1 root involvement, which is typically affected with L5/S1 disc herniation (posterolateral). An L1/L2 (posterolateral) herniation would affect the L2 root, an L4/L5 herniation would affect the L5 and cauda equina would involve more symptomatology than simple S1 involvement.", "type": "NEUROSURGERY", "options": {"1": "Herniated disc L1/L2.", "2": "Horsetail syndrome.", "3": "L4/L5 disc herniation.", "4": "Herniated disc L5/S1.", "5": NaN}, "correct_option": 4, "explanations": {"1": {"exist": true, "char_ranges": [[267, 329]], "word_ranges": [[39, 48]], "text": "An L1/L2 (posterolateral) herniation would affect the L2 root,"}, "2": {"exist": true, "char_ranges": [[374, 448]], "word_ranges": [[56, 66]], "text": "cauda equina would involve more symptomatology than simple S1 involvement."}, "3": {"exist": true, "char_ranges": [[330, 369]], "word_ranges": [[48, 55]], "text": "an L4/L5 herniation would affect the L5"}, "4": {"exist": true, "char_ranges": [[0, 266]], "word_ranges": [[0, 39]], "text": "Acute sciatica probably due to disc extrusion. Both the sensory distribution, the paresis for plantar flexion (tiptoe) and the absence of the Achilles reflex are typical of S1 root involvement, which is typically affected with L5/S1 disc herniation (posterolateral)."}, "5": {"exist": false, "char_ranges": [], "word_ranges": [], "text": ""}}} {"id": 399, "year": 2016, "question_id_specific": 141, "full_question": "20-year-old male, who consults for lumbosacral pain of inflammatory rhythm of 4 months of evolution. Also bilateral thalalgia and morning stiffness for 1 hour. In the last 2 months, onset of diarrhea with loss of 4 kg of weight. What is the most correct diagnostic approach?", "full_answer": "The patient presents the clinical features of spondylarthropathy, which is often associated with inflammatory bowel disease, as this case suggests.", "type": "TRAUMATOLOGY AND ORTHOPEDICS", "options": {"1": "Given the patient's age, he most likely suffers from non-specific low back pain and tendonitis of the feet. If the diarrhea persists, a digestive study would be performed.", "2": "Perform a digestive study to rule out tumor pathology. Low back pain may be due to visceral pathology.", "3": "The clinical picture is very suggestive of spondyloarthritis. Inflammatory bowel disease should be ruled out.", "4": "I would request a lumbar MRI to rule out disc herniation and if diarrhea persists, a digestive study.", "5": NaN}, "correct_option": 3, "explanations": {"1": {"exist": false, "char_ranges": [], "word_ranges": [], "text": ""}, "2": {"exist": false, "char_ranges": [], "word_ranges": [], "text": ""}, "3": {"exist": true, "char_ranges": [[0, 147]], "word_ranges": [[0, 20]], "text": "The patient presents the clinical features of spondylarthropathy, which is often associated with inflammatory bowel disease, as this case suggests."}, "4": {"exist": false, "char_ranges": [], "word_ranges": [], "text": ""}, "5": {"exist": false, "char_ranges": [], "word_ranges": [], "text": ""}}} {"id": 12, "year": 2011, "question_id_specific": 92, "full_question": "A 30-year-old man suffers a fall from a standing height of 2 meters. On arrival at the emergency room on a stretcher, he is conscious and oriented, with bladder catheterization and clear urine. He shows swelling of the left heel and mild tingling in the anterior aspect of the left thigh and scrotum as well as dysesthesia in both feet. The patient says goodbye to the stretcher-bearer with a handshake and a simultaneous pat on the shoulder. Rotation of the lower limbs on the stretcher grasped by the ankles is painless and compression of the pelvis is asymptomatic. Next:", "full_answer": "I believe that the correct answer is 3. The patient, due to the mechanism of injury (foot fall), the calcaneal fracture and the neurological symptoms, suggests a low spinal cord injury due to a fracture of the lumbar spine. The injury is not high because he moves his arms. It also does not appear to be a pelvic injury because of the absence of pain on mobilization or palpation. If there is a suspicion of pelvic injury in the primary evaluation of all polytraumatized (ATLS) a spinal exploration should be performed by rotating the spine en bloc to avoid producing more injury. Abdominal palpation is also part of the initial evaluation. Abdominal ultrasound may be indicated in hemodynamically unstable patients if abdominal injuries are suspected but in any case it is done after the initial evaluation.", "type": "SURGERY", "options": {"1": "The patient will be seated on the stretcher so that the cervical spine can be explored.", "2": "Refer to radiology for study of the lower extremities (feet, femurs, pelvis).", "3": "We will palpate the abdomen and roll en bloc to palpate the thoracolumbar spine.", "4": "We will place a cervical collar and refer to radiology for imaging of the lower extremities and cervical spine.", "5": "We will request an abdominal ultrasound."}, "correct_option": 3, "explanations": {"1": {"exist": false, "char_ranges": [], "word_ranges": [], "text": ""}, "2": {"exist": false, "char_ranges": [], "word_ranges": [], "text": ""}, "3": {"exist": true, "char_ranges": [[0, 223]], "word_ranges": [[0, 39]], "text": "I believe that the correct answer is 3. The patient, due to the mechanism of injury (foot fall), the calcaneal fracture and the neurological symptoms, suggests a low spinal cord injury due to a fracture of the lumbar spine."}, "4": {"exist": false, "char_ranges": [], "word_ranges": [], "text": ""}, "5": {"exist": false, "char_ranges": [], "word_ranges": [], "text": ""}}} {"id": 605, "year": 2022, "question_id_specific": 113, "full_question": "35-year-old male, letter carrier, with no history of interest, who comes to the emergency department for acute cervical pain of 24 hours of evolution, without previous trauma, which radiates to the left arm up to the hand and is accompanied by paresthesia in the radial border of the forearm. She presents no objectifiable loss of strength, preserves neck mobility although it is painful and contracture of the paravertebral musculature is appreciated. The first attitude will be:", "full_answer": "In this case we describe a case of cervicobrachialgia without alarm signs at the present time, so the most indicated treatment at this time would be conservative, based on NSAIDs, local heat and rest. This is not an urgent case, so answer 2 would not be valid. The performance of an MRI and an electromyogram are valid options for the etiological study of the picture, however, it is necessary to emphasize that we should first perform a correct pain management, so since we are asked about the first attitude, option 1 would be more valid.", "type": "TRAUMATOLOGY", "options": {"1": "Conservative treatment with non-steroidal anti-inflammatory drugs, local heat and relative rest.", "2": "Urgent call to the neurosurgeon for surgical evaluation.", "3": "Preferential request for MRI and electromyogram.", "4": "Preferential referral to traumatology outpatients.", "5": NaN}, "correct_option": 1, "explanations": {"1": {"exist": true, "char_ranges": [[0, 200]], "word_ranges": [[0, 34]], "text": "In this case we describe a case of cervicobrachialgia without alarm signs at the present time, so the most indicated treatment at this time would be conservative, based on NSAIDs, local heat and rest."}, "2": {"exist": true, "char_ranges": [[201, 260]], "word_ranges": [[34, 47]], "text": "This is not an urgent case, so answer 2 would not be valid."}, "3": {"exist": true, "char_ranges": [[261, 540]], "word_ranges": [[47, 95]], "text": "The performance of an MRI and an electromyogram are valid options for the etiological study of the picture, however, it is necessary to emphasize that we should first perform a correct pain management, so since we are asked about the first attitude, option 1 would be more valid."}, "4": {"exist": false, "char_ranges": [], "word_ranges": [], "text": ""}, "5": {"exist": false, "char_ranges": [], "word_ranges": [], "text": ""}}} {"id": 443, "year": 2018, "question_id_specific": 102, "full_question": "A 64-year-old woman, without relevant comorbidities, operated on for a right breast cancer by means of an extended lumpectomy and selective sentinel lymph node biopsy, with the following anatomopathological result: Infiltrating ductal carcinoma poorly dlferentiated (grade 3), size 2.2 cm (pT2), estrogen and progesterone receptor positive, Ki67 25%, HER2 negative. There is macroscopic involvement of the sentinel lymph node, although there is no involvement of the rest of the isolated axillary nodes (pN1). What adjuvant systemic treatment do you consider most appropriate for this patient?", "full_answer": "It is a Luminal B. According to the SEGO, this patient is a candidate for chemotherapy treatment because of the high histologic grade and high tumor proliferation.", "type": "GYNECOLOGY AND OBSTETRICS", "options": {"1": "Adjuvant hormone therapy including aromatase inhibitors.", "2": "Adjuvant chemotherapy, preferably with anthracyclines and taxanes, followed by hormone therapy.", "3": "Adjuvant chemotherapy in association with the monoclonal antibody trastuzumab, followed by hormone therapy.", "4": "Adjuvant chemotherapy, preferably with anthracyclines and taxanes.", "5": NaN}, "correct_option": 2, "explanations": {"1": {"exist": false, "char_ranges": [], "word_ranges": [], "text": ""}, "2": {"exist": true, "char_ranges": [[0, 163]], "word_ranges": [[0, 27]], "text": "It is a Luminal B. According to the SEGO, this patient is a candidate for chemotherapy treatment because of the high histologic grade and high tumor proliferation."}, "3": {"exist": false, "char_ranges": [], "word_ranges": [], "text": ""}, "4": {"exist": false, "char_ranges": [], "word_ranges": [], "text": ""}, "5": {"exist": false, "char_ranges": [], "word_ranges": [], "text": ""}}} {"id": 197, "year": 2013, "question_id_specific": 71, "full_question": "A 10-year-old boy with brief episodes of distractibility (< 1 minute) in which he does not respond to calls and blinks. An EEG shows spike-wave discharges at 3 cycles per second. First-line elective treatment would be with:", "full_answer": "Absences behave as brief episodes of abrupt loss of consciousness without alteration of postural control; they typically last seconds and can be repeated many times a day, are usually accompanied by small bilateral motor signs (blinking, chewing) and consciousness is recovered equally abruptly, without subsequent confusion or memory of the episode (MIR 03-04, 248; MIR 97-98, 51). The age of onset is usually between 4 years and early adolescence, being the most frequent cause of seizures in this age range. EEG findings are typically generalized and symmetrical spike-wave discharges at 3 Hz (MIR 99-00F, 67) coinciding with seizures, although in the interictal EEG there are more periods of abnormal activity than clinically visible. The most appropriate drugs for the treatment of absences are Valproate and Ethosuximide (CTO Manual).", "type": "NEUROLOGY", "options": {"1": "Valproate.", "2": "Carbamazepine.", "3": "Phenytoin.", "4": "Gabapentin.", "5": "Clonazepam."}, "correct_option": 1, "explanations": {"1": {"exist": true, "char_ranges": [[739, 840]], "word_ranges": [[113, 128]], "text": "The most appropriate drugs for the treatment of absences are Valproate and Ethosuximide (CTO Manual)."}, "2": {"exist": false, "char_ranges": [], "word_ranges": [], "text": ""}, "3": {"exist": false, "char_ranges": [], "word_ranges": [], "text": ""}, "4": {"exist": false, "char_ranges": [], "word_ranges": [], "text": ""}, "5": {"exist": false, "char_ranges": [], "word_ranges": [], "text": ""}}} {"id": 478, "year": 2020, "question_id_specific": 134, "full_question": "35-year-old male, transferred to the emergency department after a traffic accident. On admission, a Glasgow scale score of 15, blood pressure of 140/90 mmHg, respiratory rate of 35 rpm and heart rate of 110 bpm, with a baseline oxygen saturation of 91%. On examination there is right thoracic hypophonesis and tympanism on percussion.What is the presumptive diagnosis? :", "full_answer": "This is an easy and classic question that you can also apply to daily clinical practice. Hypophonesis in a hemithorax + tympanism = pneumothorax. If there is dullness on percussion: hemothorax. In either case, in case of hypophonesis of a hemithorax in a severe and unstable polytraumatized patient: thoracic drainage (if possible, large caliber if placed in a hospital environment, since although the predominant lesion is pneumothorax, a hemothorax component can be found which, in case of placing a fine drainage, could obstruct it).", "type": "CRITICAL CARE", "options": {"1": "Traumatic cardiac tamponade.", "2": "Right tension pneumothorax.", "3": "Massive right hemothorax.", "4": "Pulmonary contusion.", "5": NaN}, "correct_option": 2, "explanations": {"1": {"exist": false, "char_ranges": [], "word_ranges": [], "text": ""}, "2": {"exist": true, "char_ranges": [[89, 145]], "word_ranges": [[16, 24]], "text": "Hypophonesis in a hemithorax + tympanism = pneumothorax."}, "3": {"exist": true, "char_ranges": [[146, 193]], "word_ranges": [[24, 31]], "text": "If there is dullness on percussion: hemothorax."}, "4": {"exist": false, "char_ranges": [], "word_ranges": [], "text": ""}, "5": {"exist": false, "char_ranges": [], "word_ranges": [], "text": ""}}} {"id": 97, "year": 2012, "question_id_specific": 171, "full_question": "Patient presenting since 2 weeks ago with a pruritic eruption consisting of multiple polygonal erythematous-violaceous papules, flattened surface, preferentially located on the anterior aspect of wrists, pretibial area and lumbar area. She also presents whitish reticulated lesions in the jugal mucosa. What is the most probable diagnosis?", "full_answer": "They are undoubtedly describing the typical lesions (both on skin and oral mucosa) of a lichen planus (5).", "type": "DERMATOLOGY, VENEREOLOGY AND PLASTIC SURGERY", "options": {"1": "Pityriasis versicolor.", "2": "Mycosis fungoides.", "3": "Psoriasis in drops.", "4": "Pityriasis rosea of Gibert.", "5": "Lichen planus."}, "correct_option": 5, "explanations": {"1": {"exist": false, "char_ranges": [], "word_ranges": [], "text": ""}, "2": {"exist": false, "char_ranges": [], "word_ranges": [], "text": ""}, "3": {"exist": false, "char_ranges": [], "word_ranges": [], "text": ""}, "4": {"exist": false, "char_ranges": [], "word_ranges": [], "text": ""}, "5": {"exist": true, "char_ranges": [[0, 102]], "word_ranges": [[0, 17]], "text": "They are undoubtedly describing the typical lesions (both on skin and oral mucosa) of a lichen planus"}}} {"id": 426, "year": 2018, "question_id_specific": 94, "full_question": "A 73-year-old patient...swallowing a complete neurological dysphagia secondary to a cardioembolic stroke in the left middle cerebral artery territory. will require long-term nutritional support. Of the following treatment modalities, which do you consider most appropriate for this case?", "full_answer": "Enteral nutrition is a nutritional support technique that involves administering nutrients directly into the gastrointestinal tract by gavage. The choice of the route of formula administration depends on the nutritional status, age of the patient and the underlying disease. If enteral nutrition is considered to be of short duration, the route of choice is the nasogastric or nasoduodenal- nasojejunal tube... If prolonged nutrition is anticipated the route of choice is gastrostomy (option 3), with jejunostomy being reserved for cases in which it is not possible to use the gastric route.", "type": "ENDOCRINOLOGY", "options": {"1": "Enteral nutrition by nasogastric tube.", "2": "Enteral nutrition by nasoduodenal tube.", "3": "Enteral nutrition by gastrostomy.", "4": "Enteral nutrition by jejunostomy.", "5": NaN}, "correct_option": 3, "explanations": {"1": {"exist": true, "char_ranges": [[275, 410]], "word_ranges": [[39, 60]], "text": "If enteral nutrition is considered to be of short duration, the route of choice is the nasogastric or nasoduodenal- nasojejunal tube..."}, "2": {"exist": true, "char_ranges": [[275, 410]], "word_ranges": [[39, 60]], "text": "If enteral nutrition is considered to be of short duration, the route of choice is the nasogastric or nasoduodenal- nasojejunal tube..."}, "3": {"exist": true, "char_ranges": [[411, 495]], "word_ranges": [[60, 73]], "text": "If prolonged nutrition is anticipated the route of choice is gastrostomy (option 3),"}, "4": {"exist": true, "char_ranges": [[496, 591]], "word_ranges": [[73, 90]], "text": "with jejunostomy being reserved for cases in which it is not possible to use the gastric route."}, "5": {"exist": false, "char_ranges": [], "word_ranges": [], "text": ""}}} {"id": 262, "year": 2014, "question_id_specific": 58, "full_question": "A 70-year-old woman with a history of hypertension and moderate heart failure presents with a persistent, dry cough that begins as an itchy sensation in the throat. What is the drug suspected of causing the patient's clinical and laboratory alteration?", "full_answer": "These are typical adverse effects of ACEIs, in the case of cough it is usually dry and irritative and appears in up to 3% of patients. Hyperkalemia may also appear in 1%.", "type": "PHARMACOLOGY", "options": {"1": "Hydrochlorothiazide.", "2": "Bisoprolol.", "3": "Furosemide.", "4": "Enalapril.", "5": "Hydralazine."}, "correct_option": 4, "explanations": {"1": {"exist": false, "char_ranges": [], "word_ranges": [], "text": ""}, "2": {"exist": false, "char_ranges": [], "word_ranges": [], "text": ""}, "3": {"exist": false, "char_ranges": [], "word_ranges": [], "text": ""}, "4": {"exist": true, "char_ranges": [[0, 170]], "word_ranges": [[0, 32]], "text": "These are typical adverse effects of ACEIs, in the case of cough it is usually dry and irritative and appears in up to 3% of patients. Hyperkalemia may also appear in 1%."}, "5": {"exist": false, "char_ranges": [], "word_ranges": [], "text": ""}}} {"id": 346, "year": 2016, "question_id_specific": 159, "full_question": "69-year-old woman who comes to your office referring genital bleeding of several months of evolution. She denies hormone replacement therapy and anticoagulation. She provides normal cervicovaginal cytology. General and genital physical examination without findings of interest. BMI of 38kg/m2. Indicate the most correct attitude:", "full_answer": "The correct answer is 2. We are dealing with a case of postmenopausal metrorrhagia, whose most frequent cause is genital atrophy in the first place and endometrial cancer in the second place. The data given on obesity should lead us to suspect endometrial cancer, since peripheral fat is converted to estrone, and these estrogens cause the endometrium to proliferate. On the other hand, she provides a normal cytology, which rules out option 3. The hormonal evaluation is not opportune, since we understand that the patient is in menopause. We should not prescribe treatment without first ruling out organic causes.", "type": "GYNECOLOGY AND OBSTETRICS", "options": {"1": "Prescribe cyclic progesterone.", "2": "Endometrial biopsy.", "3": "Random cervical biopsies.", "4": "Hormonal evaluation with FSH, LH and estradiol.", "5": NaN}, "correct_option": 2, "explanations": {"1": {"exist": true, "char_ranges": [[541, 615]], "word_ranges": [[88, 99]], "text": "We should not prescribe treatment without first ruling out organic causes."}, "2": {"exist": true, "char_ranges": [[25, 191]], "word_ranges": [[5, 32]], "text": "We are dealing with a case of postmenopausal metrorrhagia, whose most frequent cause is genital atrophy in the first place and endometrial cancer in the second place."}, "3": {"exist": true, "char_ranges": [[387, 444]], "word_ranges": [[63, 73]], "text": "she provides a normal cytology, which rules out option 3."}, "4": {"exist": true, "char_ranges": [[445, 540]], "word_ranges": [[73, 88]], "text": "The hormonal evaluation is not opportune, since we understand that the patient is in menopause."}, "5": {"exist": false, "char_ranges": [], "word_ranges": [], "text": ""}}} {"id": 170, "year": 2013, "question_id_specific": 89, "full_question": "A 62-year-old patient who has had a pericardial bioprosthesis in mitral position for two years. He is admitted urgently for acute coronary syndrome. Two days later and after a favorable evolution, he suddenly presented heart failure with acute pulmonary edema. Select the diagnostic option in this case from the following possibilities:", "full_answer": "The valve is biological and, in general, the veils are not sewn to the papillary muscles, so we don't care what happens to them. A ventricular aneurysm depresses LVEF, but does not suddenly put you into PAD. In contrast, a VSD sharply increases flow to the pulmonary bed, causing pulmonary edema, and is also one of the most frequent complications of AMI. Ischemic ASD... I do not deny that it exists, but it does not even appear in the guidelines. And endocarditis would not require so many saddlebags.", "type": "CARDIOLOGY AND CARDIOVASCULAR SURGERY", "options": {"1": "Acute post-infarction mitral insufficiency.", "2": "Left ventricular aneurysm.", "3": "Acute ventricular septal defect.", "4": "Postinfarction atrial septal defect.", "5": "Dysfunction of the bioprosthesis due to probable acute endocarditis."}, "correct_option": 3, "explanations": {"1": {"exist": true, "char_ranges": [[356, 448]], "word_ranges": [[62, 80]], "text": "Ischemic ASD... I do not deny that it exists, but it does not even appear in the guidelines."}, "2": {"exist": true, "char_ranges": [[129, 207]], "word_ranges": [[24, 37]], "text": "A ventricular aneurysm depresses LVEF, but does not suddenly put you into PAD."}, "3": {"exist": true, "char_ranges": [[221, 355]], "word_ranges": [[39, 62]], "text": "a VSD sharply increases flow to the pulmonary bed, causing pulmonary edema, and is also one of the most frequent complications of AMI."}, "4": {"exist": true, "char_ranges": [[356, 448]], "word_ranges": [[62, 80]], "text": "Ischemic ASD... I do not deny that it exists, but it does not even appear in the guidelines."}, "5": {"exist": true, "char_ranges": [[453, 503]], "word_ranges": [[81, 88]], "text": "endocarditis would not require so many saddlebags."}}} {"id": 40, "year": 2011, "question_id_specific": 136, "full_question": "A 64-year-old smoker and heavy drinker reports painless right jugulodigastric lymphadenopathy of progressive growth in the last two months whose fine needle aspiration was reported as squamous cell carcinoma. Which of the following is the most likely location of the primary tumor:", "full_answer": "You are in the middle of the ENT block. You have already passed the Pneumology and Digestive blocks. Dermatology is still to come out, but a dermatologist (answer 1) would not ask this. So we are left with parotid and larynx; but parotid squamous cell carcinoma is rare; I would mark 5 and rest easy. Perhaps because all the adenopathies at this level that I have seen have ended up corresponding to carcinomas of the upper aerodigestive tract.", "type": "OTOLARYNGOLOGY AND MAXILLOFACIAL SURGERY", "options": {"1": "Scalp.", "2": "Parotid.", "3": "Lung.", "4": "Esophagus.", "5": "Larynx."}, "correct_option": 5, "explanations": {"1": {"exist": false, "char_ranges": [], "word_ranges": [], "text": ""}, "2": {"exist": false, "char_ranges": [], "word_ranges": [], "text": ""}, "3": {"exist": false, "char_ranges": [], "word_ranges": [], "text": ""}, "4": {"exist": false, "char_ranges": [], "word_ranges": [], "text": ""}, "5": {"exist": true, "char_ranges": [[317, 444]], "word_ranges": [[57, 78]], "text": "all the adenopathies at this level that I have seen have ended up corresponding to carcinomas of the upper aerodigestive tract."}}} {"id": 126, "year": 2012, "question_id_specific": 59, "full_question": "A 65-year-old man, a retired office worker and smoker of 1 pack of cigarettes a day, comes to the clinic with a persistent, generally dry cough and progressive dyspnea of 2 years of evolution which is currently grade 2. The patient denies other symptoms. Physical examination shows no relevant data. The attitude to follow would be:", "full_answer": "The correct answer is 2 (Perform chest X-ray and spirometry with bronchodilator test). It is probably COPD but we must confirm it and assess the severity of the obstruction with spirometry before starting treatment. He is a patient at risk for lung cancer so it is mandatory to perform a chest X-ray to rule out this or other pathologies that would justify the symptoms, although most likely they are due to the debut of his COPD.", "type": "PNEUMOLOGY AND THORACIC SURGERY", "options": {"1": "Start treatment with inhaled bronchodilators.", "2": "Chest X-ray and spirometry with bronchodilator test.", "3": "Prescribe oral corticosteroids.", "4": "Thoracic CT scan.", "5": "Perform basal arterial blood gases."}, "correct_option": 2, "explanations": {"1": {"exist": false, "char_ranges": [], "word_ranges": [], "text": ""}, "2": {"exist": true, "char_ranges": [[216, 430]], "word_ranges": [[34, 76]], "text": "He is a patient at risk for lung cancer so it is mandatory to perform a chest X-ray to rule out this or other pathologies that would justify the symptoms, although most likely they are due to the debut of his COPD."}, "3": {"exist": false, "char_ranges": [], "word_ranges": [], "text": ""}, "4": {"exist": false, "char_ranges": [], "word_ranges": [], "text": ""}, "5": {"exist": false, "char_ranges": [], "word_ranges": [], "text": ""}}} {"id": 568, "year": 2022, "question_id_specific": 151, "full_question": "A 22-year-old male presents to the emergency department with \"almost black\" urine for the past 12 hours. He has no past history of interest and is not taking any medications. He refers an upper respiratory catarrhal symptoms since 4-5 days ago for which he has taken paracetamol. Examination: temperature 37.3 ºC, BP 150/95, HR 85 bpm, slight pharyngeal erythema, the rest normal. Laboratory tests showed: Hb 12.8 g/dl, Hcto 39%, leukocytes 10,500/mm3 with normal formula, platelets 250,000/mm3, normal coagulation, urea 25 mg/dl, creatinine 0.8 mg/dl, ions, hepatic, lipid, albumin and total proteins normal. Normal autoimmunity study and negative viral serology. Urine 24 h with proteinuria of 0.75 g/24 h, sediment with 10 erythrocytes per field (90% dysmorphic), no leukocyturia. Renal ultrasound was normal. Which diagnosis do you think is more likely?", "full_answer": "A picture of macroscopic hematuria and mild proteinuria with normal creatinine levels, accompanied by an upper tract catarrhal picture, is very suggestive of IgA nephropathy (option 3 correct).", "type": "NEPHROLOGY", "options": {"1": "Nephropathy with minimal changes.", "2": "Postinfectious glomerulonephritis.", "3": "IgA nephropathy.", "4": "Acute interstitial nephritis.", "5": NaN}, "correct_option": 3, "explanations": {"1": {"exist": false, "char_ranges": [], "word_ranges": [], "text": ""}, "2": {"exist": false, "char_ranges": [], "word_ranges": [], "text": ""}, "3": {"exist": true, "char_ranges": [[0, 193]], "word_ranges": [[0, 28]], "text": "A picture of macroscopic hematuria and mild proteinuria with normal creatinine levels, accompanied by an upper tract catarrhal picture, is very suggestive of IgA nephropathy (option 3 correct)."}, "4": {"exist": false, "char_ranges": [], "word_ranges": [], "text": ""}, "5": {"exist": false, "char_ranges": [], "word_ranges": [], "text": ""}}} {"id": 286, "year": 2016, "question_id_specific": 60, "full_question": "A 60-year-old man with a history of hypertension and dyslipidemia has recently been diagnosed with heart failure. After performing the relevant tests, it was found that the left ventricular ejection fraction was preserved (>50%). With regard to treatment, indicate the CORRECT answer:", "full_answer": "Treatment with diuretics should be started at high doses.", "type": "CARDIOLOGY AND VASCULAR SURGERY", "options": {"1": "Initial treatment should be directed to the underlying pathological process.", "2": "Treatment with diuretics should be started at high doses.", "3": "If nitrate treatment is required, it should be started at low doses.", "4": "Dyspnea can be treated by reducing neurohormonal activation with angiotensin converting enzyme inhibitors or angiotensin receptor antagonists.", "5": NaN}, "correct_option": 2, "explanations": {"1": {"exist": false, "char_ranges": [], "word_ranges": [], "text": ""}, "2": {"exist": true, "char_ranges": [[0, 57]], "word_ranges": [[0, 9]], "text": "Treatment with diuretics should be started at high doses."}, "3": {"exist": false, "char_ranges": [], "word_ranges": [], "text": ""}, "4": {"exist": false, "char_ranges": [], "word_ranges": [], "text": ""}, "5": {"exist": false, "char_ranges": [], "word_ranges": [], "text": ""}}} {"id": 228, "year": 2014, "question_id_specific": 75, "full_question": "In a patient with clinical signs of heart failure who presents with ascites disproportionately elevated in relation to peripheral edema, the most likely etiology would be:", "full_answer": "They tell us about clinical symptoms of right failure, with edema and ascites. Therefore, we rule out left heart failure: EAo, dilated with DSVI and a MHO. Of the two we are left with, in constrictive pericarditis we have increased filling pressures in right chambers, which cause retrograde pressure increase with hepatic congestion, ascites, edema, etc. On the contrary, in pulmonary hypertension, the tricuspid valve is responsible (a priori) for \"containing\" this overpressure.", "type": "CARDIOLOGY", "options": {"1": "Severe aortic stenosis.", "2": "Dilated cardiomyopathy with significant left ventricular dysfunction.", "3": "Primary pulmonary hypertension.", "4": "Obstructive hypertrophic cardiomyopathy.", "5": "Constrictive pericarditis."}, "correct_option": 5, "explanations": {"1": {"exist": true, "char_ranges": [[0, 155]], "word_ranges": [[0, 27]], "text": "They tell us about clinical symptoms of right failure, with edema and ascites. Therefore, we rule out left heart failure: EAo, dilated with DSVI and a MHO."}, "2": {"exist": true, "char_ranges": [[0, 155]], "word_ranges": [[0, 27]], "text": "They tell us about clinical symptoms of right failure, with edema and ascites. Therefore, we rule out left heart failure: EAo, dilated with DSVI and a MHO."}, "3": {"exist": true, "char_ranges": [[373, 481]], "word_ranges": [[59, 73]], "text": "in pulmonary hypertension, the tricuspid valve is responsible (a priori) for \"containing\" this overpressure."}, "4": {"exist": true, "char_ranges": [[0, 155]], "word_ranges": [[0, 27]], "text": "They tell us about clinical symptoms of right failure, with edema and ascites. Therefore, we rule out left heart failure: EAo, dilated with DSVI and a MHO."}, "5": {"exist": true, "char_ranges": [[185, 355]], "word_ranges": [[34, 56]], "text": "in constrictive pericarditis we have increased filling pressures in right chambers, which cause retrograde pressure increase with hepatic congestion, ascites, edema, etc."}}} {"id": 217, "year": 2014, "question_id_specific": 122, "full_question": "A 54-year-old man is diagnosed with a left renal tumor suggestive of renal cell carcinoma. His preoperative workup shows elevated levels of GPT, alkaline phosphatase and alpha-2-globulin and prolonged prothrombin time. The liver is diffusely enlarged, but without defects of hepatic infiltration. The most likely rationale for these findings is due to:", "full_answer": "Very easy question in which we are being presented with a patient with Stauffer's syndrome, a paraneoplastic syndrome consisting of liver dysfunction secondary to toxic products secreted by a number of tumors (most frequently, renal cell carcinoma, like the one in this case). As in this case, cholestasis (elevation of alkaline phosphatase), mobilization of hepatic enzymes and lengthening of the prothrombin time indicate liver dysfunction. Metastases are ruled out (there are no defects of hepatic infiltration), intrahepatic tumor thrombosis (it would give another symptomatology and this would be more acute), and acute hepatitis and hemochromatosis (they are not in context nor is it what the author of the question wants us to think).", "type": "NEPHROLOGY", "options": {"1": "Hepatic metastases.", "2": "Intrahepatic tumor thrombosis.", "3": "Acute hepatitis.", "4": "Presence of hepatotoxic substances produced by the tumor.", "5": "Hemochromatosis."}, "correct_option": 4, "explanations": {"1": {"exist": true, "char_ranges": [[443, 515]], "word_ranges": [[65, 76]], "text": "Metastases are ruled out (there are no defects of hepatic infiltration),"}, "2": {"exist": true, "char_ranges": [[516, 614]], "word_ranges": [[76, 90]], "text": "intrahepatic tumor thrombosis (it would give another symptomatology and this would be more acute),"}, "3": {"exist": true, "char_ranges": [[619, 741]], "word_ranges": [[91, 113]], "text": "acute hepatitis and hemochromatosis (they are not in context nor is it what the author of the question wants us to think)."}, "4": {"exist": true, "char_ranges": [[276, 442]], "word_ranges": [[42, 65]], "text": "case). As in this case, cholestasis (elevation of alkaline phosphatase), mobilization of hepatic enzymes and lengthening of the prothrombin time indicate liver dysfunction."}, "5": {"exist": true, "char_ranges": [[619, 741]], "word_ranges": [[91, 113]], "text": "acute hepatitis and hemochromatosis (they are not in context nor is it what the author of the question wants us to think)."}}} {"id": 142, "year": 2012, "question_id_specific": 138, "full_question": "12-year-old girl, only child of healthy, non-consanguineous parents. There is no personal or family history of interest. In the clinical examination to start a sports activity, you find a discrete hepatomegaly of normal consistency, so you request a blood test, which highlights an ASAT of 80U / l, ALAT 105 U / l, electrophoretic proteinogram with all protein fractions in normal range and negativity of serology for hepatotropic viruses. What should you rule out and what test would you indicate for it?", "full_answer": "In fact, I would not test this patient at this time, and I would check if the transaminase level has normalized at a later time, since the most frequent cause, at least in children, is a transient elevation in the context of viral diseases (most frequently cytomegalovirus and Epstein-Barr virus). The minimal elevation of liver enzymes and the fact that it has a normal protein profile goes against chronicity... If I had to choose I would choose answer 5, which seems to me the least aggressive, and if I had to do analytical tests I would add the determination of alpha-1-antitrypsin (without CT) and markers of celiac disease, because celiac disease can also be associated with some hypertransaminemia.", "type": "PEDIATRICS", "options": {"1": "Deficiency of alpha 1 antitrypsin. Thoracic TAC to confirm emphysema.", "2": "Mucoviscidosis. Determination of chlorine in sweat.", "3": "Autoimmune hepatitis. Liver biopsy.", "4": "Glycogenosis type VI (phosphorylase deficiency). Muscle biopsy.", "5": "Wilson's disease. Determine ceruloplasmin and copper in blood and urine."}, "correct_option": 5, "explanations": {"1": {"exist": false, "char_ranges": [], "word_ranges": [], "text": ""}, "2": {"exist": false, "char_ranges": [], "word_ranges": [], "text": ""}, "3": {"exist": false, "char_ranges": [], "word_ranges": [], "text": ""}, "4": {"exist": false, "char_ranges": [], "word_ranges": [], "text": ""}, "5": {"exist": false, "char_ranges": [], "word_ranges": [], "text": ""}}} {"id": 267, "year": 2014, "question_id_specific": 139, "full_question": "In a patient with liver cirrhosis and ascites, who develops a pleural effusion with the following data in the pleural fluid: LDH 45 U/L (serum 220 U/L), pleural/serum protein ratio 0.3 and pleural/serum LDH ratio 0.2. What would be the appropriate attitude?", "full_answer": "They describe pleural fluid parameters of transudate, so if the patient is not in heart failure it is a hydropic decompensation. No other studies are needed.", "type": "PNEUMOLOGY", "options": {"1": "The study of the fluid with cell count, glucose, pH, ADA, cholesterol and culture should be extended. ADA, cholesterol and culture.", "2": "Blind pleural biopsy.", "3": "Diagnostic videothoracoscopy.", "4": "Empirical antibiotherapy for suspected parapneumonic effusion.", "5": "Continue treatment of his liver disease."}, "correct_option": 5, "explanations": {"1": {"exist": false, "char_ranges": [], "word_ranges": [], "text": ""}, "2": {"exist": false, "char_ranges": [], "word_ranges": [], "text": ""}, "3": {"exist": false, "char_ranges": [], "word_ranges": [], "text": ""}, "4": {"exist": false, "char_ranges": [], "word_ranges": [], "text": ""}, "5": {"exist": true, "char_ranges": [[0, 157]], "word_ranges": [[0, 26]], "text": "They describe pleural fluid parameters of transudate, so if the patient is not in heart failure it is a hydropic decompensation. No other studies are needed."}}} {"id": 99, "year": 2012, "question_id_specific": 84, "full_question": "A 60-year-old diabetic patient is consulting for the first time regarding the treatment of his disease. Which of the following goals would you recommend to him first?", "full_answer": "The rest are not indications. The target BP is < 130/85 mmHg.", "type": "ENDOCRINOLOGY", "options": {"1": "Maintain blood pressure below 110/70 mmHg.", "2": "Smoking cessation.", "3": "Maintain a body mass index (BMI) less than 21.", "4": "Perform daily basal capillary glycemia.", "5": "Avoidance of animal fats in the diet."}, "correct_option": 2, "explanations": {"1": {"exist": true, "char_ranges": [[0, 61]], "word_ranges": [[0, 12]], "text": "The rest are not indications. The target BP is < 130/85 mmHg."}, "2": {"exist": true, "char_ranges": [[0, 29]], "word_ranges": [[0, 5]], "text": "The rest are not indications."}, "3": {"exist": true, "char_ranges": [[0, 29]], "word_ranges": [[0, 5]], "text": "The rest are not indications."}, "4": {"exist": true, "char_ranges": [[0, 29]], "word_ranges": [[0, 5]], "text": "The rest are not indications."}, "5": {"exist": true, "char_ranges": [[0, 29]], "word_ranges": [[0, 5]], "text": "The rest are not indications."}}} {"id": 380, "year": 2016, "question_id_specific": 135, "full_question": "A 25-year-old patient, during a tennis match, has intense pain in the neck and in the left eye. The next morning he wakes up with a feeling of gait instability and has palpebral ptosis of the left eye and anisocoria, with the left pupil being smaller than the right. The patient maintains good visual acuity. Where would you most likely locate the lesion?", "full_answer": "The picture presented to us is that of a Horner syndrome (miosis with ptosis) so the possibilities are reduced to options 3 or 4, but they specify that the patient presents gait instability, which is not justified by a Horner due to carotid dissection affecting the superior cervical ganglion, which is what they want us to believe when they talk about pain in the neck and in the eye. If the patient really has ataxia, the only possible location would be the spinal cord.", "type": "NEUROLOGY", "options": {"1": "III cranial pair.", "2": "Optic chiasm.", "3": "Superior cervical ganglion.", "4": "Spinal bulb.", "5": NaN}, "correct_option": 4, "explanations": {"1": {"exist": false, "char_ranges": [], "word_ranges": [], "text": ""}, "2": {"exist": false, "char_ranges": [], "word_ranges": [], "text": ""}, "3": {"exist": true, "char_ranges": [[134, 293]], "word_ranges": [[25, 49]], "text": "they specify that the patient presents gait instability, which is not justified by a Horner due to carotid dissection affecting the superior cervical ganglion,"}, "4": {"exist": true, "char_ranges": [[386, 472]], "word_ranges": [[69, 84]], "text": "If the patient really has ataxia, the only possible location would be the spinal cord."}, "5": {"exist": false, "char_ranges": [], "word_ranges": [], "text": ""}}} {"id": 341, "year": 2016, "question_id_specific": 33, "full_question": "A 67-year-old woman diagnosed with infiltrating ductal carcinoma of the breast with no family history of neoplasia. What additional studies should be performed on the tumor because of its clinical and therapeutic implications?", "full_answer": "Study of hormone receptors and HER2. Both for the type of treatment and to study the prognostic factor and adjuvant treatment, it is necessary to know the different hormonal receptors (estrogen and progesterone) as well as HER2-neu.", "type": "GYNECOLOGY AND OBSTETRICS", "options": {"1": "Complete phenotypic study by flow cytometry.", "2": "Study of hormone receptors and HER2.", "3": "Study of hormone receptors, e-cadherin and study of first-degree relatives.", "4": "Study of BRCA 1-2 and study of first-degree relatives.", "5": NaN}, "correct_option": 2, "explanations": {"1": {"exist": false, "char_ranges": [], "word_ranges": [], "text": ""}, "2": {"exist": true, "char_ranges": [[37, 232]], "word_ranges": [[6, 37]], "text": "Both for the type of treatment and to study the prognostic factor and adjuvant treatment, it is necessary to know the different hormonal receptors (estrogen and progesterone) as well as HER2-neu."}, "3": {"exist": false, "char_ranges": [], "word_ranges": [], "text": ""}, "4": {"exist": false, "char_ranges": [], "word_ranges": [], "text": ""}, "5": {"exist": false, "char_ranges": [], "word_ranges": [], "text": ""}}} {"id": 308, "year": 2016, "question_id_specific": 215, "full_question": "A 47-year-old man with myopia magna, who underwent cataract surgery 2 years ago, comes to the emergency room reporting a profound and painless loss of vision in his right eye. Which of the following diagnoses can cause this symptomatology?", "full_answer": "He refers to a patient post cataract surgery 2 years ago with deep and painless loss of OD. Post-surgical endophthalmitis is more associated with patients in the immediate postoperative period, besides being very painful with rapidly progressive decrease in visual acuity, so it is not correct. Retinal detachment is consistent with the clinical case because it is also a myopic patient who tend to have a special predisposition to detachment. ARMD usually occurs in older patients and the decrease in vision is progressive. In PVD there may or may not be a slight decrease in visual acuity and the patient usually reports some discomfort or pain at the onset of the clinical picture.", "type": "OPHTHALMOLOGY", "options": {"1": "Post-surgical endophthalmitis.", "2": "Retinal detachment.", "3": "Age-related macular degeneration, wet form.", "4": "Posterior vitreous detachment.", "5": NaN}, "correct_option": 2, "explanations": {"1": {"exist": true, "char_ranges": [[92, 294]], "word_ranges": [[18, 46]], "text": "Post-surgical endophthalmitis is more associated with patients in the immediate postoperative period, besides being very painful with rapidly progressive decrease in visual acuity, so it is not correct."}, "2": {"exist": true, "char_ranges": [[295, 443]], "word_ranges": [[46, 70]], "text": "Retinal detachment is consistent with the clinical case because it is also a myopic patient who tend to have a special predisposition to detachment."}, "3": {"exist": true, "char_ranges": [[444, 524]], "word_ranges": [[70, 83]], "text": "ARMD usually occurs in older patients and the decrease in vision is progressive."}, "4": {"exist": true, "char_ranges": [[525, 684]], "word_ranges": [[83, 113]], "text": "In PVD there may or may not be a slight decrease in visual acuity and the patient usually reports some discomfort or pain at the onset of the clinical picture."}, "5": {"exist": false, "char_ranges": [], "word_ranges": [], "text": ""}}} {"id": 519, "year": 2021, "question_id_specific": 176, "full_question": "A 25-year-old woman comes to the ED accompanied by a friend reporting that she was the victim of a sexual assault 4 hours ago. The patient is very affected, answering her questions inconsistently, slowly and not being able to remember how the assault occurred. Her vital signs (blood pressure, heart rate and temperature) are normal. Which of the following would be WRONG:", "full_answer": "All the actions detailed are correct except the one detailed in option 2. The collection of samples of legal interest will only be carried out in the presence of the coroner, and, without the presence of the coroner, the woman must give her written consent. In addition, the taking of gynecological samples must be adapted to the psychological state of the victim, so in most cases, it is counterproductive to perform it at this time (even more so when they describe the patient's state as \"very affected, answering their questions incongruently, slowly and not being able to remember how the aggression occurred\".", "type": "CRITICAL AND EMERGENCY CARE", "options": {"1": "Contact the duty court.", "2": "Immediate gynecological examination with cervical cytology.", "3": "Administer 500 mg of intramuscular ceftriaxone, 1 g of azithromycin orally and 2 g of tinidazole orally in single doses.", "4": "Administer 1500 mg of levonorgestrel orally in a single dose.", "5": NaN}, "correct_option": 2, "explanations": {"1": {"exist": false, "char_ranges": [], "word_ranges": [], "text": ""}, "2": {"exist": true, "char_ranges": [[74, 257]], "word_ranges": [[13, 45]], "text": "The collection of samples of legal interest will only be carried out in the presence of the coroner, and, without the presence of the coroner, the woman must give her written consent."}, "3": {"exist": false, "char_ranges": [], "word_ranges": [], "text": ""}, "4": {"exist": false, "char_ranges": [], "word_ranges": [], "text": ""}, "5": {"exist": false, "char_ranges": [], "word_ranges": [], "text": ""}}} {"id": 63, "year": 2011, "question_id_specific": 200, "full_question": "A 31-year-old bricklayer comes to the emergency department with tenosynovitis caused by a penetrating wound. On examination, the ulnar bursa is found to be infected with consequent tendon necrosis. Which tendon will be affected?", "full_answer": "Both the flexor carpi radialis and the abductor pollicis longus are anatomically far from the ulnar. The palmaris longus is medial, as is the flexor digitorum profundus. The tendon that runs alongside the ulnar nerve, inserting into the pisiform, is the flexor carpi ulnaris, thus answer 1.", "type": "ANATOMY", "options": {"1": "Tendon of the flexor carpi ulnaris.", "2": "Tendon of the abductor pollicis longus.", "3": "Tendon of the deep flexor of the fingers.", "4": "Radial carpal flexor tendon.", "5": "Palmaris longus tendon."}, "correct_option": 1, "explanations": {"1": {"exist": true, "char_ranges": [[170, 290]], "word_ranges": [[27, 47]], "text": "The tendon that runs alongside the ulnar nerve, inserting into the pisiform, is the flexor carpi ulnaris, thus answer 1."}, "2": {"exist": true, "char_ranges": [[0, 100]], "word_ranges": [[0, 16]], "text": "Both the flexor carpi radialis and the abductor pollicis longus are anatomically far from the ulnar."}, "3": {"exist": true, "char_ranges": [[101, 169]], "word_ranges": [[16, 27]], "text": "The palmaris longus is medial, as is the flexor digitorum profundus."}, "4": {"exist": true, "char_ranges": [[0, 100]], "word_ranges": [[0, 16]], "text": "Both the flexor carpi radialis and the abductor pollicis longus are anatomically far from the ulnar."}, "5": {"exist": true, "char_ranges": [[101, 169]], "word_ranges": [[16, 27]], "text": "The palmaris longus is medial, as is the flexor digitorum profundus."}}} {"id": 30, "year": 2011, "question_id_specific": 61, "full_question": "A patient previously diagnosed with COPD comes to the emergency room for 3 days with increased dyspnea until he is at rest, cough with whitish expectoration and marked drowsiness. After receiving treatment with oxygen at low concentrations, bronchodilators and coticoids, a second arterial blood gas with 28% inspiratory oxygen fraction shows a pH of 7.32, pO2 61 mmHg, pCO2 58 mmHg, HCO3- 29 mmol/l. What would be the interpretation you would make of the arterial blood gas and what treatment would you use?", "full_answer": "This is a very bad question, I will bet on one answer, but depending on data that we are not given, three answers could be valid. The trap is that they give us the initial clinical picture WITHOUT GASOMETRY and then they give us the blood gases afterwards WITHOUT CLINIC, so there is no way of knowing what the evolution of the patient has been with our treatment, so it is impossible to know what to do. If the patient initially came with a blood gas with a pH of 7.05 and now has the one we are given, we are doing well (although we are not told the time interval between one thing and the other) and, in this case, it could be worthwhile to continue with the same treatment. If nothing has evolved, the clinical picture is the same and the blood gas at the beginning was similar, something should be done (also depending on the patient's baseline status, which they do not tell us either). If we decide to do something, at present we might decide to use ventilation rather than theophylline, although with \"so little\" alteration of pH (and even more so if we think that this is a chronic patient who may have a baseline blood gas similar to the one presented to us) it may seem \"exaggerated\". We have nothing \"against\" theophylline (we are not told of any contraindication), but nowadays it is rare to resort to it. 1: Wrong, he is a \"retentive\" patient and increasing O2 flow is highly likely to increase hypercapnia and, therefore, acidosis. 2: We will keep this one as correct. 3: It is not a metabolic acidosis, the bicarbonate is not decreased, on the contrary, it is slightly increased trying to compensate the respiratory acidosis. 4: He does not reach respiratory failure by definition (pO2 60), but he has hypercapnia and slight acidosis. If we knew the first blood gas and she was much worse (i.e. she was doing very well), continuing with the same medication would be an option, but this data is missing!!!, in any case it seems to be a trick answer. 5: At present and -as already mentioned- taking into account that there is no formal contraindication, it is not usually used.", "type": "NEUROLOGY AND THORACIC SURGERY", "options": {"1": "Increase oxygen flow because he has acute respiratory acidosis and hypoxemia.", "2": "I would initiate noninvasive mechanical ventilation because he has acute hypercapnic respiratory failure with moderate respiratory acidosis.", "3": "I would add sodium bicarbonate to correct the acute metabolic acidosis.", "4": "The patient does not have respiratory insufficiency so he would continue with the same pharmacological regimen.", "5": "I would add intravenous aminophylline as a respiratory stimulant, since I appreciate hypoxemia and hypercapnia."}, "correct_option": 4, "explanations": {"1": {"exist": true, "char_ranges": [[1322, 1446]], "word_ranges": [[245, 264]], "text": "Wrong, he is a \"retentive\" patient and increasing O2 flow is highly likely to increase hypercapnia and, therefore, acidosis."}, "2": {"exist": true, "char_ranges": [[893, 1195]], "word_ranges": [[168, 223]], "text": "If we decide to do something, at present we might decide to use ventilation rather than theophylline, although with \"so little\" alteration of pH (and even more so if we think that this is a chronic patient who may have a baseline blood gas similar to the one presented to us) it may seem \"exaggerated\"."}, "3": {"exist": true, "char_ranges": [[1487, 1641]], "word_ranges": [[273, 297]], "text": "It is not a metabolic acidosis, the bicarbonate is not decreased, on the contrary, it is slightly increased trying to compensate the respiratory acidosis."}, "4": {"exist": true, "char_ranges": [[1645, 1750]], "word_ranges": [[298, 315]], "text": "He does not reach respiratory failure by definition (pO2 60), but he has hypercapnia and slight acidosis."}, "5": {"exist": true, "char_ranges": [[1968, 2091]], "word_ranges": [[358, 378]], "text": "At present and -as already mentioned- taking into account that there is no formal contraindication, it is not usually used."}}} {"id": 226, "year": 2014, "question_id_specific": 71, "full_question": "An 82-year-old hypertensive woman on atenolol, hydrochlorothiazide and digoxin. She comes to the emergency room for atrial fibrillation and is administered IV verapamil. The ECG showed complete atrioventricular block. What is the most probable cause of this clinical situation?", "full_answer": "He already takes two AV node brakers and you add a third, and are you surprised that he gets complete AV block after messing with drugs from different families but same effect (pharmacodynamics)? Three. And remember that drugs are like drinks when you go to a bar: don't mix!", "type": "CARDIOLOGY", "options": {"1": "Digitalis intoxication due to pharmacokinetic interaction with verapamil.", "2": "Hypokalemia due to administration of thiazide and digoxin.", "3": "Pharmacodynamic interaction of beta-blocker, digoxin and verapamil.", "4": "Hypotensive effect of thiazide diuretic.", "5": "Cardiac arrhythmia due to verapamil."}, "correct_option": 3, "explanations": {"1": {"exist": false, "char_ranges": [], "word_ranges": [], "text": ""}, "2": {"exist": false, "char_ranges": [], "word_ranges": [], "text": ""}, "3": {"exist": true, "char_ranges": [[0, 195]], "word_ranges": [[0, 33]], "text": "He already takes two AV node brakers and you add a third, and are you surprised that he gets complete AV block after messing with drugs from different families but same effect (pharmacodynamics)?"}, "4": {"exist": false, "char_ranges": [], "word_ranges": [], "text": ""}, "5": {"exist": false, "char_ranges": [], "word_ranges": [], "text": ""}}} {"id": 565, "year": 2022, "question_id_specific": 148, "full_question": "A 48-year-old patient referred to the nephrology department for an estimated glomerular filtration rate (eGFR) of 32 ml/min/1.72 mL/min/1.72 mL. Which of the following data would NOT suggest chronic kidney disease?", "full_answer": "Adequate corticomedullary differentiation is suggestive of no chronic parenchymal impairment, and is a frequent finding in acute renal failure (option 4 correct). A family history of nephropathy could indicate the existence of a hereditary disorder (incorrect option 2). Small kidneys and hyperphosphatemia are features of chronic kidney disease (option 1 and 3 incorrect).", "type": "NEPHROLOGY", "options": {"1": "Small sized kidneys.", "2": "Family history of nephropathy.", "3": "Elevated levels of phosphorus in blood.", "4": "Good ultrasound differentiation of renal cortex and medulla.", "5": NaN}, "correct_option": 4, "explanations": {"1": {"exist": true, "char_ranges": [[271, 373]], "word_ranges": [[38, 53]], "text": "Small kidneys and hyperphosphatemia are features of chronic kidney disease (option 1 and 3 incorrect)."}, "2": {"exist": true, "char_ranges": [[163, 270]], "word_ranges": [[22, 38]], "text": "A family history of nephropathy could indicate the existence of a hereditary disorder (incorrect option 2)."}, "3": {"exist": true, "char_ranges": [[271, 373]], "word_ranges": [[38, 53]], "text": "Small kidneys and hyperphosphatemia are features of chronic kidney disease (option 1 and 3 incorrect)."}, "4": {"exist": true, "char_ranges": [[0, 162]], "word_ranges": [[0, 22]], "text": "Adequate corticomedullary differentiation is suggestive of no chronic parenchymal impairment, and is a frequent finding in acute renal failure (option 4 correct)."}, "5": {"exist": false, "char_ranges": [], "word_ranges": [], "text": ""}}} {"id": 135, "year": 2012, "question_id_specific": 38, "full_question": "A 64-year-old man with Child A liver cirrhosis with no history of decompensation. After detection of a focal hepatic lesion on screening ultrasound, the study is completed with a thoracoabdominal CT scan. This scan showed the presence of 4 liver lesions (one of them up to 6 cm) with uptake pattern typical of hepatocarcinoma, tumor vascular invasion and a metastasis in the right adrenal gland. No ascites is observed. The patient reports only asthenia, but no general syndrome. The treatment of this patient would be:", "full_answer": "Comment: The patient presents a good general condition with a recently diagnosed hepatocarcinoma, not giving him any type of treatment other than symptomatic and supportive at this stage would not be the right thing to do. TACE would not be indicated because of extrahepatic involvement. Transplantation would also not be done if he has extrahepatic disease. We are left with answers 3 and 4, both with systemic treatment (indicated because it is metastatic) and the correct answer is 4, for all the reasons given therein.", "type": "ONCOLOGY", "options": {"1": "Symptomatic and supportive treatment, since he presented an advanced hepatocarcinoma with vascular invasion and metastasis.", "2": "Transarterial chemoembolization (TACE), since the treatment increases survival in patients with advanced hepatocarcinoma.", "3": "Systemic chemotherapy with doxorubicin iv since he presents disseminated disease but he is in good general condition.", "4": "Treatment with oral sorafenib, since this is a patient with good general condition, Child A and advanced stage hepatocarcinoma BCLC-C.", "5": "The appropriate treatment is liver transplantation as it is the only procedure that completely eliminates the primary tumor and avoids future complications of cirrhosis."}, "correct_option": 4, "explanations": {"1": {"exist": true, "char_ranges": [[9, 222]], "word_ranges": [[1, 36]], "text": "The patient presents a good general condition with a recently diagnosed hepatocarcinoma, not giving him any type of treatment other than symptomatic and supportive at this stage would not be the right thing to do."}, "2": {"exist": true, "char_ranges": [[223, 287]], "word_ranges": [[36, 45]], "text": "TACE would not be indicated because of extrahepatic involvement."}, "3": {"exist": true, "char_ranges": [[393, 522]], "word_ranges": [[64, 85]], "text": "both with systemic treatment (indicated because it is metastatic) and the correct answer is 4, for all the reasons given therein."}, "4": {"exist": true, "char_ranges": [[393, 522]], "word_ranges": [[64, 85]], "text": "both with systemic treatment (indicated because it is metastatic) and the correct answer is 4, for all the reasons given therein."}, "5": {"exist": true, "char_ranges": [[288, 358]], "word_ranges": [[45, 56]], "text": "Transplantation would also not be done if he has extrahepatic disease."}}} {"id": 556, "year": 2022, "question_id_specific": 55, "full_question": "63-year-old diabetic patient whose family physician has requested a blood test to determine vitamin B12 levels.What is the antidiabetic drug he is taking that warrants such a request:", "full_answer": "Metformin is an oral antidiabetic widely used in the treatment of type 2 diabetes mellitus. The best known side effects are gastrointestinal side effects and lactic acidosis; however, vitamin B 12 malabsorption, is less well known. A decrease in vitamin B 12 levels is observed in patients treated with metformin. The mechanism by which this deficit occurs is not clear and it is known to be reversible when treatment is interrupted.", "type": "ENDOCRINOLOGY", "options": {"1": "Gliclazide.", "2": "Metformin.", "3": "Repaglinide.", "4": "Pioglitazone.", "5": NaN}, "correct_option": 2, "explanations": {"1": {"exist": false, "char_ranges": [], "word_ranges": [], "text": ""}, "2": {"exist": true, "char_ranges": [[0, 231]], "word_ranges": [[0, 36]], "text": "Metformin is an oral antidiabetic widely used in the treatment of type 2 diabetes mellitus. The best known side effects are gastrointestinal side effects and lactic acidosis; however, vitamin B 12 malabsorption, is less well known."}, "3": {"exist": false, "char_ranges": [], "word_ranges": [], "text": ""}, "4": {"exist": false, "char_ranges": [], "word_ranges": [], "text": ""}, "5": {"exist": false, "char_ranges": [], "word_ranges": [], "text": ""}}} {"id": 448, "year": 2018, "question_id_specific": 148, "full_question": "A 59-year-old woman with recent onset of epileptic seizures comes to the emergency department. The examination showed left pyramidal signs and papillary edema. Brain MRI showed a right hemispheric mass with edema, midline deviation and signs of tentorial herniation. She reports a headache that has been progressing in intensity for the past week. Which of the following features associated with headache seems most likely in this patient?", "full_answer": "They describe a recent diagnosis of cerebral LOE with signs of intracranial hypertension (edema, midline deviation, papilledema). The associated headache is typically morning headache, with increasing ICP values at night.", "type": "NEUROSURGERY", "options": {"1": "Predominance in the morning.", "2": "It does not change with effort.", "3": "Photophobia.", "4": "Sonophobia.", "5": NaN}, "correct_option": 1, "explanations": {"1": {"exist": true, "char_ranges": [[0, 221]], "word_ranges": [[0, 30]], "text": "They describe a recent diagnosis of cerebral LOE with signs of intracranial hypertension (edema, midline deviation, papilledema). The associated headache is typically morning headache, with increasing ICP values at night."}, "2": {"exist": false, "char_ranges": [], "word_ranges": [], "text": ""}, "3": {"exist": false, "char_ranges": [], "word_ranges": [], "text": ""}, "4": {"exist": false, "char_ranges": [], "word_ranges": [], "text": ""}, "5": {"exist": false, "char_ranges": [], "word_ranges": [], "text": ""}}} {"id": 591, "year": 2022, "question_id_specific": 64, "full_question": "A 68-year-old woman who wears +4.00 diopter glasses for distance vision in both eyes. She consults because since a few weeks ago she has been having eye pain with headache and blurred vision at night when she is watching television in twilight. Which of the following pathologies is she most likely to present?", "full_answer": "We are presented with a 68-year-old woman with moderate hyperopia and no history of cataract surgery. This is not a mere coincidence: the fact that the patient is hyperopic is important, because the eyeballs of hyperopic patients are shorter, with a certain conflict of space in the anterior segment when the crystalline lens undergoes a process of intumescence secondary to age (what patients call incipient cataract). In the clinical case, moreover, the ocular and headache pain occurs in scotopic situations, or in situations of dim ambient lighting, which produces an average mydriasis to optimize the light influx to the eyeball. It is evident, therefore, that the patient suffers from primary angular closure processes that possibly self-resolve when the triggering situation ceases, and not a true acute glaucoma attack. But these intermittent angular closures are contemplated within the spectrum of glaucoma due to primary angular closure (option 1 correct). Migraine, not being a Neurology question, can sometimes be a confounder in the usual clinic; however, patients are usually younger, or at least have a clear history of migraine, and, above all, they present symptomatologic worsening with intense sensory stimuli (photophobia and phonophobia), but symptoms are not worsened precisely in cases of dim lighting (option 2 incorrect). Actinic keratitis occurs precisely in patients exposed to ultraviolet radiation that is not properly filtered before reaching the cornea; it is typical of welders (it is called welder's keratitis), skiers and other people with very intense light exposures and without adequate eye protection (wrong option 3). Dry eye syndrome does not usually produce headache, and ocular discomfort does not go beyond the persistent discomfort that patients might define as pain because of its recently more studied neuropathic component. Some forms of dry eye may be worse in the evening but others are worse in the morning, and it depends mainly on the frequency of blinking. In addition, dry eye syndrome is an outdated term; nowadays it is preferred to speak of dry eye disease (incorrect option 4).", "type": "OPHTHALMOLOGY", "options": {"1": "Glaucoma due to primary angular closure.", "2": "Migraine.", "3": "Actinic keratitis.", "4": "Dry eye syndrome.", "5": NaN}, "correct_option": 1, "explanations": {"1": {"exist": true, "char_ranges": [[635, 967]], "word_ranges": [[100, 148]], "text": "It is evident, therefore, that the patient suffers from primary angular closure processes that possibly self-resolve when the triggering situation ceases, and not a true acute glaucoma attack. But these intermittent angular closures are contemplated within the spectrum of glaucoma due to primary angular closure (option 1 correct)."}, "2": {"exist": true, "char_ranges": [[968, 1347]], "word_ranges": [[148, 205]], "text": "Migraine, not being a Neurology question, can sometimes be a confounder in the usual clinic; however, patients are usually younger, or at least have a clear history of migraine, and, above all, they present symptomatologic worsening with intense sensory stimuli (photophobia and phonophobia), but symptoms are not worsened precisely in cases of dim lighting (option 2 incorrect)."}, "3": {"exist": true, "char_ranges": [[1348, 1657]], "word_ranges": [[205, 251]], "text": "Actinic keratitis occurs precisely in patients exposed to ultraviolet radiation that is not properly filtered before reaching the cornea; it is typical of welders (it is called welder's keratitis), skiers and other people with very intense light exposures and without adequate eye protection (wrong option 3)."}, "4": {"exist": true, "char_ranges": [[1658, 2136]], "word_ranges": [[251, 332]], "text": "Dry eye syndrome does not usually produce headache, and ocular discomfort does not go beyond the persistent discomfort that patients might define as pain because of its recently more studied neuropathic component. Some forms of dry eye may be worse in the evening but others are worse in the morning, and it depends mainly on the frequency of blinking. In addition, dry eye syndrome is an outdated term; nowadays it is preferred to speak of dry eye disease (incorrect option 4)."}, "5": {"exist": false, "char_ranges": [], "word_ranges": [], "text": ""}}} {"id": 291, "year": 2016, "question_id_specific": 157, "full_question": "An 8-year-old patient presenting with superficial pustular lesions, erosions and yellowish crusts around the mouth for the last 3 days:", "full_answer": "It is a typical and frequent clinical picture in children. You are describing a contagious impetigo. There is not much dispute. The most frequent causative agent in the bullous form is S. aureus phage II.", "type": "DERMATOLOGY, VENEREOLOGY AND PLASTIC SURGERY", "options": {"1": "Exudative erythema multiforme.", "2": "Contagious impetigo.", "3": "Infantile acne.", "4": "Pustular psoriasis.", "5": NaN}, "correct_option": 2, "explanations": {"1": {"exist": false, "char_ranges": [], "word_ranges": [], "text": ""}, "2": {"exist": true, "char_ranges": [[0, 100]], "word_ranges": [[0, 16]], "text": "It is a typical and frequent clinical picture in children. You are describing a contagious impetigo."}, "3": {"exist": false, "char_ranges": [], "word_ranges": [], "text": ""}, "4": {"exist": false, "char_ranges": [], "word_ranges": [], "text": ""}, "5": {"exist": false, "char_ranges": [], "word_ranges": [], "text": ""}}} {"id": 213, "year": 2014, "question_id_specific": 160, "full_question": "A 75-year-old woman, with menopause at age 52, with no family or personal history of fracture, diagnosed with temporal arteritis, who is going to start treatment with high-dose prednisone and expects to be treated for at least one year. The delay of densitometry (DXA) in her center is 4-5 months. Preventive treatment for osteoporosis is considered. Which of the following is the most appropriate approach?", "full_answer": "In the data you provide there are no clinical risk factors for osteoporosis prior to the current disease. Since a prolonged and high-dose prednisone regimen will be established, it is advisable, in addition to the non-pharmacological preventive measures, to initiate treatment in any case to minimize the secondary loss of bone mass that occurs with the administration of glucocorticoids (and, above all, in the first months). Therefore, and for this reason alone, options 1, 2 and 5 would be discarded (the three of them have another conditioning factor). In this context, and in the case of a person over 65 years of age, an appropriate attitude would be to administer calcium supplements (corticosteroids are osteopenizantes because they produce, among other mechanisms, a negative balance of this element) and vitamin D. The option of associating a bisphosphonate could also be considered, but calcium supplementation is missing in proposal 3.", "type": "RHEUMATOLOGY", "options": {"1": "Request DXA and wait for the result.", "2": "Assess absolute risk of fracture using the FRAX questionnaire without BMD and treat only if it is high.", "3": "Start treatment with bisphosphonates and vitamin D (800 IU/day).", "4": "Administer calcium supplementation (1g) and vitamin D (800 IU/day).", "5": "Assess for osteopenia on radiographs and treat if present."}, "correct_option": 4, "explanations": {"1": {"exist": true, "char_ranges": [[106, 556]], "word_ranges": [[18, 88]], "text": "Since a prolonged and high-dose prednisone regimen will be established, it is advisable, in addition to the non-pharmacological preventive measures, to initiate treatment in any case to minimize the secondary loss of bone mass that occurs with the administration of glucocorticoids (and, above all, in the first months). Therefore, and for this reason alone, options 1, 2 and 5 would be discarded (the three of them have another conditioning factor)."}, "2": {"exist": true, "char_ranges": [[106, 556]], "word_ranges": [[18, 88]], "text": "Since a prolonged and high-dose prednisone regimen will be established, it is advisable, in addition to the non-pharmacological preventive measures, to initiate treatment in any case to minimize the secondary loss of bone mass that occurs with the administration of glucocorticoids (and, above all, in the first months). Therefore, and for this reason alone, options 1, 2 and 5 would be discarded (the three of them have another conditioning factor)."}, "3": {"exist": true, "char_ranges": [[825, 947]], "word_ranges": [[130, 148]], "text": "The option of associating a bisphosphonate could also be considered, but calcium supplementation is missing in proposal 3."}, "4": {"exist": true, "char_ranges": [[557, 824]], "word_ranges": [[88, 130]], "text": "In this context, and in the case of a person over 65 years of age, an appropriate attitude would be to administer calcium supplements (corticosteroids are osteopenizantes because they produce, among other mechanisms, a negative balance of this element) and vitamin D."}, "5": {"exist": true, "char_ranges": [[106, 556]], "word_ranges": [[18, 88]], "text": "Since a prolonged and high-dose prednisone regimen will be established, it is advisable, in addition to the non-pharmacological preventive measures, to initiate treatment in any case to minimize the secondary loss of bone mass that occurs with the administration of glucocorticoids (and, above all, in the first months). Therefore, and for this reason alone, options 1, 2 and 5 would be discarded (the three of them have another conditioning factor)."}}} {"id": 442, "year": 2018, "question_id_specific": 100, "full_question": "34-year-old woman who consults for repeated coitorragia. She brings a cytology with a high grade intraepithelial lesion (U-SIL). Subsequently, colposcopy and biopsy were performed on a mosaic area and histology showed a focus of invasive squamous cell carcinoma of 2 mm in length. Which therapeutic option is the most appropriate for this patient?", "full_answer": "According to the SEGO: \"Excise the entire lesion so that it can be evaluated histologically. It involves excision of the entire transformation zone. In general, the excision should be adapted to the size and characteristics of the lesion. Three types of excision are distinguished according to the presence of the endocervical component of the lesion. Excision type 1 (applicable in cases with transformation zone type 1, in which the diathermic loop should not include endocervical canal or exceed 8 mm in depth), excision type 2 (indicated in transformation zones type 2, involves resecting a small part of endocervical canal visible by colposcopy) and excision type 3 (indicated in transformation zones type 3, includes part of endocervical epithelium)\". When they speak of excision, it is synonymous with conization; as the lesion is less than 8mm deep, this is the indicated treatment.", "type": "GYNECOLOGY AND OBSTETRICS", "options": {"1": "Conization.", "2": "Total hysterectomy without adnexectomy.", "3": "Pelvic radiotherapy with curative intent.", "4": "Repeat a more extensive biopsy.", "5": NaN}, "correct_option": 1, "explanations": {"1": {"exist": true, "char_ranges": [[821, 890]], "word_ranges": [[127, 140]], "text": "as the lesion is less than 8mm deep, this is the indicated treatment."}, "2": {"exist": false, "char_ranges": [], "word_ranges": [], "text": ""}, "3": {"exist": false, "char_ranges": [], "word_ranges": [], "text": ""}, "4": {"exist": false, "char_ranges": [], "word_ranges": [], "text": ""}, "5": {"exist": false, "char_ranges": [], "word_ranges": [], "text": ""}}} {"id": 491, "year": 2020, "question_id_specific": 106, "full_question": "65-year-old woman referred to the ED for fever and alterations in the CBC: hemoglobin 11.4 g/dL, leukocytes 0.86 x103/μL,(neutrophils 41.9 %, lymphocytes 55.8 %),platelets 48.0 x103/μL, fibrinogen 118 mg/dL,D-dimer 20.2 μg/mL. A bone marrow examination was performed and the patient was diagnosed with acute leukemia with t(15;17) in 60% of the cells. Which of the following answers is correct?", "full_answer": "I think this is a very nice question. There is no doubt about the diagnosis. Because if you don't know the translocation, nothing happens... the answers give you the diagnosis. Now it's time to find out how acute promyelocytic leukemia is currently treated (ATRA aside... which I'm sure you know). As of 2017, the PETHEMA protocol for APL in low to intermediate risk patients (and/or over 70 years of age) is ATO+ATRA. In high risk patients, it is treated with ATRA+chemotherapy. I don't think they want you to know how risk is established in promielos, but it is based on leukocyte and platelet counts. In this case, it doesn't score anything for the score since you have <10,000 leukocytes and >40,000 platelets. That is, it is a low risk. Correct answer 2. Perhaps some people have been confused by option 3. One of the main causes of mortality in the induction of a promyelocyte is coagulopathy, but option 3 is totally incorrect. We could only mess things up even more by putting heparin!!!! I don't know what they are explaining right now in the academies, but it is clear that they wanted you to know that many promyelocytes, since 3 little years ago, are not treated with chemotherapy.", "type": "HEMATOLOGY", "options": {"1": "If asymptomatic, transretinoic acid (ATRA) will be initiated and controls in day hospital will be recommended.", "2": "Treatment with arsenic trioxide, ATRA and supportive therapy will be initiated.", "3": "It is a myeloblastic leukemia type M3, so chemotherapy and heparin treatment will be initiated to control disseminated intravascular coagulation.", "4": "Antibiotic treatment should be started. When the fever disappears, leukemia treatment should be started.", "5": NaN}, "correct_option": 2, "explanations": {"1": {"exist": false, "char_ranges": [], "word_ranges": [], "text": ""}, "2": {"exist": true, "char_ranges": [[298, 418]], "word_ranges": [[50, 72]], "text": "As of 2017, the PETHEMA protocol for APL in low to intermediate risk patients (and/or over 70 years of age) is ATO+ATRA."}, "3": {"exist": true, "char_ranges": [[812, 996]], "word_ranges": [[141, 173]], "text": "One of the main causes of mortality in the induction of a promyelocyte is coagulopathy, but option 3 is totally incorrect. We could only mess things up even more by putting heparin!!!!"}, "4": {"exist": false, "char_ranges": [], "word_ranges": [], "text": ""}, "5": {"exist": false, "char_ranges": [], "word_ranges": [], "text": ""}}} {"id": 277, "year": 2016, "question_id_specific": 226, "full_question": "In a colon cancer screening program, a 52-year-old patient undergoes a colonoscopy. The entire colon is normal except for the finding of a 2 cm pedunculated polyp in the sigma which is removed with a diathermy loop. The anatomopathological result indicates that there is a carcinoma in situ limited to the head of the polyp. She has a thoracoabdominal CT scan reported as normal. Indicate the correct course of action.", "full_answer": "Carcinoma in situ is delimited by the basement membrane, i.e. it is not invasive yet, and therefore the treatment is polypectomy, which is already done. Subsequently, an endoscopic follow-up (with biopsies) should be performed at an interval of 3 to 6 months depending on the patient's history, age and concomitant pathologies.", "type": "DIGESTIVE TRACT", "options": {"1": "Segmental resection of the affected colon.", "2": "Periodic endoscopic surveillance.", "3": "Local resection of the base of the polyp.", "4": "Sigmoidectomy plus lymphadenectomy.", "5": NaN}, "correct_option": 2, "explanations": {"1": {"exist": false, "char_ranges": [], "word_ranges": [], "text": ""}, "2": {"exist": true, "char_ranges": [[167, 327]], "word_ranges": [[26, 51]], "text": "an endoscopic follow-up (with biopsies) should be performed at an interval of 3 to 6 months depending on the patient's history, age and concomitant pathologies."}, "3": {"exist": false, "char_ranges": [], "word_ranges": [], "text": ""}, "4": {"exist": false, "char_ranges": [], "word_ranges": [], "text": ""}, "5": {"exist": false, "char_ranges": [], "word_ranges": [], "text": ""}}} {"id": 155, "year": 2012, "question_id_specific": 83, "full_question": "A 62-year-old man with a history of arterial hypertension under treatment with captopril, duodenal ulcer and uric acid urolithiasis. He consults for typical podagra crisis similar to others presented in the last two years. Examination showed tophi in both pinnae. Analyses show uric acid 10.1 mg/dl, creatine 1.5 mg/dl. Indicate which of the following answers is FALSE in relation to his possible treatment with allopurinol.", "full_answer": "Allopurinol is always the treatment of choice, especially in the case of tophaceous gout. Moreover, in this case, uricosuric acid could worsen urolithiasis by increasing uricosuria (sorry for the redundancy...).", "type": "RHEUMATOLOGY", "options": {"1": "Its use is second choice when uricosurics have failed.", "2": "Its prolonged use is practically obligatory because there are tophi.", "3": "Its introduction should be delayed until the current attack has resolved with anti-inflammatory drugs or colchicine.", "4": "Its introduction should be gradual up to a dose that achieves a uricemia below 6 mg/dL.", "5": "In the first months of treatment it is advisable to associate low doses of colchicine to prevent new attacks."}, "correct_option": 1, "explanations": {"1": {"exist": true, "char_ranges": [[0, 89]], "word_ranges": [[0, 14]], "text": "Allopurinol is always the treatment of choice, especially in the case of tophaceous gout."}, "2": {"exist": false, "char_ranges": [], "word_ranges": [], "text": ""}, "3": {"exist": false, "char_ranges": [], "word_ranges": [], "text": ""}, "4": {"exist": false, "char_ranges": [], "word_ranges": [], "text": ""}, "5": {"exist": false, "char_ranges": [], "word_ranges": [], "text": ""}}} {"id": 373, "year": 2016, "question_id_specific": 86, "full_question": "We are referred from ophthalmology to a 31-year-old man who consulted for loss of vision and progressive headache. Physical examination showed temporal hemianopsia and pituitary MRI showed a 35 x 30 x 20 mm turnoration compressing chiasm and cavernous sinus. The results of the functional study are cortisol 14 microg/dL, TSH 1.4 microU/ml, T4L 1.2 ng/dL, prolactin 480 microg/L (vn < 15), testosterone 160 ng/dL (vn 300-1200), FSH 1.2 U/I., (vn 5-15) and LH 2 U/L (vn 3-15). What is the initial therapeutic approach?", "full_answer": "From what is described it is a macroprolactinoma as it is a pituitary tumor of more than one centimeter, which secretes prolactin and with an insufficiency of the rest of pituitary hormones. The first line treatment of choice is dopaminergic agonists, although when the visual field is affected, surgery may be chosen. In this case, the cavernous sinus is pressured by the mass, so the surgical risk increases. Therefore, in this particular case, the surgical option seems more risky. Taking this into account and the fact that it is a tumor that produces high levels of prolactin, which indicates that it probably responds to pharmacological blockade, the most appropriate option would be option 1, treatment with dopaminergic agonists.", "type": "NEUROLOGY", "options": {"1": "Dopaminergic agonists.", "2": "Transsphenoidal surgery.", "3": "External radiotherapy or radiosurgery.", "4": "Treatment of hypogonadotropic hypogonadism with testosterone.", "5": NaN}, "correct_option": 1, "explanations": {"1": {"exist": true, "char_ranges": [[0, 251]], "word_ranges": [[0, 41]], "text": "From what is described it is a macroprolactinoma as it is a pituitary tumor of more than one centimeter, which secretes prolactin and with an insufficiency of the rest of pituitary hormones. The first line treatment of choice is dopaminergic agonists,"}, "2": {"exist": true, "char_ranges": [[319, 484]], "word_ranges": [[52, 79]], "text": "In this case, the cavernous sinus is pressured by the mass, so the surgical risk increases. Therefore, in this particular case, the surgical option seems more risky."}, "3": {"exist": false, "char_ranges": [], "word_ranges": [], "text": ""}, "4": {"exist": false, "char_ranges": [], "word_ranges": [], "text": ""}, "5": {"exist": false, "char_ranges": [], "word_ranges": [], "text": ""}}} {"id": 523, "year": 2021, "question_id_specific": 124, "full_question": "A 45-year-old patient was diagnosed with HIV infection three months ago, presenting at that time with CD4 45 lymphocytes/µl and an HIV-1 viral load of 500,000 copies/ml. At that time he had a negative Mantoux test. He started treatment with an integrase inhibitor and two reverse transcriptase inhibitors, and at one month he had 25,000 copies and CD4 had risen to 80/µL. She consulted for presenting a picture of cervical lymphadenopathy and fever of two weeks of evolution. The puncture of one of the lymph nodes showed isolated acid-fast bacilli and epithelioid granulomas:", "full_answer": "We are dealing with a patient with HIV infection in a situation of severe immunosuppression (<50 CD4+) and a very high viral load, who has followed the expected evolution from the immunovirological point of view after the start of treatment (after 12 weeks, the viral load has ostensibly decreased - answer 2 would not be correct, remember that it is desirable that it becomes negative 24 weeks after the start of treatment, although in patients with higher CV like ours, it could take longer - and the CD4+ count has begun to rise). A negative Mantoux test does not rule out tuberculosis (incorrect answer 4), since in patients with severe immunosuppression it can give false negatives. The diagnosis of lymph node tuberculosis is immediate when detecting \"isolated acid-fast bacilli and epithelioid granulomas\" in the puncture of one of the lymph nodes, however: considering the patient profile, this process will have been clinically unmasked during the immunological reconstitution and the infection is not the consequence of an incomplete immunological recovery (correct answer number 3).", "type": "INFECTIOUS DISEASES", "options": {"1": "It is a lymph node tuberculosis related to an incomplete immunologic recovery.", "2": "Treatment failure and it is an opportunistic infection.", "3": "This is an opportunistic infection unmasked within a picture of immune reconstitution.", "4": "A negative Mantoux test rules out tuberculosis.", "5": NaN}, "correct_option": 3, "explanations": {"1": {"exist": true, "char_ranges": [[688, 1093]], "word_ranges": [[116, 173]], "text": "The diagnosis of lymph node tuberculosis is immediate when detecting \"isolated acid-fast bacilli and epithelioid granulomas\" in the puncture of one of the lymph nodes, however: considering the patient profile, this process will have been clinically unmasked during the immunological reconstitution and the infection is not the consequence of an incomplete immunological recovery (correct answer number 3)."}, "2": {"exist": true, "char_ranges": [[300, 533]], "word_ranges": [[50, 93]], "text": "answer 2 would not be correct, remember that it is desirable that it becomes negative 24 weeks after the start of treatment, although in patients with higher CV like ours, it could take longer - and the CD4+ count has begun to rise)."}, "3": {"exist": true, "char_ranges": [[688, 1093]], "word_ranges": [[116, 173]], "text": "The diagnosis of lymph node tuberculosis is immediate when detecting \"isolated acid-fast bacilli and epithelioid granulomas\" in the puncture of one of the lymph nodes, however: considering the patient profile, this process will have been clinically unmasked during the immunological reconstitution and the infection is not the consequence of an incomplete immunological recovery (correct answer number 3)."}, "4": {"exist": true, "char_ranges": [[534, 687]], "word_ranges": [[93, 116]], "text": "A negative Mantoux test does not rule out tuberculosis (incorrect answer 4), since in patients with severe immunosuppression it can give false negatives."}, "5": {"exist": false, "char_ranges": [], "word_ranges": [], "text": ""}}} {"id": 417, "year": 2018, "question_id_specific": 76, "full_question": "78-year-old woman with dementia and institutionalized is brought by her caregivers for significant abdominal pain with deterioration of general condition and abdominal distension. CBC shows leukocytosis, elevated hematocrit, renal failure and metabolic acidosis. ECG shows atrial fibrillation. Abdominal tomography shows edematous small bowel loops, with intestinal and portal accumulation. The most probable diagnosis is:", "full_answer": "Typical case of acute mesenteric ischemia, which should be suspected in elderly patients with abdominal pain and distension of sudden onset, diarrhea or vomiting and who present cardiovascular risk factors, especially emboligenic heart disease. In this case we are given characteristic biochemical data (leukocytosis, metabolic acidosis due to lactic acid, hemoconcentration due to loss of fluid to the third space...) and imaging tests (dilated loops, pneumatosis, gas at portal level...) suggestive of mesenteric ischemia.", "type": "GENERAL SURGERY", "options": {"1": "Perforation of gastric or duodenal ulcus.", "2": "Biliary leak.", "3": "Obstructive neoplasia of the sigma with perforation.", "4": "Mesenteric ischemia.", "5": NaN}, "correct_option": 4, "explanations": {"1": {"exist": false, "char_ranges": [], "word_ranges": [], "text": ""}, "2": {"exist": false, "char_ranges": [], "word_ranges": [], "text": ""}, "3": {"exist": false, "char_ranges": [], "word_ranges": [], "text": ""}, "4": {"exist": true, "char_ranges": [[0, 244]], "word_ranges": [[0, 34]], "text": "Typical case of acute mesenteric ischemia, which should be suspected in elderly patients with abdominal pain and distension of sudden onset, diarrhea or vomiting and who present cardiovascular risk factors, especially emboligenic heart disease."}, "5": {"exist": false, "char_ranges": [], "word_ranges": [], "text": ""}}} {"id": 403, "year": 2016, "question_id_specific": 135, "full_question": "A 25-year-old patient, during a tennis match, has severe pain in his neck and left eye. The next morning he wakes up with a feeling of gait instability and has palpebral ptosis of the left eye and anisocoria, with the left pupil being smaller than the right. The patient maintains good visual acuity, where would you most likely locate the lesion?", "full_answer": "Claude-Bernard-Horner syndrome is an old acquaintance of the MIR exam. We do not see it daily in clinical practice, but it appears in many exams. Characteristic is upper eyelid ptosis and miosis of that eye. This occurs due to a deficit of sympathetic innervation of the orbital territory. The sympathetic nervous system innervates the Müller muscle, which is an accessory muscle to the levator of the upper eyelid. If the Müller ceases to function, the eyelid descends slightly. It also innervates the iris dilator muscle. Therefore, the sphincter muscle, its antagonist, is unopposed and the balance of the pupil shifts toward miosis. It explains ptosis and miosis of the left eye, so we suspect Horner's in that eye. She also describes what may have happened: after a possible sudden movement while playing tennis, she has neck pain and a Horner's. We should suspect an aneurysm. We should suspect a dissecting carotid aneurysm, which is one of the best known (and most serious) causes of an acquired Horner's syndrome. The superior cervical ganglion is a ganglion of the sympathetic nervous system located in the sheath of the carotid artery, and at this level it has been affected. We can also try to get this answer right by ruling out. An alteration of the III cranial nerve can produce ptosis, but the pupil, if affected, should be in mydriasis. And damage to the third cranial nerve should also produce some type of ophthalmoplegia, with diplopia. An involvement of the optic chiasm would produce a bilateral loss of vision (probably a bitemporal hemianopsia). We are told in the statement that he maintains good visual acuity. On the other hand, there would also be no ptosis or anisocoria. And finally, an alteration of the medulla oblongata would produce different neurological symptoms not described, and there should be no ptosis or pupillary alterations, because the third pair is located in the midbrain and the sympathetic nuclei in the spinal cord.", "type": "OPHTHALMOLOGY (ECTOPIC)", "options": {"1": "III cranial nerve.", "2": "Optic chiasm.", "3": "Upper cervical ganglion.", "4": "Spinal cord.", "5": NaN}, "correct_option": 3, "explanations": {"1": {"exist": true, "char_ranges": [[1243, 1353]], "word_ranges": [[208, 227]], "text": "An alteration of the III cranial nerve can produce ptosis, but the pupil, if affected, should be in mydriasis."}, "2": {"exist": true, "char_ranges": [[1457, 1636]], "word_ranges": [[243, 272]], "text": "An involvement of the optic chiasm would produce a bilateral loss of vision (probably a bitemporal hemianopsia). We are told in the statement that he maintains good visual acuity."}, "3": {"exist": true, "char_ranges": [[852, 1186]], "word_ranges": [[140, 196]], "text": "We should suspect an aneurysm. We should suspect a dissecting carotid aneurysm, which is one of the best known (and most serious) causes of an acquired Horner's syndrome. The superior cervical ganglion is a ganglion of the sympathetic nervous system located in the sheath of the carotid artery, and at this level it has been affected."}, "4": {"exist": true, "char_ranges": [[1714, 1966]], "word_ranges": [[286, 325]], "text": "an alteration of the medulla oblongata would produce different neurological symptoms not described, and there should be no ptosis or pupillary alterations, because the third pair is located in the midbrain and the sympathetic nuclei in the spinal cord."}, "5": {"exist": false, "char_ranges": [], "word_ranges": [], "text": ""}}} {"id": 42, "year": 2011, "question_id_specific": 150, "full_question": "A one-and-a-half year old boy comes to the emergency room with abdominal pain and jaundice. On examination, an abdominal mass is palpated. An ultrasound shows the presence of a common bile duct cyst. What therapeutic approach should we adopt?", "full_answer": "The correct answer is 4. A common bile duct cyst is a congenital pathology that usually presents clinically beyond the neonatal period. It is appropriate to have a quality image that delimits the lesion in order to decide the surgical approach.", "type": "PEDIATRICS", "options": {"1": "A percutaneous puncture for peritoneal lavage will be performed to verify that the cyst contains bile in order to leave a drainage to improve the abdominal pain.", "2": "An exploratory laparotomy will be indicated and a radical resection of the entire biliary tract will be performed to replace it with a loop of intestine.", "3": "A laparotomy will be indicated to drain the cyst and when the dilatation remits the drainage will be removed.", "4": "A cholangio-resonance will be performed to delimit the cyst and a laparotomy will be indicated for cyst resection and anastomosis of the biliary tract.", "5": "A HIDA gammagraphic study is necessary to delimit the cyst and to be able to perform percutaneous drainage safely."}, "correct_option": 4, "explanations": {"1": {"exist": false, "char_ranges": [], "word_ranges": [], "text": ""}, "2": {"exist": false, "char_ranges": [], "word_ranges": [], "text": ""}, "3": {"exist": false, "char_ranges": [], "word_ranges": [], "text": ""}, "4": {"exist": true, "char_ranges": [[25, 244]], "word_ranges": [[5, 41]], "text": "A common bile duct cyst is a congenital pathology that usually presents clinically beyond the neonatal period. It is appropriate to have a quality image that delimits the lesion in order to decide the surgical approach."}, "5": {"exist": false, "char_ranges": [], "word_ranges": [], "text": ""}}} {"id": 445, "year": 2018, "question_id_specific": 233, "full_question": "45-year-old woman. Two previous pregnancies with normal deliveries (G2PN2). Heavy menstrual bleeding for approximately 1 year. Ultrasound shows a 2 cm subserosal myoma that has been stable for several years. Endometrial biopsy was performed and it was normal. The patient has hemoglobin 10 g/dL despite oral ferrotherapy and the amount of menstrual flow has not decreased despite treatment with tranexamic acid and mefenamic acid. Which of the following is the treatment of first choice in this patient?", "full_answer": "According to the SEGO, in perimenopausal women with heavy and/or prolonged menstrual bleeding who require contraception, the levonorgestrel IUD (also called MIRENA IUD) is the first option.", "type": "GYNECOLOGY AND OBSTETRICS", "options": {"1": "Total hysterectomy preserving appendages.", "2": "Combined oral contraceptives.", "3": "Endometrial ablation.", "4": "Levonorgestrel IUD.", "5": NaN}, "correct_option": 4, "explanations": {"1": {"exist": false, "char_ranges": [], "word_ranges": [], "text": ""}, "2": {"exist": false, "char_ranges": [], "word_ranges": [], "text": ""}, "3": {"exist": false, "char_ranges": [], "word_ranges": [], "text": ""}, "4": {"exist": true, "char_ranges": [[0, 189]], "word_ranges": [[0, 27]], "text": "According to the SEGO, in perimenopausal women with heavy and/or prolonged menstrual bleeding who require contraception, the levonorgestrel IUD (also called MIRENA IUD) is the first option."}, "5": {"exist": false, "char_ranges": [], "word_ranges": [], "text": ""}}} {"id": 511, "year": 2021, "question_id_specific": 152, "full_question": "A 79-year-old hypertensive woman on treatment with olmesartan comes for consultation for watery diarrhea of 4 to 6 stools per day for the past two months. Three months ago she was treated with nonsteroidal anti-inflammatory drugs for 3 weeks for lumbosciatica. Gastroscopy and colonoscopy were performed, both without macroscopic alterations. Duodenal biopsies were normal, while colon biopsies showed a chronic inflammatory infiltrate of the lamina propria with an irregular band of collagen immediately below the mucosal surface epithelium with a thickness greater than 10 mm and a number of intraepithelial lymphocytes >20 per 100 epithelial cells. Which is the most likely diagnosis?", "full_answer": "The diagnostic criteria for microscopic colitis are: a)Chronic or intermittent non-bloody watery diarrhea b)Colonic mucosa assessed by colonoscopy macroscopically normal or near normal c)Characteristic histopathological findings. In our case, the patient presents findings compatible with collagenous colitis (band of collagen underlying the epithelium greater than 10 microns) and lymphocytic (more than 20 intraepithelial lymphocytes). The treatment of this pathology is topical corticosteroids with low oral bioavailability (mainly budesonide). Olmesartan enteropathy produces a picture similar to celiac disease and is characterized by duodenal biopsy with villous atrophy, mucosal inflammation with increased intraepithelial lymphocytes and cryptitis. In this case a normal duodenal biopsy is described, but it would be interesting to keep the concept for future examinations.", "type": "DIGESTIVE", "options": {"1": "Microscopic colitis.", "2": "Enteropathy due to NSAIDs.", "3": "Olmesartan-associated enteropathy.", "4": "Irritable bowel syndrome.", "5": NaN}, "correct_option": 1, "explanations": {"1": {"exist": true, "char_ranges": [[243, 377]], "word_ranges": [[29, 47]], "text": "the patient presents findings compatible with collagenous colitis (band of collagen underlying the epithelium greater than 10 microns)"}, "2": {"exist": false, "char_ranges": [], "word_ranges": [], "text": ""}, "3": {"exist": true, "char_ranges": [[548, 756]], "word_ranges": [[68, 94]], "text": "Olmesartan enteropathy produces a picture similar to celiac disease and is characterized by duodenal biopsy with villous atrophy, mucosal inflammation with increased intraepithelial lymphocytes and cryptitis."}, "4": {"exist": false, "char_ranges": [], "word_ranges": [], "text": ""}, "5": {"exist": false, "char_ranges": [], "word_ranges": [], "text": ""}}} {"id": 33, "year": 2011, "question_id_specific": 65, "full_question": "A 64-year-old man, diagnosed with myasthenia gravis 1 year ago, treated with low-dose steroids (3 mg/day of deflazacort) and anticholinesterase drugs, asymptomatic for 6 months, consults for mild difficulty in swallowing and evening diplopia for a few days. He was admitted to the hospital for suspected myasthenic crisis and treatment was started. The next day the patient alerts at 03 hours in the morning because the patient makes a strange noise when breathing in, like a soft snoring, the patient is sound asleep and very sweaty, but does not seem to be fatigued. What is the most correct attitude?", "full_answer": "This is a patient with decompensated myasthenia with bulbar symptoms (dysphagia) and therefore risk of respiratory muscle involvement. The respiratory symptoms correspond to an exhaustion of the muscles, first inspiratory and then accessory (the patient does not appear fatigued or tachypneic because the muscles are exhausted), progressing to a carbonic coma.", "type": "NEUROLOGY AND NEUROSURGERY", "options": {"1": "Reassure the nurse and family that the patient is a habitual snorer and is peacefully asleep. The patient should be placed in lateral decubitus.", "2": "Notify intensive care for suspected acute respiratory failure, to assess possible orotracheal intubation and assisted ventilation.", "3": "Perform polysomnographic study to rule out sleep apnea.", "4": "Decrease steroid dose; if he has a steroid myopathy he will improve.", "5": "Order a thoracic CT scan to rule out compressive thymoma over the trachea associated with myasthenia."}, "correct_option": 2, "explanations": {"1": {"exist": false, "char_ranges": [], "word_ranges": [], "text": ""}, "2": {"exist": true, "char_ranges": [[0, 134]], "word_ranges": [[0, 18]], "text": "This is a patient with decompensated myasthenia with bulbar symptoms (dysphagia) and therefore risk of respiratory muscle involvement."}, "3": {"exist": false, "char_ranges": [], "word_ranges": [], "text": ""}, "4": {"exist": false, "char_ranges": [], "word_ranges": [], "text": ""}, "5": {"exist": false, "char_ranges": [], "word_ranges": [], "text": ""}}} {"id": 129, "year": 2012, "question_id_specific": 81, "full_question": "A 78-year-old male consults for a year-long history of progressive cognitive impairment with memory and orientation lapses. His family reports recurrent visual hallucinations, occasional falls and striking motor slowing. What is the most likely diagnosis?", "full_answer": "Dementia with visual hallucinations at the beginning of the course (in the first 2 years) is very suggestive of Lewy bodies, and is the guiding symptom in these questions. If we add to this the falls and motor slowing suggestive of parkinsonism, it makes the question easier. Therefore, number 3 is correct.", "type": "NEUROLOGY AND NEUROSURGERY", "options": {"1": "Multi-infarct vascular dementia.", "2": "Alzheimer's type dementia.", "3": "Dementia with diffuse Lewy bodies.", "4": "Frontotemporal dementia.", "5": "Normal pressure hydrocephalus."}, "correct_option": 3, "explanations": {"1": {"exist": false, "char_ranges": [], "word_ranges": [], "text": ""}, "2": {"exist": false, "char_ranges": [], "word_ranges": [], "text": ""}, "3": {"exist": true, "char_ranges": [[0, 307]], "word_ranges": [[0, 52]], "text": "Dementia with visual hallucinations at the beginning of the course (in the first 2 years) is very suggestive of Lewy bodies, and is the guiding symptom in these questions. If we add to this the falls and motor slowing suggestive of parkinsonism, it makes the question easier. Therefore, number 3 is correct."}, "4": {"exist": false, "char_ranges": [], "word_ranges": [], "text": ""}, "5": {"exist": false, "char_ranges": [], "word_ranges": [], "text": ""}}} {"id": 503, "year": 2020, "question_id_specific": 80, "full_question": "An 8-year-old boy consults for the appearance of pubic hair at the base of the penis for the last 12 months. The testicle has not increased in volume and the penis has not increased in size. There is no increase in growth rate in the last year. The bone age is one year older than the chronological age. What is the most likely diagnosis?", "full_answer": "In the male, the onset of puberty is considered normal from the age of 9 years, therefore in this case it is a premature puberty. The onset of puberty is defined as an increase in testicular volume (greater than or equal to 4cc); by specifying that the testicular size has not increased, we can rule out options 1 and 2. The excessive secretion of androgens produced by an adrenal tumor would cause a greater amount of hair and an increase in penile size, which our patient does not present. Therefore, the correct option is early adrenarche.", "type": "PEDIATRICS", "options": {"1": "Central precocious puberty.", "2": "Peripheral precocious puberty.", "3": "Early adrenarche.", "4": "Adrenal tumor.", "5": NaN}, "correct_option": 3, "explanations": {"1": {"exist": true, "char_ranges": [[321, 542]], "word_ranges": [[60, 96]], "text": "The excessive secretion of androgens produced by an adrenal tumor would cause a greater amount of hair and an increase in penile size, which our patient does not present. Therefore, the correct option is early adrenarche."}, "2": {"exist": true, "char_ranges": [[321, 542]], "word_ranges": [[60, 96]], "text": "The excessive secretion of androgens produced by an adrenal tumor would cause a greater amount of hair and an increase in penile size, which our patient does not present. Therefore, the correct option is early adrenarche."}, "3": {"exist": true, "char_ranges": [[321, 542]], "word_ranges": [[60, 96]], "text": "The excessive secretion of androgens produced by an adrenal tumor would cause a greater amount of hair and an increase in penile size, which our patient does not present. Therefore, the correct option is early adrenarche."}, "4": {"exist": false, "char_ranges": [], "word_ranges": [], "text": ""}, "5": {"exist": false, "char_ranges": [], "word_ranges": [], "text": ""}}} {"id": 300, "year": 2016, "question_id_specific": 106, "full_question": "A 32-year-old man from Cameroon consults for fever cough and left hemithorax pain of 1 month's evolution. He took amoxicillin-clavulanic acid for 1 week without improvement of symptoms. A blood test showed a white blood cell count of 8000/microL and a hemoglobin of 12.8 g/dL. Chest X-ray showed a loculated left pleural effusion occupying one third of the hemithorax. A thoracentesis shows a yellowish fluid with the following features: red blood cells 2000/uL, leukocytes 2500/uL, with 90% lymphocytes, protein 4.9 g/dL, lactate dehydrogenase 550 U/L, glucose 67 mg/dL, and absence of malignant cells on cytology. Which of the following tests would be most useful in diagnosing the cause of the pleural effusion?", "full_answer": "The determination of ADA, in the presence of a pneumonia of long evolution that does not improve with antibiotic treatment, with an effusion with abundant leukocytes and lymphocytic predominance, in which the presence of neoplasia is ruled out, with values higher than 70 UI/L, should lead us to have a high degree of suspicion about tuberculous etiology and send the pleural fluid to microbiology for tuberculosis study.", "type": "INFECTIOUS DISEASES", "options": {"1": "A thoracic computed tomography (CT) scan.", "2": "Pleural fluid pH measurement.", "3": "Measurement of adenosine deaminase in pleural fluid.", "4": "Tuberculin test.", "5": NaN}, "correct_option": 3, "explanations": {"1": {"exist": false, "char_ranges": [], "word_ranges": [], "text": ""}, "2": {"exist": false, "char_ranges": [], "word_ranges": [], "text": ""}, "3": {"exist": true, "char_ranges": [[0, 421]], "word_ranges": [[0, 67]], "text": "The determination of ADA, in the presence of a pneumonia of long evolution that does not improve with antibiotic treatment, with an effusion with abundant leukocytes and lymphocytic predominance, in which the presence of neoplasia is ruled out, with values higher than 70 UI/L, should lead us to have a high degree of suspicion about tuberculous etiology and send the pleural fluid to microbiology for tuberculosis study."}, "4": {"exist": false, "char_ranges": [], "word_ranges": [], "text": ""}, "5": {"exist": false, "char_ranges": [], "word_ranges": [], "text": ""}}} {"id": 434, "year": 2018, "question_id_specific": 119, "full_question": "A 38-year-old woman of veterinary profession, in charge of monitoring wild animals and assisting in the delivery of domestic livestock. She starts with a high fever with chills, headache, myalgia and non-productive cough that she interprets as a flu-like process. She presented with chest pain. Chest X-ray showed bilateral pulmonary infiltrates in lower fields. A serologic test was performed with elevated antibody titers against phase I antigens. Which of the following statements is TRUE?", "full_answer": "Acute infection by Coxiella burnetii manifested in this case by Pneumonia. Coxiella infection is not transmitted by ticks. The contact can be with high probability at the time of delivery of an infected farm animal, in which the bacterium is disseminated in aerosols. The treatment of choice is with doxycycline in the acute phase (if meningitis is present, quinolones are preferred due to penetration of the blood-brain barrier). The combination of doxycycline plus hicroxychloroquine can be used, but for chronic Q fever (which is where elevation of antibodies against phase I antigens is observed and carries a higher risk of mortality).", "type": "INFECTIOUS DISEASES AND MICROBIOLOGY", "options": {"1": "The way of transmission of this entity is by ticks.", "2": "Both doxycycline and hydroxychloroquine are effective in treating acute forms of this disease.", "3": "In its acute form it also presents, generally, elevation of antibodies against phase I antigens.", "4": "Mortality in acute forms is almost nonexistent.", "5": NaN}, "correct_option": 4, "explanations": {"1": {"exist": true, "char_ranges": [[75, 122]], "word_ranges": [[11, 18]], "text": "Coxiella infection is not transmitted by ticks."}, "2": {"exist": true, "char_ranges": [[431, 640]], "word_ranges": [[68, 101]], "text": "The combination of doxycycline plus hicroxychloroquine can be used, but for chronic Q fever (which is where elevation of antibodies against phase I antigens is observed and carries a higher risk of mortality)."}, "3": {"exist": false, "char_ranges": [], "word_ranges": [], "text": ""}, "4": {"exist": false, "char_ranges": [], "word_ranges": [], "text": ""}, "5": {"exist": false, "char_ranges": [], "word_ranges": [], "text": ""}}} {"id": 563, "year": 2022, "question_id_specific": 175, "full_question": "A 62-year-old patient who consults because of close contact in recent weeks with a person with active tuberculosis. Mantoux test is positive (12 mm). His history includes treatment with oral anticoagulants (acenocoumarol) for a deep vein thrombosis a few months ago. He does not report fever or cough and is asymptomatic. Chest X-ray is normal. Which of the following treatments is the most advisable?", "full_answer": "As of 2018, there are four dosing schedules for treatment of latent TB infection recommended by the CDC using isoniazid (INH), rifapentine (RPT) or rifampin (RIF). All treatments are effective. Health care providers should prescribe the shorter treatments, which are more convenient, whenever possible. The shortest combination would be Isoniazid + Rifapentine for three months or Rifampicin for four months. However, the fact that he is taking acenocoumarol means that the indication for this patient is Isoniazid for 9 months, since there is interaction between the rifamycin derivatives (rifapentine and rifampicin).", "type": "PREVENTIVE MEDICINE", "options": {"1": "Isoniazid, for nine months.", "2": "Rifampicin, for four months.", "3": "Isoniazid and rifampicin, for three months.", "4": "Isoniazid and rifapentine, for three months.", "5": NaN}, "correct_option": 4, "explanations": {"1": {"exist": false, "char_ranges": [], "word_ranges": [], "text": ""}, "2": {"exist": false, "char_ranges": [], "word_ranges": [], "text": ""}, "3": {"exist": false, "char_ranges": [], "word_ranges": [], "text": ""}, "4": {"exist": false, "char_ranges": [], "word_ranges": [], "text": ""}, "5": {"exist": false, "char_ranges": [], "word_ranges": [], "text": ""}}} {"id": 89, "year": 2012, "question_id_specific": 57, "full_question": "A 64-year-old man consults for right calf pain when climbing hills and stairs and when walking on level ground for a prolonged period of time. He reports that the pain becomes so intense with exertion that he is forced to stand up, which causes the pain to improve in a matter of minutes. As personal history, the patient is a smoker of 20 cigarettes a day, diabetic on metformin and hypertensive. On examination, there is good peripheral perfusion of the right foot, but the patient has a decreased pedal pulse. Which of the following complementary tests should be ordered initially for the diagnosis and assessment of the severity of the patient's disease?", "full_answer": "Although the diagnosis of peripheral arteriopathy is fundamentally clinical (in this patient the picture is very typical), we can objectively assess the severity of the disease with the use of bloodless techniques. The most important is the ankle-brachial index. Therefore, answer 2 is correct. Imaging techniques (arteriography, angioCT or angioMRI) are used when surgical repair of arterial occlusion is considered.", "type": "CARDIOLOGY AND VASCULAR SURGERY", "options": {"1": "Arteriography of lower extremities.", "2": "Ankle-brachial index.", "3": "Computerized axial tomography with vascular reconstruction of lower extremities.", "4": "Tape stress test.", "5": "Magnetic resonance imaging."}, "correct_option": 2, "explanations": {"1": {"exist": true, "char_ranges": [[295, 417]], "word_ranges": [[44, 60]], "text": "Imaging techniques (arteriography, angioCT or angioMRI) are used when surgical repair of arterial occlusion is considered."}, "2": {"exist": true, "char_ranges": [[123, 262]], "word_ranges": [[17, 39]], "text": "we can objectively assess the severity of the disease with the use of bloodless techniques. The most important is the ankle-brachial index."}, "3": {"exist": false, "char_ranges": [], "word_ranges": [], "text": ""}, "4": {"exist": false, "char_ranges": [], "word_ranges": [], "text": ""}, "5": {"exist": false, "char_ranges": [], "word_ranges": [], "text": ""}}} {"id": 586, "year": 2022, "question_id_specific": 75, "full_question": "A 30-year-old woman requesting a contraceptive method. She has a history of an emergency cesarean section 7 months ago, giving birth to a 3,550 g baby girl. She is exclusively breastfeeding. Point out the correct statement:", "full_answer": "Given her circumstances and taking into account that she is exclusively breastfeeding, the best thing to do would be to prescribe only gestagens.", "type": "OBSTETRICS AND GYNECOLOGY", "options": {"1": "Explain to her that, if she is amenorrheic, she does not require another contraceptive method since breastfeeding is sufficient as a contraceptive method.", "2": "He informed her that the levonogestrel IUD is contraindicated because she had a cesarean section less than a year ago.", "3": "He explains that she could use a gestagen-only hormonal method.", "4": "She is offered combined hormonal contraception.", "5": NaN}, "correct_option": 3, "explanations": {"1": {"exist": false, "char_ranges": [], "word_ranges": [], "text": ""}, "2": {"exist": false, "char_ranges": [], "word_ranges": [], "text": ""}, "3": {"exist": true, "char_ranges": [[0, 145]], "word_ranges": [[0, 23]], "text": "Given her circumstances and taking into account that she is exclusively breastfeeding, the best thing to do would be to prescribe only gestagens."}, "4": {"exist": false, "char_ranges": [], "word_ranges": [], "text": ""}, "5": {"exist": false, "char_ranges": [], "word_ranges": [], "text": ""}}} {"id": 461, "year": 2018, "question_id_specific": 148, "full_question": "A 59-year-old woman with recent onset of epileptic seizures comes to the emergency department. On examination she presents left pyramidal signs and papillary edema. A brain MRI shows a right hemispheric mass with edema, midline deviation and signs of tentorial herniation. She reports that for the past week she has been suffering from headache that has been progressing in intensity. Which of the following features associated with headache seems most likely for this patient?", "full_answer": "It is a headache in the context of papilledema. In this case the papilledema is due to increased intracranial pressure due to a tumor. The headache due to increased intracranial pressure is of positional type (it increases with Valsalva maneuvers and when bending down), so option 2 is not correct. In addition, it is predominantly morning (option 1 is correct) because in the morning intracranial pressure naturally rises. Migraine-type headaches may be associated with photophobia or sonophobia (answers 3 and 4). But this headache is not a migraine headache.", "type": "OPHTHALMOLOGY (ECTOPIC)", "options": {"1": "Morning predominance.", "2": "It does not change with effort.", "3": "Photophobia.", "4": "Sonophobia.", "5": NaN}, "correct_option": 1, "explanations": {"1": {"exist": true, "char_ranges": [[299, 423]], "word_ranges": [[50, 68]], "text": "In addition, it is predominantly morning (option 1 is correct) because in the morning intracranial pressure naturally rises."}, "2": {"exist": true, "char_ranges": [[135, 298]], "word_ranges": [[24, 50]], "text": "The headache due to increased intracranial pressure is of positional type (it increases with Valsalva maneuvers and when bending down), so option 2 is not correct."}, "3": {"exist": true, "char_ranges": [[424, 561]], "word_ranges": [[68, 89]], "text": "Migraine-type headaches may be associated with photophobia or sonophobia (answers 3 and 4). But this headache is not a migraine headache."}, "4": {"exist": true, "char_ranges": [[424, 561]], "word_ranges": [[68, 89]], "text": "Migraine-type headaches may be associated with photophobia or sonophobia (answers 3 and 4). But this headache is not a migraine headache."}, "5": {"exist": false, "char_ranges": [], "word_ranges": [], "text": ""}}} {"id": 547, "year": 2022, "question_id_specific": 125, "full_question": "A 23-year-old male diagnosed with Wolff-Parkinson-White syndrome who comes to the emergency department for palpitations. The ECG shows an irregular wide QRS tachycardia, with a heart rate of 205 bpm, compatible with pre-excited atrial fibrillation. Which of the following drugs do you consider appropriate to stop the tachycardia:", "full_answer": "Procainamide. The treatment of choice for Wolff-Parkinson-White syndrome is cardioversion with direct current. Heart rate slowing drugs, commonly used in atrial fibrillation, are ineffective, and digoxin and non-dihydropyridine calcium channel blockers (e.g., verapamil, diltiazem) are contraindicated because they may increase the ventricular rate and cause ventricular fibrillation. If cardioversion is not possible, drugs that prolong the refractory period of accessory connection should be used. IV procainamide or amiodarone is preferred, but any class Ia, Ic or III antiarrhythmic drug can be used.", "type": "CARDIOLOGY", "options": {"1": "Verapamil.", "2": "Digoxin.", "3": "Adenosine.", "4": "Procainamide.", "5": NaN}, "correct_option": 4, "explanations": {"1": {"exist": true, "char_ranges": [[196, 384]], "word_ranges": [[25, 47]], "text": "digoxin and non-dihydropyridine calcium channel blockers (e.g., verapamil, diltiazem) are contraindicated because they may increase the ventricular rate and cause ventricular fibrillation."}, "2": {"exist": true, "char_ranges": [[196, 384]], "word_ranges": [[25, 47]], "text": "digoxin and non-dihydropyridine calcium channel blockers (e.g., verapamil, diltiazem) are contraindicated because they may increase the ventricular rate and cause ventricular fibrillation."}, "3": {"exist": true, "char_ranges": [[111, 191]], "word_ranges": [[13, 24]], "text": "Heart rate slowing drugs, commonly used in atrial fibrillation, are ineffective,"}, "4": {"exist": true, "char_ranges": [[385, 543]], "word_ranges": [[47, 70]], "text": "If cardioversion is not possible, drugs that prolong the refractory period of accessory connection should be used. IV procainamide or amiodarone is preferred,"}, "5": {"exist": false, "char_ranges": [], "word_ranges": [], "text": ""}}} {"id": 193, "year": 2013, "question_id_specific": 161, "full_question": "In a 3-month-old infant all the clinical manifestations outlined below would make it necessary to rule out cystic fibrosis, EXCEPT:", "full_answer": "Dehydration may be due to diarrhea of other etiologies. I have doubts about answer 3 since it does not specify which secretions are involved. We have to understand that the growth of Pseudomona aeruginosa is in the culture of bronchial secretions. Because clearly the indication to perform a sweat chlorine (to rule out FQP) are the HIPOnatremic dehydrations and not the hypernatremic ones as the answer says.", "type": "PEDIATRICS", "options": {"1": "Delayed meconium evacuation.", "2": "Salty taste of the skin.", "3": "Isolation of secretions of Pseudomonas aeruginosa.", "4": "Deterioration/detention of the weight curve.", "5": "Hypernatremic dehydration."}, "correct_option": 5, "explanations": {"1": {"exist": false, "char_ranges": [], "word_ranges": [], "text": ""}, "2": {"exist": false, "char_ranges": [], "word_ranges": [], "text": ""}, "3": {"exist": false, "char_ranges": [], "word_ranges": [], "text": ""}, "4": {"exist": false, "char_ranges": [], "word_ranges": [], "text": ""}, "5": {"exist": true, "char_ranges": [[256, 409]], "word_ranges": [[42, 67]], "text": "clearly the indication to perform a sweat chlorine (to rule out FQP) are the HIPOnatremic dehydrations and not the hypernatremic ones as the answer says."}}} {"id": 431, "year": 2018, "question_id_specific": 116, "full_question": "A 38-year-old man comes to the emergency department with fever above 39ºC and deterioration of consciousness after a trip to Equatorial Guinea without antimalarial prophylaxis. Laboratory tests showed creatinine 3.4 mg/dL, AST 764 UlL, ALT 678 UlL. The laboratory reports the visualization of Plasmodium falciparum in the blood with a parasitemia level of 6%. What treatment would you initiate at this time?", "full_answer": "In severe cases of Plasmodium infection, the greatest risk of mortality occurs in the first 24 hours of clinical presentation, so treatment should be started immediately. Treatment with parenteral or intramuscular artesunate is more indicated in severe Plasmodium infection, both in adults and children and in pregnant women.", "type": "INFECTIOUS DISEASES AND MICROBIOLOGY", "options": {"1": "Mefloquine orally.", "2": "Intravenous artesunate.", "3": "Doxycycline orally.", "4": "Oral quinine sulfate.", "5": NaN}, "correct_option": 2, "explanations": {"1": {"exist": false, "char_ranges": [], "word_ranges": [], "text": ""}, "2": {"exist": true, "char_ranges": [[0, 325]], "word_ranges": [[0, 48]], "text": "In severe cases of Plasmodium infection, the greatest risk of mortality occurs in the first 24 hours of clinical presentation, so treatment should be started immediately. Treatment with parenteral or intramuscular artesunate is more indicated in severe Plasmodium infection, both in adults and children and in pregnant women."}, "3": {"exist": false, "char_ranges": [], "word_ranges": [], "text": ""}, "4": {"exist": false, "char_ranges": [], "word_ranges": [], "text": ""}, "5": {"exist": false, "char_ranges": [], "word_ranges": [], "text": ""}}} {"id": 569, "year": 2022, "question_id_specific": 152, "full_question": "24-year-old male presenting with dysmorphic microhematuria, proteinuria of 3 g/24 h, eGFR (CKD-EPI) 85 ml/min and sensorineural hearing loss. He reports that his maternal grandmother required dialysis at 70 years of age and both his mother and younger sister have isolated microhematuria. Which of the following diseases is more likely?", "full_answer": "Textbook case, little to add. Microhematuria, proteinuria and sensorineural hearing loss in a young male with a family history of women with hematuria = Alport syndrome.", "type": "NEPHROLOGY", "options": {"1": "Alport syndrome.", "2": "IgA nephropathy.", "3": "Fabry disease.", "4": "Autosomal dominant polycystic kidney disease.", "5": NaN}, "correct_option": 1, "explanations": {"1": {"exist": true, "char_ranges": [[30, 169]], "word_ranges": [[5, 26]], "text": "Microhematuria, proteinuria and sensorineural hearing loss in a young male with a family history of women with hematuria = Alport syndrome."}, "2": {"exist": false, "char_ranges": [], "word_ranges": [], "text": ""}, "3": {"exist": false, "char_ranges": [], "word_ranges": [], "text": ""}, "4": {"exist": false, "char_ranges": [], "word_ranges": [], "text": ""}, "5": {"exist": false, "char_ranges": [], "word_ranges": [], "text": ""}}} {"id": 501, "year": 2020, "question_id_specific": 76, "full_question": "A 5-year-old boy diagnosed with chronic renal failure. He comes to the emergency department with vomiting, malaise, and palpitations. An electrocardiogram was performed with T-wave elevation and blood tests showed potassium levels of 5.9 mEq/L. Immediate administration of calcium gluconate is decided. What is the purpose of this treatment?", "full_answer": "Calcium gluconate has no effect on blood potassium levels (therefore options 1, 2 and 3 are ruled out). Its function is to decrease the excitability of cardiac myocytes to decrease the likelihood of developing heart rhythm disorders.", "type": "PEDIATRICS", "options": {"1": "Encourage the displacement of potassium from the plasma space to the intracellular space.", "2": "Chelate circulating potassium to favor its hepatic elimination.", "3": "Chelate circulating potassium to favor its renal elimination.", "4": "Antagonize the action of potassium on the myocardial cell membrane.", "5": NaN}, "correct_option": 4, "explanations": {"1": {"exist": true, "char_ranges": [[0, 103]], "word_ranges": [[0, 18]], "text": "Calcium gluconate has no effect on blood potassium levels (therefore options 1, 2 and 3 are ruled out)."}, "2": {"exist": true, "char_ranges": [[0, 103]], "word_ranges": [[0, 18]], "text": "Calcium gluconate has no effect on blood potassium levels (therefore options 1, 2 and 3 are ruled out)."}, "3": {"exist": true, "char_ranges": [[0, 103]], "word_ranges": [[0, 18]], "text": "Calcium gluconate has no effect on blood potassium levels (therefore options 1, 2 and 3 are ruled out)."}, "4": {"exist": true, "char_ranges": [[104, 233]], "word_ranges": [[18, 37]], "text": "Its function is to decrease the excitability of cardiac myocytes to decrease the likelihood of developing heart rhythm disorders."}, "5": {"exist": false, "char_ranges": [], "word_ranges": [], "text": ""}}} {"id": 242, "year": 2014, "question_id_specific": 111, "full_question": "A 50-year-old man with chronic bronchitis is admitted for pneumonia with positive blood culture for Streptococcus pneumoniae, with an MIC to penicillin of 0.0125 mg/l. Treatment was started with penicillin 2 million every 4h. On the fifth day, she still has a fever of 38ºC. Which of the following decisions do you think is correct?", "full_answer": "The patient is a man with pneumonia with isolated germ in blood culture and antibiogram. According to the data provided, it is a pneumococcus sensitive to penicillin (MIC < 0.125), so treatment is started with penicillin at high doses every 4 h, despite which the fever persists. If we analyze the case carefully, the treatment should be effective since the germ is totally sensitive to the drug and the dosage is adequate (so that high concentrations of the drug should be maintained at all times, remember that the efficacy of beta-lactams is time-dependent and the antibiotic concentrations in blood should be above the MIC permanently to achieve effectiveness and avoid the production of resistance). In such a case, what could be happening, probably, although the antibiotic is the right one, it is not reaching the infectious focus adequately because an empyema has probably occurred. We should rule out the existence of an empyema and drain it while continuing with the antibiotic treatment. If the dosage had not been adequate and the presence of drug resistance was possible, one option could have been to treat with ceftriaxone at high doses, since we should remember that streptococcus does not produce beta-lactamase and beta-lactam resistance is produced by another mechanism (using amoxicillin-clavulanate would not provide us with anything). With quinolone we do not increase the spectrum and it is evident that the poor evolution of the patient is not only a problem of time.", "type": "INFECTIOUS DISEASES", "options": {"1": "I would change the treatment to ceftriaxone because of its greater efficacy.", "2": "I would add a quinolone to the treatment.", "3": "I would switch to amoxicillin/clavulanic acid.", "4": "It would rule out the presence of pleural empyema.", "5": "I would continue with the same treatment, assuming it is simply a timing issue."}, "correct_option": 4, "explanations": {"1": {"exist": true, "char_ranges": [[999, 1228]], "word_ranges": [[162, 199]], "text": "If the dosage had not been adequate and the presence of drug resistance was possible, one option could have been to treat with ceftriaxone at high doses, since we should remember that streptococcus does not produce beta-lactamase"}, "2": {"exist": true, "char_ranges": [[1357, 1491]], "word_ranges": [[215, 241]], "text": "With quinolone we do not increase the spectrum and it is evident that the poor evolution of the patient is not only a problem of time."}, "3": {"exist": true, "char_ranges": [[1233, 1356]], "word_ranges": [[200, 215]], "text": "beta-lactam resistance is produced by another mechanism (using amoxicillin-clavulanate would not provide us with anything)."}, "4": {"exist": true, "char_ranges": [[756, 998]], "word_ranges": [[123, 162]], "text": "although the antibiotic is the right one, it is not reaching the infectious focus adequately because an empyema has probably occurred. We should rule out the existence of an empyema and drain it while continuing with the antibiotic treatment."}, "5": {"exist": false, "char_ranges": [], "word_ranges": [], "text": ""}}} {"id": 487, "year": 2020, "question_id_specific": 68, "full_question": "What genetic and reproductive counseling would you indicate to a 30-year-old woman who consults because she is a carrier of a premutation in the FMR1 gene, responsible for Fragile X syndrome, and who wishes to have offspring. Point out the correct answer:", "full_answer": "Another classic MIR, asked in 2019 and in 2017, fragile X syndrome. Also, on the same concept as question 44, the phenomenon of genetic anticipation. Triplet expansion diseases, such as this one, can have the anticipation phenomenon whereby females with premutations (between 55-200 repeats) can have children with full mutations (more than 200 repeats of the triplet) and more obvious, more severe and/or earlier clinical, in both males and females (Fragile X syndrome, early ovarian failure, tremor/ataxia associated with Fragile X).", "type": "GENETICS", "options": {"1": "All her male children will be carriers and, therefore, will manifest the disease.", "2": "There is a phenomenon of genetic anticipation, so their offspring will present more severe and earlier symptoms.", "3": "Preimplantation genetic diagnosis for sex selection of embryos by fluorescence in situ hybridization (FISH) is the best option for healthy offspring.", "4": "50% of her daughters will be carriers, but will not present clinical manifestations of the disease.", "5": NaN}, "correct_option": 2, "explanations": {"1": {"exist": false, "char_ranges": [], "word_ranges": [], "text": ""}, "2": {"exist": true, "char_ranges": [[150, 535]], "word_ranges": [[25, 81]], "text": "Triplet expansion diseases, such as this one, can have the anticipation phenomenon whereby females with premutations (between 55-200 repeats) can have children with full mutations (more than 200 repeats of the triplet) and more obvious, more severe and/or earlier clinical, in both males and females (Fragile X syndrome, early ovarian failure, tremor/ataxia associated with Fragile X)."}, "3": {"exist": false, "char_ranges": [], "word_ranges": [], "text": ""}, "4": {"exist": false, "char_ranges": [], "word_ranges": [], "text": ""}, "5": {"exist": false, "char_ranges": [], "word_ranges": [], "text": ""}}} {"id": 364, "year": 2016, "question_id_specific": 176, "full_question": "A 78-year-old woman with a history of type 2 diabetes treated with oral antidiabetics, hypertension treated with beta-blockers and ACE inhibitors, and NYHA grade 1 congestive heart failure with LVEF of 48%, and currently asymptomatic. Functional status: ECOG 0. History of 2 years of evolution of small lumps in the neck. Biopsy of cervical adenopathy: follicular lymphoma grade 2. Extension study: Hb 12 g/dL, Leukocytes 6,900/microL (Neutrophils 60%, Lymphocytes 27%, Monocytes 6%, Eosinophils 4%, Basophils 4%), Platelets 220,000/microL. MO: infiltrate by follicular lymphoma. Creatinine 1.5 mg/dL, LDH 235 U/L, Beta2 microglobulin 2.1 microg/mL. CT: lymphadenopathies less than 3 cm in cervical, axillary, retroperitoneal, iliac and inguinal territories; liver and spleen are normal. Which of the following treatments is the most appropriate?", "full_answer": "There may be debate about whether to start Rituximab with another cytostatic drug or in monotherapy, but given the lack of survival benefit with early initiation of treatment, the NCCN guidelines recommend waiting and seeing, unless GELF criteria are met to initiate treatment: GELF criteria (Follicular Lymphoma Study Group): - Involvement of ≥3 nodal areas, each with a diameter ≥ 3 cm. - Any nodal or extranodal mass with a diameter ≥ 7 cm. - B symptoms. - Splenomegaly. - Pleural effusion or ascites. - Cytopenias (leukocytes < 1.0 x 109 / L and/or platelets < 100 x 109 / L). - Leukemia (> 5.0 x 109 /L of malignant cells).", "type": "HEMATOLOGY", "options": {"1": "Rituximab-CHOP (Cyclophosphamide, Adriamycin, Vincristine, Prednisone).", "2": "Rituximab-CVP (Cyclophosphamide, Vincristine, Prednisone).", "3": "Do not treat and monitor (wait and see).", "4": "Rituxirnab-Bendamustine.", "5": NaN}, "correct_option": 3, "explanations": {"1": {"exist": true, "char_ranges": [[0, 225]], "word_ranges": [[0, 35]], "text": "There may be debate about whether to start Rituximab with another cytostatic drug or in monotherapy, but given the lack of survival benefit with early initiation of treatment, the NCCN guidelines recommend waiting and seeing,"}, "2": {"exist": true, "char_ranges": [[0, 225]], "word_ranges": [[0, 35]], "text": "There may be debate about whether to start Rituximab with another cytostatic drug or in monotherapy, but given the lack of survival benefit with early initiation of treatment, the NCCN guidelines recommend waiting and seeing,"}, "3": {"exist": true, "char_ranges": [[0, 225]], "word_ranges": [[0, 35]], "text": "There may be debate about whether to start Rituximab with another cytostatic drug or in monotherapy, but given the lack of survival benefit with early initiation of treatment, the NCCN guidelines recommend waiting and seeing,"}, "4": {"exist": true, "char_ranges": [[0, 225]], "word_ranges": [[0, 35]], "text": "There may be debate about whether to start Rituximab with another cytostatic drug or in monotherapy, but given the lack of survival benefit with early initiation of treatment, the NCCN guidelines recommend waiting and seeing,"}, "5": {"exist": false, "char_ranges": [], "word_ranges": [], "text": ""}}} {"id": 90, "year": 2012, "question_id_specific": 32, "full_question": "A 35-year-old woman is diagnosed with colon cancer located in the hepatic angle. She has a family history of colon cancer in her mother, an aunt at the age of 45 and a grandfather. The most accepted surgical intervention is:", "full_answer": "The statement with the family history indicates that this is a case of hereditary nonpolyposis colorectal cancer (Lynch syndrome). The age and right location data lead us to answer 4: total colectomy with ileorectal anastomosis. If a conventional right hemicolectomy type surgery is performed, the chances of recurrence are higher than 30%. Excision of the rectum is not necessary prophylactically.", "type": "GENERAL SURGERY", "options": {"1": "Right hemicolectomy.", "2": "Extended right hemicolectomy.", "3": "Subtotal colectomy.", "4": "Total colectomy with ileorectal anatomosis.", "5": "Total colectomy with ileoanal anatomy."}, "correct_option": 4, "explanations": {"1": {"exist": false, "char_ranges": [], "word_ranges": [], "text": ""}, "2": {"exist": false, "char_ranges": [], "word_ranges": [], "text": ""}, "3": {"exist": false, "char_ranges": [], "word_ranges": [], "text": ""}, "4": {"exist": true, "char_ranges": [[131, 228]], "word_ranges": [[19, 35]], "text": "The age and right location data lead us to answer 4: total colectomy with ileorectal anastomosis."}, "5": {"exist": false, "char_ranges": [], "word_ranges": [], "text": ""}}} {"id": 528, "year": 2021, "question_id_specific": 134, "full_question": "53-year-old woman with a personal history of obesity and migraine. Her blood pressure has been taken repeatedly at the doctor's and nurse's office, presenting figures below 140/90 mmHg. However, an approved blood pressure measuring device has been purchased and she has been taught how to use it correctly. He comes in showing blood pressure recordings taken at home over several weeks with values above 140/90 mmHg. Mark the correct answer:", "full_answer": "The patient described is the definition of masked HT, normal figures in consultation and elevated on AMPA or ABPM (option 2 correct). It is not secondary HT because no study has been done to confirm it (option 1 incorrect). It is not isolated HT because the elevated figures are maintained for several weeks (option 3 incorrect). It is not refractory HT because it has not yet been treated (option 4 incorrect).", "type": "NEPHROLOGY", "options": {"1": "She has secondary hypertension.", "2": "Presents masked arterial hypertension.", "3": "Presents isolated clinical hypertension.", "4": "Presents refractory arterial hypertension.", "5": NaN}, "correct_option": 2, "explanations": {"1": {"exist": true, "char_ranges": [[134, 223]], "word_ranges": [[22, 39]], "text": "It is not secondary HT because no study has been done to confirm it (option 1 incorrect)."}, "2": {"exist": true, "char_ranges": [[0, 133]], "word_ranges": [[0, 22]], "text": "The patient described is the definition of masked HT, normal figures in consultation and elevated on AMPA or ABPM (option 2 correct)."}, "3": {"exist": true, "char_ranges": [[224, 329]], "word_ranges": [[39, 56]], "text": "It is not isolated HT because the elevated figures are maintained for several weeks (option 3 incorrect)."}, "4": {"exist": true, "char_ranges": [[330, 411]], "word_ranges": [[56, 71]], "text": "It is not refractory HT because it has not yet been treated (option 4 incorrect)."}, "5": {"exist": false, "char_ranges": [], "word_ranges": [], "text": ""}}} {"id": 583, "year": 2022, "question_id_specific": 70, "full_question": "A 43-year-old woman with a levonorgestrel intrauterine device who consults for lack of rest at night with the appearance of heat and palpitations. In the analytical analysis highlights free T4 10.5 pmol/L (9-19), thyrotropin 2.1 mIU/L (0.30-5.00), FSH 95.6 IU/L (1.38-16.7), LH 21 IU/L (2.4-9.3), estradiol < 0.07 nmol/L (0.07 - 1.14). What is the most appropriate treatment?", "full_answer": "The hormonal profile we are told is in menopausal range (FSH >20; estradiol <5) and the patient has systemic vasomotor symptoms. Therefore, it would be more appropriate to administer systemic estrogens. To compensate, she already has a local uterine progestin (levonorgestrel IUD).", "type": "OBSTETRICS AND GYNECOLOGY", "options": {"1": "Benzodiazepines.", "2": "Vaginal estrogens.", "3": "Oral estrogens and progestogens in continuous regimen.", "4": "Transdermal estrogens in a continuous regimen.", "5": NaN}, "correct_option": 4, "explanations": {"1": {"exist": false, "char_ranges": [], "word_ranges": [], "text": ""}, "2": {"exist": false, "char_ranges": [], "word_ranges": [], "text": ""}, "3": {"exist": false, "char_ranges": [], "word_ranges": [], "text": ""}, "4": {"exist": true, "char_ranges": [[0, 202]], "word_ranges": [[0, 31]], "text": "The hormonal profile we are told is in menopausal range (FSH >20; estradiol <5) and the patient has systemic vasomotor symptoms. Therefore, it would be more appropriate to administer systemic estrogens."}, "5": {"exist": false, "char_ranges": [], "word_ranges": [], "text": ""}}} {"id": 195, "year": 2013, "question_id_specific": 163, "full_question": "A 15-month-old boy, who during dinner, after eating a bite of tortilla, suddenly presented with facial reddening predominantly perioral, habonous lesions on the trunk and extremities and cough. On arrival at the center, the patient was conscious and, in addition to what was described above, the following were observed: suprasternal twitching, abundant watery rhinorrhea, bilateral hypoventilation without wheezing and capillary refill of less than 2 seconds. Of the following statements, indicate the CORRECT answer:", "full_answer": "She is describing egg anaphylaxis, a potentially fatal condition. The treatment of choice is intramuscular adrenaline, and effectively, without wasting too much time. The first thing is INTRAMUSCULAR adrenaline NOW! The venous route, corticosteroids, antihistamines, etc. will come later.", "type": "PEDIATRICS", "options": {"1": "The priority is to cannulate a venous line.", "2": "Intramuscular methylprednisolone is the treatment of choice.", "3": "It is a picture of urticaria associated with asthma, and should be treated with antihistamines and inhaled bronchodilators.", "4": "Parents should be advised to transfer to a hospital emergency department.", "5": "Intramuscular adrenaline should be administered without further delay."}, "correct_option": 5, "explanations": {"1": {"exist": true, "char_ranges": [[167, 288]], "word_ranges": [[23, 39]], "text": "The first thing is INTRAMUSCULAR adrenaline NOW! The venous route, corticosteroids, antihistamines, etc. will come later."}, "2": {"exist": true, "char_ranges": [[167, 288]], "word_ranges": [[23, 39]], "text": "The first thing is INTRAMUSCULAR adrenaline NOW! The venous route, corticosteroids, antihistamines, etc. will come later."}, "3": {"exist": true, "char_ranges": [[167, 288]], "word_ranges": [[23, 39]], "text": "The first thing is INTRAMUSCULAR adrenaline NOW! The venous route, corticosteroids, antihistamines, etc. will come later."}, "4": {"exist": true, "char_ranges": [[167, 288]], "word_ranges": [[23, 39]], "text": "The first thing is INTRAMUSCULAR adrenaline NOW! The venous route, corticosteroids, antihistamines, etc. will come later."}, "5": {"exist": true, "char_ranges": [[0, 166]], "word_ranges": [[0, 23]], "text": "She is describing egg anaphylaxis, a potentially fatal condition. The treatment of choice is intramuscular adrenaline, and effectively, without wasting too much time."}}} {"id": 84, "year": 2012, "question_id_specific": 49, "full_question": "A 75-year-old man, hypertensive and dyslipidemic with a history of heart failure due to left ventricular systolic dysfunction (EF < 30%). He comes to our office for worsening of his usual dyspnea in recent weeks. His systolic blood pressure is 160/95 mmHg with a heart rate of 65 bpm. In the physical examination no crackles were heard and his jugular venous pressure was normal. He provides a creatinine determination that is 3.7 mg/dL with ions within normal. Which of the following drugs is the most appropriate to improve the prognosis of his heart failure?", "full_answer": "In a patient with multiple CVRF with concomitant heart and renal failure (presumably of multiple etiology: renal hypoperfusion, arteriosclerosis...), we must be very careful when using diuretics and ACEI/ARA-II, since we can worsen renal function and/or increase potassium levels (among diuretics, spironolactone and eplerenone are \"potassium-sparing\"). Therefore, in this case we will use bisoprolol.", "type": "CARDIOLOGY AND VASCULAR SURGERY", "options": {"1": "Enalapril.", "2": "Losartan.", "3": "Spironolactone.", "4": "Bisoprolol.", "5": "Eplerenone."}, "correct_option": 4, "explanations": {"1": {"exist": true, "char_ranges": [[0, 353]], "word_ranges": [[0, 46]], "text": "In a patient with multiple CVRF with concomitant heart and renal failure (presumably of multiple etiology: renal hypoperfusion, arteriosclerosis...), we must be very careful when using diuretics and ACEI/ARA-II, since we can worsen renal function and/or increase potassium levels (among diuretics, spironolactone and eplerenone are \"potassium-sparing\")."}, "2": {"exist": true, "char_ranges": [[0, 353]], "word_ranges": [[0, 46]], "text": "In a patient with multiple CVRF with concomitant heart and renal failure (presumably of multiple etiology: renal hypoperfusion, arteriosclerosis...), we must be very careful when using diuretics and ACEI/ARA-II, since we can worsen renal function and/or increase potassium levels (among diuretics, spironolactone and eplerenone are \"potassium-sparing\")."}, "3": {"exist": true, "char_ranges": [[0, 353]], "word_ranges": [[0, 46]], "text": "In a patient with multiple CVRF with concomitant heart and renal failure (presumably of multiple etiology: renal hypoperfusion, arteriosclerosis...), we must be very careful when using diuretics and ACEI/ARA-II, since we can worsen renal function and/or increase potassium levels (among diuretics, spironolactone and eplerenone are \"potassium-sparing\")."}, "4": {"exist": false, "char_ranges": [], "word_ranges": [], "text": ""}, "5": {"exist": true, "char_ranges": [[0, 353]], "word_ranges": [[0, 46]], "text": "In a patient with multiple CVRF with concomitant heart and renal failure (presumably of multiple etiology: renal hypoperfusion, arteriosclerosis...), we must be very careful when using diuretics and ACEI/ARA-II, since we can worsen renal function and/or increase potassium levels (among diuretics, spironolactone and eplerenone are \"potassium-sparing\")."}}} {"id": 427, "year": 2018, "question_id_specific": 95, "full_question": "A 45-year-old woman comes to the emergency department with confusional symptoms accompanied by dysphonia. On examination she has a palpable neck mass and a blood test shows plasma calcium levels of 15 mg/dL (normal up to 10.2 mg/dL). Given these findings, which of the following diagnoses should be suspected?", "full_answer": "Palpable mass in the neck: suspect thyroid/parathyroid pathology. It is associated with hypercalcemia, therefore, the option of medullary thyroid cancer is ruled out (it does not affect the level of calcemia). To think of MEN type I we should associate other pathologies (pituitary, pancreatic or duodenal tumors). The presence of a calcemia higher than 13mgr/dl, together with dysphonia, indicates a malignant tumor (higher calcemia level and infiltration), therefore we opt for option 1.", "type": "ENDOCRINOLOGY", "options": {"1": "Parathyroid carcinoma.", "2": "Medullary thyroid carcinoma.", "3": "MEN type I.", "4": "Parathyroid adenoma.", "5": NaN}, "correct_option": 1, "explanations": {"1": {"exist": true, "char_ranges": [[315, 489]], "word_ranges": [[47, 73]], "text": "The presence of a calcemia higher than 13mgr/dl, together with dysphonia, indicates a malignant tumor (higher calcemia level and infiltration), therefore we opt for option 1."}, "2": {"exist": true, "char_ranges": [[66, 209]], "word_ranges": [[8, 31]], "text": "It is associated with hypercalcemia, therefore, the option of medullary thyroid cancer is ruled out (it does not affect the level of calcemia)."}, "3": {"exist": true, "char_ranges": [[210, 314]], "word_ranges": [[31, 47]], "text": "To think of MEN type I we should associate other pathologies (pituitary, pancreatic or duodenal tumors)."}, "4": {"exist": false, "char_ranges": [], "word_ranges": [], "text": ""}, "5": {"exist": false, "char_ranges": [], "word_ranges": [], "text": ""}}} {"id": 462, "year": 2018, "question_id_specific": 151, "full_question": "A 13-year-old boy presents with subacute headache and diplopia. Neurologic examination shows vertical gaze palsy and MRI shows a contrast-enhancing lesion in the pineal region obstructing the aqueduct of Sylvian. The most likely diagnosis is:", "full_answer": "Diplopia is usually caused by acute or secondary strabismus. Vertical gaze palsy is due to a midbrain lesion. In this case the MRI already places the origin of the symptoms in the pineal gland. The most frequent tumor in this region is the germ cell tumor (answer 3).", "type": "OPHTHALMOLOGY (ECTOPIC)", "options": {"1": "Glioblastoma.", "2": "Medulloblastoma.", "3": "Germ cell tumor.", "4": "Meningioma.", "5": NaN}, "correct_option": 3, "explanations": {"1": {"exist": false, "char_ranges": [], "word_ranges": [], "text": ""}, "2": {"exist": false, "char_ranges": [], "word_ranges": [], "text": ""}, "3": {"exist": true, "char_ranges": [[110, 267]], "word_ranges": [[18, 48]], "text": "In this case the MRI already places the origin of the symptoms in the pineal gland. The most frequent tumor in this region is the germ cell tumor (answer 3)."}, "4": {"exist": false, "char_ranges": [], "word_ranges": [], "text": ""}, "5": {"exist": false, "char_ranges": [], "word_ranges": [], "text": ""}}} {"id": 557, "year": 2022, "question_id_specific": 168, "full_question": "58-year-old male, obese, asymptomatic, moderate drinker, who presents a first baseline blood glucose of 153 mg/d with negative glycosuria. In the following weeks he has had two other baseline blood glucose readings of 118 and 136 mg/dl. Which of the following is the most appropriate approach to confirm the diagnosis of diabetes mellitus:", "full_answer": "Diagnostic Criteria for DM according to ISPAD 2018 *Classic symptoms of diabetes with blood glucose > 200mgr/dl or: * Fasting glycemia ≥ 126mgr/dl (fasting at least 8h) or * Glycemia at 2h SOG ≥ 200mgr/dl (on 2 occasions) or * HbA1C ≥ 6.5%.", "type": "ENDOCRINOLOGY", "options": {"1": "Practice a blood glucose curve with 75 g of glucose.", "2": "Already meets diagnostic criteria for diabetes mellitus.", "3": "Request a determination of basal insulinemia or C-peptide.", "4": "Request a determination of glycosylated hemoglobin.", "5": NaN}, "correct_option": 2, "explanations": {"1": {"exist": false, "char_ranges": [], "word_ranges": [], "text": ""}, "2": {"exist": true, "char_ranges": [[0, 240]], "word_ranges": [[0, 43]], "text": "Diagnostic Criteria for DM according to ISPAD 2018 *Classic symptoms of diabetes with blood glucose > 200mgr/dl or: * Fasting glycemia ≥ 126mgr/dl (fasting at least 8h) or * Glycemia at 2h SOG ≥ 200mgr/dl (on 2 occasions) or * HbA1C ≥ 6.5%."}, "3": {"exist": false, "char_ranges": [], "word_ranges": [], "text": ""}, "4": {"exist": false, "char_ranges": [], "word_ranges": [], "text": ""}, "5": {"exist": false, "char_ranges": [], "word_ranges": [], "text": ""}}} {"id": 35, "year": 2011, "question_id_specific": 68, "full_question": "A 56-year-old patient with no personal or family history of interest is studied because he has been having difficulty walking with his right leg for six months. He reports no other symptoms. The examination shows a 4/5 weakness for dorsal flexion and eversion of the foot and 4/5 for flexion and inversion of the foot, with increased muscle reflexes and Babinski present, the rest being normal. The syndromic diagnosis would be:", "full_answer": "Answer 1 incorrect: It does not explain pyramidalism. Answer 2 correct: A nonspecific answer, but probably the correct answer. Answer 3 incorrect: We would lack data to think of a Brown Sequard hemimedullary syndrome. It would explain the pyramidalism, but we would be missing data suggestive of this picture such as a contralateral hypoesthesia. Incorrect answer 4: It could explain the difficulty in dorsiflexion and plantar flexion of the foot, but not the pyramidalism. Incorrect answer 5: In a similar way to the previous one, it could explain part of the clinical picture, but not the pyramidalism.", "type": "NEUROLOGY AND NEUROSURGERY", "options": {"1": "Mononeuropathy of the right common peroneal nerve.", "2": "Focal involvement of the first motor neuron and probably of the second.", "3": "Right hemimedullary involvement (Brown-Sequard).", "4": "Multiple mononeuropathy with involvement of the common peroneal and right posterior tibial nerve.", "5": "Right lumbar plexopathy."}, "correct_option": 2, "explanations": {"1": {"exist": true, "char_ranges": [[20, 53]], "word_ranges": [[3, 8]], "text": "It does not explain pyramidalism."}, "2": {"exist": false, "char_ranges": [], "word_ranges": [], "text": ""}, "3": {"exist": true, "char_ranges": [[147, 346]], "word_ranges": [[22, 54]], "text": "We would lack data to think of a Brown Sequard hemimedullary syndrome. It would explain the pyramidalism, but we would be missing data suggestive of this picture such as a contralateral hypoesthesia."}, "4": {"exist": true, "char_ranges": [[367, 473]], "word_ranges": [[57, 74]], "text": "It could explain the difficulty in dorsiflexion and plantar flexion of the foot, but not the pyramidalism."}, "5": {"exist": true, "char_ranges": [[494, 604]], "word_ranges": [[77, 97]], "text": "In a similar way to the previous one, it could explain part of the clinical picture, but not the pyramidalism."}}} {"id": 271, "year": 2016, "question_id_specific": 170, "full_question": "An 80-year-old man comes to your office for evaluation of a scheduled cholecystectomy by laparoscopy. He has a history of arterial hypertension under treatment for 10 years. He denies heart or pulmonary disease. She has no chest pain. She has an active life and goes daily to the gym where she alternates swimming and walking on the treadmill for at least one hour. Usual treatment: nebivolol 5 mg every 24 hours and hydrochlorothiazide 12.5 mg daily. Physical examination: weight 73 kgs; height 179 cm; blood pressure 138/80 mmHg; heart rate 60 beats/minute. No murmurs are auscultated Which of the following is the most appropriate preoperative approach?", "full_answer": "This is a completely gratuitous question, from a healthier gentleman than me who is going to undergo low-risk surgery on a scheduled basis. The answer is obviously 4. Had there been a fifth option it would have been that nothing needs to be done, which might have generated some doubt in the student, but since the question does not exist it is a piece of candy. Although there are many protocols on preoperative procedures, almost as many as there are hospitals, the question makes it easy by putting a very old patient, 80 years old. The age range from which it is considered mandatory to request an ECG varies between 45, 50, 60 or even some say 70 years. For example: > 50 years: always, except if a previous ECG is available < 50 years: only if there is known or suspected heart disease (CVRF), severe DM, hyperthyroidism or severe COPD.", "type": "ANESTHESIOLOGY AND CRITICAL CARE", "options": {"1": "Perform a stress test.", "2": "Perform echocardiography", "3": "Perform a thallium and dipyridamole scan.", "4": "Perform electrocardiogram.", "5": NaN}, "correct_option": 4, "explanations": {"1": {"exist": false, "char_ranges": [], "word_ranges": [], "text": ""}, "2": {"exist": false, "char_ranges": [], "word_ranges": [], "text": ""}, "3": {"exist": false, "char_ranges": [], "word_ranges": [], "text": ""}, "4": {"exist": true, "char_ranges": [[363, 658]], "word_ranges": [[66, 119]], "text": "Although there are many protocols on preoperative procedures, almost as many as there are hospitals, the question makes it easy by putting a very old patient, 80 years old. The age range from which it is considered mandatory to request an ECG varies between 45, 50, 60 or even some say 70 years."}, "5": {"exist": false, "char_ranges": [], "word_ranges": [], "text": ""}}} {"id": 306, "year": 2016, "question_id_specific": 207, "full_question": "A 34-year-old patient playing tennis is hit by a tennis ball in the left orbit. On examination he presents significant palpebral hematoma, hyposphagmia, diplopia to the upper vision with limitation of the upper version of the eyeball. What would you suspect?", "full_answer": "The statement describes diplopia to superior vision with limitation of upper eyeball extension. As the mechanical function of the left inferior rectus is compromised, it would not allow to complete the supraduccion movement generating vertical diplopia, this plus the traumatic antecedent allows to suspect a fracture of the orbital floor with muscular entrapment of the inferior rectus. Fracture of the roof is much less frequent and the entrapment of the superior rectus would cause limitation for infraduction.", "type": "OPHTHALMOLOGY", "options": {"1": "Fracture of the inferior wall of the floor of the orbit with entrapment of the inferior rectus muscle.", "2": "Zygomatic arch fracture.", "3": "Fracture of the superior wall of the orbit with entrapment of the superior rectus muscle.", "4": "Dentoalveolar fracture.", "5": NaN}, "correct_option": 1, "explanations": {"1": {"exist": true, "char_ranges": [[96, 387]], "word_ranges": [[13, 57]], "text": "As the mechanical function of the left inferior rectus is compromised, it would not allow to complete the supraduccion movement generating vertical diplopia, this plus the traumatic antecedent allows to suspect a fracture of the orbital floor with muscular entrapment of the inferior rectus."}, "2": {"exist": false, "char_ranges": [], "word_ranges": [], "text": ""}, "3": {"exist": true, "char_ranges": [[388, 513]], "word_ranges": [[57, 77]], "text": "Fracture of the roof is much less frequent and the entrapment of the superior rectus would cause limitation for infraduction."}, "4": {"exist": false, "char_ranges": [], "word_ranges": [], "text": ""}, "5": {"exist": false, "char_ranges": [], "word_ranges": [], "text": ""}}} {"id": 328, "year": 2016, "question_id_specific": 83, "full_question": "A 49-year-old man who consults for severe polyuria and polydipsia and unintentional weight loss of 10 kg and is diagnosed with diabetes mellitus due to a plasma blood glucose of 322 mg/dL and a glycosylated hemoglobin of 9.8%. His physician gave him dietary recommendations, the convenience of physical exercise, and he started treatment with metformin 850 mg/12 hours and glimepiride 6 mg/day. In the following weeks the glycemic controls are progressively reduced. At 4 months glycemia is 94 mg/dL and HbA1c is 5.9%. The patient complains of frequent episodes of dizziness, epigastric pain, blurred vision, sweating and tremor, which improve with food and occur mainly in the late morning and late afternoon. What modification would you propose in his treatment?", "full_answer": "Discontinue sulfonylurea because of Risk of hypoglycemia.", "type": "ENDOCRINOLOGY", "options": {"1": "Review dietary carbohydrate distribution.", "2": "Discontinue metformin.", "3": "Discontinue sulfonylurea.", "4": "Substitute metformin with a DPP4 inhibitor.", "5": NaN}, "correct_option": 3, "explanations": {"1": {"exist": false, "char_ranges": [], "word_ranges": [], "text": ""}, "2": {"exist": false, "char_ranges": [], "word_ranges": [], "text": ""}, "3": {"exist": true, "char_ranges": [[0, 57]], "word_ranges": [[0, 7]], "text": "Discontinue sulfonylurea because of Risk of hypoglycemia."}, "4": {"exist": false, "char_ranges": [], "word_ranges": [], "text": ""}, "5": {"exist": false, "char_ranges": [], "word_ranges": [], "text": ""}}} {"id": 223, "year": 2014, "question_id_specific": 62, "full_question": "In a 60-year-old diabetic patient with stable angina secondary to ischemic heart disease due to subtotal obstruction in the middle third of the anterior descending coronary artery, what treatment would you propose?", "full_answer": "Stable angina in diabetics: this question is a nod to the BARI-2D study (NEJM 2009), which showed that medical management was not inferior to revascularization (in fact, the new guidelines reflect the trend that not all coronary stenosis should be systematically opened). Answer 1 is along these lines, although I miss statins, ACE inhibitors and antiplatelet agents, and I have too many vasodilators (nitrates are class IIb, if my memory serves me correctly). 2 is absurd: sitting down to avoid angina? That's like cutting down the forest to prevent fires. 3 to 5 are about revascularization: since it is a single vessel and not a proximal LAD, revascularization would have to be percutaneous, so we discard 3. Between 4 and 5, the fifth is more complete: not only would it be necessary to dilate, but also to implant a stent. So the doubt would be between 1 and 5. Applying the general rules of response in the MIR, if two options are similar except in one detail, the correct one is one of these, so we would mark 5 (and this is what would be done in any hospital nowadays: a suboccluded artery is dilated directly). On the other hand, and although 1 is incomplete, I suppose the author wanted to be clever and show that he has read the latest articles. Conclusion: I would mark 5... but I would not be surprised if it was 1.", "type": "CARDIOLOGY", "options": {"1": "Medical with vasodilators and beta-blockers to avoid angina.", "2": "Expectorant with rigorous rest, since decreasing myocardial oxygen demand should decrease angina.", "3": "Surgical revascularization of the ischemic myocardium by means of a left mammary artery bypass distal to the lesion in the diseased coronary artery.", "4": "Dilatation of the coronary artery lesion by therapeutic catheterization.", "5": "Therapeutic catheterization to dilate the lesion of the diseased artery and implantation of a stent in the dilated area."}, "correct_option": 5, "explanations": {"1": {"exist": true, "char_ranges": [[867, 1119]], "word_ranges": [[149, 196]], "text": "Applying the general rules of response in the MIR, if two options are similar except in one detail, the correct one is one of these, so we would mark 5 (and this is what would be done in any hospital nowadays: a suboccluded artery is dilated directly)."}, "2": {"exist": true, "char_ranges": [[461, 503]], "word_ranges": [[73, 81]], "text": "2 is absurd: sitting down to avoid angina?"}, "3": {"exist": true, "char_ranges": [[594, 711]], "word_ranges": [[96, 117]], "text": "since it is a single vessel and not a proximal LAD, revascularization would have to be percutaneous, so we discard 3."}, "4": {"exist": true, "char_ranges": [[712, 827]], "word_ranges": [[117, 140]], "text": "Between 4 and 5, the fifth is more complete: not only would it be necessary to dilate, but also to implant a stent."}, "5": {"exist": true, "char_ranges": [[867, 1119]], "word_ranges": [[149, 196]], "text": "Applying the general rules of response in the MIR, if two options are similar except in one detail, the correct one is one of these, so we would mark 5 (and this is what would be done in any hospital nowadays: a suboccluded artery is dilated directly)."}}} {"id": 468, "year": 2020, "question_id_specific": 122, "full_question": "A 70-year-old man consults for non-radiating lumbar pain of 1 week's duration, of progressive intensity, accompanied by marked limitation of spinal mobility and febrile fever. The lumbar spine X-ray shows no alterations. What diagnosis should we rule out first and with what technique?", "full_answer": "We suspect spondylodiscitis - the febrile fever and the limitation of mobility of the spine with the acute course give the key -, and the indicated test is the MRI. \"The most demonstrative test is magnetic resonance imaging (MRI), which is positive even in the first 2 weeks of the picture, which facilitates early diagnosis, with a sensitivity of 90%\". Vertebral crushing would be seen in the simple X-ray, as well as vertebral ankylosing hyperostosis and many metastases. In addition, there is no data in the history pointing to bone metastases (oncologic history, weight loss etc).", "type": "ORTHOPEDIC SURGERY AND TRAUMATOLOGY", "options": {"1": "Vertebral crush with computed tomography (CT).", "2": "Bone metastases with technetium bone scan.", "3": "Vertebral ankylosing hyperostosis with dorsolumbar spine x-ray.", "4": "Pyogenic spondylodiscitis with magnetic resonance imaging (MRI).", "5": NaN}, "correct_option": 4, "explanations": {"1": {"exist": true, "char_ranges": [[354, 473]], "word_ranges": [[60, 78]], "text": "Vertebral crushing would be seen in the simple X-ray, as well as vertebral ankylosing hyperostosis and many metastases."}, "2": {"exist": true, "char_ranges": [[487, 584]], "word_ranges": [[80, 96]], "text": "there is no data in the history pointing to bone metastases (oncologic history, weight loss etc)."}, "3": {"exist": true, "char_ranges": [[354, 473]], "word_ranges": [[60, 78]], "text": "Vertebral crushing would be seen in the simple X-ray, as well as vertebral ankylosing hyperostosis and many metastases."}, "4": {"exist": true, "char_ranges": [[0, 164]], "word_ranges": [[0, 30]], "text": "We suspect spondylodiscitis - the febrile fever and the limitation of mobility of the spine with the acute course give the key -, and the indicated test is the MRI."}, "5": {"exist": false, "char_ranges": [], "word_ranges": [], "text": ""}}} {"id": 96, "year": 2012, "question_id_specific": 170, "full_question": "60-year-old male patient who refers 10 days ago the appearance of blistering lesions on dorsum of hands after sun exposure. The picture is accompanied by skin fragility. Histopathologically there is a subepidermal blister with PAS+ deposits in and around the superficial dermal vessels. The most plausible diagnosis is:", "full_answer": "Porphyria cutanea tarda (PCT) is the most common porphyria and can present with three major clinical forms: familial, sporadic and toxic. Clinically, it is characterized by marked skin fragility with the appearance of erosions, vesicles and blisters after minimal trauma. The most frequent sites are the back of the hands and the face, i.e. the areas of greatest photoexposure. Clinically, the most important differential diagnosis should be established with pseudoporphyria or phototoxic bullous dermatitis, in which the blisters and cutaneous erosions are produced by photosensitivity, induced by drugs such as tetracyclines, sulfonamides, etc. However, unlike porphyria, porphyrin levels in serum, feces and urine are normal. The histopathological differential diagnosis should be established with those diseases showing dermal hyaline deposits, such as: hyalinosis cutis, in which unlike porphyria, PAS-positive hyaline deposits are extensive and massively occupy the deep dermis and subcutaneous cellular tissue; Milium colloid in which hyaline material, usually PAS negative expands the papillary dermis, forming characteristic clefts and fissures; and lipoid proteinosis, in which the material is deposited throughout the dermis, has a distribution not only perivascular but more extensive than in porphyria, and usually affects sweat glands.", "type": "DERMATOLOGY, VENEREOLOGY AND PLASTIC SURGERY", "options": {"1": "Familial benign pemphigus.", "2": "Pemphigoid.", "3": "Porphyria cutanea tarda.", "4": "Pemphigus vulgaris.", "5": "Scalded skin syndrome."}, "correct_option": 3, "explanations": {"1": {"exist": false, "char_ranges": [], "word_ranges": [], "text": ""}, "2": {"exist": false, "char_ranges": [], "word_ranges": [], "text": ""}, "3": {"exist": true, "char_ranges": [[0, 377]], "word_ranges": [[0, 59]], "text": "Porphyria cutanea tarda (PCT) is the most common porphyria and can present with three major clinical forms: familial, sporadic and toxic. Clinically, it is characterized by marked skin fragility with the appearance of erosions, vesicles and blisters after minimal trauma. The most frequent sites are the back of the hands and the face, i.e. the areas of greatest photoexposure."}, "4": {"exist": false, "char_ranges": [], "word_ranges": [], "text": ""}, "5": {"exist": false, "char_ranges": [], "word_ranges": [], "text": ""}}} {"id": 603, "year": 2022, "question_id_specific": 112, "full_question": "A 61-year-old woman, administrative, with a history of overweight, hypertension, dyslipidemia and metabolic syndrome, who consults for pain in both buttocks, left trochanteric region, lateral aspect of the left thigh up to the knee and left leg up to the middle third. The pain appears when the lower limb is lifted with the knee extended, but is relieved when the knee is flexed. What is the first clinical suspicion?", "full_answer": "The most common symptomatology of a patient with coxarthrosis includes: hip pain that limits ambulation, nocturnal discomfort and even at rest, hip stiffness and locking. Coxo-femoral osteoarthrosis presents with a negative Lasegue test. Wrong answer 2. Low back pain of a neuropathic nature usually presents with unilateral and distributed radiculopathy according to dermatomes. The recognized risk factors are most frequently: women, obesity, smoking and sedentary lifestyle or situations of prolonged sitting. The lumbosciatica is usually referred to the buttock, posterior aspect of the thighs and even inguinal. The clinical case describes the Lasegue maneuver or test; a provocation test that evidences radicular irritation in the lumbosacral region (Answer 3 correct). Claudication due to canal stenosis typically presents with pain that worsens with trunk extension (walking, standing) and is relieved with flexion (sitting, sleeping in the fetal position...) Wrong answer 4.", "type": "TRAUMATOLOGY", "options": {"1": "Gouty arthritis of the left hip.", "2": "Left coxofemoral arthrosis.", "3": "Radiated low back pain / lumbosciatica.", "4": "Claudication due to canal stenosis.", "5": NaN}, "correct_option": 3, "explanations": {"1": {"exist": false, "char_ranges": [], "word_ranges": [], "text": ""}, "2": {"exist": true, "char_ranges": [[171, 253]], "word_ranges": [[25, 36]], "text": "Coxo-femoral osteoarthrosis presents with a negative Lasegue test. Wrong answer 2."}, "3": {"exist": true, "char_ranges": [[617, 775]], "word_ranges": [[87, 110]], "text": "The clinical case describes the Lasegue maneuver or test; a provocation test that evidences radicular irritation in the lumbosacral region (Answer 3 correct)."}, "4": {"exist": true, "char_ranges": [[776, 983]], "word_ranges": [[110, 140]], "text": "Claudication due to canal stenosis typically presents with pain that worsens with trunk extension (walking, standing) and is relieved with flexion (sitting, sleeping in the fetal position...) Wrong answer 4."}, "5": {"exist": false, "char_ranges": [], "word_ranges": [], "text": ""}}} {"id": 574, "year": 2022, "question_id_specific": 100, "full_question": "An 80-year-old woman with a history of arterial hypertension and diabetes mellitus who suddenly presents with language impairment. On arrival at the hospital, an unknown atrial fibrillation is observed, and on examination she presents fluent speech with a tendency to slurred speech, poor language comprehension, inability to repeat and dysnomia. Semiologically she presents a picture compatible with:", "full_answer": "It is a semiology question that helps us to remember the types of aphasia. The following aspects should be taken into account: language emission (fluent, non-fluent, non-emission), comprehension (yes or no), repetition (yes or no) and nomination. Wernicke's aphasia or sensitive aphasia has a fluent language, does not understand, does not repeat and does not nominate, just like the patient in the question. Sensory transcortical aphasia differs from sensory aphasia in that patients are able to repeat. Conduction aphasia differs from sensory aphasia in that comprehension is preserved, all else being equal. Finally, in broca's aphasia the language is non-fluent, comprehension is preserved, it does not repeat and does not nominate.", "type": "NEUROLOGY", "options": {"1": "Broca's aphasia.", "2": "Conduction aphasia.", "3": "Sensitive transcortical aphasia.", "4": "Wernicke's aphasia.", "5": NaN}, "correct_option": 4, "explanations": {"1": {"exist": true, "char_ranges": [[611, 736]], "word_ranges": [[92, 111]], "text": "Finally, in broca's aphasia the language is non-fluent, comprehension is preserved, it does not repeat and does not nominate."}, "2": {"exist": true, "char_ranges": [[505, 610]], "word_ranges": [[77, 92]], "text": "Conduction aphasia differs from sensory aphasia in that comprehension is preserved, all else being equal."}, "3": {"exist": true, "char_ranges": [[409, 504]], "word_ranges": [[63, 77]], "text": "Sensory transcortical aphasia differs from sensory aphasia in that patients are able to repeat."}, "4": {"exist": true, "char_ranges": [[247, 408]], "word_ranges": [[37, 63]], "text": "Wernicke's aphasia or sensitive aphasia has a fluent language, does not understand, does not repeat and does not nominate, just like the patient in the question."}, "5": {"exist": false, "char_ranges": [], "word_ranges": [], "text": ""}}} {"id": 67, "year": 2012, "question_id_specific": 111, "full_question": "A 45-year-old male patient with a history of uric lithiasis and repeated expulsive nephritic colic for the past 25 years comes to the emergency room reporting palpitations and severe right lumbar pain for the past 2 hours. Urinalysis shows a pH of 5.5 and leukocyturia without proteinuria. The electrocardiogram confirms a previously unknown AF. Blood biochemistry shows a Creat of 0.9, Ca of 11 and LDH of 950. What is the most useful diagnostic maneuver to perform?", "full_answer": "In principle, what you tell us does not fit with a nephritic colic. The urine is normal (without hematuria, caused by the lithiasis as it progresses), the Creat is normal (it can rise up to 1.9 in nephritic colic). The increase in LDH is non-specific and can be found in hepatic parenchymal lesions, AMI or pulmonary embolism, as well as in many other less frequent cases (tumors, myopathies, etc.). In this case, we have an important risk factor for embolism, which is atrial fibrillation, which, in addition, being a 45-year-old male, with no previous history, does not seem to respond to an underlying structural heart disease together with right lumbar pain, which could correspond to irritation of the pleura at the level of the right lumbar spine. Spiral CT with contrast has acquired great value in the diagnosis of PTE, due to its rapidity and good sensitivity, higher than 83-92% in most studies, and according to some it can reach 100% for central thrombi, located in main, lobar or segmental pulmonary arteries. Its specificity is greater than 90%. Nodes can produce false positives. It can also provide an alternative diagnosis in case PTE is not confirmed, so it would be a very useful test. Doppler ultrasound would be the most useful test to diagnose the source of the emboli, so it could also be a valid answer depending on what they mean by \"useful\". It is a noninvasive technique that has demonstrated its good sensitivity and specificity in the diagnosis of DVT compared to phlebography. It also allows the diagnosis of other pathologies that could justify the patient's symptoms. Its limitations are the assessment of infrapopliteal thrombi and, in many cases, in patients who are not thin, of the iliac and cava veins.", "type": "ANESTHESIOLOGY, CRITICAL CARE AND EMERGENCY MEDICINE", "options": {"1": "Plain abdominal x-ray.", "2": "Intravenous urography.", "3": "Spiral CT with contrast.", "4": "Abdominal ultrasound.", "5": "Doppler ultrasound."}, "correct_option": 3, "explanations": {"1": {"exist": false, "char_ranges": [], "word_ranges": [], "text": ""}, "2": {"exist": false, "char_ranges": [], "word_ranges": [], "text": ""}, "3": {"exist": true, "char_ranges": [[754, 1059]], "word_ranges": [[127, 178]], "text": "Spiral CT with contrast has acquired great value in the diagnosis of PTE, due to its rapidity and good sensitivity, higher than 83-92% in most studies, and according to some it can reach 100% for central thrombi, located in main, lobar or segmental pulmonary arteries. Its specificity is greater than 90%."}, "4": {"exist": false, "char_ranges": [], "word_ranges": [], "text": ""}, "5": {"exist": false, "char_ranges": [], "word_ranges": [], "text": ""}}} {"id": 77, "year": 2012, "question_id_specific": 31, "full_question": "A 17-year-old boy with Crohn's disease with colonic involvement of 2 years of evolution, in maintenance treatment with azathioprine, consults for the appearance since 5 days ago of purplish-red, hot, painful, bilateral subcutaneous nodules of pretibial location, associated with an increase in the number of stools and abdominal pain. The most appropriate approach in this case is:", "full_answer": "It appears that the patient has erythema nodosum lesions associated with a flare of his disease, which is common. Treatment of the disease and its control usually improves the cutaneous lesions. The lesions should not be biopsied.", "type": "DIGESTIVE SYSTEM", "options": {"1": "Recommend relative rest and warm cloths on both legs and add antidepressant treatment.", "2": "Biopsy skin areas away from the injured areas and prescribe opioid analgesics at the beginning.", "3": "Biopsy skin areas away from the injured areas and prescribe opioid analgesics at the beginning.", "4": "Suspect bilateral lower extremity ischemia of drug origin.", "5": "Adjustment of the treatment of the disease."}, "correct_option": 5, "explanations": {"1": {"exist": false, "char_ranges": [], "word_ranges": [], "text": ""}, "2": {"exist": true, "char_ranges": [[0, 230]], "word_ranges": [[0, 37]], "text": "It appears that the patient has erythema nodosum lesions associated with a flare of his disease, which is common. Treatment of the disease and its control usually improves the cutaneous lesions. The lesions should not be biopsied."}, "3": {"exist": true, "char_ranges": [[0, 230]], "word_ranges": [[0, 37]], "text": "It appears that the patient has erythema nodosum lesions associated with a flare of his disease, which is common. Treatment of the disease and its control usually improves the cutaneous lesions. The lesions should not be biopsied."}, "4": {"exist": false, "char_ranges": [], "word_ranges": [], "text": ""}, "5": {"exist": true, "char_ranges": [[0, 230]], "word_ranges": [[0, 37]], "text": "It appears that the patient has erythema nodosum lesions associated with a flare of his disease, which is common. Treatment of the disease and its control usually improves the cutaneous lesions. The lesions should not be biopsied."}}} {"id": 440, "year": 2018, "question_id_specific": 97, "full_question": "A 24-year-old woman consults for lower abdominal pain of two weeks of evolution that has worsened with the last sexual intercourse. She reports having a recent partner with whom she only occasionally uses a condom. On examination, she was in fair general condition. Temperature 38.6ºC, leukocytes 16,000/uL (85% neutrophils); CRP 30 mg/L. Speculum examination shows abundant abnormal vaginal discharge and painful cervical mobilization. Indicate the FALSE answer:", "full_answer": "Pelvic inflammatory disease is suspected, given the patient's symptoms. According to the 2006 SEGO protocol for Pelvic Inflammatory Disease, answer 2 is the false answer and, therefore, the one that should be marked. This is because hysterosalpingography can spread the infection to the pelvic and abdominal cavity. The rest of the answers are correct.", "type": "GYNECOLOGY AND OBSTETRICS", "options": {"1": "We would start antibiotic treatment as soon as we suspect the diagnosis.", "2": "We would indicate as a preferential complementary test a hysterosalpingography for tubal assessment.", "3": "Delay in diagnosis and treatment increases the occurrence of sequelae.", "4": "The presence of abscess would be a criterion for hospitalization.", "5": NaN}, "correct_option": 2, "explanations": {"1": {"exist": false, "char_ranges": [], "word_ranges": [], "text": ""}, "2": {"exist": true, "char_ranges": [[141, 315]], "word_ranges": [[19, 47]], "text": "answer 2 is the false answer and, therefore, the one that should be marked. This is because hysterosalpingography can spread the infection to the pelvic and abdominal cavity."}, "3": {"exist": false, "char_ranges": [], "word_ranges": [], "text": ""}, "4": {"exist": false, "char_ranges": [], "word_ranges": [], "text": ""}, "5": {"exist": false, "char_ranges": [], "word_ranges": [], "text": ""}}} {"id": 203, "year": 2013, "question_id_specific": 47, "full_question": "55-year-old woman, operated on for appendicitis 24 years ago, who comes in because while previously well, she has started with repeated vomiting after breakfast, about 12 hours ago, associated with abdominal distention. She presented a diarrhea stool a few hours after the onset of the symptoms. Examination revealed abdominal distension, tympanism and increased bowel sounds, but no peritoneal irritation. Laboratory tests showed no abnormalities. The abdominal X-ray showed dilatation of the small bowel loops, without gas in the rectal ampulla. Regarding this case it is true that:", "full_answer": "This is a typical description of a picture of intestinal obstruction due to bridging or adhesions due to previous surgery. The correct answer is 5 and not 1 because most of these situations resolve spontaneously with conservative treatment.", "type": "GENERAL SURGERY", "options": {"1": "This is a picture of intestinal flange obstruction requiring emergency laparotomy.", "2": "Most likely we are facing a case of obstruction at the level of the large intestine.", "3": "The fact that the patient presented a diarrheal stool allows us to exclude the diagnosis of intestinal obstruction. Further imaging studies are necessary to ensure a diagnosis.", "4": "We should indicate the practice of a decompressive colonoscopy.", "5": "Initial management should be conservative, with serum therapy, nasogastric aspiration and periodic clinical, analytical and radiographic monitoring."}, "correct_option": 5, "explanations": {"1": {"exist": true, "char_ranges": [[0, 240]], "word_ranges": [[0, 38]], "text": "This is a typical description of a picture of intestinal obstruction due to bridging or adhesions due to previous surgery. The correct answer is 5 and not 1 because most of these situations resolve spontaneously with conservative treatment."}, "2": {"exist": false, "char_ranges": [], "word_ranges": [], "text": ""}, "3": {"exist": false, "char_ranges": [], "word_ranges": [], "text": ""}, "4": {"exist": false, "char_ranges": [], "word_ranges": [], "text": ""}, "5": {"exist": true, "char_ranges": [[0, 240]], "word_ranges": [[0, 38]], "text": "This is a typical description of a picture of intestinal obstruction due to bridging or adhesions due to previous surgery. The correct answer is 5 and not 1 because most of these situations resolve spontaneously with conservative treatment."}}} {"id": 158, "year": 2012, "question_id_specific": 133, "full_question": "Obese 12-year-old boy (87 kg). He has been complaining of mechanical pain in the right thigh and knee for 4 months. Clinical examination reveals a 1 cm discrepancy in the length of the lower extremities and an attitude of the right lower extremity in external rotation with loss of internal rotation. We should suspect that he presents:", "full_answer": "We are presented with an adolescent patient who is overweight with pain in the thigh and knee, a length discrepancy of 1 cm and the limb in external rotation. This picture should automatically lead us to mark epiphysiolysis capitis femoris (proximal femoral epiphysiolysis, the fact of reading it in Latin nomina is a bit of a misunderstanding but it should not mislead us). We should think of this one because it is the most frequent and possible (correct answer 2). The other options are not impossible but they are less frequent and they would not ask us this if they wanted us to choose another option. If they were sequelae of hip dysplasia, it would not have this 4-month onset and the age of onset would be earlier. Complications of dysplasia are dislocations and subluxations that may require surgery and avascular necrosis of the head (false answer 5). Perthes disease occurs in younger children (4-9 years) with episodes of limping and pain in the hip and knee (false answer 4). In a child the most typical would be a fracture at the level of the physis, not at the level of the femoral neck, so we discard 3. For us to consider this option we would have to have been presented with a bone cyst, something that would justify a weakness in the femoral neck area. As for juvenile chronic arthritis, it is a rheumatoid option surrounded by trauma options. They do not speak to us of rheumatologic factors in an analytical analysis, nor of a polyarticular picture. Perhaps we could think about this one because they talk about pain in the right knee, but this picture that they present us is limited to the right lower limb, they do not talk about other joints or fever or other accompanying systemic symptoms, so it is not an option to evaluate, especially with the other options presented to us that would be more likely in view of the picture presented to us (option 1 false). In addition, the knee pain in this case refers to a referred pain of the hip, without involvement of the knee.", "type": "TRAUMATOLOGY AND ORTHOPEDICS", "options": {"1": "Juvenile chronic arthritis.", "2": "Epiphysiolysis capitis femoris.", "3": "Fracture of the femoral neck due to overload.", "4": "Perthes disease.", "5": "Sequelae of developmental dysplasia of the hip."}, "correct_option": 2, "explanations": {"1": {"exist": true, "char_ranges": [[1471, 1885]], "word_ranges": [[258, 334]], "text": "Perhaps we could think about this one because they talk about pain in the right knee, but this picture that they present us is limited to the right lower limb, they do not talk about other joints or fever or other accompanying systemic symptoms, so it is not an option to evaluate, especially with the other options presented to us that would be more likely in view of the picture presented to us (option 1 false)."}, "2": {"exist": true, "char_ranges": [[375, 467]], "word_ranges": [[63, 80]], "text": "We should think of this one because it is the most frequent and possible (correct answer 2)."}, "3": {"exist": true, "char_ranges": [[989, 1119]], "word_ranges": [[170, 198]], "text": "In a child the most typical would be a fracture at the level of the physis, not at the level of the femoral neck, so we discard 3."}, "4": {"exist": true, "char_ranges": [[862, 988]], "word_ranges": [[148, 170]], "text": "Perthes disease occurs in younger children (4-9 years) with episodes of limping and pain in the hip and knee (false answer 4)."}, "5": {"exist": true, "char_ranges": [[723, 861]], "word_ranges": [[128, 148]], "text": "Complications of dysplasia are dislocations and subluxations that may require surgery and avascular necrosis of the head (false answer 5)."}}} {"id": 46, "year": 2011, "question_id_specific": 155, "full_question": "A 1-year-old boy whose mother has just been diagnosed with bacilliferous pulmonary tuberculosis, having started correct treatment. The child is asymptomatic with a normal examination, negative tuberculin test (PT) and normal chest Rx. What would be the attitude to adopt in the child?", "full_answer": "The correct answer is 5. The mother should be isolated for the first 2-4 weeks of treatment, after which she would no longer be contagious. If the result of the repeat PT is still negative, the child should not be tested again. If it is positive, the X-ray (or pulmonary CT in younger children) should be repeated.", "type": "PEDIATRICS", "options": {"1": "Only separate him from the mother until her treatment is completed (at least 6 months).", "2": "Perform a high-resolution CT scan of the lung, and if normal, repeat PT at 8-12 weeks.", "3": "Chemoprophylaxis with isoniazid (INH) until the mother's smear test is negative.", "4": "Chemoprophylaxis with INH 6-9 months. New PT and chest X-ray at the end of treatment, to decide whether or not to conclude treatment.", "5": "Chemoprophylaxis with INH, 8-12 weeks. Repeat PT. If negative, stop treatment. If positive, perform chest X-ray to decide whether or not there is disease."}, "correct_option": 5, "explanations": {"1": {"exist": false, "char_ranges": [], "word_ranges": [], "text": ""}, "2": {"exist": false, "char_ranges": [], "word_ranges": [], "text": ""}, "3": {"exist": false, "char_ranges": [], "word_ranges": [], "text": ""}, "4": {"exist": false, "char_ranges": [], "word_ranges": [], "text": ""}, "5": {"exist": true, "char_ranges": [[25, 314]], "word_ranges": [[5, 57]], "text": "The mother should be isolated for the first 2-4 weeks of treatment, after which she would no longer be contagious. If the result of the repeat PT is still negative, the child should not be tested again. If it is positive, the X-ray (or pulmonary CT in younger children) should be repeated."}}} {"id": 593, "year": 2022, "question_id_specific": 146, "full_question": "A 40-year-old woman, with no personal or family history of interest, presents several days of watery diarrhea, very abundant, accompanied by loss of 10 kg of weight, flushing and facial reddening, together with lipothymias and colicky abdominal pain, with hypokalemia in the laboratory data. An abdominal ultrasound shows a mass of 1.5 cm in diameter in the pancreatic tail. Among the following, your primary diagnostic suspicion is:", "full_answer": "Vasoactive intestinal peptide (VIP). VIPoma-diarrhea. The 3.", "type": "ONCOLOGY", "options": {"1": "Insulinoma.", "2": "PPoma.", "3": "VIPoma.", "4": "Glucagonoma.", "5": NaN}, "correct_option": 3, "explanations": {"1": {"exist": false, "char_ranges": [], "word_ranges": [], "text": ""}, "2": {"exist": false, "char_ranges": [], "word_ranges": [], "text": ""}, "3": {"exist": true, "char_ranges": [[0, 60]], "word_ranges": [[0, 7]], "text": "Vasoactive intestinal peptide (VIP). VIPoma-diarrhea. The 3."}, "4": {"exist": false, "char_ranges": [], "word_ranges": [], "text": ""}, "5": {"exist": false, "char_ranges": [], "word_ranges": [], "text": ""}}} {"id": 395, "year": 2016, "question_id_specific": 136, "full_question": "A 41-year-old man comes to the Emergency Department with three days of swelling and pain in the right knee, with functional impotence and fever Two weeks earlier he had had a self-limited diarrhea. On examination there is joint effusion, so we proceed to perform an arthrocentesis and obtain 50 cc of cloudy liquid, with decreased viscosity and the following analytical parameters: leukocytes 40. 000/microL (85% of neutrophils), glucose 40 mg/dL, absence of crystals, Gram stain: no microorganisms are observed. Which of the following statements about this patient is WRONG:", "full_answer": "A gram negative stain never rules out the presence of infection.", "type": "TRAUMATOLOGY AND ORTHOPEDICS", "options": {"1": "Treatment with cloxacillin and ceftriaxone should be initiated pending the result of the fluid culture.", "2": "It is advisable to perform daily arthrocentesis to relieve symptoms and prevent joint destruction.", "3": "If the culture is negative, it is likely to be reactive arthritis.", "4": "Negative Gram stain rules out septic arthritis.", "5": NaN}, "correct_option": 4, "explanations": {"1": {"exist": false, "char_ranges": [], "word_ranges": [], "text": ""}, "2": {"exist": false, "char_ranges": [], "word_ranges": [], "text": ""}, "3": {"exist": false, "char_ranges": [], "word_ranges": [], "text": ""}, "4": {"exist": true, "char_ranges": [[0, 64]], "word_ranges": [[0, 11]], "text": "A gram negative stain never rules out the presence of infection."}, "5": {"exist": false, "char_ranges": [], "word_ranges": [], "text": ""}}} {"id": 424, "year": 2018, "question_id_specific": 91, "full_question": "A 27-year-old woman consults for secondary amenorrhea of one year of evolution. She has no iatrogenic history. Analytically, the estradiol concentration is below normal and the prolactin value is 12 ng/mL. Indicate which of the following examinations would you request as a first step to try to identify the origin of the disorder:", "full_answer": "Young woman with secondary amenorrhea, with no history of interest. Prolactin values are normal (values above 20ngr/ml are considered hyperprolectinemia), thus ruling out hyperprolactinemia as the cause of amenorrhea. They tell us that the estrogen level is low. In this case we should suspect early ovarian failure. For its diagnosis we will ask for a basal FSH (in cases of premature ovarian failure it would be elevated).", "type": "ENDOCRINOLOGY", "options": {"1": "Hypothalamohypophyseal MRI.", "2": "Karyotype.", "3": "Basal FSH.", "4": "Chorionic gonadotropin stimulation.", "5": NaN}, "correct_option": 3, "explanations": {"1": {"exist": false, "char_ranges": [], "word_ranges": [], "text": ""}, "2": {"exist": false, "char_ranges": [], "word_ranges": [], "text": ""}, "3": {"exist": true, "char_ranges": [[218, 424]], "word_ranges": [[29, 67]], "text": "They tell us that the estrogen level is low. In this case we should suspect early ovarian failure. For its diagnosis we will ask for a basal FSH (in cases of premature ovarian failure it would be elevated)."}, "4": {"exist": false, "char_ranges": [], "word_ranges": [], "text": ""}, "5": {"exist": false, "char_ranges": [], "word_ranges": [], "text": ""}}} {"id": 169, "year": 2013, "question_id_specific": 87, "full_question": "A 70-year-old woman, hypertensive and diabetic, who has worked as a sales clerk all her life. She consults for an internal malleolar lesion in the right leg with intense pain throughout the day that worsens at night. On examination, there are uncomplicated varicose cords in both extremities and normal femoral and popliteal pulses with absence of distal pulses. What examination should we perform first?", "full_answer": "At least for me, as a family physician, I usually start with the least invasive and most cost-effective tests, and in the case described, an ischemic problem in the lower limbs is the ankle-brachial index.", "type": "CARDIOLOGY AND CARDIOVASCULAR SURGERY", "options": {"1": "Venous echo-Doppler of lower limbs.", "2": "Angioresonance of lower limbs.", "3": "Arteriography.", "4": "Ankle-brachial index.", "5": "AngioCT."}, "correct_option": 4, "explanations": {"1": {"exist": false, "char_ranges": [], "word_ranges": [], "text": ""}, "2": {"exist": false, "char_ranges": [], "word_ranges": [], "text": ""}, "3": {"exist": false, "char_ranges": [], "word_ranges": [], "text": ""}, "4": {"exist": true, "char_ranges": [[40, 205]], "word_ranges": [[8, 35]], "text": "I usually start with the least invasive and most cost-effective tests, and in the case described, an ischemic problem in the lower limbs is the ankle-brachial index."}, "5": {"exist": false, "char_ranges": [], "word_ranges": [], "text": ""}}} {"id": 95, "year": 2012, "question_id_specific": 170, "full_question": "A 60-year-old male patient who reported 10 days ago the appearance of blistering lesions on the dorsum of the hands after sun exposure. The picture is accompanied by skin fragility. Histopathologically there is a subepidermal blister with PAS+ deposits in and around the superficial dermal vessels. The most plausible diagnosis is:", "full_answer": "The fact of being a photoinduced picture, the histology and the age make the diagnosis of PCT more probable (3). The rest of the answers do not fit with the clinical picture or the histology.", "type": "DERMATOLOGY, VENEREOLOGY AND PLASTIC SURGERY", "options": {"1": "Familial benign pemphigus.", "2": "Pemphigoid.", "3": "Porphyria cutanea tarda.", "4": "Pemphigus vulgaris.", "5": "Scalded skin syndrome."}, "correct_option": 3, "explanations": {"1": {"exist": true, "char_ranges": [[113, 191]], "word_ranges": [[20, 35]], "text": "The rest of the answers do not fit with the clinical picture or the histology."}, "2": {"exist": true, "char_ranges": [[113, 191]], "word_ranges": [[20, 35]], "text": "The rest of the answers do not fit with the clinical picture or the histology."}, "3": {"exist": true, "char_ranges": [[0, 112]], "word_ranges": [[0, 20]], "text": "The fact of being a photoinduced picture, the histology and the age make the diagnosis of PCT more probable (3)."}, "4": {"exist": true, "char_ranges": [[113, 191]], "word_ranges": [[20, 35]], "text": "The rest of the answers do not fit with the clinical picture or the histology."}, "5": {"exist": true, "char_ranges": [[113, 191]], "word_ranges": [[20, 35]], "text": "The rest of the answers do not fit with the clinical picture or the histology."}}} {"id": 537, "year": 2021, "question_id_specific": 1, "full_question": "44-year-old patient with a rapidly growing lesion (16x8x12 cm) involving the soft tissues of the proximal third of the right thigh. He reports that he has started to notice the mass, which has a hard consistency and is adhered to deep planes, in the last 6 months. Previously, she had been a regular athlete and had even finished a marathon. On examination, collateral circulation was observed, but no right inguinal lymph nodes were palpable. After biopsy it was confirmed to be a high-grade undifferentiated pleomorphic sarcoma, which in the imaging studies was found proximal to the lesser trochanter. The extension CT scan is negative. What would be the therapeutic approach?", "full_answer": "In resection margins, we have: intralesional → marginal (option 3) → Extended → Radical (options 1 and 4). This is a high-grade tumor, in which extended resection including tumor-free margins is indicated. Radical amputation surgery has not been shown to improve survival being reserved for selected cases (invasion of neurovascular structures, pathologic fracture...). Therefore, we discard both option 1 and 4; option 3 is also discarded, because it is a smaller margin than the extended one; by elimination we are left with option 2.", "type": "TRAUMATOLOGY", "options": {"1": "Hip disarticulation.", "2": "Treatment with ifosfamide and tamoxifen for 6 cycles prior to resection surgery to reduce the size of the lesion.", "3": "Marginal surgery removing the pseudocapsule that these tumors usually form in response to their rapid growth.", "4": "Modified right hemipelvectomy.", "5": NaN}, "correct_option": 2, "explanations": {"1": {"exist": true, "char_ranges": [[206, 412]], "word_ranges": [[32, 61]], "text": "Radical amputation surgery has not been shown to improve survival being reserved for selected cases (invasion of neurovascular structures, pathologic fracture...). Therefore, we discard both option 1 and 4;"}, "2": {"exist": false, "char_ranges": [], "word_ranges": [], "text": ""}, "3": {"exist": true, "char_ranges": [[413, 494]], "word_ranges": [[61, 76]], "text": "option 3 is also discarded, because it is a smaller margin than the extended one;"}, "4": {"exist": true, "char_ranges": [[206, 412]], "word_ranges": [[32, 61]], "text": "Radical amputation surgery has not been shown to improve survival being reserved for selected cases (invasion of neurovascular structures, pathologic fracture...). Therefore, we discard both option 1 and 4;"}, "5": {"exist": false, "char_ranges": [], "word_ranges": [], "text": ""}}} {"id": 363, "year": 2016, "question_id_specific": 97, "full_question": "A 35-year-old patient who, due to hematemesis due to ulcus, receives a transfusion of 2 packed red blood cells. 5-10 minutes after starting the transfusion, she begins with fever, chills, hypotension and pain in the lumbar region. What would be the most likely diagnosis?", "full_answer": "Acute hemolytic transfusion reactions can occur within minutes. Clinically they are characterized by chills, fever, urticaria, tachycardia, nausea and vomiting, lumbar pain, hemorrhage due to disseminated intravascular coagulation (DIC)..... We are told of a patient who, within minutes of starting a transfusion, presented with an acute and very serious condition (with shock). Such an acute and severe situation only fits option 4.", "type": "HEMATOLOGY", "options": {"1": "Bacterial contamination of the blood.", "2": "Febrile reaction secondary to transfusion.", "3": "Febrile reaction due to plasma contaminating red blood cells.", "4": "Hemolytic transfusion reaction.", "5": NaN}, "correct_option": 4, "explanations": {"1": {"exist": false, "char_ranges": [], "word_ranges": [], "text": ""}, "2": {"exist": false, "char_ranges": [], "word_ranges": [], "text": ""}, "3": {"exist": false, "char_ranges": [], "word_ranges": [], "text": ""}, "4": {"exist": true, "char_ranges": [[242, 433]], "word_ranges": [[29, 62]], "text": "We are told of a patient who, within minutes of starting a transfusion, presented with an acute and very serious condition (with shock). Such an acute and severe situation only fits option 4."}, "5": {"exist": false, "char_ranges": [], "word_ranges": [], "text": ""}}} {"id": 339, "year": 2016, "question_id_specific": 33, "full_question": "67-year-old woman diagnosed with an infiltrating ductal carcinoma of the breast and no family history of neoplasia. What additional studies should be performed on the tumor for its clinical-therapeutic implications?", "full_answer": "All breast carcinomas should be accompanied by the study of hormone receptors and HER-2 for the prognostic implications of the tumor, as well as the adequacy of the treatment of the same, since depending on the result will apply some drugs or others. The study of relatives and the BCRA gene is not indicated since she does not have a family history of neoplasia.", "type": "GYNECOLOGY AND OBSTETRICS", "options": {"1": "Complete phenotypic study by flow cytometry.", "2": "Study of hormone receptors and HER2.", "3": "Study of hormone receptors, e-cadherin and study of first-degree relatives.", "4": "BRCA 1-2 study and study of first-degree relatives.", "5": NaN}, "correct_option": 2, "explanations": {"1": {"exist": false, "char_ranges": [], "word_ranges": [], "text": ""}, "2": {"exist": true, "char_ranges": [[0, 187]], "word_ranges": [[0, 32]], "text": "All breast carcinomas should be accompanied by the study of hormone receptors and HER-2 for the prognostic implications of the tumor, as well as the adequacy of the treatment of the same,"}, "3": {"exist": true, "char_ranges": [[251, 363]], "word_ranges": [[43, 64]], "text": "The study of relatives and the BCRA gene is not indicated since she does not have a family history of neoplasia."}, "4": {"exist": true, "char_ranges": [[251, 363]], "word_ranges": [[43, 64]], "text": "The study of relatives and the BCRA gene is not indicated since she does not have a family history of neoplasia."}, "5": {"exist": false, "char_ranges": [], "word_ranges": [], "text": ""}}} {"id": 144, "year": 2012, "question_id_specific": 142, "full_question": "A 13-year-old girl consults us for weight loss, asthenia, secondary amenorrhea and constipation. Examination revealed a temperature of 35ºC, weight below the 3rd percentile, bradycardia and orthostatic hypotension. Laboratory tests showed anemia with mild leukopenia, cholesterol of 230 mg/dL and on the electrocardiogram a flattened T wave with ST-segment elevation and prolonged QTc. What process would you suspect and what would you do next?", "full_answer": "The correct answer is 1. The symptoms and signs described are the biological findings associated with anorexia nervosa. The ECG disturbances are derived from electrolyte disturbances.", "type": "PEDIATRICS", "options": {"1": "Suspect anorexia nervosa and inquire about intake, body image, etc.", "2": "Suspect hypothyroidism and request TSH and thyroid hormones.", "3": "Suspect cardiomyopathy and request echocardiography.", "4": "I would suspect celiac disease and request anti-transglutaminase IgA antibodies.", "5": "Suspect panhypopituitarism and request a cranial CT scan."}, "correct_option": 1, "explanations": {"1": {"exist": true, "char_ranges": [[25, 119]], "word_ranges": [[5, 18]], "text": "The symptoms and signs described are the biological findings associated with anorexia nervosa."}, "2": {"exist": false, "char_ranges": [], "word_ranges": [], "text": ""}, "3": {"exist": false, "char_ranges": [], "word_ranges": [], "text": ""}, "4": {"exist": false, "char_ranges": [], "word_ranges": [], "text": ""}, "5": {"exist": false, "char_ranges": [], "word_ranges": [], "text": ""}}} {"id": 356, "year": 2016, "question_id_specific": 166, "full_question": "A pregnant woman, 10 weeks pregnant, comes to the emergency room with a blood pressure of 160/105 mmHg. She is in good general condition, with only mild headache, which is the reason for taking her blood pressure. After 4 hours of rest she presented with a blood pressure of 150/95mmHg. A complete blood count is normal and proteinuria is negative. What type of hypertension does she have?", "full_answer": "Chronic hypertension. This is hypertension that is detected before pregnancy or before the 20th week of pregnancy and/or persists at 12 weeks postpartum.", "type": "GYNECOLOGY AND OBSTETRICS", "options": {"1": "Moderate pre-eclampsia.", "2": "Pregnancy-induced hypertension.", "3": "Chronic hypertension.", "4": "Eclampsia.", "5": NaN}, "correct_option": 3, "explanations": {"1": {"exist": false, "char_ranges": [], "word_ranges": [], "text": ""}, "2": {"exist": false, "char_ranges": [], "word_ranges": [], "text": ""}, "3": {"exist": true, "char_ranges": [[22, 153]], "word_ranges": [[2, 23]], "text": "This is hypertension that is detected before pregnancy or before the 20th week of pregnancy and/or persists at 12 weeks postpartum."}, "4": {"exist": false, "char_ranges": [], "word_ranges": [], "text": ""}, "5": {"exist": false, "char_ranges": [], "word_ranges": [], "text": ""}}} {"id": 495, "year": 2020, "question_id_specific": 64, "full_question": "65-year-old woman with no comorbidities, undergoing left breast-conserving surgery with selective sentinel lymph node biopsy. The pathological anatomy of the specimen shows an infiltrating ductal carcinoma grade III of 11 mm, two negative sentinel nodes, estrogen receptors: 0%, progesterone receptors: 0%, Ki 67: 70%, HER2: negative. The extension study is negative. Adjuvant treatment would be:", "full_answer": "Typical question, conservative surgery implies radiate, for sure. High grade, and no hormone receptors or her2 (the dreaded triple negative). I was there when there were no taxanes or herceptin and I was in the trials that showed that more and more are being cured, unfortunately still not all. That is why today, anthracyclines (does not suffer from heart) and taxanes before RT. My specialty is beautiful, and will be more so in the coming years.", "type": "MEDICAL ONCOLOGY", "options": {"1": "Chemotherapy based on anthracyclines and taxanes, radiotherapy and trastuzumab.", "2": "Chemotherapy based on anthracyclines and taxanes, radiotherapy and tamoxifen.", "3": "Chemotherapy based on anthracyclines and taxanes, and radiotherapy.", "4": "Chemotherapy based on anthracyclines and taxanes, radiotherapy and aromatase inhibitors, since it is postmenopausal.", "5": NaN}, "correct_option": 3, "explanations": {"1": {"exist": false, "char_ranges": [], "word_ranges": [], "text": ""}, "2": {"exist": false, "char_ranges": [], "word_ranges": [], "text": ""}, "3": {"exist": true, "char_ranges": [[307, 380]], "word_ranges": [[52, 63]], "text": "today, anthracyclines (does not suffer from heart) and taxanes before RT."}, "4": {"exist": false, "char_ranges": [], "word_ranges": [], "text": ""}, "5": {"exist": false, "char_ranges": [], "word_ranges": [], "text": ""}}} {"id": 355, "year": 2016, "question_id_specific": 166, "full_question": "A pregnant woman, 10 weeks pregnant, comes to the emergency room with a blood pressure of 160/105 mmHg. She is in good general condition, with only mild headache, which is the reason for taking her blood pressure. After 4 hours of rest she presented with a blood pressure of 150/95mmHg. A complete blood count is normal and proteinuria is negative. What type of hypertension is present?", "full_answer": "The correct answer is 3. The SEGO in its 2006 protocol \"Hypertensive disorders of pregnancy\" refers that this is chronic hypertension, although diagnosed in pregnancy. This is because it has been diagnosed at 10 weeks of gestation. Above 20 weeks' gestation, we would be talking about preeclampsia (if HT + proteinuria or uterine artery pulsatility index above the 95th percentile), pregnancy-induced hypertension (only HT) or eclampsia (a form of severe preeclampsia).", "type": "GYNECOLOGY AND OBSTETRICS", "options": {"1": "Moderate preeclampsia.", "2": "Pregnancy-induced hypertension.", "3": "Chronic hypertension.", "4": "Eclampsia.", "5": NaN}, "correct_option": 3, "explanations": {"1": {"exist": true, "char_ranges": [[232, 469]], "word_ranges": [[37, 71]], "text": "Above 20 weeks' gestation, we would be talking about preeclampsia (if HT + proteinuria or uterine artery pulsatility index above the 95th percentile), pregnancy-induced hypertension (only HT) or eclampsia (a form of severe preeclampsia)."}, "2": {"exist": true, "char_ranges": [[232, 469]], "word_ranges": [[37, 71]], "text": "Above 20 weeks' gestation, we would be talking about preeclampsia (if HT + proteinuria or uterine artery pulsatility index above the 95th percentile), pregnancy-induced hypertension (only HT) or eclampsia (a form of severe preeclampsia)."}, "3": {"exist": true, "char_ranges": [[25, 231]], "word_ranges": [[5, 37]], "text": "The SEGO in its 2006 protocol \"Hypertensive disorders of pregnancy\" refers that this is chronic hypertension, although diagnosed in pregnancy. This is because it has been diagnosed at 10 weeks of gestation."}, "4": {"exist": true, "char_ranges": [[232, 469]], "word_ranges": [[37, 71]], "text": "Above 20 weeks' gestation, we would be talking about preeclampsia (if HT + proteinuria or uterine artery pulsatility index above the 95th percentile), pregnancy-induced hypertension (only HT) or eclampsia (a form of severe preeclampsia)."}, "5": {"exist": false, "char_ranges": [], "word_ranges": [], "text": ""}}} {"id": 94, "year": 2012, "question_id_specific": 31, "full_question": "A 17-year-old boy with Crohn's disease with colonic involvement of 2 years of evolution, in maintenance treatment with azathioprine, consults for the appearance for 5 days of subcutaneous red purplish, hot, painful, bilateral, pretibial localization nodules, associated with an increase in the number of stools and abdominal pain. The most appropriate approach in this case is:", "full_answer": "The patient's presenting picture is presumably erythema nodosum. About 10% of cases of erythema nodosum are associated with inflammatory bowel disease, both ulcerative colitis and Crohn's disease. As mentioned, in most cases, erythema nodosum has a self-limited course. When associated with inflammatory bowel disease, erythema nodosum usually resolves with treatment of the intestinal flare, and recurs with disease recurrences. Local measures include elevation of the legs and bed rest. In particularly severe cases, oral administration of potassium iodide has been shown to be helpful. Although the use of aspirin and other NSAIDs is effective for erythema nodosum, their use in patients with inflammatory bowel disease should be limited, as they may worsen the intestinal picture, the usefulness of antidepressants in this picture has not been described, so answer nº1 would not be correct.", "type": "DERMATOLOGY, VENEREOLOGY AND PLASTIC SURGERY", "options": {"1": "Recommend relative rest and warm cloths on both legs and add antidepressant treatment.", "2": "Biopsy areas of skin away from the injured areas and prescribe opioid analgesics for entry.", "3": "Suspect the existence of a malignant intestinal tumor lesion as a trigger of the cutaneous process.", "4": "Suspect bilateral lower extremity ischemia of drug-induced origin.", "5": "Adjustment of bowel disease treatment."}, "correct_option": 5, "explanations": {"1": {"exist": true, "char_ranges": [[589, 894]], "word_ranges": [[84, 133]], "text": "Although the use of aspirin and other NSAIDs is effective for erythema nodosum, their use in patients with inflammatory bowel disease should be limited, as they may worsen the intestinal picture, the usefulness of antidepressants in this picture has not been described, so answer nº1 would not be correct."}, "2": {"exist": false, "char_ranges": [], "word_ranges": [], "text": ""}, "3": {"exist": false, "char_ranges": [], "word_ranges": [], "text": ""}, "4": {"exist": false, "char_ranges": [], "word_ranges": [], "text": ""}, "5": {"exist": true, "char_ranges": [[270, 429]], "word_ranges": [[38, 59]], "text": "When associated with inflammatory bowel disease, erythema nodosum usually resolves with treatment of the intestinal flare, and recurs with disease recurrences."}}} {"id": 39, "year": 2011, "question_id_specific": 134, "full_question": "60-year-old woman, diagnosed with breast cancer treated with surgery and radiotherapy 10 years ago. Currently with metastases in the dorsal spine and sacroiliac joint, for which she has received monthly treatment with i.v. zoledronic acid for the last two years. The patient presented spontaneous bone exposure at the level of the mandibular bone of approximately 2 cm. There are no signs of acute infection and the gum around the bone exposure is strictly normal. The mandibular CT scan shows an area of bone sclerosis. What would be the diagnosis?", "full_answer": "This one has been difficult for me, we are going to rule it out. 1 cannot be, because there would be infectious signs. Answer 2 is possible because bisphosphonates create mandibular necrosis. 3 seems unlikely to me because the radiotherapy that the patient received would be oriented on the breast and not on the mandible. I rule out number 4, as it is not a case of dental root involvement. 5 is possible; after all, the patient already has bone metastases. So, I hesitate between 2 and 5, which one to choose? I am certainly not sure what a bone sclerosis means on a CT scan, but a bone metastasis would be more destructive than sclerotizing. On the other hand, using reverse psychology, what did the questioner want me to know? That bisphosphonates can create mandibular necrosis? Or that a woman with several bone metastases can have a metastasis also in the mandible? The latter seems too simple, and would make me lean more towards 2, but it's not clear to me. Be that as it may, between two possible answers you have to always answer, and between 2 and 5, you have to check 2, which is more likely.", "type": "OTORHINOLARYNGOLOGY AND MAXILLOFACIAL SURGERY", "options": {"1": "Odontogenic abscess.", "2": "Osteonecrosis due to bisphosphonates.", "3": "Osteoradionecrosis.", "4": "Dental root included.", "5": "Metastasis of breast carcinoma."}, "correct_option": 2, "explanations": {"1": {"exist": true, "char_ranges": [[64, 118]], "word_ranges": [[13, 23]], "text": "out. 1 cannot be, because there would be infectious signs."}, "2": {"exist": true, "char_ranges": [[577, 644]], "word_ranges": [[105, 116]], "text": "scan, but a bone metastasis would be more destructive than sclerotizing."}, "3": {"exist": true, "char_ranges": [[192, 322]], "word_ranges": [[32, 55]], "text": "3 seems unlikely to me because the radiotherapy that the patient received would be oriented on the breast and not on the mandible."}, "4": {"exist": true, "char_ranges": [[323, 458]], "word_ranges": [[55, 81]], "text": "I rule out number 4, as it is not a case of dental root involvement. 5 is possible; after all, the patient already has bone metastases."}, "5": {"exist": true, "char_ranges": [[577, 644]], "word_ranges": [[105, 116]], "text": "scan, but a bone metastasis would be more destructive than sclerotizing."}}} {"id": 252, "year": 2014, "question_id_specific": 83, "full_question": "What disease would we suspect in a young patient with a frequent history of dysphagia to solids and liquids with repeated food impactions, without symptoms of heartburn and in whom oral endoscopy shows multiple concentric esophageal rings with normal mucosa (trachealized appearance)?", "full_answer": "The endoscopic appearance of trachealized esophagus is very typical of eosinophilic esophagitis, which presents with dysphagia and repeated food impactions.", "type": "DIGESTIVE SYSTEM", "options": {"1": "Herpetic esophagitis.", "2": "Eosinophilic esophagitis.", "3": "Candidiasis esophagitis.", "4": "Cytomegalovirus esophagitis.", "5": "Adenocarcinoma of the esophagus."}, "correct_option": 2, "explanations": {"1": {"exist": false, "char_ranges": [], "word_ranges": [], "text": ""}, "2": {"exist": true, "char_ranges": [[0, 156]], "word_ranges": [[0, 20]], "text": "The endoscopic appearance of trachealized esophagus is very typical of eosinophilic esophagitis, which presents with dysphagia and repeated food impactions."}, "3": {"exist": false, "char_ranges": [], "word_ranges": [], "text": ""}, "4": {"exist": false, "char_ranges": [], "word_ranges": [], "text": ""}, "5": {"exist": false, "char_ranges": [], "word_ranges": [], "text": ""}}} {"id": 57, "year": 2011, "question_id_specific": 77, "full_question": "In a 42-year-old obese woman, cortisol levels in blood, urine and saliva are determined and in all the determinations they are elevated. After performing the nocturnal suppression test with dexamethasone, cortisol levels in blood remain elevated, so ACTH independent Cushing's syndrome is suspected, what additional test would you request to confirm the diagnosis at this time?", "full_answer": "Anyway, I think that the clinical case is poorly stated, since before doing the CT scan, ACTH should be measured and, once ACTH is suppressed, the CT scan should be done. But since the question is \"what would you do to confirm the diagnosis of suspected ACTH independent Cushing's syndrome\", the answer is CT.", "type": "ENDOCRINOLOGY", "options": {"1": "A petrosal sinus catheterization.", "2": "A gammagraphy with labeled somatostatin (Octreoscan).", "3": "A scan with Sesta-MIBI.", "4": "An abdominal ultrasound.", "5": "An adrenal CT scan."}, "correct_option": 5, "explanations": {"1": {"exist": false, "char_ranges": [], "word_ranges": [], "text": ""}, "2": {"exist": false, "char_ranges": [], "word_ranges": [], "text": ""}, "3": {"exist": false, "char_ranges": [], "word_ranges": [], "text": ""}, "4": {"exist": false, "char_ranges": [], "word_ranges": [], "text": ""}, "5": {"exist": false, "char_ranges": [], "word_ranges": [], "text": ""}}} {"id": 595, "year": 2022, "question_id_specific": 88, "full_question": "A 23-month-old boy presenting with fever, hypotension, repeated vomiting, meningeal signs, petechiae, and ecchymosis. The CBC shows 25,000 leukocytes (75% neutrophils, 10% white blood cells) and 12,000 platelets/microL, and the coagulation study shows a prothrombin activity of 35%. Which option is NOT appropriate for management?", "full_answer": "Patient with suspected meningococcal sepsis. Stabilization with fluids (hypotension) according to ABCDE is essential. Blood culture can be obtained together with the initial blood test. Early initiation of antibiotic therapy (cefotaxime) is essential. However, in an unstable patient such as ours, a lumbar puncture is not indicated.", "type": "PEDIATRICS", "options": {"1": "Intravenous cefotaxime administration.", "2": "Lumbar puncture.", "3": "Collection of blood culture.", "4": "Intravenous fluids.", "5": NaN}, "correct_option": 2, "explanations": {"1": {"exist": true, "char_ranges": [[186, 251]], "word_ranges": [[25, 33]], "text": "Early initiation of antibiotic therapy (cefotaxime) is essential."}, "2": {"exist": true, "char_ranges": [[252, 333]], "word_ranges": [[33, 47]], "text": "However, in an unstable patient such as ours, a lumbar puncture is not indicated."}, "3": {"exist": true, "char_ranges": [[118, 185]], "word_ranges": [[14, 25]], "text": "Blood culture can be obtained together with the initial blood test."}, "4": {"exist": true, "char_ranges": [[0, 117]], "word_ranges": [[0, 14]], "text": "Patient with suspected meningococcal sepsis. Stabilization with fluids (hypotension) according to ABCDE is essential."}, "5": {"exist": false, "char_ranges": [], "word_ranges": [], "text": ""}}} {"id": 98, "year": 2012, "question_id_specific": 171, "full_question": "The patient presents since 2 weeks ago a pruritic eruption consisting of multiple polygonal erythematous-violaceous papules, with flattened surface, preferentially located on the anterior aspect of the wrists, pretibial area and lumbar area. She also presents whitish reticulated lesions in the jugal mucosa. What is the most probable diagnosis?", "full_answer": "Lichen planus is a very common dermatosis of unknown etiology characterized by the appearance of pruritic polygonal papules, with a purplish color and shiny surface, preferentially located in the distal areas of the extremities. Characteristically these papules show a symmetrical distribution and are preferentially located on the anterior aspect of the wrists (Figure 1) and ankles. Oral mucosal involvement is very frequent, and approximately 75% of patients with lichen planus show an asymptomatic whitish reticulation affecting the jugal or lingual mucosa.", "type": "DERMATOLOGY, VENEREOLOGY AND PLASTIC SURGERY", "options": {"1": "Pityriasis versicolor.", "2": "Mycosis fungoides.", "3": "Psoriasis in drops.", "4": "Pityriasis rosea of Gibert.", "5": "Lichen planus."}, "correct_option": 5, "explanations": {"1": {"exist": false, "char_ranges": [], "word_ranges": [], "text": ""}, "2": {"exist": false, "char_ranges": [], "word_ranges": [], "text": ""}, "3": {"exist": false, "char_ranges": [], "word_ranges": [], "text": ""}, "4": {"exist": false, "char_ranges": [], "word_ranges": [], "text": ""}, "5": {"exist": true, "char_ranges": [[0, 384]], "word_ranges": [[0, 56]], "text": "Lichen planus is a very common dermatosis of unknown etiology characterized by the appearance of pruritic polygonal papules, with a purplish color and shiny surface, preferentially located in the distal areas of the extremities. Characteristically these papules show a symmetrical distribution and are preferentially located on the anterior aspect of the wrists (Figure 1) and ankles."}}} {"id": 118, "year": 2012, "question_id_specific": 117, "full_question": "A 14-year-old asymptomatic boy whose father has just been diagnosed with pulmonary TB undergoes a Mantoux test, with a negative result. What is the appropriate course of action?", "full_answer": "In young contacts, chemoprophylaxis should be started even if the mantoux test is negative and repeated after 2-3 months to decide whether to continue treatment for 6 months (if it turns positive) or to stop it.", "type": "INFECTOLOGY", "options": {"1": "Reassure her that she is not at risk.", "2": "Chest X-ray.", "3": "Start chemoprophylaxis and repeat skin test after 3 months.", "4": "Repeat skin test at one month.", "5": "A sputum study should be performed before starting chemoprophylaxis with Isoniazid for 1 year."}, "correct_option": 3, "explanations": {"1": {"exist": false, "char_ranges": [], "word_ranges": [], "text": ""}, "2": {"exist": false, "char_ranges": [], "word_ranges": [], "text": ""}, "3": {"exist": true, "char_ranges": [[0, 211]], "word_ranges": [[0, 36]], "text": "In young contacts, chemoprophylaxis should be started even if the mantoux test is negative and repeated after 2-3 months to decide whether to continue treatment for 6 months (if it turns positive) or to stop it."}, "4": {"exist": false, "char_ranges": [], "word_ranges": [], "text": ""}, "5": {"exist": false, "char_ranges": [], "word_ranges": [], "text": ""}}} {"id": 388, "year": 2016, "question_id_specific": 233, "full_question": "A 13-year-old boy presents to the pediatrics office with a feeling of heartburn during and after meals and occasional dysphagia, which increases with the ingestion of solid foods such as meat. In the family history, his mother is diagnosed with hiatal hernia. In the personal history she refers to allergy to pollens, mites and latex. Physical examination shows no significant findings except for eczematous lesions in the popliteal and antecubital hollows. Which of the following complementary tests would you NOT perform according to the most probable diagnoses?", "full_answer": "We are being presented with a case of eosinophilic esophagitis (adolescent male, with heartburn and dysphagia, and a history of atopic dermatitis). Of all the tests proposed to us, the least indicated in this case would be option 3.", "type": "PEDIATRICS", "options": {"1": "Upper gastrointestinal endoscopy.", "2": "Esophageal pHmetry of 24 hours.", "3": "Carbon 13 breath test for Helicobacter pylori.", "4": "Esophago-gastro-duodenal barium study.", "5": NaN}, "correct_option": 3, "explanations": {"1": {"exist": false, "char_ranges": [], "word_ranges": [], "text": ""}, "2": {"exist": false, "char_ranges": [], "word_ranges": [], "text": ""}, "3": {"exist": true, "char_ranges": [[0, 232]], "word_ranges": [[0, 39]], "text": "We are being presented with a case of eosinophilic esophagitis (adolescent male, with heartburn and dysphagia, and a history of atopic dermatitis). Of all the tests proposed to us, the least indicated in this case would be option 3."}, "4": {"exist": false, "char_ranges": [], "word_ranges": [], "text": ""}, "5": {"exist": false, "char_ranges": [], "word_ranges": [], "text": ""}}} {"id": 421, "year": 2018, "question_id_specific": 78, "full_question": "A 52-year-old man was referred to the gastroenterology department for hematochezia, tenesmus and reduction of stool diameter. A series of tests were performed and a diagnosis of adenocarcinoma of the sigma without distant metastasis was made. The patient underwent surgery and was referred to the medical oncology department for evaluation of complementary chemotherapy treatment. Which of the following is a poor prognostic factor after surgical resection and should be taken into account when planning chemotherapy treatment?", "full_answer": "On this occasion, we are asked about which factors have the greatest influence in order to carry out adjuvant chemotherapy in colorectal cancer. In order to do so, we must know which are the poor prognostic factors in this pathology where obstruction or intestinal perforation due to the lesion, invasion of adjacent organs, elevation of tumor markers such as CEA and the histology of the neoplasm stand out. Among these, less importance is given to the size of the lesion than to its degree of invasion of the intestinal wall or other adjacent structures, a criterion that must be taken into account for staging by TNM or Astler-Coller classification, which guide the indication of adjuvant treatment after the surgical approach.", "type": "GENERAL SURGERY", "options": {"1": "The presence of anemia at diagnosis.", "2": "The existence of a family history of colorectal cancer.", "3": "The size of the primary lesion and histological differentiation.", "4": "Perforation or adhesion of the tumor to adjacent organs.", "5": NaN}, "correct_option": 4, "explanations": {"1": {"exist": false, "char_ranges": [], "word_ranges": [], "text": ""}, "2": {"exist": false, "char_ranges": [], "word_ranges": [], "text": ""}, "3": {"exist": true, "char_ranges": [[422, 472]], "word_ranges": [[70, 80]], "text": "less importance is given to the size of the lesion"}, "4": {"exist": true, "char_ranges": [[18, 295]], "word_ranges": [[3, 49]], "text": "we are asked about which factors have the greatest influence in order to carry out adjuvant chemotherapy in colorectal cancer. In order to do so, we must know which are the poor prognostic factors in this pathology where obstruction or intestinal perforation due to the lesion,"}, "5": {"exist": false, "char_ranges": [], "word_ranges": [], "text": ""}}} {"id": 500, "year": 2020, "question_id_specific": 45, "full_question": "15-year-old female presenting with delayed menarche and short stature. She does not have intellectual disability. Which of the following genetic tests would be routinely used for the diagnosis of this patient:", "full_answer": "In a woman with delayed puberty and short stature we should always think about and rule out Turner syndrome, whose diagnosis is made by karyotyping (45X0).", "type": "PEDIATRICS", "options": {"1": "Massive sequencing (NGS).", "2": "FISH.", "3": "DNA and/or RNA microarrays.", "4": "Karyotype.", "5": NaN}, "correct_option": 4, "explanations": {"1": {"exist": false, "char_ranges": [], "word_ranges": [], "text": ""}, "2": {"exist": false, "char_ranges": [], "word_ranges": [], "text": ""}, "3": {"exist": false, "char_ranges": [], "word_ranges": [], "text": ""}, "4": {"exist": true, "char_ranges": [[0, 155]], "word_ranges": [[0, 26]], "text": "In a woman with delayed puberty and short stature we should always think about and rule out Turner syndrome, whose diagnosis is made by karyotyping (45X0)."}, "5": {"exist": false, "char_ranges": [], "word_ranges": [], "text": ""}}} {"id": 611, "year": 2022, "question_id_specific": 116, "full_question": "95-year-old woman living in a nursing home, independent for her basic activities of daily living, goes out to the garden for walks. She has a history of hypertension, dyslipidemia, osteoporosis and mild cognitive impairment. She suffers a fall when getting up at night to go to the bathroom. X-ray shows a displaced subcapital fracture of the right hip. What is the recommended treatment?", "full_answer": "We are told of a case of a 95-year-old patient with a displaced intracapsular (subcapital) hip fracture. In this case, the surgical options may be the use of cannulated screws or hip arthroplasty. Cannulated screws are reserved for cases of non-displaced fractures (Wrong answer 2) so the most indicated option would be arthroplasty, in this case hemiarthroplasty (Correct answer 1). Trochanteric pins are reserved for extracapsular fractures (Wrong answer 2) and conservative treatment is only considered in patients with very low functional demand or surgical contraindication (Wrong answer 4).", "type": "TRAUMATOLOGY", "options": {"1": "Hip hemiarthroplasty.", "2": "Trochanteric nail fixation.", "3": "Fixation with cannulated screws.", "4": "Conservative: bed-chair life.", "5": NaN}, "correct_option": 1, "explanations": {"1": {"exist": true, "char_ranges": [[197, 383]], "word_ranges": [[33, 60]], "text": "Cannulated screws are reserved for cases of non-displaced fractures (Wrong answer 2) so the most indicated option would be arthroplasty, in this case hemiarthroplasty (Correct answer 1)."}, "2": {"exist": true, "char_ranges": [[197, 459]], "word_ranges": [[33, 70]], "text": "Cannulated screws are reserved for cases of non-displaced fractures (Wrong answer 2) so the most indicated option would be arthroplasty, in this case hemiarthroplasty (Correct answer 1). Trochanteric pins are reserved for extracapsular fractures (Wrong answer 2)"}, "3": {"exist": false, "char_ranges": [], "word_ranges": [], "text": ""}, "4": {"exist": true, "char_ranges": [[464, 596]], "word_ranges": [[71, 89]], "text": "conservative treatment is only considered in patients with very low functional demand or surgical contraindication (Wrong answer 4)."}, "5": {"exist": false, "char_ranges": [], "word_ranges": [], "text": ""}}} {"id": 542, "year": 2022, "question_id_specific": 38, "full_question": "A 25-year-old woman with a history of allergic bronchial asthma due to sensitization to grass pollens and atopic dermatitis. She suddenly presents hypotension, tachycardia, urticaria and bronchospasm after ingesting peanuts and is transferred to the emergency department. What would be the order of drugs to be administered and what laboratory test could be requested for further evaluation?", "full_answer": "The patient is suffering from anaphylactic shock secondary to peanut ingestion. The treatment of anaphylaxis and anaphylactic shock of first choice is intramuscular adrenaline 1mg/ml (dose 0.01 mg/kg, in adults 0.5 mg maximum). Followed by symptomatic treatment to stabilize the airway with 100% high flow O2, ensure large caliber venous access, fluid replacement and continuous monitoring (HR, BP, SatO2 , diuresis). As adjuvant treatment, an inhaled bronchodilator such as SABA, intramuscular dexchlorpheniramine or corticosteroids such as hydrocortisone IV or methylprednisolone IV can be administered. Adrenaline can be repeated at the same doses every 5 minutes a maximum of 3 times and if after 15 minutes there is no response, IV adrenaline infusion should be started, Glucagon (if treatment with beta-blockers), Atropine (if prolonged bradycardia) or Vasopressors (if refractory hypotension). The laboratory marker that should be requested if anaphylaxis is suspected is serum tryptase in the first hour for a basal determination and another at 4-6 hours to observe its elevation curve. It is not an emergency marker but it is very useful for future allergological study.", "type": "ALLERGOLOGY", "options": {"1": "Intramuscular adrenaline, followed by an inhaled bronchodilator of the SABA type and intramuscular dexchlorpheniramine. Then request serum tryptase.", "2": "An inhaled SABA-type bronchodilator, followed by intramuscular dexchlorpheniramine and intramuscular adrenaline. Then order serum tryptase.", "3": "Intramuscular dexchlorpheniramine, followed by an inhaled bronchodilator of the SABA type and intramuscular adrenaline. Then request eosinophil cationic protein.", "4": "Intramuscular adrenaline, followed by an inhaled bronchodilator of the SABA type and intramuscular dexchlorpheniramine. Then request eosinophil cationic protein.", "5": NaN}, "correct_option": 1, "explanations": {"1": {"exist": true, "char_ranges": [[901, 1094]], "word_ranges": [[129, 161]], "text": "The laboratory marker that should be requested if anaphylaxis is suspected is serum tryptase in the first hour for a basal determination and another at 4-6 hours to observe its elevation curve."}, "2": {"exist": false, "char_ranges": [], "word_ranges": [], "text": ""}, "3": {"exist": false, "char_ranges": [], "word_ranges": [], "text": ""}, "4": {"exist": true, "char_ranges": [[901, 1094]], "word_ranges": [[129, 161]], "text": "The laboratory marker that should be requested if anaphylaxis is suspected is serum tryptase in the first hour for a basal determination and another at 4-6 hours to observe its elevation curve."}, "5": {"exist": false, "char_ranges": [], "word_ranges": [], "text": ""}}} {"id": 79, "year": 2012, "question_id_specific": 36, "full_question": "Which of the following clinical data, recorded during the clinical history taking of a patient with diarrhea of more than 4 weeks of evolution, can be found in people without organic pathology:", "full_answer": "Although mucus is usually more frightening in the consultation, it falls within the Rome II criteria for the diagnosis of Irritable Bowel Syndrome. All the rest are typical signs of organicity, although I think the question is poorly phrased, since any of these data (especially age) can be found in patients who in the end have nothing, what happens is that in these cases we must always look for organicity.", "type": "DIGESTIVE SYSTEM", "options": {"1": "Weight loss with or without rectorrhagia.", "2": "Persistence of diarrhea during the night.", "3": "Onset of symptoms after age 50.", "4": "Presence of dermatitis and/or arthritis during the examination.", "5": "Expulsion of mucus in more than 25% of defecations."}, "correct_option": 5, "explanations": {"1": {"exist": true, "char_ranges": [[148, 193]], "word_ranges": [[23, 31]], "text": "All the rest are typical signs of organicity,"}, "2": {"exist": true, "char_ranges": [[148, 193]], "word_ranges": [[23, 31]], "text": "All the rest are typical signs of organicity,"}, "3": {"exist": true, "char_ranges": [[148, 193]], "word_ranges": [[23, 31]], "text": "All the rest are typical signs of organicity,"}, "4": {"exist": true, "char_ranges": [[148, 193]], "word_ranges": [[23, 31]], "text": "All the rest are typical signs of organicity,"}, "5": {"exist": true, "char_ranges": [[0, 147]], "word_ranges": [[0, 23]], "text": "Although mucus is usually more frightening in the consultation, it falls within the Rome II criteria for the diagnosis of Irritable Bowel Syndrome."}}} {"id": 524, "year": 2021, "question_id_specific": 54, "full_question": "An 86-year-old woman in whom nonvalvular atrial fibrillation has been detected. She has a CHADS2 score of 3 points. In the literature, similar patients on warfarin therapy have a stroke risk of 2.2% vs. 5.2% in patients without warfarin. What would be the number needed to treat (NNT) to prevent an emboligenic stroke with anticoagulation therapy:", "full_answer": "On average, if 100 patients like this woman are treated with warfarin, the number of strokes is reduced by 3 (from 5.2 untreated to 3.2 treated). Therefore, if you treat 33.3 patients like this woman with warfarin, you reduce the number of strokes by 1: the NNT is 33.3.", "type": "STATISTICS", "options": {"1": "3", "2": "19,2.", "3": "33,3.", "4": "49,5.", "5": NaN}, "correct_option": 3, "explanations": {"1": {"exist": false, "char_ranges": [], "word_ranges": [], "text": ""}, "2": {"exist": false, "char_ranges": [], "word_ranges": [], "text": ""}, "3": {"exist": true, "char_ranges": [[0, 270]], "word_ranges": [[0, 49]], "text": "On average, if 100 patients like this woman are treated with warfarin, the number of strokes is reduced by 3 (from 5.2 untreated to 3.2 treated). Therefore, if you treat 33.3 patients like this woman with warfarin, you reduce the number of strokes by 1: the NNT is 33.3."}, "4": {"exist": false, "char_ranges": [], "word_ranges": [], "text": ""}, "5": {"exist": false, "char_ranges": [], "word_ranges": [], "text": ""}}} {"id": 319, "year": 2016, "question_id_specific": 141, "full_question": "A 20-year-old boy consults for lumbosacral pain of inflammatory rhythm of 4 months of evolution. Also bilateral thalalgia and morning stiffness for 1 hour. In the last 2 months, onset of diarrhea with loss of 4 kg of weight. What is the most correct diagnostic approach?", "full_answer": "The clinical picture is typical of spondyloarthritis, and together with the digestive symptoms it is probably associated with inflammatory bowel disease.", "type": "RHEUMATOLOGY", "options": {"1": "Given the patient's age, he most likely suffers from non-specific low back pain and tendonitis in the feet. If the diarrhea persists, a digestive study would be performed.", "2": "I would perform a digestive study to rule out tumor pathology. The lumbar pain may be due to visceral pathology.", "3": "The clinical picture is very suggestive of spondyloarthritis. Inflammatory bowel disease should be ruled out.", "4": "I would request a lumbar MRI to rule out disc herniation and if diarrhea persists, a digestive study.", "5": NaN}, "correct_option": 3, "explanations": {"1": {"exist": false, "char_ranges": [], "word_ranges": [], "text": ""}, "2": {"exist": false, "char_ranges": [], "word_ranges": [], "text": ""}, "3": {"exist": true, "char_ranges": [[0, 153]], "word_ranges": [[0, 21]], "text": "The clinical picture is typical of spondyloarthritis, and together with the digestive symptoms it is probably associated with inflammatory bowel disease."}, "4": {"exist": false, "char_ranges": [], "word_ranges": [], "text": ""}, "5": {"exist": false, "char_ranges": [], "word_ranges": [], "text": ""}}} {"id": 400, "year": 2016, "question_id_specific": 147, "full_question": "A 13-year-old boy presents dorsal pain of several months of evolution, only on standing and walking, what diagnosis should we consider?", "full_answer": "The pain is of mechanical characteristics, and the most frequent cause of mechanical back pain in adolescents is mechanical overload. There is no data in the statement to suggest any of the other etiologies.", "type": "TRAUMATOLOGY AND ORTHOPEDICS", "options": {"1": "Scheuermann's disease.", "2": "Thoracic scoliosis.", "3": "Malignant tumor of the vertebral body.", "4": "It is a mechanical overload.", "5": NaN}, "correct_option": 4, "explanations": {"1": {"exist": true, "char_ranges": [[134, 207]], "word_ranges": [[20, 34]], "text": "There is no data in the statement to suggest any of the other etiologies."}, "2": {"exist": true, "char_ranges": [[134, 207]], "word_ranges": [[20, 34]], "text": "There is no data in the statement to suggest any of the other etiologies."}, "3": {"exist": true, "char_ranges": [[134, 207]], "word_ranges": [[20, 34]], "text": "There is no data in the statement to suggest any of the other etiologies."}, "4": {"exist": true, "char_ranges": [[0, 133]], "word_ranges": [[0, 20]], "text": "The pain is of mechanical characteristics, and the most frequent cause of mechanical back pain in adolescents is mechanical overload."}, "5": {"exist": false, "char_ranges": [], "word_ranges": [], "text": ""}}} {"id": 196, "year": 2013, "question_id_specific": 164, "full_question": "A 4-year-old boy consulted for the appearance over the last three days of skin lesions on the legs and buttocks. His parents reported that his pediatrician had diagnosed an upper respiratory tract infection 10 days earlier. In the last 12 hours she presented intense colicky abdominal pain and had two diarrheal stools. Afebrile. No weight loss. On examination she has numerous petechiae and palpable purpuric lesions predominantly on the buttocks and lower extremities. Good general condition although she has intense abdominal pain. Abdominal palpation is difficult to assess due to diffuse pain. No visceromegaly. No other findings of interest on examination. Which of the following data does NOT support your diagnostic suspicion?", "full_answer": "You are describing a Schönlein-Henoch purpura which is a vasculitic phenomenon, so the lesions are not due to plateletopenia. Scrotal edema (sometimes even torsion of Morgani's capsid of the testis) is not exceptional in males with S. Schönlein-Henoch.", "type": "PEDIATRICS", "options": {"1": "Knee and ankle arthritis.", "2": "Hematuria.", "3": "Plateletopenia.", "4": "Fecal occult blood.", "5": "Scrotal edema."}, "correct_option": 3, "explanations": {"1": {"exist": false, "char_ranges": [], "word_ranges": [], "text": ""}, "2": {"exist": false, "char_ranges": [], "word_ranges": [], "text": ""}, "3": {"exist": false, "char_ranges": [], "word_ranges": [], "text": ""}, "4": {"exist": false, "char_ranges": [], "word_ranges": [], "text": ""}, "5": {"exist": true, "char_ranges": [[0, 252]], "word_ranges": [[0, 38]], "text": "You are describing a Schönlein-Henoch purpura which is a vasculitic phenomenon, so the lesions are not due to plateletopenia. Scrotal edema (sometimes even torsion of Morgani's capsid of the testis) is not exceptional in males with S. Schönlein-Henoch."}}} {"id": 509, "year": 2021, "question_id_specific": 150, "full_question": "83-year-old male who consults for a recurrent picture of abdominal pain in the left iliac fossa, accompanied by diarrhea of up to 6 stools per day without pathological products, of 10-15 days of evolution. No fever or general condition was reported. A recent blood test showed no leukocytosis and a negative fecal occult blood test. On examination there is mild pain on palpation of the left iliac fossa without peritoneal irritation. He had a colonoscopy three years ago in which diverticula were reported along the entire colon, more numerous in the left colon, with no other associated lesions. Which of the following would be the best approach to follow?", "full_answer": "They refer to a concept that has emerged relatively recently and is not present in all clinical guidelines as uncomplicated symptomatic diverticular disease. In these cases, the therapeutic trial with rifaximin is recommended, being the true number 1. The absence of acute phase reactants in the CBC suggests that she does not have acute diverticulitis at the time of consultation that would lead us to request an imaging test to rule it out.", "type": "DIGESTIVE", "options": {"1": "Empirical treatment with antibiotics.", "2": "Request a preferential colonoscopy.", "3": "Request urgent abdominal ultrasound.", "4": "Refer to surgery to evaluate sigmoidectomy.", "5": NaN}, "correct_option": 1, "explanations": {"1": {"exist": true, "char_ranges": [[0, 251]], "word_ranges": [[0, 38]], "text": "They refer to a concept that has emerged relatively recently and is not present in all clinical guidelines as uncomplicated symptomatic diverticular disease. In these cases, the therapeutic trial with rifaximin is recommended, being the true number 1."}, "2": {"exist": true, "char_ranges": [[252, 442]], "word_ranges": [[38, 73]], "text": "The absence of acute phase reactants in the CBC suggests that she does not have acute diverticulitis at the time of consultation that would lead us to request an imaging test to rule it out."}, "3": {"exist": true, "char_ranges": [[252, 442]], "word_ranges": [[38, 73]], "text": "The absence of acute phase reactants in the CBC suggests that she does not have acute diverticulitis at the time of consultation that would lead us to request an imaging test to rule it out."}, "4": {"exist": false, "char_ranges": [], "word_ranges": [], "text": ""}, "5": {"exist": false, "char_ranges": [], "word_ranges": [], "text": ""}}} {"id": 549, "year": 2022, "question_id_specific": 128, "full_question": "75-year-old woman with a history of heart failure with an ejection fraction of 25% who comes for a check-up, being stable in NYHA functional class III. She has an implanted cardioverter defibrillator. Current medication consists of lisinopril, carvedilol and spironolactone at maximum tolerated doses. On physical examination he has BP 118/74 mmHg, HR 78 bpm. On cardiac auscultation a third tone is detected, pulmonary auscultation is normal and there is no edema. Which of the following is the most appropriate step in his management?", "full_answer": "Discontinue lisinopril and initiate sacubitril/valsartan.", "type": "CARDIOLOGY", "options": {"1": "Add sacubitril/valsartan.", "2": "Add ivabradine.", "3": "Discontinue lisinopril and start sacubitril/valsartan.", "4": "Discontinue carvedilol and start ivabradine.", "5": NaN}, "correct_option": 3, "explanations": {"1": {"exist": false, "char_ranges": [], "word_ranges": [], "text": ""}, "2": {"exist": false, "char_ranges": [], "word_ranges": [], "text": ""}, "3": {"exist": true, "char_ranges": [[0, 57]], "word_ranges": [[0, 5]], "text": "Discontinue lisinopril and initiate sacubitril/valsartan."}, "4": {"exist": false, "char_ranges": [], "word_ranges": [], "text": ""}, "5": {"exist": false, "char_ranges": [], "word_ranges": [], "text": ""}}} {"id": 221, "year": 2014, "question_id_specific": 48, "full_question": "A 20-year-old man with ataxia, headaches and solid-cystic mass in the right cerebellar hemisphere underwent surgery and a lesion was resected, histologically showing cells with long thin cytoplasmic processes, fascicular and microcystic pattern, numerous vessels and Rosenthal's fibers. The most probable anatomopathologic diagnosis is:", "full_answer": "A question that does not seem to have a second intention, as there are no confusing elements. The statement concisely summarizes the description of pilocytic astrocytoma. The Rosenthal fibers may confuse us as they are not very specific, and make us think, for example, of pleomorphic xanthoastrocytoma in which they can also be found, but the cell morphology described (long and thin cytoplasmic processes, which are the ones that give the name \"pilocytic\") keeps us the first option as the correct one. We can find a good description of the entity, explaining what distinguishes it from other glial tumors in Practical Surgical Neuropathology. A Diagnostic Approach. A. Perry, D.J. Brat. p82-88. Churchill Livingstone Elsevier. 2010. (Philadephia).", "type": "ANATOMIC PATHOLOGY", "options": {"1": "Pilocytic astrocytoma.", "2": "Pleomorphic xanthoastrocytoma.", "3": "Central neurocytoma.", "4": "Liponeurocytoma.", "5": "Prion disease."}, "correct_option": 1, "explanations": {"1": {"exist": true, "char_ranges": [[171, 504]], "word_ranges": [[26, 82]], "text": "The Rosenthal fibers may confuse us as they are not very specific, and make us think, for example, of pleomorphic xanthoastrocytoma in which they can also be found, but the cell morphology described (long and thin cytoplasmic processes, which are the ones that give the name \"pilocytic\") keeps us the first option as the correct one."}, "2": {"exist": true, "char_ranges": [[171, 504]], "word_ranges": [[26, 82]], "text": "The Rosenthal fibers may confuse us as they are not very specific, and make us think, for example, of pleomorphic xanthoastrocytoma in which they can also be found, but the cell morphology described (long and thin cytoplasmic processes, which are the ones that give the name \"pilocytic\") keeps us the first option as the correct one."}, "3": {"exist": false, "char_ranges": [], "word_ranges": [], "text": ""}, "4": {"exist": false, "char_ranges": [], "word_ranges": [], "text": ""}, "5": {"exist": false, "char_ranges": [], "word_ranges": [], "text": ""}}} {"id": 606, "year": 2022, "question_id_specific": 113, "full_question": "35-year-old male, letter carrier, with no history of interest, who comes to the emergency department for acute cervical pain of 24 hours of evolution, without previous trauma, which radiates to the left arm up to the hand and is accompanied by paresthesia in the radial border of the forearm. She presents no objectifiable loss of strength, preserves neck mobility although it is painful and contracture of the paravertebral musculature is appreciated. The first attitude will be:", "full_answer": "Correct answer 1. 75% of patients with radiculopathy improve with non-surgical treatment, this being the treatment of choice in the initial stages. Magnetic resonance imaging is of choice in certain \"red flags\": fever, weight loss, nocturnal pain, persistence of symptoms despite conservative treatment and loss of strength (incorrect answer 3).", "type": "TRAUMATOLOGY", "options": {"1": "Conservative treatment with non-steroidal anti-inflammatory drugs, local heat and relative rest.", "2": "Urgent call to the neurosurgeon for surgical evaluation.", "3": "Preferential request for MRI and electromyogram.", "4": "Preferential referral to traumatology outpatients.", "5": NaN}, "correct_option": 1, "explanations": {"1": {"exist": true, "char_ranges": [[0, 147]], "word_ranges": [[0, 22]], "text": "Correct answer 1. 75% of patients with radiculopathy improve with non-surgical treatment, this being the treatment of choice in the initial stages."}, "2": {"exist": false, "char_ranges": [], "word_ranges": [], "text": ""}, "3": {"exist": true, "char_ranges": [[148, 345]], "word_ranges": [[22, 50]], "text": "Magnetic resonance imaging is of choice in certain \"red flags\": fever, weight loss, nocturnal pain, persistence of symptoms despite conservative treatment and loss of strength (incorrect answer 3)."}, "4": {"exist": false, "char_ranges": [], "word_ranges": [], "text": ""}, "5": {"exist": false, "char_ranges": [], "word_ranges": [], "text": ""}}} {"id": 436, "year": 2018, "question_id_specific": 223, "full_question": "A 35-year-old Bolivian woman, 3 months pregnant, who has been living in Spain for 10 years with her partner, who is Spanish and has never traveled to Latin America. The woman has two other children with her partner, born in Spain, and a brother who lives in Bolivia and another in Spain. During her pregnancy she has been diagnosed with Trypanosoma cruzi infection. Which of the following is WRONG?", "full_answer": "The sexual route is not a means of transmission of Trypanosoma cruzi. In our environment, since the transmitting vector is not found, the most important means of transmission is the vertical route from mother to child. The woman was already infected by the parasite when she arrived in Spain, it may be correct to suggest that her siblings have a T. cruzi serology test because of the risk that her mother may have transmitted the infection vertically to all her siblings. Likewise, having had two pregnancies in these years of stay in Spain, it should be suggested that her children also be tested for the probable vertical transmission.", "type": "INFECTIOUS DISEASES AND MICROBIOLOGY", "options": {"1": "Recommend that their children have a T. cruzi serology because of the risk of transplacental transmission.", "2": "Inform the patient of the risk of T. cruzi transmission and the attitude to be taken during pregnancy, delivery and postpartum.", "3": "Recommend that her husband have T. cruzi serology because of the risk of having sexually transmitted the disease to her.", "4": "Recommend that her siblings have T. cruzi serology because of the risk of vectorial transmission.", "5": NaN}, "correct_option": 3, "explanations": {"1": {"exist": true, "char_ranges": [[483, 638]], "word_ranges": [[82, 108]], "text": "having had two pregnancies in these years of stay in Spain, it should be suggested that her children also be tested for the probable vertical transmission."}, "2": {"exist": false, "char_ranges": [], "word_ranges": [], "text": ""}, "3": {"exist": true, "char_ranges": [[0, 69]], "word_ranges": [[0, 12]], "text": "The sexual route is not a means of transmission of Trypanosoma cruzi."}, "4": {"exist": true, "char_ranges": [[219, 472]], "word_ranges": [[36, 81]], "text": "The woman was already infected by the parasite when she arrived in Spain, it may be correct to suggest that her siblings have a T. cruzi serology test because of the risk that her mother may have transmitted the infection vertically to all her siblings."}, "5": {"exist": false, "char_ranges": [], "word_ranges": [], "text": ""}}} {"id": 220, "year": 2014, "question_id_specific": 126, "full_question": "A 42-year-old man came to consultation for edema. Initial tests showed the presence of proteinuria of more than 10 grams/24 hours. Which of the following measures is NOT indicated in the treatment of this patient?", "full_answer": "The patient in the case presents a nephrotic syndrome, so far so good. As for the diet, to reduce edema in these patients, it is recommended that it be low-protein and low-salt, so options 1 and 3 would be true. ACE inhibitors are also administered as primary treatment for nephrotic syndrome, since they decrease urinary protein excretion, as do ARA II and loop diuretics, so options 2 and 5 would also be correct. Therefore, simply by discarding, we are left with option 4 as false, and therefore, the correct answer. Moreover, it should stand out from the beginning as the option to be marked, since precisely one of the causes of nephrotic syndrome is the intake of NSAIDs, so how could they be indicated in the treatment?", "type": "NEPHROLOGY", "options": {"1": "Low-protein diet.", "2": "Administration of loop diuretics.", "3": "Restriction of salt in the diet.", "4": "Administration of NSAIDs.", "5": "Administration of angiotensin-converting enzyme inhibitors."}, "correct_option": 4, "explanations": {"1": {"exist": true, "char_ranges": [[88, 211]], "word_ranges": [[17, 40]], "text": "to reduce edema in these patients, it is recommended that it be low-protein and low-salt, so options 1 and 3 would be true."}, "2": {"exist": true, "char_ranges": [[212, 415]], "word_ranges": [[40, 73]], "text": "ACE inhibitors are also administered as primary treatment for nephrotic syndrome, since they decrease urinary protein excretion, as do ARA II and loop diuretics, so options 2 and 5 would also be correct."}, "3": {"exist": true, "char_ranges": [[88, 211]], "word_ranges": [[17, 40]], "text": "to reduce edema in these patients, it is recommended that it be low-protein and low-salt, so options 1 and 3 would be true."}, "4": {"exist": true, "char_ranges": [[597, 677]], "word_ranges": [[104, 118]], "text": "since precisely one of the causes of nephrotic syndrome is the intake of NSAIDs,"}, "5": {"exist": true, "char_ranges": [[212, 415]], "word_ranges": [[40, 73]], "text": "ACE inhibitors are also administered as primary treatment for nephrotic syndrome, since they decrease urinary protein excretion, as do ARA II and loop diuretics, so options 2 and 5 would also be correct."}}} {"id": 483, "year": 2020, "question_id_specific": 174, "full_question": "A 22-year-old woman presents with sudden cardiac arrest with a rhythm that is considered amenable to defibrillation. Chest compressions and ventilations are being performed, a defibrillator shock has been delivered, and a peripheral venous line has been cannulated. Which of the following would be correct next:", "full_answer": "Controversial question, since amiodarone (the dose is correct, 300mg), is administered after the third shock, and not after the first defibrillation, which is what we are told in the statement. Although, be careful, that is what the guidelines say. I am sure that more than one of us has administered amiodarone as soon as we have had venous access available in the event of a shockable rhythm. The other options are incorrect.", "type": "CRITICAL CARE", "options": {"1": "Administration of 150 mg of intravenous amiodarone.", "2": "Administration of 300 mg of intravenous amiodarone.", "3": "Administration of 1 mg of intravenous atropine.", "4": "Administer 2 mg of intravenous atropine.", "5": NaN}, "correct_option": 2, "explanations": {"1": {"exist": false, "char_ranges": [], "word_ranges": [], "text": ""}, "2": {"exist": true, "char_ranges": [[30, 70]], "word_ranges": [[3, 9]], "text": "amiodarone (the dose is correct, 300mg),"}, "3": {"exist": false, "char_ranges": [], "word_ranges": [], "text": ""}, "4": {"exist": false, "char_ranges": [], "word_ranges": [], "text": ""}, "5": {"exist": false, "char_ranges": [], "word_ranges": [], "text": ""}}} {"id": 113, "year": 2012, "question_id_specific": 93, "full_question": "A 35-year-old healthy woman consults for asthenia. The CBC shows microcytic and hypochromic anemia (hemoglobin 7 g/dL, mean corpuscular volume 68 fl, mean corpuscular hemoglobin 24 pg) with an elevated red cell distribution amplitude (ADE 20%), decreased reticulocytes (0.3%, 30000/l absolute), decreased hemoglobin content in reticulocytes (17 pg) and discrete thrombocytosis (500000 platelets). With the CBC data, what would be your diagnostic suspicion even without having the patient's biochemistry?", "full_answer": "With the data we are given, the first thing that comes to mind is that it is an arregenerative anemia, since the bone marrow has not responded with an exaltation of erythropoiesis; in this way we have already eliminated the options that speak of regenerative anemia. The next thing that orients us towards the cause is that it is microcytic and hypochromic, which is characteristic of iron deficiency anemia, since folic deficiency gives rise to megaloblastic anemia and active hemorrhage does not give microcytosis or hypochromia, but normocytosis and normochromia. We are almost inclined to answer 5, central arregenerative anemia due to iron deficiency...but there remains answer 1, \"thalassemic trait\", which makes us uncomfortable and makes us doubt if we are not going for the apparently easy thing...until we remember that in thalassemia the ADE is normal. Definitely, this girl of childbearing age has iron deficiency anemia, something quite common. Those who have doubts can consult \"Hematology. Manual básico razonado\" by Jesús San Miguel.", "type": "HEMATOLOGY", "options": {"1": "Thalassemic trait.", "2": "Central anemia, arregenerative, due to folic acid deficiency.", "3": "Peripheral, regenerative, hemolytic anemia.", "4": "Peripheral anemia, regenerative, due to acute active hemorrhage.", "5": "Central anemia, arregenerative, due to iron deficiency."}, "correct_option": 5, "explanations": {"1": {"exist": true, "char_ranges": [[830, 863]], "word_ranges": [[131, 137]], "text": "in thalassemia the ADE is normal."}, "2": {"exist": true, "char_ranges": [[109, 266]], "word_ranges": [[21, 46]], "text": "the bone marrow has not responded with an exaltation of erythropoiesis; in this way we have already eliminated the options that speak of regenerative anemia."}, "3": {"exist": true, "char_ranges": [[109, 266]], "word_ranges": [[21, 46]], "text": "the bone marrow has not responded with an exaltation of erythropoiesis; in this way we have already eliminated the options that speak of regenerative anemia."}, "4": {"exist": false, "char_ranges": [], "word_ranges": [], "text": ""}, "5": {"exist": true, "char_ranges": [[287, 566]], "word_ranges": [[50, 90]], "text": "orients us towards the cause is that it is microcytic and hypochromic, which is characteristic of iron deficiency anemia, since folic deficiency gives rise to megaloblastic anemia and active hemorrhage does not give microcytosis or hypochromia, but normocytosis and normochromia."}}} {"id": 59, "year": 2011, "question_id_specific": 51, "full_question": "A 47-year-old male patient who consults the emergency department for sudden loss of consciousness Which of the following has a higher diagnostic yield?", "full_answer": "Obviously, it is the only answer that encompasses all the causes of sudden loss of consciousness and that allows us to somehow filter the diagnosis.", "type": "ANESTHESIOLOGY AND CRITICAL CARE", "options": {"1": "Clinical history.", "2": "Electrocardiogram.", "3": "Holter recording.", "4": "EEG.", "5": "Cranial CT scan."}, "correct_option": 1, "explanations": {"1": {"exist": true, "char_ranges": [[0, 148]], "word_ranges": [[0, 25]], "text": "Obviously, it is the only answer that encompasses all the causes of sudden loss of consciousness and that allows us to somehow filter the diagnosis."}, "2": {"exist": false, "char_ranges": [], "word_ranges": [], "text": ""}, "3": {"exist": false, "char_ranges": [], "word_ranges": [], "text": ""}, "4": {"exist": false, "char_ranges": [], "word_ranges": [], "text": ""}, "5": {"exist": false, "char_ranges": [], "word_ranges": [], "text": ""}}} {"id": 412, "year": 2018, "question_id_specific": 54, "full_question": "Elizabeth has three sons and one daughter (Mary), all of whom are healthy. A brother and a maternal uncle of Isabel died of Duchenne disease: an X-linked recessive disease. At present, Maria wants to become pregnant and wants to know the risk of transmitting the disease to her offspring. With the data collected, what is the probability that Maria is a carrier of Duchenne disease?", "full_answer": "The maternal grandmother carried the mutation in one copy of the gene on an X chromosome (we are not told that she had the disease, but a son of hers, Isabel's maternal uncle, did have the disease). Isabel's mother continues to carry it, since one of Isabel's brothers has suffered from the disease but not her father. Thus, Isabel has a 50% chance of having inherited the mutated gene from her mother (as her children are all healthy, we are not sure if she is a carrier), and therefore her daughter has a 25% chance of having inherited the mutated gene from her grandmother.", "type": "BIOSTATISTICS", "options": {"1": "2/3.", "2": "1/2.", "3": "1/4.", "4": "Less than 1/4.", "5": NaN}, "correct_option": 3, "explanations": {"1": {"exist": false, "char_ranges": [], "word_ranges": [], "text": ""}, "2": {"exist": false, "char_ranges": [], "word_ranges": [], "text": ""}, "3": {"exist": true, "char_ranges": [[0, 576]], "word_ranges": [[0, 104]], "text": "The maternal grandmother carried the mutation in one copy of the gene on an X chromosome (we are not told that she had the disease, but a son of hers, Isabel's maternal uncle, did have the disease). Isabel's mother continues to carry it, since one of Isabel's brothers has suffered from the disease but not her father. Thus, Isabel has a 50% chance of having inherited the mutated gene from her mother (as her children are all healthy, we are not sure if she is a carrier), and therefore her daughter has a 25% chance of having inherited the mutated gene from her grandmother."}, "4": {"exist": false, "char_ranges": [], "word_ranges": [], "text": ""}, "5": {"exist": false, "char_ranges": [], "word_ranges": [], "text": ""}}} {"id": 347, "year": 2016, "question_id_specific": 159, "full_question": "69-year-old woman who comes to your office referring genital bleeding of several months of evolution. She denies hormone replacement therapy and anticoagulation. She provides normal cervicovaginal cytology. General and genital physical examination without findings of interest. BMI of 38kg/m2. Indicate the most correct attitude:", "full_answer": "Endometrial biopsy. Peri- or menopause: - Due to the high prevalence of organic pathology during this period it is necessary to systematically perform endometrial biopsy (Cornier or Hysteroscopy).", "type": "GYNECOLOGY AND OBSTETRICS", "options": {"1": "Prescribe cyclic progesterone.", "2": "Endometrial biopsy.", "3": "Random biopsies of the cervix.", "4": "Hormonal assessment with FSH, LH and estradiol.", "5": NaN}, "correct_option": 2, "explanations": {"1": {"exist": false, "char_ranges": [], "word_ranges": [], "text": ""}, "2": {"exist": true, "char_ranges": [[20, 196]], "word_ranges": [[2, 28]], "text": "Peri- or menopause: - Due to the high prevalence of organic pathology during this period it is necessary to systematically perform endometrial biopsy (Cornier or Hysteroscopy)."}, "3": {"exist": false, "char_ranges": [], "word_ranges": [], "text": ""}, "4": {"exist": false, "char_ranges": [], "word_ranges": [], "text": ""}, "5": {"exist": false, "char_ranges": [], "word_ranges": [], "text": ""}}} {"id": 325, "year": 2016, "question_id_specific": 56, "full_question": "A 54-year-old patient is admitted for thermometric fever of 38°C in the previous five days and dyspnea at rest (NYHA lV) that appeared 6 hours before coming to the hospital. Examination in the emergency room was compatible with heart failure and the ECG showed complete atrioventricular block with an escape ventricular rate of 45 bpm. Signs of heart failure are refractory to medical treatment and transesophageal echocardiography shows an aortic valve with an effective regurgitant orifice of 0.5 cm2. Serial cultures are positive for Streptococcus gallolyticus. Indicate the best course of action:", "full_answer": "The correct answer is 1, since the indications for cardiac surgery include the appearance of heart failure or new conduction disorders, and in this case it presents both.", "type": "CRITICAL, PALLIATIVE AND EMERGENCY CARE", "options": {"1": "Cardiac surgery for aortic valve replacement by mechanical prosthesis with antibiotic therapy according to antibiogram.", "2": "Antibiotic therapy according to antibiogram and implantation of intra-aortic balloon counterpulsation and transient pacemaker for up to 3 weeks, after which a permanent pacemaker will be implanted.", "3": "Implantation of transient pacemaker, antibiotic therapy according to antibiogram and percutaneous implantation of aortic valve prosthesis.", "4": "Urgent implantation of definitive pacemaker with antibiotic therapy according to antibiogram for 6 weeks.", "5": NaN}, "correct_option": 1, "explanations": {"1": {"exist": true, "char_ranges": [[31, 170]], "word_ranges": [[6, 28]], "text": "the indications for cardiac surgery include the appearance of heart failure or new conduction disorders, and in this case it presents both."}, "2": {"exist": false, "char_ranges": [], "word_ranges": [], "text": ""}, "3": {"exist": false, "char_ranges": [], "word_ranges": [], "text": ""}, "4": {"exist": false, "char_ranges": [], "word_ranges": [], "text": ""}, "5": {"exist": false, "char_ranges": [], "word_ranges": [], "text": ""}}} {"id": 335, "year": 2016, "question_id_specific": 175, "full_question": "An 88-year-old man comes for consultation because he has fallen 3 times in the last 6 months. None of the falls was accompanied by dizziness or syncope. One fall occurred while he was walking in the garden. His medical history includes hypertension without postural changes in blood pressure, gout, osteoarthritis, and depression. She takes 5 medications regularly. Which of the following is most likely to contribute to falls in this patient?", "full_answer": "Paroxetine; complicated question since both hydrochlorothiazide and lisinopril can also produce falls in the elderly, but they would generally be accompanied by dizziness.", "type": "PHARMACOLOGY", "options": {"1": "Allopurinol.", "2": "Hydrochlorothiazide.", "3": "Lisinopril.", "4": "Paroxetine.", "5": NaN}, "correct_option": 4, "explanations": {"1": {"exist": false, "char_ranges": [], "word_ranges": [], "text": ""}, "2": {"exist": true, "char_ranges": [[39, 171]], "word_ranges": [[4, 23]], "text": "both hydrochlorothiazide and lisinopril can also produce falls in the elderly, but they would generally be accompanied by dizziness."}, "3": {"exist": true, "char_ranges": [[39, 171]], "word_ranges": [[4, 23]], "text": "both hydrochlorothiazide and lisinopril can also produce falls in the elderly, but they would generally be accompanied by dizziness."}, "4": {"exist": true, "char_ranges": [[39, 171]], "word_ranges": [[4, 23]], "text": "both hydrochlorothiazide and lisinopril can also produce falls in the elderly, but they would generally be accompanied by dizziness."}, "5": {"exist": false, "char_ranges": [], "word_ranges": [], "text": ""}}} {"id": 19, "year": 2011, "question_id_specific": 109, "full_question": "A 32-year-old woman has traveled to Cuba from where she arrived three days ago. The day after her return, she goes to her family doctor for high fever, intense arthromyalgia and headache that had started before the return trip and he prescribes her paracetamol. Three days later and with no improvement, he presented in the morning with a generalized pruritic maculo-papular rash, which was more intense on the lower limbs where it developed into petechiae, for which he went to the emergency department. There is no relevant data in the EC except platelets 75000/mm3 (htco 36%, leukocytes 4100 79% neutrophils). What is the most likely diagnosis?", "full_answer": "In this case I believe that the correct answer is 5. The 1 should not be because although the clinical picture is suggestive, it does not present lymphocytosis. It does not seem to be an allergic exanthema from the clinical picture. And certainly she does not have symptoms or clinical signs compatible with malaria or typhoid fever. What points to dengue, in addition to the clinical signs, is the plateletopenia.", "type": "INFECTIOUS DISEASES", "options": {"1": "Infectious mononucleosis.", "2": "Allergic exanthema.", "3": "P. falciparum malaria.", "4": "Typhoid fever.", "5": "Dengue fever."}, "correct_option": 5, "explanations": {"1": {"exist": false, "char_ranges": [], "word_ranges": [], "text": ""}, "2": {"exist": true, "char_ranges": [[233, 291]], "word_ranges": [[41, 51]], "text": "And certainly she does not have symptoms or clinical signs"}, "3": {"exist": true, "char_ranges": [[237, 332]], "word_ranges": [[42, 57]], "text": "certainly she does not have symptoms or clinical signs compatible with malaria or typhoid fever."}, "4": {"exist": true, "char_ranges": [[237, 332]], "word_ranges": [[42, 57]], "text": "certainly she does not have symptoms or clinical signs compatible with malaria or typhoid fever."}, "5": {"exist": true, "char_ranges": [[53, 159], [334, 414]], "word_ranges": [[11, 28], [57, 70]], "text": "The 1 should not be because although the clinical picture is suggestive, it does not present lymphocytosis. What points to dengue, in addition to the clinical signs, is the plateletopenia."}}} {"id": 401, "year": 2016, "question_id_specific": 147, "full_question": "A 13-year-old boy presents dorsal pain of several months of evolution, only when standing and walking, what diagnosis should we consider?", "full_answer": "A malignant tumor would also hurt at rest. As for Scheuermann and scoliosis, we should be given some physical data because the patient presents with a specific examination that would give us the diagnosis.", "type": "TRAUMATOLOGY AND ORTHOPEDICS", "options": {"1": "Scheuermann's disease.", "2": "Thoracic scoliosis.", "3": "Malignant tumor of the vertebral body.", "4": "It is a mechanical overload.", "5": NaN}, "correct_option": 4, "explanations": {"1": {"exist": true, "char_ranges": [[43, 205]], "word_ranges": [[8, 34]], "text": "As for Scheuermann and scoliosis, we should be given some physical data because the patient presents with a specific examination that would give us the diagnosis."}, "2": {"exist": true, "char_ranges": [[43, 205]], "word_ranges": [[8, 34]], "text": "As for Scheuermann and scoliosis, we should be given some physical data because the patient presents with a specific examination that would give us the diagnosis."}, "3": {"exist": true, "char_ranges": [[0, 42]], "word_ranges": [[0, 8]], "text": "A malignant tumor would also hurt at rest."}, "4": {"exist": false, "char_ranges": [], "word_ranges": [], "text": ""}, "5": {"exist": false, "char_ranges": [], "word_ranges": [], "text": ""}}} {"id": 607, "year": 2022, "question_id_specific": 115, "full_question": "A 27-year-old male, a regular athlete, complains of pain in the right leg after continuous running. He has visited a physiotherapist on several occasions and has been diagnosed with an overload in the calf muscles. Several months have passed, he has not improved and refers intense pain after physical activity that subsides with rest in the following hours of exercise. What test can help in the diagnosis?", "full_answer": "Clinical suspicion of chronic compartment syndrome (CCS). The diagnosis is made by obtaining compartment pressures at rest, during exercise and post-exercise.", "type": "TRAUMATOLOGY", "options": {"1": "Positron emission tomography with 18 FDG.", "2": "Determination of posterior compartment pressure immediately after activity.", "3": "Doppler ultrasound to rule out a circulatory disorder of the lower extremity.", "4": "Magnetic resonance spectroscopy.", "5": NaN}, "correct_option": 2, "explanations": {"1": {"exist": false, "char_ranges": [], "word_ranges": [], "text": ""}, "2": {"exist": true, "char_ranges": [[0, 158]], "word_ranges": [[0, 21]], "text": "Clinical suspicion of chronic compartment syndrome (CCS). The diagnosis is made by obtaining compartment pressures at rest, during exercise and post-exercise."}, "3": {"exist": false, "char_ranges": [], "word_ranges": [], "text": ""}, "4": {"exist": false, "char_ranges": [], "word_ranges": [], "text": ""}, "5": {"exist": false, "char_ranges": [], "word_ranges": [], "text": ""}}} {"id": 453, "year": 2018, "question_id_specific": 150, "full_question": "A 25-year-old man consults for tremor. Examination reveals dysarthria and dystonia. He has a family history of psychiatric illness and movement disorders. Which diagnostic test do you consider most accurate?", "full_answer": "If they are talking about a young man 20-30 years old, with tremor and dystonia, whether or not he has a family history, and especially if one of the options tells me about copper in urine, they are telling me to think of Wilson's disease. I do not think of juvenile Parkinson's, nor an epileptic clinic. So the correct answer is 4.", "type": "NEUROLOGY", "options": {"1": "DATS CAN.", "2": "Nerve conduction study.", "3": "Electroencephalogram.", "4": "Copper in urine of 24.", "5": NaN}, "correct_option": 4, "explanations": {"1": {"exist": true, "char_ranges": [[0, 332]], "word_ranges": [[0, 62]], "text": "If they are talking about a young man 20-30 years old, with tremor and dystonia, whether or not he has a family history, and especially if one of the options tells me about copper in urine, they are telling me to think of Wilson's disease. I do not think of juvenile Parkinson's, nor an epileptic clinic. So the correct answer is 4."}, "2": {"exist": false, "char_ranges": [], "word_ranges": [], "text": ""}, "3": {"exist": true, "char_ranges": [[0, 332]], "word_ranges": [[0, 62]], "text": "If they are talking about a young man 20-30 years old, with tremor and dystonia, whether or not he has a family history, and especially if one of the options tells me about copper in urine, they are telling me to think of Wilson's disease. I do not think of juvenile Parkinson's, nor an epileptic clinic. So the correct answer is 4."}, "4": {"exist": true, "char_ranges": [[0, 332]], "word_ranges": [[0, 62]], "text": "If they are talking about a young man 20-30 years old, with tremor and dystonia, whether or not he has a family history, and especially if one of the options tells me about copper in urine, they are telling me to think of Wilson's disease. I do not think of juvenile Parkinson's, nor an epileptic clinic. So the correct answer is 4."}, "5": {"exist": false, "char_ranges": [], "word_ranges": [], "text": ""}}} {"id": 529, "year": 2021, "question_id_specific": 166, "full_question": "An 18-year-old patient consulted for edema. A complete blood test showed proteinuria of 8 g/day without microhematuria, hypoalbuminemia and hypercholesterolemia with normal renal function. He was empirically administered corticosteroids. After one month, the clinical picture has completely disappeared. What is your diagnostic hypothesis?", "full_answer": "We describe a young patient with criteria of nephrotic syndrome with excellent response to corticosteroids. Without biopsy, the clinical presentation and evolution suggest a nephropathy with minimal changes, which usually presents with normal or slightly altered renal function together with nephrotic syndrome, and which in 85-90% of cases resolves with steroid treatment. The age is the only data that is a bit shaky, since although it is the most frequent cause of idiopathic nephrotic syndrome in children and adolescents, it is usually recommended to perform a biopsy prior to treatment in those older than 16 years, in any case it does not admit much discussion.", "type": "NEPHROLOGY", "options": {"1": "Amyloidosis.", "2": "IgA nephropathy or Berger's disease.", "3": "Alport syndrome.", "4": "Nephropathy with minimal changes.", "5": NaN}, "correct_option": 4, "explanations": {"1": {"exist": false, "char_ranges": [], "word_ranges": [], "text": ""}, "2": {"exist": false, "char_ranges": [], "word_ranges": [], "text": ""}, "3": {"exist": false, "char_ranges": [], "word_ranges": [], "text": ""}, "4": {"exist": true, "char_ranges": [[0, 311]], "word_ranges": [[0, 42]], "text": "We describe a young patient with criteria of nephrotic syndrome with excellent response to corticosteroids. Without biopsy, the clinical presentation and evolution suggest a nephropathy with minimal changes, which usually presents with normal or slightly altered renal function together with nephrotic syndrome,"}, "5": {"exist": false, "char_ranges": [], "word_ranges": [], "text": ""}}} {"id": 258, "year": 2014, "question_id_specific": 96, "full_question": "A 62-year-old man with type 2 diabetes mellitus of 10 years of evolution is treated with metformin and sitagliptin. He does little physical exercise and follows an adequate diet. In the last 6 months he has lost weight and has more asthenia. Her glycemic controls have deteriorated from baseline blood glucose levels of 110-140 mg/dL to 170-200 mg/dL, as well as her glycosylated hemoglobin which has gone from 7.1 to 8.5%. The most appropriate therapeutic measure to be taken is:", "full_answer": "To associate a basal insulin dose to the treatment. Expected question on a very important subject: diabetes and its treatment. Patient poorly controlled with metformin and sitagliptin with discrete cardinal symptoms (weight loss, asthenia). The best option is insulin to control cardinal symptoms and decrease HbA1c to < 7%.", "type": "ENDOCRINOLOGY", "options": {"1": "Increase dietary protein and long-chain carbohydrate intake to improve asthenia and weight loss.", "2": "To associate a basal insulin dose to the treatment.", "3": "Associate with ascarbose treatment.", "4": "Substitute sitagliptin for pioglitazone.", "5": "Substitute metformin for glimepiride."}, "correct_option": 2, "explanations": {"1": {"exist": false, "char_ranges": [], "word_ranges": [], "text": ""}, "2": {"exist": true, "char_ranges": [[127, 324]], "word_ranges": [[20, 49]], "text": "Patient poorly controlled with metformin and sitagliptin with discrete cardinal symptoms (weight loss, asthenia). The best option is insulin to control cardinal symptoms and decrease HbA1c to < 7%."}, "3": {"exist": false, "char_ranges": [], "word_ranges": [], "text": ""}, "4": {"exist": false, "char_ranges": [], "word_ranges": [], "text": ""}, "5": {"exist": false, "char_ranges": [], "word_ranges": [], "text": ""}}} {"id": 375, "year": 2016, "question_id_specific": 128, "full_question": "A 45-year-old patient under study for possible meningitis, with fever, headache and vomiting of 2 days of evolution. A brain MRI and lumbar puncture were performed. Twenty hours later, on getting up to go to the bathroom, he complains of severe headache, very marked when he sits up but disappears when he lies down. She no longer has fever or vomiting. What is most likely the origin of this headache?", "full_answer": "The picture they describe of intense headache with orthostatism that disappears when lying down, although it is true that the most typical presentation is between the first 24-48 hours and in this case it is somewhat earlier. Even so, the rest of the answers are improbable: the first cannot be because we have not been told what the initial headache was like; the third, although it could be due to the time of onset (as I said a little earlier than expected), the reality is that the headache with orthostasis is so typical that it would be the most probable cause; the last one could not be either because SAH does not follow this pattern in relation to orthostasis.", "type": "NEUROLOGY", "options": {"1": "Meningitis remains the fundamental cause of her headache, as it is the same type of headache as at the onset of symptoms.", "2": "It is a post lumbar puncture headache.", "3": "It is necessary to look for a different cause to this headache, because it is not typical of post-lumbar puncture syndrome or meningitis.", "4": "Most likely it was not a viral meningitis, but a subarachnoid hemorrhage. For this reason, the initial headache disappears almost completely when lying down.", "5": NaN}, "correct_option": 2, "explanations": {"1": {"exist": true, "char_ranges": [[275, 360]], "word_ranges": [[46, 62]], "text": "the first cannot be because we have not been told what the initial headache was like;"}, "2": {"exist": true, "char_ranges": [[0, 96]], "word_ranges": [[0, 14]], "text": "The picture they describe of intense headache with orthostatism that disappears when lying down,"}, "3": {"exist": true, "char_ranges": [[361, 567]], "word_ranges": [[62, 101]], "text": "the third, although it could be due to the time of onset (as I said a little earlier than expected), the reality is that the headache with orthostasis is so typical that it would be the most probable cause;"}, "4": {"exist": true, "char_ranges": [[568, 669]], "word_ranges": [[101, 119]], "text": "the last one could not be either because SAH does not follow this pattern in relation to orthostasis."}, "5": {"exist": false, "char_ranges": [], "word_ranges": [], "text": ""}}} {"id": 352, "year": 2016, "question_id_specific": 161, "full_question": "A 27-year-old woman referred to the gynecology office for evaluation referring dyspareunia for about 8 months, along with dyschezia and occasional rectorrhagia coinciding with menstruation for 3-4 months. She also reports dysmenorrhea for years, which she controls well with ibuprofen. She has been trying to get pregnant for 16 months without success. In the gynecological examination she only shows pain when pressing on the posterior vaginal fornix. Which test do you consider would allow you to reach a diagnosis of certainty of her pathology?", "full_answer": "The correct answer is 2. We are facing a suspicion of endometriosis, due to the symptoms and signs given in the statement. We are asked for a diagnosis of certainty, and the Gold Standard for the diagnosis of this disease is diagnostic laparoscopy.", "type": "GYNECOLOGY AND OBSTETRICS", "options": {"1": "Transvaginal ultrasound.", "2": "Diagnostic laparoscopy.", "3": "Magnetic resonance imaging.", "4": "Colonoscopy.", "5": NaN}, "correct_option": 2, "explanations": {"1": {"exist": false, "char_ranges": [], "word_ranges": [], "text": ""}, "2": {"exist": true, "char_ranges": [[25, 248]], "word_ranges": [[5, 43]], "text": "We are facing a suspicion of endometriosis, due to the symptoms and signs given in the statement. We are asked for a diagnosis of certainty, and the Gold Standard for the diagnosis of this disease is diagnostic laparoscopy."}, "3": {"exist": false, "char_ranges": [], "word_ranges": [], "text": ""}, "4": {"exist": false, "char_ranges": [], "word_ranges": [], "text": ""}, "5": {"exist": false, "char_ranges": [], "word_ranges": [], "text": ""}}} {"id": 575, "year": 2022, "question_id_specific": 102, "full_question": "A 29-year-old woman with a history of febrile seizures and a heavy smoker. She consults for frequent, stereotyped episodes that begin with an ascending epigastric sensation and an unpleasant odor, followed by disconnection from the environment. Family members who are with her observe that she makes sucking movements and repeated opening and closing movements of the left hand. After two minutes the movements subside, but she has difficulty responding appropriately and only partially remembers what happened. Which of the following is the most likely diagnosis?", "full_answer": "The clinical case makes a textbook description of a medial temporal seizure. The aura of ascending epigastric sensation and olfactory alteration and the subsequent description of the seizure with alteration of the level of consciousness (disconnection from the environment) with oral and manual automatisms. The current classification is the 2017 ILAE classification that would describe this seizure as focal seizure with altered level of consciousness, but in the MIR they still use the old terminology (complex is what altered level of consciousness was called).", "type": "NEUROLOGY", "options": {"1": "Complex focal temporal lobe seizures.", "2": "Transient ischemic attacks in right carotid territory.", "3": "Simple focal motor seizures.", "4": "Episodes of transient global amnesia.", "5": NaN}, "correct_option": 1, "explanations": {"1": {"exist": true, "char_ranges": [[308, 564]], "word_ranges": [[44, 84]], "text": "The current classification is the 2017 ILAE classification that would describe this seizure as focal seizure with altered level of consciousness, but in the MIR they still use the old terminology (complex is what altered level of consciousness was called)."}, "2": {"exist": false, "char_ranges": [], "word_ranges": [], "text": ""}, "3": {"exist": false, "char_ranges": [], "word_ranges": [], "text": ""}, "4": {"exist": false, "char_ranges": [], "word_ranges": [], "text": ""}, "5": {"exist": false, "char_ranges": [], "word_ranges": [], "text": ""}}} {"id": 278, "year": 2016, "question_id_specific": 31, "full_question": "A 20-year-old woman with a 15 cm solid-cystic ovarian tumor detected by ultrasound after presenting with non-specific abdominal symptoms. In the anatomopathological study of the corresponding specimen, teeth, hairs, areas of intestinal epithelium, areas of squamous epithelium (15%) and bronchial epithelium, as well as neuroectodermal and embryonic elements are found in several of the histological preparations. In reference to this case, point out the correct diagnosis:", "full_answer": "After reading the question and the possible answers, we have two tumor and two non-tumor options. At no time are atypical cellular features mentioned, so we can safely rule out tumor lesions, leaving doubts between answers 2 and 3. In this case, we would have to opt for IMMATURE TERATOMA, mentioning the presence of embryonic remains.", "type": "PATHOLOGICAL ANATOMY", "options": {"1": "Teratocarcinoma.", "2": "Immature teratoma.", "3": "Mature cystic teratoma.", "4": "Dysgerminoma.", "5": NaN}, "correct_option": 2, "explanations": {"1": {"exist": true, "char_ranges": [[98, 191]], "word_ranges": [[16, 32]], "text": "At no time are atypical cellular features mentioned, so we can safely rule out tumor lesions,"}, "2": {"exist": true, "char_ranges": [[246, 335]], "word_ranges": [[42, 56]], "text": "we would have to opt for IMMATURE TERATOMA, mentioning the presence of embryonic remains."}, "3": {"exist": true, "char_ranges": [[246, 335]], "word_ranges": [[42, 56]], "text": "we would have to opt for IMMATURE TERATOMA, mentioning the presence of embryonic remains."}, "4": {"exist": true, "char_ranges": [[98, 191]], "word_ranges": [[16, 32]], "text": "At no time are atypical cellular features mentioned, so we can safely rule out tumor lesions,"}, "5": {"exist": false, "char_ranges": [], "word_ranges": [], "text": ""}}} {"id": 225, "year": 2014, "question_id_specific": 64, "full_question": "An 82-year-old patient presents with sudden and frequent dizziness, without prodromes, of short duration and which subsides spontaneously, without ever presenting syncope. Physical examination and baseline electrocardiogram are normal. In an ambulatory electrocardiogram recording during 24 hours, short phases of absence of P waves prior to QRS with an escape rhythm of the atrioventricular junction with narrow QRS at 40 bpm and a wave at the beginning of the ST segment corresponding to a retrograde P were detected in the vigil period. No asystole periods longer than 3 seconds are detected. Given this you would say:", "full_answer": "What are these findings in the Holter? If the sinus node fails, the AV node takes over and stimulates the ventricle. In 10% of people there is retrograde conduction, so that beat generated in the AVN stimulates both ventricles and atria, giving a retrograde P as reported in the study. In this sense, no problem. Likewise, there are also no pauses longer than 3 seconds that could cause cardiogenic syncope (which, in fact, he has never suffered). Option 1 is ruled out: it is not a complete AVB. Option 2 cannot be answered either: to exclude, what is said to exclude... It is like never/always. Option 3 does not make much sense: we could suspend those that impair conduction (beta-blockers, calcium antagonists), but drugs that facilitate, as long as we do not give him a perfusion of atropine to take home... In option 4, stating that he has sinoatrial block is a bit of a rap: without an electrophysiological study it cannot be said, he does not necessarily have SA block, and this alone would not indicate the implantation of a pacemaker. So, finally, I would check option 5.", "type": "CARDIOLOGY", "options": {"1": "The patient has a third-degree AVB and requires pacemaker implantation.", "2": "The absence of periods of asystole >3 seconds excludes a cardiac cause of dizziness.", "3": "Drug treatment to increase conduction in the AV node would be indicated.", "4": "The patient presents sinus dysfunction with sinoatrial block and requires pacemaker implantation due to symptoms.", "5": "The alterations detected in the ambulatory electrocardiogram are typical of patients of this age and there is no indication for therapeutic intervention."}, "correct_option": 5, "explanations": {"1": {"exist": true, "char_ranges": [[448, 496]], "word_ranges": [[77, 88]], "text": "Option 1 is ruled out: it is not a complete AVB."}, "2": {"exist": true, "char_ranges": [[497, 596]], "word_ranges": [[88, 105]], "text": "Option 2 cannot be answered either: to exclude, what is said to exclude... It is like never/always."}, "3": {"exist": true, "char_ranges": [[597, 812]], "word_ranges": [[105, 141]], "text": "Option 3 does not make much sense: we could suspend those that impair conduction (beta-blockers, calcium antagonists), but drugs that facilitate, as long as we do not give him a perfusion of atropine to take home..."}, "4": {"exist": true, "char_ranges": [[813, 1044]], "word_ranges": [[141, 182]], "text": "In option 4, stating that he has sinoatrial block is a bit of a rap: without an electrophysiological study it cannot be said, he does not necessarily have SA block, and this alone would not indicate the implantation of a pacemaker."}, "5": {"exist": false, "char_ranges": [], "word_ranges": [], "text": ""}}} {"id": 370, "year": 2016, "question_id_specific": 125, "full_question": "A former smoker patient, previously diagnosed with COPD comes in for a checkup. The physical examination reveals digital hypokratism of recent onset. What is the most acceptable explanation for this finding in the clinical context described?", "full_answer": "In a digital hypokratitis of recent onset in a smoking patient, the first option to rule out is bronchopulmonary carcinoma. Other pathologies can also occur but not so suddenly.", "type": "PULMONOLOGY AND THORACIC SURGERY", "options": {"1": "Pulmonary carcinoma.", "2": "Bronchiectasis.", "3": "Pulmonary fibrosis.", "4": "Cyanotic heart disease.", "5": NaN}, "correct_option": 1, "explanations": {"1": {"exist": true, "char_ranges": [[0, 123]], "word_ranges": [[0, 20]], "text": "In a digital hypokratitis of recent onset in a smoking patient, the first option to rule out is bronchopulmonary carcinoma."}, "2": {"exist": true, "char_ranges": [[124, 177]], "word_ranges": [[20, 29]], "text": "Other pathologies can also occur but not so suddenly."}, "3": {"exist": true, "char_ranges": [[124, 177]], "word_ranges": [[20, 29]], "text": "Other pathologies can also occur but not so suddenly."}, "4": {"exist": true, "char_ranges": [[124, 177]], "word_ranges": [[20, 29]], "text": "Other pathologies can also occur but not so suddenly."}, "5": {"exist": false, "char_ranges": [], "word_ranges": [], "text": ""}}} {"id": 332, "year": 2016, "question_id_specific": 88, "full_question": "In a routine workup of a 59-year-old woman, a smoker of 20 cigarettes/day for 25 years, hypercalcemia of 11.3 mg/dL with a phosphorus of 3.4 mg/dL is detected. It would NOT be efficient at the outset:", "full_answer": "Urinary hydroxyproline: Urinary hydroxyproline excretion reflects the degradation of bone collagen, but is also influenced by the metabolism of other tissues (cartilage, skin) and by the absorption of products rich in collagen, such as meat or gelatins. Due to its diverse tissue origin and metabolic pattern, it correlates poorly with bone resorption.", "type": "ENDOCRINOLOGY", "options": {"1": "Determine serum PTH levels.", "2": "Determine serum vitamin D levels.", "3": "Determination of hydroxyprolinuria.", "4": "Plain chest X-ray.", "5": NaN}, "correct_option": 3, "explanations": {"1": {"exist": false, "char_ranges": [], "word_ranges": [], "text": ""}, "2": {"exist": false, "char_ranges": [], "word_ranges": [], "text": ""}, "3": {"exist": true, "char_ranges": [[0, 253]], "word_ranges": [[0, 37]], "text": "Urinary hydroxyproline: Urinary hydroxyproline excretion reflects the degradation of bone collagen, but is also influenced by the metabolism of other tissues (cartilage, skin) and by the absorption of products rich in collagen, such as meat or gelatins."}, "4": {"exist": false, "char_ranges": [], "word_ranges": [], "text": ""}, "5": {"exist": false, "char_ranges": [], "word_ranges": [], "text": ""}}} {"id": 78, "year": 2012, "question_id_specific": 33, "full_question": "A 55-year-old postmenopausal woman consults for asthenia and dyspnea on exertion. In the anamnesis she reported slight epigastralgia and occasional heartburn. No metrorrhagia. Laboratory tests: Hb6 gr/dL, MCV 69 fl, sideremia 13 micrograms/dL, ferritin 4 ngr/mL. Gastrointestinal endoscopy: small sliding hiatal hernia with no signs of esophagitis. What is the most correct approach?", "full_answer": "We have a case of chronic iron deficiency anemia, without obvious bleeding. The first cause of iron deficiency anemia is occult bleeding of digestive origin. The existence of clinical symptoms suggestive of pathology of the upper digestive tract has led us to begin the study with upper endoscopy, but there is no pathology to justify the anemia, so further investigation is necessary, which rules out answers 1 and 2. Between 3 and 4, large bowel lesions are much more common and should be ruled out before small bowel lesions. In fact the recommendations of capsule endoscopy in anemia studies are restricted.", "type": "DIGESTIVE SYSTEM", "options": {"1": "Administer oral iron and monitor the evolution of anemia.", "2": "Treat with proton pump inhibitors and evaluate at three months.", "3": "Recommend a complete colonoscopy.", "4": "Perform an endoscopic capsule study.", "5": "Request a gynecological evaluation."}, "correct_option": 3, "explanations": {"1": {"exist": true, "char_ranges": [[158, 418]], "word_ranges": [[25, 69]], "text": "The existence of clinical symptoms suggestive of pathology of the upper digestive tract has led us to begin the study with upper endoscopy, but there is no pathology to justify the anemia, so further investigation is necessary, which rules out answers 1 and 2."}, "2": {"exist": true, "char_ranges": [[158, 418]], "word_ranges": [[25, 69]], "text": "The existence of clinical symptoms suggestive of pathology of the upper digestive tract has led us to begin the study with upper endoscopy, but there is no pathology to justify the anemia, so further investigation is necessary, which rules out answers 1 and 2."}, "3": {"exist": true, "char_ranges": [[419, 528]], "word_ranges": [[69, 89]], "text": "Between 3 and 4, large bowel lesions are much more common and should be ruled out before small bowel lesions."}, "4": {"exist": true, "char_ranges": [[419, 528]], "word_ranges": [[69, 89]], "text": "Between 3 and 4, large bowel lesions are much more common and should be ruled out before small bowel lesions."}, "5": {"exist": false, "char_ranges": [], "word_ranges": [], "text": ""}}} {"id": 397, "year": 2016, "question_id_specific": 140, "full_question": "You assess a 66-year-old patient with accentuated groin pain with prolonged standing a few days a month. A plain radiograph of the hips shows narrowing of the femoroacetabular joint space, sclerosis and osteophytes. What is your attitude?", "full_answer": "You present with coxarthrosis (no further imaging tests are needed), and conservative options are exhausted before opting for surgical treatment. Weak opioids are not the initial treatment of choice.", "type": "TRAUMATOLOGY AND ORTHOPEDICS", "options": {"1": "I make the diagnosis of coxarthrosis and send to the traumatologist to place a hip prosthesis.", "2": "Start treatment with weak opioids that have shown evidence of halting disease progression.", "3": "I instigate treatment with paracetamol, explain that the evolution is very variable and the surgical indication depends on functionality and pain control.", "4": "Because of the radiological features described, I need a hip MRI before making a therapeutic decision.", "5": NaN}, "correct_option": 3, "explanations": {"1": {"exist": false, "char_ranges": [], "word_ranges": [], "text": ""}, "2": {"exist": true, "char_ranges": [[0, 199]], "word_ranges": [[0, 29]], "text": "You present with coxarthrosis (no further imaging tests are needed), and conservative options are exhausted before opting for surgical treatment. Weak opioids are not the initial treatment of choice."}, "3": {"exist": true, "char_ranges": [[0, 199]], "word_ranges": [[0, 29]], "text": "You present with coxarthrosis (no further imaging tests are needed), and conservative options are exhausted before opting for surgical treatment. Weak opioids are not the initial treatment of choice."}, "4": {"exist": false, "char_ranges": [], "word_ranges": [], "text": ""}, "5": {"exist": false, "char_ranges": [], "word_ranges": [], "text": ""}}} {"id": 275, "year": 2016, "question_id_specific": 74, "full_question": "A 67-year-old man, diagnosed with cirrhosis of the liver and an active drinker, is admitted for progressive abdominal distension with diffuse malaise, of two weeks of evolution. He reported no fever or other symptoms. On examination, there was a changeable dullness on abdominal percussion, with absence of edema. A diagnostic paracentesis is performed, finding a slightly cloudy fluid, with 2,300 cells/mL, of which 30% are lymphocytes, 60% polymorphonuclear and 10% red blood cells. What is the first therapeutic measure you would prescribe for this patient immediately?", "full_answer": "This is spontaneous bacterial peritonitis (SBP), defined by the presence of > 250 polymorphonuclears/mL in the ascitic fluid (in this case there are 1,380, which is 60% of 2,300). It is an indication to start 3rd generation cephalosporins (ceftriaxone or cefotaxime) and albumin. The rest are treatments that can wait for improvement (they do not speak of tension ascites).", "type": "DIGESTIVE SYSTEM", "options": {"1": "Salt and fluid restriction.", "2": "Diuretic treatment with oral spironolactone.", "3": "Treatment with a third generation cephalosporin.", "4": "Evacuative paracentesis.", "5": NaN}, "correct_option": 3, "explanations": {"1": {"exist": true, "char_ranges": [[280, 373]], "word_ranges": [[43, 59]], "text": "The rest are treatments that can wait for improvement (they do not speak of tension ascites)."}, "2": {"exist": true, "char_ranges": [[280, 373]], "word_ranges": [[43, 59]], "text": "The rest are treatments that can wait for improvement (they do not speak of tension ascites)."}, "3": {"exist": true, "char_ranges": [[0, 279]], "word_ranges": [[0, 43]], "text": "This is spontaneous bacterial peritonitis (SBP), defined by the presence of > 250 polymorphonuclears/mL in the ascitic fluid (in this case there are 1,380, which is 60% of 2,300). It is an indication to start 3rd generation cephalosporins (ceftriaxone or cefotaxime) and albumin."}, "4": {"exist": true, "char_ranges": [[280, 373]], "word_ranges": [[43, 59]], "text": "The rest are treatments that can wait for improvement (they do not speak of tension ascites)."}, "5": {"exist": false, "char_ranges": [], "word_ranges": [], "text": ""}}} {"id": 418, "year": 2018, "question_id_specific": 77, "full_question": "An 80-year-old woman comes to the emergency room with abdominal pain starting in the epigastrium and radiating later to the left iliac fossa. An abdominal CT scan is performed, showing inflammation in the walls of the sigma and mesenteric abscess of 2 cm. The treatment of choice is:", "full_answer": "We are asked about the management of a very common pathology, diverticulitis. And we are asked about the different therapeutic options depending on how advanced it is. The correct answer is 1, it does not meet the criteria to require drainage by guided puncture (the abscess is not larger than 5-6 cm). Discharge colostomy in this case makes no sense whatsoever, since it is not a question of solving an obstructive condition. Drainage by laparoscopic surgery is not indicated in this case. It could be indicated in the absence of the possibility of percutaneous drainage after failure of conservative treatment, although it is a very controversial option at this time. Sigmoidectomy and anastomosis is also not required. In a high number of cases, after the resolution of the symptoms, the outbreaks are reduced and they can be prevented with diet and hygienic dietary habits.", "type": "GENERAL SURGERY", "options": {"1": "Admission to the ward with absolute diet and broad-spectrum antibiotic treatment.", "2": "Discharge colostomy.", "3": "Drainage by laparoscopic surgery.", "4": "Urgent surgery with sigmoidectomy and colorectal anastomosis.", "5": NaN}, "correct_option": 1, "explanations": {"1": {"exist": true, "char_ranges": [[722, 877]], "word_ranges": [[117, 144]], "text": "In a high number of cases, after the resolution of the symptoms, the outbreaks are reduced and they can be prevented with diet and hygienic dietary habits."}, "2": {"exist": true, "char_ranges": [[303, 426]], "word_ranges": [[52, 72]], "text": "Discharge colostomy in this case makes no sense whatsoever, since it is not a question of solving an obstructive condition."}, "3": {"exist": true, "char_ranges": [[193, 302]], "word_ranges": [[32, 52]], "text": "it does not meet the criteria to require drainage by guided puncture (the abscess is not larger than 5-6 cm)."}, "4": {"exist": true, "char_ranges": [[670, 721]], "word_ranges": [[110, 117]], "text": "Sigmoidectomy and anastomosis is also not required."}, "5": {"exist": false, "char_ranges": [], "word_ranges": [], "text": ""}}} {"id": 200, "year": 2013, "question_id_specific": 75, "full_question": "A 75-year-old woman consults for insomnia. She reports that for years she has been waking up at night with a tingling, burning sensation in the legs and sometimes in the arms; these symptoms also occur in the afternoon. She notices improvement when moving her legs, but the symptoms recur during rest, so she cannot fall asleep again. This picture could lead us to the diagnosis of restless legs syndrome Which of the following is FALSE?", "full_answer": "The diagnosis is based on clinical criteria, so muscle biopsy is only performed to rule out other diseases, not to confirm this one. An iron profile, thyroid hormone and vitamin determinations should be performed to rule out other diseases with similar symptomatology. Treatment is symptomatic and is instituted when it interferes with sleep or quality of life, and the first choice treatment is dopamine agonists such as pramipexole and ropirinol.", "type": "NEUROLOGY", "options": {"1": "The diagnosis of this affectation is based on clinical criteria.", "2": "A basic blood test including iron profile, thyroid hormones and B 12 and folic acid should be performed.", "3": "The treatment would be indicated in patients who present alteration of sleep or quality of life but does not alter the course of the disease.", "4": "The diagnosis is confirmed by muscle biopsy.", "5": "Pramipexole and ropirinol are used in the treatment."}, "correct_option": 4, "explanations": {"1": {"exist": false, "char_ranges": [], "word_ranges": [], "text": ""}, "2": {"exist": true, "char_ranges": [[133, 268]], "word_ranges": [[23, 42]], "text": "An iron profile, thyroid hormone and vitamin determinations should be performed to rule out other diseases with similar symptomatology."}, "3": {"exist": true, "char_ranges": [[269, 448]], "word_ranges": [[42, 70]], "text": "Treatment is symptomatic and is instituted when it interferes with sleep or quality of life, and the first choice treatment is dopamine agonists such as pramipexole and ropirinol."}, "4": {"exist": true, "char_ranges": [[0, 132]], "word_ranges": [[0, 23]], "text": "The diagnosis is based on clinical criteria, so muscle biopsy is only performed to rule out other diseases, not to confirm this one."}, "5": {"exist": true, "char_ranges": [[269, 448]], "word_ranges": [[42, 70]], "text": "Treatment is symptomatic and is instituted when it interferes with sleep or quality of life, and the first choice treatment is dopamine agonists such as pramipexole and ropirinol."}}} {"id": 469, "year": 2020, "question_id_specific": 137, "full_question": "93-year-old woman with a history of COPD, AHT, dyslipidemia, atrial fibrillation and ischemic heart disease. She lives alone. She has ventricular ejection fraction 53%, creatinine 1.5 mg/dL, normal coagulation. She receives treatment with inhaled bronchodilators and ASA 100 mg per day. She suffers a fall in the street, after which it is impossible for her to stand or sit up, with intense pain in the right hip on mobilization, great shortening and external rotation of the leg. When and how should she be treated?", "full_answer": "A new approach is currently being developed for the treatment of these patients, which tries to coordinate from the beginning the activity of the different sectors involved, avoiding that the transfer of the patient from one to another takes place at the end of the treatment of each part. This idea gave rise to the so-called orthogeriatric units\", which combine the surgeon's surgical activity with that of the patient's immediate rehabilitation and the controls and follow-up of internal medicine. From the very beginning of the process, the social workers and the various vital and support aids are put in place to ensure that recovery is as rapid and complete as possible. ... In most cases, surgery should be performed as soon as possible, as soon as the patient is medically stabilized. There is a contraindication to surgery in patients with unstable medical comorbidity, but several studies have shown that delaying surgery beyond 48 h after admission increases mortality. This is why the ideal time to intervene in these patients is after achieving stabilization of their medical pathologies, taking care that this takes place before 48 h have elapsed.\" There is no data to suggest that he has any unstable comorbidity at the present time, so surgery in less than 48 hours and orthogeriatric management is of interest.", "type": "ORTHOPEDIC SURGERY AND TRAUMATOLOGY", "options": {"1": "Admission for transthoracic ultrasound, assessment by Cardiology and then closed reduction and fixation with plate screw.", "2": "Admission for evaluation by Nephrology and Cardiology, and then Traumatology will decide whether to use an arthroplasty or a trochanteric nail.", "3": "Admission to Internal Medicine / Geriatrics, progressive stabilization of comorbidities, conservative orthopedic treatment.", "4": "Closed reduction and fixation of the fracture within 48 hours, very early sedation, orthogeriatric management.", "5": NaN}, "correct_option": 4, "explanations": {"1": {"exist": false, "char_ranges": [], "word_ranges": [], "text": ""}, "2": {"exist": false, "char_ranges": [], "word_ranges": [], "text": ""}, "3": {"exist": false, "char_ranges": [], "word_ranges": [], "text": ""}, "4": {"exist": true, "char_ranges": [[982, 1328]], "word_ranges": [[158, 217]], "text": "This is why the ideal time to intervene in these patients is after achieving stabilization of their medical pathologies, taking care that this takes place before 48 h have elapsed.\" There is no data to suggest that he has any unstable comorbidity at the present time, so surgery in less than 48 hours and orthogeriatric management is of interest."}, "5": {"exist": false, "char_ranges": [], "word_ranges": [], "text": ""}}} {"id": 378, "year": 2016, "question_id_specific": 132, "full_question": "A 72-year-old patient presents with progressive memory loss and word finding difficulty of one year's duration. What is the most useful examination to characterize the type of cognitive deficit he presents with?", "full_answer": "The characterization of cognitive deterioration processes is still clinical, with imaging tests (MRI or PET) being a support tool in some occasions. The first option would be the correct one because it allows us to measure in a standardized way each of the cognitive domains, which will allow us to frame the patient's deficits in a specific syndrome and to monitor the deterioration with repeated scans over time.", "type": "NEUROLOGY", "options": {"1": "Neuropsychological evaluation.", "2": "Brain magnetic resonance imaging.", "3": "Positron Emission Tomography.", "4": "Electroencephalogram.", "5": NaN}, "correct_option": 1, "explanations": {"1": {"exist": true, "char_ranges": [[0, 275]], "word_ranges": [[0, 45]], "text": "The characterization of cognitive deterioration processes is still clinical, with imaging tests (MRI or PET) being a support tool in some occasions. The first option would be the correct one because it allows us to measure in a standardized way each of the cognitive domains,"}, "2": {"exist": false, "char_ranges": [], "word_ranges": [], "text": ""}, "3": {"exist": true, "char_ranges": [[0, 275]], "word_ranges": [[0, 45]], "text": "The characterization of cognitive deterioration processes is still clinical, with imaging tests (MRI or PET) being a support tool in some occasions. The first option would be the correct one because it allows us to measure in a standardized way each of the cognitive domains,"}, "4": {"exist": false, "char_ranges": [], "word_ranges": [], "text": ""}, "5": {"exist": false, "char_ranges": [], "word_ranges": [], "text": ""}}} {"id": 582, "year": 2022, "question_id_specific": 208, "full_question": "A 53-year-old man with no history of interest who after a bad movement in the gym notices low back pain that after a few hours radiates to the back of the right leg and reaches the lateral edge of the foot. On examination he presents positive Lasègue maneuver at 40o, abolished hamstring reflex and he cannot stand on tiptoe. The most probable etiological diagnosis is:", "full_answer": "He describes S1 root involvement by the inability to tiptoe and the absence of the Achilles reflex, in addition to the distribution of pain down the leg. Of our options, the cause that can produce damage to S1 is L5-S1 disc herniation.", "type": "NEUROLOGY", "options": {"1": "Right L2-L3 disc herniation.", "2": "Right L3-L4 disc herniation.", "3": "Right L4-L5 disc herniation.", "4": "Right L5-S1 disc herniation.", "5": NaN}, "correct_option": 4, "explanations": {"1": {"exist": false, "char_ranges": [], "word_ranges": [], "text": ""}, "2": {"exist": false, "char_ranges": [], "word_ranges": [], "text": ""}, "3": {"exist": false, "char_ranges": [], "word_ranges": [], "text": ""}, "4": {"exist": true, "char_ranges": [[0, 235]], "word_ranges": [[0, 42]], "text": "He describes S1 root involvement by the inability to tiptoe and the absence of the Achilles reflex, in addition to the distribution of pain down the leg. Of our options, the cause that can produce damage to S1 is L5-S1 disc herniation."}, "5": {"exist": false, "char_ranges": [], "word_ranges": [], "text": ""}}} {"id": 160, "year": 2013, "question_id_specific": 117, "full_question": "A 35-year-old man presenting hematuria after respiratory infections for several years, a blood test shows creatinine 1 mg/dl with no other alterations and urine hematocytes 50/field being 80% dysmorphic, with proteinuria of 0.8 grams in 24 hours. What is the most likely diagnosis?", "full_answer": "The correct answer is: 3. IgA nephropathy. Given that the clinical presentation of this nephropathy appears in a very unspecific way, to identify the responsible disease we must rely on its epidemiology. Since he is a young male, with a history of previous respiratory infections, a relationship between these infections and renal pathology is established. Based on this we think about IgA nephropathy, which besides being the most frequent glomerulonephritis, it is presented in most of the cases as described.", "type": "NEPHROLOGY", "options": {"1": "Minimal change nephropathy.", "2": "Membranous glomerulonephritis.", "3": "IgA nephropathy.", "4": "Proliferative and diffuse glomerulonephritis.", "5": "Focal and primary segmental glomerulosclerosis."}, "correct_option": 3, "explanations": {"1": {"exist": false, "char_ranges": [], "word_ranges": [], "text": ""}, "2": {"exist": false, "char_ranges": [], "word_ranges": [], "text": ""}, "3": {"exist": true, "char_ranges": [[204, 511]], "word_ranges": [[32, 80]], "text": "Since he is a young male, with a history of previous respiratory infections, a relationship between these infections and renal pathology is established. Based on this we think about IgA nephropathy, which besides being the most frequent glomerulonephritis, it is presented in most of the cases as described."}, "4": {"exist": false, "char_ranges": [], "word_ranges": [], "text": ""}, "5": {"exist": false, "char_ranges": [], "word_ranges": [], "text": ""}}} {"id": 312, "year": 2016, "question_id_specific": 217, "full_question": "An 84-year-old woman presents with loss of vision in the left eye of 4 days of evolution accompanied by metamorphopsia. The macula shows abundant hard exudates, two small deep hemorrhages and a localized neurosensory retinal detachment. In the contralateral eye there are abundant soft drusen. Given this picture, which of the following diagnoses do you think is the most likely?", "full_answer": "In principle, this is an easy question, as it deals with a disease that is frequently repeated in the MIR exams. The description is typical of option 2. It is an exudative AMD (although as we said in question 215, with the new classification, we would speak of an advanced AMD in its neovascular variant). In any case, we have an elderly patient with vision loss and metamorphopsia in one eye. In the examination there are lesions in the macula. With that we can rule out acute posterior vitreous detachment, which does not produce visual loss, nor metamorphopsia, nor lesions in the macula. And also non-arteritic anterior ischemic optic neuropathy, because although it produces visual loss and is typical of older people, it does not produce metamorphopsia. And there are no lesions in the macula but papilla edema. In central retinal artery obstruction, metamorphopsia is not particularly characteristic (visual loss and advanced age are), but the lesions in the fundus of the eye vary. In the case of arterial occlusion, there is intracellular edema that is seen as absence of hemorrhages and a whitish, pale fundus. In addition, the entire retina is globally affected, and a cherry-red spot may appear in the macula. In this case the lesions are totally different. They describe hard exudates, deep hemorrhages and localized neurosensory retinal detachment. All this in the macula. These are the characteristic findings of neovascular (or exudative) AMD. As a clue, in the other eye there are abundant soft drusen. Drusen are the typical findings of AMD. So he has AMD in both eyes and in the right eye it has recently become complicated by the appearance of neovascularization.", "type": "OPHTHALMOLOGY", "options": {"1": "Acute posterior vitreous detachment.", "2": "Exudative age-related macular degeneration (AMD).", "3": "Central retinal artery obstruction.", "4": "Non-arteritic anterior ischemic optic neuropathy.", "5": NaN}, "correct_option": 2, "explanations": {"1": {"exist": true, "char_ranges": [[394, 591]], "word_ranges": [[71, 103]], "text": "In the examination there are lesions in the macula. With that we can rule out acute posterior vitreous detachment, which does not produce visual loss, nor metamorphopsia, nor lesions in the macula."}, "2": {"exist": true, "char_ranges": [[1270, 1459]], "word_ranges": [[211, 237]], "text": "They describe hard exudates, deep hemorrhages and localized neurosensory retinal detachment. All this in the macula. These are the characteristic findings of neovascular (or exudative) AMD."}, "3": {"exist": true, "char_ranges": [[990, 1269]], "word_ranges": [[164, 211]], "text": "In the case of arterial occlusion, there is intracellular edema that is seen as absence of hemorrhages and a whitish, pale fundus. In addition, the entire retina is globally affected, and a cherry-red spot may appear in the macula. In this case the lesions are totally different."}, "4": {"exist": true, "char_ranges": [[592, 759]], "word_ranges": [[103, 127]], "text": "And also non-arteritic anterior ischemic optic neuropathy, because although it produces visual loss and is typical of older people, it does not produce metamorphopsia."}, "5": {"exist": false, "char_ranges": [], "word_ranges": [], "text": ""}}} {"id": 24, "year": 2011, "question_id_specific": 115, "full_question": "A 68-year-old man is evaluated for fever, weight loss and dyspnea on exertion. The EP reveals a new aortic diastolic murmur and stigmata of peripheral embolisms. Echocardiography showed a 1 cm vegetation on the aortic valve and two blood cultures isolated penicillin-sensitive Streptococcus bovis. After starting treatment for endocarditis, which of the following examinations could help us in the diagnosis?", "full_answer": "Another gift of a question, the association of S.bovis endocarditis and colon cancer. By the way, it is funny that they give Chest Rx as a test to be performed..... are you telling me that a patient with fever and dyspnea will have an echocardiogram before a simple chest x-ray? And on the other hand... why don't those who write the MIR ask shorter questions without so much back and forth? If what they want is to ask the test to be performed on a patient with S. bovis endocarditis, they could ask the question directly without telling us about the patient's moles...", "type": "INFECTIOUS", "options": {"1": "Glucose tolerance test.", "2": "Chest X-ray.", "3": "Upper gastrointestinal endoscopy.", "4": "Colonoscopy.", "5": "Abdominal ultrasound."}, "correct_option": 4, "explanations": {"1": {"exist": false, "char_ranges": [], "word_ranges": [], "text": ""}, "2": {"exist": false, "char_ranges": [], "word_ranges": [], "text": ""}, "3": {"exist": false, "char_ranges": [], "word_ranges": [], "text": ""}, "4": {"exist": true, "char_ranges": [[28, 84]], "word_ranges": [[5, 13]], "text": "the association of S.bovis endocarditis and colon cancer."}, "5": {"exist": false, "char_ranges": [], "word_ranges": [], "text": ""}}} {"id": 260, "year": 2014, "question_id_specific": 99, "full_question": "A 45-year-old woman comes to the clinic referred from surgery with the diagnosis of a neuroendocrine tumor diagnosed after partial pancreatectomy for a 2-cm tumor in the tail of the pancreas. The tumor had been detected by chance in an abdominal CT scan requested to complete the study of a simple hepatic cyst. On questioning the patient, the history of irregular menstruation, amenorrhea for the last 6 months, and repeated renal colic since she was 20 years old, for which she has required lithotripsy on several occasions, stand out. She also has a family history of renoureteral colic. What is her suspected diagnosis?", "full_answer": "Very nice clinical question from MEN. Pancreatic neuroendocrine tumor + pituitary adenoma (prolactinoma) + hyperparathyroidism (renoureteral colic in young people): MEN 1. Medium difficulty because you have to know the MEN triad and recognize the different tumors.", "type": "ENDOCRINOLOGY", "options": {"1": "Multiple endocrine neoplasia type 1 or Wermer's syndrome.", "2": "Multiple endocrine neoplasia type 2 A or Sipple's syndrome.", "3": "Multiple endocrine neoplasia type 2B.", "4": "Somatostatinoma.", "5": "A PTH-producing neuroendocrine tumor."}, "correct_option": 1, "explanations": {"1": {"exist": true, "char_ranges": [[38, 171]], "word_ranges": [[6, 22]], "text": "Pancreatic neuroendocrine tumor + pituitary adenoma (prolactinoma) + hyperparathyroidism (renoureteral colic in young people): MEN 1."}, "2": {"exist": false, "char_ranges": [], "word_ranges": [], "text": ""}, "3": {"exist": false, "char_ranges": [], "word_ranges": [], "text": ""}, "4": {"exist": false, "char_ranges": [], "word_ranges": [], "text": ""}, "5": {"exist": false, "char_ranges": [], "word_ranges": [], "text": ""}}} {"id": 211, "year": 2014, "question_id_specific": 87, "full_question": "A 78-year-old woman comes to the emergency department for left iliac fossa pain of 24 hours of evolution associated with fever and occasional vomiting. On examination, there is selective pain on palpation in the left iliac fossa with a feeling of occupation, defense and positive decompression. When acute diverticulitis is suspected, which of the following statements is correct?", "full_answer": "Answer 1 is incorrect because the imaging tests of choice would be ultrasound or CT; answer 3 is incorrect because the laparoscopic approach is indicated from the start; answer 4 is incorrect because the indication in the first episode is for those under 50 years of age, in the rest it is indicated after the second episode or in cases of persistent symptoms or suspected neoplasia; and answer 5 is incorrect because surgery in cases of perforation with peritonitis should always resect the affected segment of colon to control the focus of the infection.", "type": "GENERAL SURGERY", "options": {"1": "The safest and best performing complementary examination is the barium contrast enema.", "2": "In case of contained pelvic abscess, percutaneous drainage guided by CT or ultrasound is indicated.", "3": "If surgical intervention is required after resolution of the acute episode, the laparoscopic approach is contraindicated.", "4": "In case of acute uncomplicated diverticulitis, elective sigmoidectomy is indicated after cure of the first acute episode.", "5": "If generalized peritonitis occurs, the most appropriate surgical technique is the practice of a derivative colostomy without resection of the affected sigmoid segment."}, "correct_option": 2, "explanations": {"1": {"exist": true, "char_ranges": [[0, 84]], "word_ranges": [[0, 15]], "text": "Answer 1 is incorrect because the imaging tests of choice would be ultrasound or CT;"}, "2": {"exist": false, "char_ranges": [], "word_ranges": [], "text": ""}, "3": {"exist": true, "char_ranges": [[85, 169]], "word_ranges": [[15, 28]], "text": "answer 3 is incorrect because the laparoscopic approach is indicated from the start;"}, "4": {"exist": true, "char_ranges": [[170, 383]], "word_ranges": [[28, 66]], "text": "answer 4 is incorrect because the indication in the first episode is for those under 50 years of age, in the rest it is indicated after the second episode or in cases of persistent symptoms or suspected neoplasia;"}, "5": {"exist": true, "char_ranges": [[384, 556]], "word_ranges": [[66, 94]], "text": "and answer 5 is incorrect because surgery in cases of perforation with peritonitis should always resect the affected segment of colon to control the focus of the infection."}}} {"id": 162, "year": 2013, "question_id_specific": 121, "full_question": "A 52-year-old patient with stage V chronic kidney disease, secondary to autosomal dominant polycystic kidney disease, receives a cadaver donor kidney graft. A 34-year-old brother of his comes to visit him and says he does not know if he has the disease because he has never seen a doctor since he was a teenager. What attitude or test do you think is the most appropriate to recommend at that moment?", "full_answer": "The correct answer is: 2. Abdomino-pelvic ultrasound. Abdomino-pelvic ultrasound is undoubtedly the most specific and sensitive test for the diagnosis of polycystic kidney disease and, above all, the cheapest to demonstrate whether or not the patient's sibling has inherited polycystic kidney disease.", "type": "NEPHROLOGY", "options": {"1": "Genetic mutational and linkage study.", "2": "Abdomino-pelvic ultrasound.", "3": "Helical computed axial tomography with iodinated contrast.", "4": "Abdominal and cerebral MRI.", "5": "Periodic clinical controls."}, "correct_option": 2, "explanations": {"1": {"exist": false, "char_ranges": [], "word_ranges": [], "text": ""}, "2": {"exist": true, "char_ranges": [[54, 301]], "word_ranges": [[7, 42]], "text": "Abdomino-pelvic ultrasound is undoubtedly the most specific and sensitive test for the diagnosis of polycystic kidney disease and, above all, the cheapest to demonstrate whether or not the patient's sibling has inherited polycystic kidney disease."}, "3": {"exist": false, "char_ranges": [], "word_ranges": [], "text": ""}, "4": {"exist": false, "char_ranges": [], "word_ranges": [], "text": ""}, "5": {"exist": false, "char_ranges": [], "word_ranges": [], "text": ""}}} {"id": 207, "year": 2014, "question_id_specific": 181, "full_question": "A 43-year-old woman referred to the Hospital's Lower Genital Tract Pathology Clinic for presenting with a cervico-vaginal cytology reported as \"L-SIL\" (Low Grade Intraepithelial Lesion). She refers repeated vaginal infections (Trichomoniasis,...) and having had more than 5 sexual partners in her life. The best care strategy for this woman is:", "full_answer": "The best answer to this question is number 5. Most of the lesions identified as L-SIL are associated with HPV infection: for this reason, it is advisable to stop smoking, as this is a factor in the progression of the said infection. On the other hand, according to the latest studies, 70% of the cytologies reported as LSIL will remit, 15% will persist and another 15% will show a more severe cytology, without being able to differentiate whether it is a biological progression or a lesion hidden in the initial cytology. Because of this, it is best to return for a follow-up at 6 months. Given the woman's personal history (more than 5 sexual partners and repeat infections) and her age, a referral for colposcopy would be ideal. However, answer 5 says control in 6 months, but does not specify which control (it could be a colposcopy or cytology).", "type": "GYNECOLOGY AND OBSTETRICS", "options": {"1": "Conization with diathermy loop.", "2": "Endocervical curettage with Kevorkian curettage.", "3": "Endometrial biopsy with Cornier cannula.", "4": "Hysterectomy without adnexectomy.", "5": "Advise her to stop smoking and contraceptives and to have a control in 6 months."}, "correct_option": 5, "explanations": {"1": {"exist": false, "char_ranges": [], "word_ranges": [], "text": ""}, "2": {"exist": false, "char_ranges": [], "word_ranges": [], "text": ""}, "3": {"exist": false, "char_ranges": [], "word_ranges": [], "text": ""}, "4": {"exist": false, "char_ranges": [], "word_ranges": [], "text": ""}, "5": {"exist": true, "char_ranges": [[46, 588]], "word_ranges": [[9, 105]], "text": "Most of the lesions identified as L-SIL are associated with HPV infection: for this reason, it is advisable to stop smoking, as this is a factor in the progression of the said infection. On the other hand, according to the latest studies, 70% of the cytologies reported as LSIL will remit, 15% will persist and another 15% will show a more severe cytology, without being able to differentiate whether it is a biological progression or a lesion hidden in the initial cytology. Because of this, it is best to return for a follow-up at 6 months."}}} {"id": 584, "year": 2022, "question_id_specific": 73, "full_question": "A 51-year-old woman, menopausal for a year and a half, who consults for vaginal spotting of 2 weeks' evolution. She reports that the spotting is less than a menstrual period. The ultrasound shows a 7 mm endometrium. Mark the correct option:", "full_answer": "The endometrium is thickened for a menopausal woman (some guidelines put the limit at 5 and others at 3mm). And the first complementary test to perform would be an endometrial biopsy. If this is not decisive, hysteroscopy will be considered as a second option.", "type": "OBSTETRICS AND GYNECOLOGY", "options": {"1": "The endometrium is thickened and in view of the patient's symptoms, an endometrial biopsy is taken with a Cornier cannula.", "2": "The endometrium is not thickened so she is prescribed tranexamic acid and ambulatory follow-up.", "3": "Indications for diagnostic hysteroscopy without the need for endometrial biopsy.", "4": "The endometrium is not thickened, but since she has clinical signs of metrorrhagia, an endometrial biopsy is indicated.", "5": NaN}, "correct_option": 1, "explanations": {"1": {"exist": true, "char_ranges": [[0, 183]], "word_ranges": [[0, 31]], "text": "The endometrium is thickened for a menopausal woman (some guidelines put the limit at 5 and others at 3mm). And the first complementary test to perform would be an endometrial biopsy."}, "2": {"exist": false, "char_ranges": [], "word_ranges": [], "text": ""}, "3": {"exist": false, "char_ranges": [], "word_ranges": [], "text": ""}, "4": {"exist": false, "char_ranges": [], "word_ranges": [], "text": ""}, "5": {"exist": false, "char_ranges": [], "word_ranges": [], "text": ""}}} {"id": 138, "year": 2012, "question_id_specific": 165, "full_question": "A child who comes to the outpatient clinic because he brings his hand to his ear, without fever or acute inflammatory signs and who on examination shows a pinkish/amber eardrum. As history he refers that one month ago he presented an episode of otalgia, fever accompanied by irritability and that he was treated with amoxicillin 40 mg/kg/day. What is the most probable diagnosis?", "full_answer": "Reading the case, we are discarding options: child with otalgia, but without fever or acute inflammatory signs: it is neither 1 (recurrent acute otitis media) nor 5 (mastoiditis). On otoscopy, a pinkish and amber eardrum is observed: there are no scales, so it is not 4 (cholesteatomatous chronic otitis media); nor perforations or adhesions, so it is not 3 (simple chronic otitis media). Otoscopy is characteristic of serous otitis media (answer 2 correct). It is known by ENT, pediatricians and family physicians that in the month following an acute otitis media, the middle ear is occupied by a mucous content whose evolution should be monitored. I do not think that this is a contestable issue either.", "type": "OTORHINOLARYNGOLOGY AND MAXILLOFACIAL SURGERY", "options": {"1": "Recurrent acute otitis media.", "2": "Serous otitis media.", "3": "Simple chronic otitis media.", "4": "Chronic cholesteatomatous otitis media.", "5": "Mastoiditis."}, "correct_option": 2, "explanations": {"1": {"exist": true, "char_ranges": [[45, 179]], "word_ranges": [[7, 28]], "text": "child with otalgia, but without fever or acute inflammatory signs: it is neither 1 (recurrent acute otitis media) nor 5 (mastoiditis)."}, "2": {"exist": true, "char_ranges": [[389, 458]], "word_ranges": [[63, 73]], "text": "Otoscopy is characteristic of serous otitis media (answer 2 correct)."}, "3": {"exist": true, "char_ranges": [[312, 388]], "word_ranges": [[50, 63]], "text": "nor perforations or adhesions, so it is not 3 (simple chronic otitis media)."}, "4": {"exist": true, "char_ranges": [[180, 311]], "word_ranges": [[28, 50]], "text": "On otoscopy, a pinkish and amber eardrum is observed: there are no scales, so it is not 4 (cholesteatomatous chronic otitis media);"}, "5": {"exist": true, "char_ranges": [[45, 179]], "word_ranges": [[7, 28]], "text": "child with otalgia, but without fever or acute inflammatory signs: it is neither 1 (recurrent acute otitis media) nor 5 (mastoiditis)."}}} {"id": 136, "year": 2012, "question_id_specific": 132, "full_question": "A 60-year-old man comes to the emergency department for a coma crisis. Ex-smoker for 3 years, with no other history of interest. A CT scan shows multiple metastases. What is the most probable origin?", "full_answer": "Pretty easy, isn't it? Lung cancer is the most frequent cause of brain metastases and in 20-30% of cases it is as a result of them that the primary tumor is diagnosed (as here).", "type": "ONCOLOGY", "options": {"1": "Head and neck cancer.", "2": "Lung cancer.", "3": "Urinary bladder cancer.", "4": "Colon cancer.", "5": "Pancreatic cancer."}, "correct_option": 2, "explanations": {"1": {"exist": false, "char_ranges": [], "word_ranges": [], "text": ""}, "2": {"exist": true, "char_ranges": [[23, 177]], "word_ranges": [[4, 34]], "text": "Lung cancer is the most frequent cause of brain metastases and in 20-30% of cases it is as a result of them that the primary tumor is diagnosed (as here)."}, "3": {"exist": false, "char_ranges": [], "word_ranges": [], "text": ""}, "4": {"exist": false, "char_ranges": [], "word_ranges": [], "text": ""}, "5": {"exist": false, "char_ranges": [], "word_ranges": [], "text": ""}}} {"id": 518, "year": 2021, "question_id_specific": 105, "full_question": "A 24-year-old woman with a history of migraine, on propranolol and oral contraceptives, presents with severe dyspnea, hoarseness, rash, nausea and vomiting 30 minutes after taking metamizole. His blood pressure is 90/40 mmHg and SatO2 is 90%. The most correct initial treatment would be to administer:", "full_answer": "We are faced with an anaphylactic reaction to metamizole, in which it is necessary to act quickly or the patient could progress to cardiac arrest. It is important to assess the severity of the allergic reactions, since, in this case, early treatment is essential to prevent high mortality if not applied. The treatment of choice is IM adrenaline (only switch to IV if anaphylaxis is refractory), which has been shown to increase survival, and administration can be repeated every 5-15 minutes if symptoms do not subside. As adjuvant treatment, dexchlorpheniramine could be administered. In this particular case, since the patient was previously treated with propranolol, this indicates that the correct answer is 4, which includes the administration of glucagon, which is administered because patients taking beta-blockers can be resistant to treatment with adrenaline and develop refractory hypotension and prolonged bradycardia. The dose in adults is 1 to 2mg, IV or IM, which can be repeated in 5 minutes or followed by an infusion at 5-15mcg/min.", "type": "CRITICAL AND EMERGENCY CARE", "options": {"1": "Adrenaline.", "2": "Adrenaline and dexchlorpheniramine.", "3": "Adrenaline, dexchlorpheniramine and methylprednisolone.", "4": "Adrenaline, dexchlorpheniramine and glucagon.", "5": NaN}, "correct_option": 4, "explanations": {"1": {"exist": false, "char_ranges": [], "word_ranges": [], "text": ""}, "2": {"exist": false, "char_ranges": [], "word_ranges": [], "text": ""}, "3": {"exist": false, "char_ranges": [], "word_ranges": [], "text": ""}, "4": {"exist": true, "char_ranges": [[587, 930]], "word_ranges": [[93, 140]], "text": "In this particular case, since the patient was previously treated with propranolol, this indicates that the correct answer is 4, which includes the administration of glucagon, which is administered because patients taking beta-blockers can be resistant to treatment with adrenaline and develop refractory hypotension and prolonged bradycardia."}, "5": {"exist": false, "char_ranges": [], "word_ranges": [], "text": ""}}} {"id": 581, "year": 2022, "question_id_specific": 205, "full_question": "61-year-old male, smoker and hypertensive, who comes to the emergency department for sudden loss of strength and tingling sensation in the right hand lasting about 15 minutes, with almost complete recovery afterwards. ECG: sinus rhythm at 93 bpm. In relation to the most probable diagnosis, indicate the FALSE answer:", "full_answer": "The case is a TIA (transient ischemic attack). The basic etiological study includes echodoppler of supra-aortic trunks +/- cerebral arteries, echocardiogram and directed blood analysis.", "type": "NEUROLOGY", "options": {"1": "A probable cause is an arterioarterial embolism due to detachment of a carotid plaque.", "2": "Doppler ultrasound of supra-aortic trunks is not very useful for diagnosis.", "3": "A cranial CT scan is necessary to assess the impact on the brain parenchyma.", "4": "Surgical treatment is indicated if imaging tests reveal carotid stenosis >70%.", "5": NaN}, "correct_option": 2, "explanations": {"1": {"exist": false, "char_ranges": [], "word_ranges": [], "text": ""}, "2": {"exist": false, "char_ranges": [], "word_ranges": [], "text": ""}, "3": {"exist": false, "char_ranges": [], "word_ranges": [], "text": ""}, "4": {"exist": false, "char_ranges": [], "word_ranges": [], "text": ""}, "5": {"exist": false, "char_ranges": [], "word_ranges": [], "text": ""}}} {"id": 543, "year": 2022, "question_id_specific": 39, "full_question": "A 35-year-old patient consults to confirm a suspected allergy to Anisakis simplex. She has previously suffered a clinical picture of gastrointestinal anisakiasis after eating fish. The tests performed show specific IgE levels of 10 KU/l against Anisakis simplex. Which of the following would be the dietary recommendation for this patient?", "full_answer": "Anisakis simplex is a parasite frequently found in fish. To avoid reactions with this parasite, a diet free of fresh and raw fish (anchovies in vinegar, smoked, salted fish, sushi, etc.) should be followed. Deep-frozen fish or fresh fish previously frozen at -20ºC for at least 5 days before cooking can be consumed, as long as it is cooked at high temperatures >60ºC (roasted, fried, etc.), avoiding raw, grilled or microwaved preparations. It is recommended to consume preferably the tails because the larvae of the parasite are usually close to the viscera of the head.", "type": "ALLERGOLOGY", "options": {"1": "He cannot eat any fish, crustaceans, mollusks or cephalopods.", "2": "He can eat fresh fish cooked on the grill.", "3": "May eat commercially flash-frozen fish.", "4": "You can eat fresh marinated or cold smoked fish.", "5": NaN}, "correct_option": 3, "explanations": {"1": {"exist": false, "char_ranges": [], "word_ranges": [], "text": ""}, "2": {"exist": false, "char_ranges": [], "word_ranges": [], "text": ""}, "3": {"exist": false, "char_ranges": [], "word_ranges": [], "text": ""}, "4": {"exist": false, "char_ranges": [], "word_ranges": [], "text": ""}, "5": {"exist": false, "char_ranges": [], "word_ranges": [], "text": ""}}} {"id": 2, "year": 2011, "question_id_specific": 36, "full_question": "An 87-year-old patient with a history of chronic bronchitis and heart failure has been diagnosed with acute calculous cholecystitis. After 4 days of hospitalization and treatment with absolute diet, piperacillin-tazobactam serum therapy, the patient continues with fever, persistent abdominal pain and leukocytosis. The most appropriate attitude at this time would be:", "full_answer": "In this question, they insist on the age and comorbidity of the patient, as well as the time of evolution, so the answer will probably not be 1. In any case, the ideal would be to drain the gallbladder that has not gone up again, so the answer I would give would be 2.", "type": "DIGESTIVE", "options": {"1": "Urgent surgical treatment.", "2": "Biliary drainage by percutaneous cholecystostomy.", "3": "Replace piperacillin-tazobactam with metronidazole-cefotaxime.", "4": "Replace piperacillin-tazobactam with amikacin-clindamycin.", "5": "Add an aminoglycoside such as gentamicin to the treatment."}, "correct_option": 2, "explanations": {"1": {"exist": true, "char_ranges": [[0, 143]], "word_ranges": [[0, 28]], "text": "In this question, they insist on the age and comorbidity of the patient, as well as the time of evolution, so the answer will probably not be 1."}, "2": {"exist": true, "char_ranges": [[146, 269]], "word_ranges": [[28, 54]], "text": "In any case, the ideal would be to drain the gallbladder that has not gone up again, so the answer I would give would be 2."}, "3": {"exist": false, "char_ranges": [], "word_ranges": [], "text": ""}, "4": {"exist": false, "char_ranges": [], "word_ranges": [], "text": ""}, "5": {"exist": false, "char_ranges": [], "word_ranges": [], "text": ""}}} {"id": 122, "year": 2012, "question_id_specific": 119, "full_question": "A 17-year-old woman came to the emergency department with acute symptoms of high fever, pharyngeal pain and cervical lymphadenopathy. She had previously been diagnosed with acute pharyngitis and was treated with amoxicillin, and later presented with a generalized macular skin rash. Laboratory tests showed slight leukocytosis and presence of activated leukocytes, slight thrombopenia and slightly increased transaminases. What would be the most likely diagnosis of this clinical picture?", "full_answer": "The correct answer is the first one. It is a typical picture of infectious mononucleosis. Although a similar picture may occur in acute toxoplasmosis, less than 1% of acute toxoplasma infections present as mononucleosis. Characteristic of Epstein-Barr infectious mononucleosis is the appearance of macular rash after treatment with amoxicillin.", "type": "MICROBIOLOGY", "options": {"1": "It is a typical picture of infectious mononucleosis.", "2": "Varicella zoster infection.", "3": "Acute toxoplasmosis.", "4": "Lyme disease.", "5": "Infection by herpes virus 8."}, "correct_option": 1, "explanations": {"1": {"exist": true, "char_ranges": [[221, 344]], "word_ranges": [[34, 49]], "text": "Characteristic of Epstein-Barr infectious mononucleosis is the appearance of macular rash after treatment with amoxicillin."}, "2": {"exist": false, "char_ranges": [], "word_ranges": [], "text": ""}, "3": {"exist": true, "char_ranges": [[90, 220]], "word_ranges": [[15, 34]], "text": "Although a similar picture may occur in acute toxoplasmosis, less than 1% of acute toxoplasma infections present as mononucleosis."}, "4": {"exist": false, "char_ranges": [], "word_ranges": [], "text": ""}, "5": {"exist": false, "char_ranges": [], "word_ranges": [], "text": ""}}} {"id": 36, "year": 2011, "question_id_specific": 130, "full_question": "An 85-year-old woman consults for fatigue and weakness especially in the mornings. She is sometimes unsteady when walking and has to sit down to regain her balance. On two occasions she has had to sit down to keep from falling but denies symptoms of dizziness. She has arterial hypertension, urinary incontinence and osteoarthritis. Her treatment is hydrochlorothiazide (25 mg/d), oxybutynin (10 mg/d), lisinopril (10 mg/d), calcium (1500 mg/d) and paracetamol (3 g/d). On examination he has a blood pressure of 115/70 mm Hg, pulse 80 bpm. His movements are slow. He has a moderate hand tremor. She can get up from the chair slowly but without needing to lean on her arms. She walks slightly leaning forward with little swinging of the arms. Turns slowly but without losing balance. She is unable to stand on one foot. Which of the following possible actions would you perform first?", "full_answer": "The correct answer is to initiate treatment with L-dopa, since the clinical picture described is of a parkinsonism whose diagnosis is exclusively clinical. The rest of the answers can be made for differential diagnosis, but the performance of complementary tests does not justify delaying the initiation of treatment.", "type": "NEUROLOGY AND NEUROSURGERY", "options": {"1": "Assess visual acuity.", "2": "Perform an MRI.", "3": "Tilt table study.", "4": "Measure blood pressure lying down and standing up.", "5": "Therapeutic attempt with L-dopa."}, "correct_option": 5, "explanations": {"1": {"exist": true, "char_ranges": [[156, 317]], "word_ranges": [[23, 48]], "text": "The rest of the answers can be made for differential diagnosis, but the performance of complementary tests does not justify delaying the initiation of treatment."}, "2": {"exist": true, "char_ranges": [[156, 317]], "word_ranges": [[23, 48]], "text": "The rest of the answers can be made for differential diagnosis, but the performance of complementary tests does not justify delaying the initiation of treatment."}, "3": {"exist": true, "char_ranges": [[156, 317]], "word_ranges": [[23, 48]], "text": "The rest of the answers can be made for differential diagnosis, but the performance of complementary tests does not justify delaying the initiation of treatment."}, "4": {"exist": true, "char_ranges": [[156, 317]], "word_ranges": [[23, 48]], "text": "The rest of the answers can be made for differential diagnosis, but the performance of complementary tests does not justify delaying the initiation of treatment."}, "5": {"exist": true, "char_ranges": [[0, 155]], "word_ranges": [[0, 23]], "text": "The correct answer is to initiate treatment with L-dopa, since the clinical picture described is of a parkinsonism whose diagnosis is exclusively clinical."}}} {"id": 338, "year": 2016, "question_id_specific": 31, "full_question": "20-year-old woman with ovarian tumor of 15 cm, solid-cystic, detected by ultrasound after presenting with unspecific abdominal symptoms. In the histopathological study of the corresponding specimen, teeth, hairs, areas of intestinal epithelium, areas of squamous epithelium (15%) and bronchial epithelium, as well as neuroectodermal and embryonic elements are found in several of the histological preparations. In reference to this case, point out the correct diagnosis:", "full_answer": "Mature cystic teratoma (common in the ovary of the adult female). Mass with a large cyst occupying most of it; in its cavity there is sebaceous material and hairs. The wall, generally a few millimeters thick, has the structure of skin, with its epidermal surface towards the cavity. Because of this preponderant development of cutaneous structures it is usually called dermoid cyst. The tissues that compose the tumor are well differentiated (mature or adult type); apart from skin structures many other tissues can be observed, particularly in a thickening or spur that eminences into the cavity, in which there are often teeth, cartilage and bone.", "type": "GYNECOLOGY AND OBSTETRICS", "options": {"1": "Teratocarcinoma.", "2": "Immature teratoma.", "3": "Mature cystic teratoma.", "4": "Dysgerminoma.", "5": NaN}, "correct_option": 3, "explanations": {"1": {"exist": false, "char_ranges": [], "word_ranges": [], "text": ""}, "2": {"exist": false, "char_ranges": [], "word_ranges": [], "text": ""}, "3": {"exist": true, "char_ranges": [[383, 649]], "word_ranges": [[62, 105]], "text": "The tissues that compose the tumor are well differentiated (mature or adult type); apart from skin structures many other tissues can be observed, particularly in a thickening or spur that eminences into the cavity, in which there are often teeth, cartilage and bone."}, "4": {"exist": false, "char_ranges": [], "word_ranges": [], "text": ""}, "5": {"exist": false, "char_ranges": [], "word_ranges": [], "text": ""}}} {"id": 432, "year": 2018, "question_id_specific": 117, "full_question": "An 85-year-old man with a peripheral venous catheter who, one week after being hospitalized for a stroke, begins with shivering and fever. Blood cultures are performed and microbiology reports that gram-positive cocci are growing in clusters. Pending the antibiogram, what is the most appropriate antibiotic treatment?", "full_answer": "The most frequent cause of catheter infection is a bacterium that colonizes the skin. The Gram stain tells us that it is a staphylococcus, most of the species of this genus acquired in-hospital are methicillin resistant, so they are considered resistant to Cloxacillin, Cefazolin and there are no clinical data about the efficacy of linezolid for the treatment of systemic infection related to catheter. This, plus the experience with vancomycin use in this setting is considered the empirical choice for vancomycin.", "type": "INFECTIOUS DISEASES AND MICROBIOLOGY", "options": {"1": "Cefazolin.", "2": "Cloxacillin.", "3": "Vancomycin.", "4": "Linezolid.", "5": NaN}, "correct_option": 3, "explanations": {"1": {"exist": true, "char_ranges": [[86, 279]], "word_ranges": [[14, 44]], "text": "The Gram stain tells us that it is a staphylococcus, most of the species of this genus acquired in-hospital are methicillin resistant, so they are considered resistant to Cloxacillin, Cefazolin"}, "2": {"exist": true, "char_ranges": [[86, 279]], "word_ranges": [[14, 44]], "text": "The Gram stain tells us that it is a staphylococcus, most of the species of this genus acquired in-hospital are methicillin resistant, so they are considered resistant to Cloxacillin, Cefazolin"}, "3": {"exist": true, "char_ranges": [[415, 516]], "word_ranges": [[66, 81]], "text": "the experience with vancomycin use in this setting is considered the empirical choice for vancomycin."}, "4": {"exist": true, "char_ranges": [[284, 403]], "word_ranges": [[45, 64]], "text": "there are no clinical data about the efficacy of linezolid for the treatment of systemic infection related to catheter."}, "5": {"exist": false, "char_ranges": [], "word_ranges": [], "text": ""}}} {"id": 513, "year": 2021, "question_id_specific": 38, "full_question": "A 72-year-old patient presents to the emergency department with severe lingual angioedema. She has no associated urticaria and no history of drug or food allergy. His personal history includes type 2 diabetes mellitus, dyslipidemia, hypertension, hypothyroidism and Parkinson's disease. In regular treatment with metformin, simvastatin, enalapril, thyroxine and levodopa. Which of the following drugs is most likely the causal drug of the clinical picture described?", "full_answer": "Of the proposed drugs, the least likely options would be 3, since metformin very rarely causes lingual angioedema (in March 2020 there was only one case described worldwide) and 4, since, among the adverse reactions of levodopa, angioedema is classified among the very rare ones (frequency less than 0.1%). Of the two remaining options, the drugs most frequently and classically associated with lingual angioedema are ACE inhibitors (frequency around 0.2%), and their association with simvastatin is also less than 0.1%.", "type": "CRITICAL AND EMERGENCY CARE", "options": {"1": "Enalapril.", "2": "Simvastatin.", "3": "Metformin or levodopa equally.", "4": "Levodopa.", "5": NaN}, "correct_option": 1, "explanations": {"1": {"exist": true, "char_ranges": [[337, 457]], "word_ranges": [[54, 70]], "text": "the drugs most frequently and classically associated with lingual angioedema are ACE inhibitors (frequency around 0.2%),"}, "2": {"exist": true, "char_ranges": [[462, 520]], "word_ranges": [[71, 80]], "text": "their association with simvastatin is also less than 0.1%."}, "3": {"exist": true, "char_ranges": [[66, 173]], "word_ranges": [[12, 28]], "text": "metformin very rarely causes lingual angioedema (in March 2020 there was only one case described worldwide)"}, "4": {"exist": true, "char_ranges": [[188, 306]], "word_ranges": [[31, 49]], "text": "among the adverse reactions of levodopa, angioedema is classified among the very rare ones (frequency less than 0.1%)."}, "5": {"exist": false, "char_ranges": [], "word_ranges": [], "text": ""}}} {"id": 580, "year": 2022, "question_id_specific": 198, "full_question": "A 35-year-old woman referred to the emergency department for head trauma due to a fall on a scooter, with no loss of consciousness or amnesia. The examination shows a closed head injury with concussion and mild headache without nausea and vomiting. The neurological examination is normal and the Glasgow index is 15. In this situation, which imaging test is indicated first?", "full_answer": "After a TBI in a patient who is not anticoagulated or antiplatelet, with no known risk factors for hemorrhage, and who does not present alarming data in the anamnesis or physical examination, it is not necessary to perform a cranial imaging test.", "type": "NEUROLOGY", "options": {"1": "Cranial CT scan without contrast.", "2": "Cranial CT with contrast.", "3": "No imaging test.", "4": "Simple skull radiography.", "5": NaN}, "correct_option": 3, "explanations": {"1": {"exist": true, "char_ranges": [[0, 246]], "word_ranges": [[0, 42]], "text": "After a TBI in a patient who is not anticoagulated or antiplatelet, with no known risk factors for hemorrhage, and who does not present alarming data in the anamnesis or physical examination, it is not necessary to perform a cranial imaging test."}, "2": {"exist": true, "char_ranges": [[0, 246]], "word_ranges": [[0, 42]], "text": "After a TBI in a patient who is not anticoagulated or antiplatelet, with no known risk factors for hemorrhage, and who does not present alarming data in the anamnesis or physical examination, it is not necessary to perform a cranial imaging test."}, "3": {"exist": true, "char_ranges": [[0, 246]], "word_ranges": [[0, 42]], "text": "After a TBI in a patient who is not anticoagulated or antiplatelet, with no known risk factors for hemorrhage, and who does not present alarming data in the anamnesis or physical examination, it is not necessary to perform a cranial imaging test."}, "4": {"exist": true, "char_ranges": [[0, 246]], "word_ranges": [[0, 42]], "text": "After a TBI in a patient who is not anticoagulated or antiplatelet, with no known risk factors for hemorrhage, and who does not present alarming data in the anamnesis or physical examination, it is not necessary to perform a cranial imaging test."}, "5": {"exist": false, "char_ranges": [], "word_ranges": [], "text": ""}}} {"id": 248, "year": 2014, "question_id_specific": 118, "full_question": "A 34-year-old man consults for fever and malaise. The medical history includes a history of homosexual relations for the last 4 months with a new partner. 2 months before the current consultation he had a painless ulcerative lesion on the glans penis with bilateral inguinal lymphadenopathy, all self-limited. Serological studies were requested with the following results: HIV negative, RPR 1/320, HAART 1/128. What treatment would you indicate for this patient?", "full_answer": "Direct response question. This is syphilis (both by antecendent and serology) in the secondary stage, so treatment is with Benzathine Penicillin 2.4 MU in a single dose.", "type": "INFECTIOUS DISEASES", "options": {"1": "None.", "2": "Intravenous penicillin G, 24 MU every day for 14 days.", "3": "Benzathine penicillin 2.4 MU intramuscular, 3 doses in three consecutive weeks.", "4": "Ceftriaxone 2 grams intramuscular in a single dose.", "5": "Benzathine penicillin 2.4 MU intramuscularly in a single dose."}, "correct_option": 5, "explanations": {"1": {"exist": false, "char_ranges": [], "word_ranges": [], "text": ""}, "2": {"exist": false, "char_ranges": [], "word_ranges": [], "text": ""}, "3": {"exist": false, "char_ranges": [], "word_ranges": [], "text": ""}, "4": {"exist": false, "char_ranges": [], "word_ranges": [], "text": ""}, "5": {"exist": true, "char_ranges": [[26, 169]], "word_ranges": [[3, 27]], "text": "This is syphilis (both by antecendent and serology) in the secondary stage, so treatment is with Benzathine Penicillin 2.4 MU in a single dose."}}} {"id": 222, "year": 2014, "question_id_specific": 197, "full_question": "30-year-old HIV (+) homosexual male who participates as a volunteer in an AIDS patient support center. According to his medical history he received diphtheria toxoid (Td) 6 years ago, MMR vaccine in childhood and adolescence, and hepatitis B 3 years ago. He is currently asymptomatic with a CD4 count above 200 cls/microliter. Which vaccines should we recommend?", "full_answer": "Vaccination recommendations for HIV+ adults include hepatitis B, influenza, MMR, pneumococcal, Td and Tdap, and for some adults, hepatitis A, or combined A and B, bacterial meningitis, HPV and meningococcal (Source [1]). Considering that the individual has received some vaccines, has a CD4 count >100 (if lower there would be a contraindication) and works in a center that helps AIDS patients (he is equated to \"health personnel\") answers 2, 4 and 5 are wrong. All four vaccines in option 1 are administrable. In a little more detail: Vaccine against tetanus and diphtheria: all persons should be vaccinated against these two diseases. Hepatitis B vaccine: hepatitis B and HIV viruses share the same routes of transmission, so it is common for HIV-infected patients to also be infected with hepatitis B virus. It is therefore important to know whether the HIV-infected person has markers of hepatitis B infection and to proceed with vaccination if he/she does not have them. Vaccination against hepatitis A: Hepatitis A in a person with hepatitis B, hepatitis C or other liver disease can be very serious. Since these infections are more frequent in HIV-infected patients, vaccination against hepatitis A is recommended. Vaccination against influenza: influenza in a patient with a lack of immune response is more serious and presents a greater risk of complications, so annual vaccination against this disease is indicated. If the patient's immune status is very deteriorated, the response to vaccination is diminished. In this case, it is essential to vaccinate all the people living with the HIV-infected person to prevent them from transmitting the disease. Pneumococcal vaccine: pneumococcal infections (especially pneumonia) are up to 10 times more frequent in HIV-infected persons than in HIV-uninfected adults, so vaccination is recommended, although in patients with counts below 200/mm3 the protective response is usually not sufficient. They should receive a second dose at 3-5 years of age. HIV-infected children should be vaccinated with pneumococcal conjugate vaccine with the corresponding number of doses according to age. Vaccination against Haemophilus Influenzae type b: this bacterium causes pneumonia and meningitis, especially in children. Although in HIV-infected adults it represents a small number of the causes of infection, vaccination is recommended. Vaccination against measles, rubella and mumps (MMR): all three diseases can become more serious in HIV-infected patients, especially measles, so all of them should be vaccinated, provided that immunosuppression is not severe (count less than 200/mm3).", "type": "EPIDEMIOLOGY", "options": {"1": "Seasonal influenza, pneumococcal, tetravalent meningitis and hepatitis A.", "2": "Seasonal influenza, Td, pneumococcal and tetravalent meningitis.", "3": "Tetravalent meningitis, pneumococcal and seasonal flu.", "4": "Td, tetravalent meningitis, pneumococcal.", "5": "Triple viral, seasonal flu, pneumococcal."}, "correct_option": 1, "explanations": {"1": {"exist": true, "char_ranges": [[462, 1948]], "word_ranges": [[74, 300]], "text": "All four vaccines in option 1 are administrable. In a little more detail: Vaccine against tetanus and diphtheria: all persons should be vaccinated against these two diseases. Hepatitis B vaccine: hepatitis B and HIV viruses share the same routes of transmission, so it is common for HIV-infected patients to also be infected with hepatitis B virus. It is therefore important to know whether the HIV-infected person has markers of hepatitis B infection and to proceed with vaccination if he/she does not have them. Vaccination against hepatitis A: Hepatitis A in a person with hepatitis B, hepatitis C or other liver disease can be very serious. Since these infections are more frequent in HIV-infected patients, vaccination against hepatitis A is recommended. Vaccination against influenza: influenza in a patient with a lack of immune response is more serious and presents a greater risk of complications, so annual vaccination against this disease is indicated. If the patient's immune status is very deteriorated, the response to vaccination is diminished. In this case, it is essential to vaccinate all the people living with the HIV-infected person to prevent them from transmitting the disease. Pneumococcal vaccine: pneumococcal infections (especially pneumonia) are up to 10 times more frequent in HIV-infected persons than in HIV-uninfected adults, so vaccination is recommended, although in patients with counts below 200/mm3 the protective response is usually not sufficient."}, "2": {"exist": true, "char_ranges": [[221, 461]], "word_ranges": [[32, 74]], "text": "Considering that the individual has received some vaccines, has a CD4 count >100 (if lower there would be a contraindication) and works in a center that helps AIDS patients (he is equated to \"health personnel\") answers 2, 4 and 5 are wrong."}, "3": {"exist": false, "char_ranges": [], "word_ranges": [], "text": ""}, "4": {"exist": true, "char_ranges": [[221, 461]], "word_ranges": [[32, 74]], "text": "Considering that the individual has received some vaccines, has a CD4 count >100 (if lower there would be a contraindication) and works in a center that helps AIDS patients (he is equated to \"health personnel\") answers 2, 4 and 5 are wrong."}, "5": {"exist": true, "char_ranges": [[221, 461]], "word_ranges": [[32, 74]], "text": "Considering that the individual has received some vaccines, has a CD4 count >100 (if lower there would be a contraindication) and works in a center that helps AIDS patients (he is equated to \"health personnel\") answers 2, 4 and 5 are wrong."}}} {"id": 244, "year": 2014, "question_id_specific": 114, "full_question": "A 16-year-old boy consults for presenting with pustular tonsillitis, fever up to 38.5°C, painful cervical lymphadenopathy, non-pruritic macular rash on the chest and mild hepatosplenomegaly, of 4-5 days of evolution. Paul-Bunnell test and Ig M for Epstein-Barr virus are positive. During his admission he developed continuous fever up to 40ºC, pancytopenia, icteric hepatitis and coagulopathy of progressive intensity. One week after admission, he was transferred to the ICU due to confusion and respiratory failure. Blood cultures and a urine culture were negative, CSF was normal and the chest x-ray showed no infiltrates. Procalcitonin is normal, but CRP and ferritin are very elevated. Of the following statements, which would be the most correct diagnostic and therapeutic approach?", "full_answer": "Difficult question in which the correct answer could give rise to discussion. The patient is a 16-year-old male who is admitted for a fairly clear case of mononucleosis due to EBV infection that becomes complicated during admission. The data available to us are: - Pancytopenia. - Hepatopathy. - Coagulopathy. - Absence of infectious focus: negative blood cultures, negative urine cultures, normal X-ray, normal CSF. - CRP and Ferritin elevated / normal Procalcitonin. The possible complications that could have occurred (and that are considered in the answers) are sepsis or a bad evolution of mononucleosis. The clinical data could be consistent with a septic process, however, the main infectious foci have been reasonably ruled out and one of the most sensitive and specific markers of bacteremia (procalcitonin) is in the normal range, which should draw our attention. On the other hand, a complication of the EBV mononucleosis suffered by our patient should be evaluated, among the possible complications are hemolytic anemia, meningoecephalitis (normal CSF) or Guillain-Barré sd. Guillain-Barré syndrome, which are not very consistent with the clinical picture presented to us. Another possible complication of exceptional severity would be hemophagocytic syndrome; this is a rare entity that occurs in certain predisposed subjects and is characterized by an uncontrolled activation and proliferation of histiocytes and T lymphocytes, which produces a state of hypercytokinemia. The diagnostic criteria, revised in 2004, include persistent high fever, hepatosplenomegaly, cytopenias, hypertriglyceridemia, hyperferritinemia, and hypofibrinogenemia. Hyperferritinemia >3000 in a suggestive clinical context necessitates treatment of the patient (mainly with immunosuppressants and corticosteroids) since mortality is very high and early treatment is essential. The definitive diagnosis is made by microscopic study of an OM biopsy. Although in this question they do not quantify the value of ferritinemia, it is a remarkable data and given that the clinical picture is suggestive, meeting the diagnostic criteria, it seems to me that answer 5 would be the most accurate.", "type": "INFECTIOUS DISEASES", "options": {"1": "She has a bacterial sepsis of undetermined origin and should be given ceftriaxone and supportive care.", "2": "She has bacterial sepsis of undetermined origin and should be given vancomycin, ceftacidime and supportive care.", "3": "This is infectious mononucleosis with a severe course and glucocorticoids should be administered.", "4": "This is an infectious mononucleosis of severe course and treatment with Acyclovir should be initiated.", "5": "I would perform a bone marrow biopsy/aspirate and, if hemophagocytosis is confirmed, initiate treatment with immunosuppressants."}, "correct_option": 5, "explanations": {"1": {"exist": true, "char_ranges": [[610, 873]], "word_ranges": [[94, 136]], "text": "The clinical data could be consistent with a septic process, however, the main infectious foci have been reasonably ruled out and one of the most sensitive and specific markers of bacteremia (procalcitonin) is in the normal range, which should draw our attention."}, "2": {"exist": true, "char_ranges": [[610, 873]], "word_ranges": [[94, 136]], "text": "The clinical data could be consistent with a septic process, however, the main infectious foci have been reasonably ruled out and one of the most sensitive and specific markers of bacteremia (procalcitonin) is in the normal range, which should draw our attention."}, "3": {"exist": true, "char_ranges": [[874, 1184]], "word_ranges": [[136, 180]], "text": "On the other hand, a complication of the EBV mononucleosis suffered by our patient should be evaluated, among the possible complications are hemolytic anemia, meningoecephalitis (normal CSF) or Guillain-Barré sd. Guillain-Barré syndrome, which are not very consistent with the clinical picture presented to us."}, "4": {"exist": true, "char_ranges": [[874, 1184]], "word_ranges": [[136, 180]], "text": "On the other hand, a complication of the EBV mononucleosis suffered by our patient should be evaluated, among the possible complications are hemolytic anemia, meningoecephalitis (normal CSF) or Guillain-Barré sd. Guillain-Barré syndrome, which are not very consistent with the clinical picture presented to us."}, "5": {"exist": true, "char_ranges": [[1657, 1938]], "word_ranges": [[237, 276]], "text": "Hyperferritinemia >3000 in a suggestive clinical context necessitates treatment of the patient (mainly with immunosuppressants and corticosteroids) since mortality is very high and early treatment is essential. The definitive diagnosis is made by microscopic study of an OM biopsy."}}} {"id": 420, "year": 2018, "question_id_specific": 78, "full_question": "A 52-year-old man was referred to the gastroenterology department for hematochezia, tenesmus and reduction of stool diameter. A series of tests were performed and a diagnosis of adenocarcinoma of the sigma without distant metastasis was made. The patient underwent surgery and was referred to the medical oncology department for evaluation of complementary chemotherapy treatment. Which of the following is a poor prognostic factor after surgical resection and should be taken into account when planning chemotherapy treatment?", "full_answer": "They are not asking about factors that affect oncologic prognosis and therefore imply a change in QT strategy. This question is straightforward. Option 4 is describing to us a T4 stage, either by invasion of adjacent organs or perforation. The T4s carry a much higher chance of recurrence, in fact there are ongoing studies that propose prophylactic HIPEC in T4 and second look surgery + HIPEC in perforated colon tumors.", "type": "GENERAL SURGERY", "options": {"1": "The presence of anemia at diagnosis.", "2": "The existence of a family history of colorectal cancer.", "3": "The size of the primary lesion and histological differentiation.", "4": "Perforation or adhesion of the tumor to adjacent organs.", "5": NaN}, "correct_option": 4, "explanations": {"1": {"exist": false, "char_ranges": [], "word_ranges": [], "text": ""}, "2": {"exist": false, "char_ranges": [], "word_ranges": [], "text": ""}, "3": {"exist": false, "char_ranges": [], "word_ranges": [], "text": ""}, "4": {"exist": true, "char_ranges": [[145, 421]], "word_ranges": [[22, 70]], "text": "Option 4 is describing to us a T4 stage, either by invasion of adjacent organs or perforation. The T4s carry a much higher chance of recurrence, in fact there are ongoing studies that propose prophylactic HIPEC in T4 and second look surgery + HIPEC in perforated colon tumors."}, "5": {"exist": false, "char_ranges": [], "word_ranges": [], "text": ""}}} {"id": 321, "year": 2016, "question_id_specific": 144, "full_question": "A 50-year-old man diagnosed with polyangiitis with granulomatosis 10 years earlier and untreated for the past 5 years is asymptomatic at a scheduled check-up. Chest X-ray, blood tests and urine sediment are normal except for positive anti-neutrophil cytoplasmic antibodies (ANCA) at titer 1/320, with anti-proteinase 3 specificity, which had previously been negative. What is the most advisable therapeutic approach?", "full_answer": "In ANCA vasculitis, whatever it is, these antibodies have been shown to be related to the activity of the disease, but at no time are changes in treatment obligatory due to their levels. In this case, with the patient being asymptomatic and with objective evidence of lack of activity, close follow-up should be performed without restarting or modifying treatment.", "type": "RHEUMATOLOGY", "options": {"1": "Start treatment with corticosteroids.", "2": "Initiate treatment with cyclophosphamide.", "3": "Initiate treatment with mycophenolate mofetil.", "4": "Watchful waiting.", "5": NaN}, "correct_option": 4, "explanations": {"1": {"exist": true, "char_ranges": [[0, 186]], "word_ranges": [[0, 33]], "text": "In ANCA vasculitis, whatever it is, these antibodies have been shown to be related to the activity of the disease, but at no time are changes in treatment obligatory due to their levels."}, "2": {"exist": true, "char_ranges": [[0, 186]], "word_ranges": [[0, 33]], "text": "In ANCA vasculitis, whatever it is, these antibodies have been shown to be related to the activity of the disease, but at no time are changes in treatment obligatory due to their levels."}, "3": {"exist": true, "char_ranges": [[0, 186]], "word_ranges": [[0, 33]], "text": "In ANCA vasculitis, whatever it is, these antibodies have been shown to be related to the activity of the disease, but at no time are changes in treatment obligatory due to their levels."}, "4": {"exist": true, "char_ranges": [[201, 364]], "word_ranges": [[36, 59]], "text": "with the patient being asymptomatic and with objective evidence of lack of activity, close follow-up should be performed without restarting or modifying treatment."}, "5": {"exist": false, "char_ranges": [], "word_ranges": [], "text": ""}}} {"id": 516, "year": 2021, "question_id_specific": 103, "full_question": "A 56-year-old patient is found comatose at home. He has a history of arterial hypertension and diabetes mellitus. He has no toxic habits or any other chronic disease. He is on treatment with irbesartan and empaglifozin. He shows no signs of malnutrition. His blood pressure is 110/60 mmHg, heart rate 110 bpm, 90% SatO2, capillary glucose 120 mg/dl and respiratory rate 7 rpm. Which initial therapeutic approach do you think is the most correct?", "full_answer": "We find a patient with hypoventilation. Glycemia is correct (we rule out hyperosmolar coma and hypoglycemia, option 2 and 4), so, although there is no history of intoxication, we have to go to the most frequent and that we can antagonize with antidotes: benzodiazepines (flumazenil) and opiates (naloxone). If we suspect ethylene glycol intoxication, we would administer thiamine, but there are no data to make us suspect.", "type": "CRITICAL AND EMERGENCY CARE", "options": {"1": "Administer naloxone, flumazenil and thiamine.", "2": "Administer naloxone, flumazenil and hypertonic glucose.", "3": "Administer naloxone and flumazenil.", "4": "Administer thiamine and hypertonic glucose.", "5": NaN}, "correct_option": 3, "explanations": {"1": {"exist": false, "char_ranges": [], "word_ranges": [], "text": ""}, "2": {"exist": false, "char_ranges": [], "word_ranges": [], "text": ""}, "3": {"exist": true, "char_ranges": [[130, 306]], "word_ranges": [[21, 48]], "text": "although there is no history of intoxication, we have to go to the most frequent and that we can antagonize with antidotes: benzodiazepines (flumazenil) and opiates (naloxone)."}, "4": {"exist": true, "char_ranges": [[307, 422]], "word_ranges": [[48, 67]], "text": "If we suspect ethylene glycol intoxication, we would administer thiamine, but there are no data to make us suspect."}, "5": {"exist": false, "char_ranges": [], "word_ranges": [], "text": ""}}} {"id": 44, "year": 2011, "question_id_specific": 153, "full_question": "Child 28 days old, born at term. History of normal pregnancy and delivery. He has been breastfed since birth. He consults because 8 days ago he started vomiting, initially sporadically and since 5 days ago, after all feedings. He is permanently hungry. The vomiting is of \"squirting\" food content. On palpation, the abdomen is soft and depressible, without visceromegaly. The blood gas analysis showed pH 7.49, bicarbonate 30 mEq/l, pCO2 53 mmHg, base excess +8 mEq/l. Ions: Na 137 mEq/l, K 3.1 mEq/l, Cl 94 mEq/l. Taking into account the most probable diagnosis in the clinical picture described, what is the complementary test of choice to confirm this clinical diagnosis?", "full_answer": "The correct answer is 3. Easy question if pathology is suspected, which I also think is simple because the clinical description is textbook hypertrophic pyloric stenosis.", "type": "PEDIATRICS", "options": {"1": "Simple abdominal X-ray.", "2": "pHmetry.", "3": "Abdominal ultrasound.", "4": "Determination of electrolytes in sweat.", "5": "Esophagogastroscopy."}, "correct_option": 3, "explanations": {"1": {"exist": false, "char_ranges": [], "word_ranges": [], "text": ""}, "2": {"exist": false, "char_ranges": [], "word_ranges": [], "text": ""}, "3": {"exist": true, "char_ranges": [[103, 170]], "word_ranges": [[18, 26]], "text": "the clinical description is textbook hypertrophic pyloric stenosis."}, "4": {"exist": false, "char_ranges": [], "word_ranges": [], "text": ""}, "5": {"exist": false, "char_ranges": [], "word_ranges": [], "text": ""}}} {"id": 54, "year": 2011, "question_id_specific": 72, "full_question": "A 22-year-old female student with no history of pathology or use of medications except anovulatory drugs comes to the emergency department for deterioration of general condition and need to take deep breaths. She reports weight loss in the last 2-3 days, polydipsia, polyuria and nausea. No cough or febrile sensation. Examination: appearance of gravity, blood pressure 100/60 mmHg, deep and rapid breathing (28rpm), level of consciousness preserved, dry mucous membranes. No fever. CBC: blood glucose 420 mg/dL, Na+ 131 mEq/L, K+ normal, pH 7.08, bicarbonate 8 mEq/L and ketonuria (+++) Which answer do you think is more correct?", "full_answer": "Debut of DM with ketoacidosis. Priority is iv insulin administration and serum therapy. Bicarbonate if pH < 7. Insulin administration should never be delayed.", "type": "ENDOCRINOLOGY", "options": {"1": "Debut of type 2 diabetes mellitus, with dyspnea probably due to pneumonia or thromboembolism, since she is taking anovulatory drugs.", "2": "It is a diabetic ketoacidosis. It must be treated with intravenous insulin, serum therapy, general measures and search for precipitating cause.", "3": "Debut of type 1 diabetes with ketoacidosis. Treat with bicarbonate and after correction of acidosis, add intravenous insulin.", "4": "It looks like diabetic ketoacidosis, but it could be alcoholic. BAC should be determined before starting insulin therapy.", "5": "Treat with rapid subcutaneous insulin, serum therapy and ask him to drink plenty of fluids."}, "correct_option": 2, "explanations": {"1": {"exist": false, "char_ranges": [], "word_ranges": [], "text": ""}, "2": {"exist": true, "char_ranges": [[31, 158]], "word_ranges": [[5, 24]], "text": "Priority is iv insulin administration and serum therapy. Bicarbonate if pH < 7. Insulin administration should never be delayed."}, "3": {"exist": false, "char_ranges": [], "word_ranges": [], "text": ""}, "4": {"exist": false, "char_ranges": [], "word_ranges": [], "text": ""}, "5": {"exist": false, "char_ranges": [], "word_ranges": [], "text": ""}}} {"id": 553, "year": 2022, "question_id_specific": 177, "full_question": "33-year-old patient, resident of Valencia, who consults for presenting on the right cheek an erythematous-orange plaque after an insect bite that has been growing slowly until reaching 1.5 cm and that in the last weeks has ulcerated and covered with a scammocostra. The lesion only causes discrete discomfort if scratched. The patient has a history of plaque psoriasis treated with subcutaneous adalimumab. She is in good general condition and does not take any other drugs and does not report any drug allergies. Indicate the treatment you would use:", "full_answer": "The fact that the place of residence is mentioned already gives us many clues. A \"bite\" that transforms into a lesion in an uncovered area with a crusty surface and that bothers little or nothing... in the Mediterranean basin this is cutaneous leishmaniasis or oriental button. And while it is true that the lesion is < 4 cm and unique, we are told that the patient is being treated with adalimumab, an anti-TNF monoclonal antibody. With the exception of meglumine antimoniate, none of the other treatments proposed in the different options are indicated in leishmaniasis, so it is clear. The only thing that is more debatable is that, as this is an immunosuppressed patient, systemic treatment would probably be more appropriate. I don't know if for that reason contestable, but at least, debatable.", "type": "DERMATOLOGY", "options": {"1": "Oral isotretinoin.", "2": "Intralesional meglumine antimoniate.", "3": "Systemic corticosteroids.", "4": "Oral amoxicillin.", "5": NaN}, "correct_option": 2, "explanations": {"1": {"exist": true, "char_ranges": [[433, 588]], "word_ranges": [[75, 99]], "text": "With the exception of meglumine antimoniate, none of the other treatments proposed in the different options are indicated in leishmaniasis, so it is clear."}, "2": {"exist": true, "char_ranges": [[433, 588]], "word_ranges": [[75, 99]], "text": "With the exception of meglumine antimoniate, none of the other treatments proposed in the different options are indicated in leishmaniasis, so it is clear."}, "3": {"exist": true, "char_ranges": [[433, 588]], "word_ranges": [[75, 99]], "text": "With the exception of meglumine antimoniate, none of the other treatments proposed in the different options are indicated in leishmaniasis, so it is clear."}, "4": {"exist": true, "char_ranges": [[433, 588]], "word_ranges": [[75, 99]], "text": "With the exception of meglumine antimoniate, none of the other treatments proposed in the different options are indicated in leishmaniasis, so it is clear."}, "5": {"exist": false, "char_ranges": [], "word_ranges": [], "text": ""}}} {"id": 608, "year": 2022, "question_id_specific": 115, "full_question": "A 27-year-old male, a regular athlete, refers pain in the right leg after continuous running. He has visited a physiotherapist on several occasions and has been diagnosed with calf overload. Several months have passed, he has not improved and refers intense pain after physical activity that subsides with rest in the following hours of exercise. What test can help in the diagnosis?", "full_answer": "We present a case of chronic compartment syndrome. For its diagnosis it is necessary to measure the pressure of the affected compartments at rest and after activity (answer 2 correct).", "type": "TRAUMATOLOGY", "options": {"1": "Positron emission tomography with 18 FDG.", "2": "Determination of posterior compartment pressure immediately after activity.", "3": "Doppler ultrasound to rule out a circulatory disorder of the lower extremity.", "4": "Spectrometry by magnetic resonance.", "5": NaN}, "correct_option": 2, "explanations": {"1": {"exist": false, "char_ranges": [], "word_ranges": [], "text": ""}, "2": {"exist": true, "char_ranges": [[0, 184]], "word_ranges": [[0, 30]], "text": "We present a case of chronic compartment syndrome. For its diagnosis it is necessary to measure the pressure of the affected compartments at rest and after activity (answer 2 correct)."}, "3": {"exist": false, "char_ranges": [], "word_ranges": [], "text": ""}, "4": {"exist": false, "char_ranges": [], "word_ranges": [], "text": ""}, "5": {"exist": false, "char_ranges": [], "word_ranges": [], "text": ""}}} {"id": 294, "year": 2016, "question_id_specific": 44, "full_question": "Amalia is a 3-year-old girl who has undergone surgery for tetralogy of Fallot. She also presented thymic dysfunction in the neonatal period and hypocalcemia as well as a nasal voice and psychomotor retardation. Which genetic test do you consider most appropriate to reach an etiological diagnosis?", "full_answer": "Question in which the clinical picture must be recognized: cardiovascular anomalies - Tetralogy of Fallot -; thymic aplasia/hypoplasia - leading to primary T-cell immunodeficiency; parathyroid aplasia/hypoplasia - hence hypocalcemia -; palatal anomalies - nasal voice; psychomotor retardation. In my opinion, just with the words \"thymic dysfunction\", hypocalcemia and cardiopathy, we would have to go straight to DiGeorge syndrome. Indeed, the genetic defect is a 22q11.2 deletion (which if we know it, the better, but with the clinic it would be enough for us). The only option with which there could be doubt is Noonan syndrome, which can also be associated with cardiopathy, but above all it is characterized by short stature, cardiopathy and bone alterations; moreover, in most cases it is caused by an alteration in the PTPN11 gene, not in those mentioned in the question.", "type": "GENETICS AND IMMUNOLOGY", "options": {"1": "22q11 deletion study (Di George or Velo-cardio-facial syndrome).", "2": "Study of FMR1 gene expansion, responsible for Fragile X syndrome.", "3": "Sequencing of genes related to HRAS (Noonan syndrome).", "4": "Genetic study of Williams Beuren syndrome.", "5": NaN}, "correct_option": 1, "explanations": {"1": {"exist": true, "char_ranges": [[309, 431]], "word_ranges": [[40, 58]], "text": "just with the words \"thymic dysfunction\", hypocalcemia and cardiopathy, we would have to go straight to DiGeorge syndrome."}, "2": {"exist": false, "char_ranges": [], "word_ranges": [], "text": ""}, "3": {"exist": true, "char_ranges": [[637, 763]], "word_ranges": [[96, 115]], "text": "can also be associated with cardiopathy, but above all it is characterized by short stature, cardiopathy and bone alterations;"}, "4": {"exist": false, "char_ranges": [], "word_ranges": [], "text": ""}, "5": {"exist": false, "char_ranges": [], "word_ranges": [], "text": ""}}} {"id": 156, "year": 2012, "question_id_specific": 230, "full_question": "A 40-year-old patient who comes to the emergency room with aphthous stomatitis, conjunctivitis, urethritis, balanitis and joint pain. Which of the following diagnoses is the most likely?", "full_answer": "Given this clinical picture and without having complementary tests available, the most likely diagnosis is reactive arthritis. The only problem is that the statement speaks of joint pain and not arthritis... Reiter's syndrome (triad of conjunctivitis, urethritis and arthritis) is one of the forms in which reactive arthritis can present. In Behçet's disease oral and genital aphthous ulcers may occur but urethritis is not a feature. And the characteristic ocular condition is uveitis.", "type": "RHEUMATOLOGY", "options": {"1": "Behçet's disease.", "2": "Reactive arthritis syndrome.", "3": "Sweet's syndrome.", "4": "Vitamin A deficiency.", "5": "Gonococcal infection."}, "correct_option": 2, "explanations": {"1": {"exist": true, "char_ranges": [[339, 434]], "word_ranges": [[50, 66]], "text": "In Behçet's disease oral and genital aphthous ulcers may occur but urethritis is not a feature."}, "2": {"exist": true, "char_ranges": [[208, 338]], "word_ranges": [[31, 50]], "text": "Reiter's syndrome (triad of conjunctivitis, urethritis and arthritis) is one of the forms in which reactive arthritis can present."}, "3": {"exist": false, "char_ranges": [], "word_ranges": [], "text": ""}, "4": {"exist": false, "char_ranges": [], "word_ranges": [], "text": ""}, "5": {"exist": false, "char_ranges": [], "word_ranges": [], "text": ""}}} {"id": 82, "year": 2012, "question_id_specific": 45, "full_question": "A 60-year-old man reports precordial pain after medium exertion. Coronary angiography shows significant stenosis in the proximal segments of the three main vessels with good distal bed. Ventricular function is depressed (<30%). What is the best therapeutic option?", "full_answer": "We are talking about a patient with 3-vessel coronary artery disease and, very importantly, with severe ventricular dysfunction. Under these conditions, the treatment of choice would be surgical (myocardial revascularization surgery or by-pass). In the case of a patient with preserved LV function, the choice of surgical vs. percutaneous treatment would be more questionable, since today, thanks to drug-eluting stents, the differences in terms of morbidity and mortality between by-pass surgery and angioplasty are minimal.", "type": "CARDIOLOGY AND VASCULAR SURGERY", "options": {"1": "Percutaneous revascularization.", "2": "Medical treatment.", "3": "Myocardial revascularization surgery.", "4": "Implantation of balloon counterpulsation.", "5": "Cardiac transplantation."}, "correct_option": 3, "explanations": {"1": {"exist": false, "char_ranges": [], "word_ranges": [], "text": ""}, "2": {"exist": false, "char_ranges": [], "word_ranges": [], "text": ""}, "3": {"exist": true, "char_ranges": [[0, 245]], "word_ranges": [[0, 33]], "text": "We are talking about a patient with 3-vessel coronary artery disease and, very importantly, with severe ventricular dysfunction. Under these conditions, the treatment of choice would be surgical (myocardial revascularization surgery or by-pass)."}, "4": {"exist": false, "char_ranges": [], "word_ranges": [], "text": ""}, "5": {"exist": false, "char_ranges": [], "word_ranges": [], "text": ""}}} {"id": 485, "year": 2020, "question_id_specific": 49, "full_question": "A 60-year-old man on carbamazepine treatment for epilepsy presenting with fever (38.8°C), odynophagia, conjunctivitis, brownish skin lesions spread over a large body surface, target-like in appearance and accompanied by epidermal detachment (greater than 30%) at the slightest friction. It is very likely that she is presenting a clinical picture of:", "full_answer": "We are dealing with a severe patient, with target lesions and Nikolsky's sign in > 30% of the body surface area. Together with the history of treatment with carbamazepine, the suspicion will be of toxic epidermal necrolysis (answer 4). If the epidermal detachment is less than 10% then we would be dealing with Stevens-Johnson (and intermediate cases are referred to as overlap, since it is a whole spectrum). Incidentally, we would recommend immediate admission to a major burn unit.", "type": "DERMATOLOGY", "options": {"1": "Acute urticaria of pharmacological cause.", "2": "Erythema minor exudative secondary to drugs.", "3": "Pharmacological Stevens-Johnson syndrome.", "4": "Epidermal toxic necrolysis.", "5": NaN}, "correct_option": 4, "explanations": {"1": {"exist": false, "char_ranges": [], "word_ranges": [], "text": ""}, "2": {"exist": false, "char_ranges": [], "word_ranges": [], "text": ""}, "3": {"exist": true, "char_ranges": [[236, 326]], "word_ranges": [[39, 54]], "text": "If the epidermal detachment is less than 10% then we would be dealing with Stevens-Johnson"}, "4": {"exist": true, "char_ranges": [[0, 235]], "word_ranges": [[0, 39]], "text": "We are dealing with a severe patient, with target lesions and Nikolsky's sign in > 30% of the body surface area. Together with the history of treatment with carbamazepine, the suspicion will be of toxic epidermal necrolysis (answer 4)."}, "5": {"exist": false, "char_ranges": [], "word_ranges": [], "text": ""}}} {"id": 219, "year": 2014, "question_id_specific": 124, "full_question": "A 72-year-old woman with type 2 diabetes and stage 5 chronic renal insufficiency. Given the suspicion of pulmonary thromboembolism, indicate which diagnostic test is contraindicated:", "full_answer": "This is a very simple question and very useful for the management of PTE in the ED. There is no need to worry: a patient with advanced renal failure (stage 5) is contraindicated for any imaging study that requires the administration of IV contrast, such as CT angiography. All other examinations do not require it and therefore would be more than indicated in the presence of a PTE.", "type": "NEPHROLOGY", "options": {"1": "Disordered echocardiogram.", "2": "Pulmonary scintigraphy.", "3": "Pulmonary angio-CT.", "4": "Electrocardiogram.", "5": "Chest X-ray."}, "correct_option": 3, "explanations": {"1": {"exist": false, "char_ranges": [], "word_ranges": [], "text": ""}, "2": {"exist": false, "char_ranges": [], "word_ranges": [], "text": ""}, "3": {"exist": true, "char_ranges": [[111, 272]], "word_ranges": [[23, 48]], "text": "a patient with advanced renal failure (stage 5) is contraindicated for any imaging study that requires the administration of IV contrast, such as CT angiography."}, "4": {"exist": false, "char_ranges": [], "word_ranges": [], "text": ""}, "5": {"exist": false, "char_ranges": [], "word_ranges": [], "text": ""}}} {"id": 578, "year": 2022, "question_id_specific": 107, "full_question": "A 55-year-old male who suffers a subarachnoid hemorrhage due to a ruptured middle cerebral artery aneurysm. He is treated surgically. After a good evolution in which he recovers the ability to walk, one month after suffering the hemorrhage, he presents a progressive deterioration, becoming cloudy and with difficulty to obey orders. Mark the correct statement:", "full_answer": "Vasospasm as a complication of subarachnoid hemorrhage usually occurs early, in the week after the hemorrhage. The progressive deterioration with alteration of the level of consciousness makes us suspect one of the complications that can occur in this condition: hydrocephalus. The diagnostic technique of choice in this case is neuroimaging with cranial CT.", "type": "NEUROLOGY", "options": {"1": "She is in the moment of evolution in which vasospasm is more frequent.", "2": "This progressive deterioration is probably secondary to aneurysm rebleeding.", "3": "Subacute deterioration is most likely due to hydrocephalus.", "4": "The diagnostic technique of choice is cerebral angiography.", "5": NaN}, "correct_option": 3, "explanations": {"1": {"exist": false, "char_ranges": [], "word_ranges": [], "text": ""}, "2": {"exist": false, "char_ranges": [], "word_ranges": [], "text": ""}, "3": {"exist": true, "char_ranges": [[0, 277]], "word_ranges": [[0, 40]], "text": "Vasospasm as a complication of subarachnoid hemorrhage usually occurs early, in the week after the hemorrhage. The progressive deterioration with alteration of the level of consciousness makes us suspect one of the complications that can occur in this condition: hydrocephalus."}, "4": {"exist": false, "char_ranges": [], "word_ranges": [], "text": ""}, "5": {"exist": false, "char_ranges": [], "word_ranges": [], "text": ""}}} {"id": 199, "year": 2013, "question_id_specific": 74, "full_question": "In a patient presenting in the second decade of life with a progressive picture of parkinsonism, tremor, dystonia and behavioral alteration, you should always perform a study to rule out the presence of a disease whose appropriate treatment can improve neurological symptoms and stop the clinical course. What disease is it?", "full_answer": "The key in the question is \"stopping the clinical course\", as Hungtinton's disease, Tourette's and Parkinson's disease do not have treatments that can stop the clinical course. Sydenham's chorea is typical of younger children with a history of streptococcal infection and resolution is usually spontaneous. On the other hand, in clinical cases it is recommended to rule out Wilson's disease in young patients with movement and neuropsychiatric disorders.", "type": "NEUROLOGY", "options": {"1": "Huntington's disease.", "2": "Wilson's disease.", "3": "Tourette's syndrome.", "4": "Sydenham's chorea.", "5": "Juvenile-onset Parkinson's disease."}, "correct_option": 2, "explanations": {"1": {"exist": true, "char_ranges": [[0, 176]], "word_ranges": [[0, 27]], "text": "The key in the question is \"stopping the clinical course\", as Hungtinton's disease, Tourette's and Parkinson's disease do not have treatments that can stop the clinical course."}, "2": {"exist": true, "char_ranges": [[326, 454]], "word_ranges": [[49, 68]], "text": "in clinical cases it is recommended to rule out Wilson's disease in young patients with movement and neuropsychiatric disorders."}, "3": {"exist": true, "char_ranges": [[0, 176]], "word_ranges": [[0, 27]], "text": "The key in the question is \"stopping the clinical course\", as Hungtinton's disease, Tourette's and Parkinson's disease do not have treatments that can stop the clinical course."}, "4": {"exist": true, "char_ranges": [[177, 306]], "word_ranges": [[27, 45]], "text": "Sydenham's chorea is typical of younger children with a history of streptococcal infection and resolution is usually spontaneous."}, "5": {"exist": true, "char_ranges": [[0, 176]], "word_ranges": [[0, 27]], "text": "The key in the question is \"stopping the clinical course\", as Hungtinton's disease, Tourette's and Parkinson's disease do not have treatments that can stop the clinical course."}}} {"id": 331, "year": 2016, "question_id_specific": 87, "full_question": "In a hospitalized patient who is unable to feed orally for more than 6 days, in which of the following clinical situations are we necessarily to use parenteral nutrition?", "full_answer": "Paralytic ileus is a failure of intestinal propulsion that appears acutely in the absence of mechanical obstruction; it is secondary to various causes and affects intestinal contractility. Therefore, parenteral administration is necessary.", "type": "ENDOCRINOLOGY", "options": {"1": "Cardioembolic stroke with complete neurologic dysphagia.", "2": "Cachexia due to chronic empyema in immunocompromised patient.", "3": "Prolonged paralytic syndrome.", "4": "Advanced Alzheimer's disease with severe risk of bronchial aspiration.", "5": NaN}, "correct_option": 3, "explanations": {"1": {"exist": false, "char_ranges": [], "word_ranges": [], "text": ""}, "2": {"exist": false, "char_ranges": [], "word_ranges": [], "text": ""}, "3": {"exist": true, "char_ranges": [[0, 239]], "word_ranges": [[0, 32]], "text": "Paralytic ileus is a failure of intestinal propulsion that appears acutely in the absence of mechanical obstruction; it is secondary to various causes and affects intestinal contractility. Therefore, parenteral administration is necessary."}, "4": {"exist": false, "char_ranges": [], "word_ranges": [], "text": ""}, "5": {"exist": false, "char_ranges": [], "word_ranges": [], "text": ""}}} {"id": 205, "year": 2014, "question_id_specific": 214, "full_question": "A 24-year-old young man who 3 days after a risky sexual contact presents numerous pustular lesions, small, very pruritic and evolving to tiny erosions affecting the entire glans and the inner face of the foreskin. Which of the following is the most likely diagnostic approach?", "full_answer": "Chancroid is a solitary lesion. Secondary syphilis usually occurs as a generalized rash or as characteristic lesions on palms and soles (syphilitic nails) with a longer latency period. Trichomonas balanitis is always associated with urethritis, which is not referred to in the question. Therefore, the answer is 1. However, the clinical picture suggests herpes simplex rather than candidiasis and 5 also refers to fungi (although dermatophytes do not cause balanitis).... from our point of view, even if we consider 1 to be correct, the question is misleading and could be open to challenge. Moreover, genital candidiasis is not necessarily sexually transmitted.", "type": "DERMATOLOGY", "options": {"1": "Genital candidiasis.", "2": "Chancroid.", "3": "Secondary syphilis.", "4": "Trichomonas balanitis.", "5": "Dermatophyte fungal infection."}, "correct_option": 1, "explanations": {"1": {"exist": false, "char_ranges": [], "word_ranges": [], "text": ""}, "2": {"exist": true, "char_ranges": [[0, 31]], "word_ranges": [[0, 5]], "text": "Chancroid is a solitary lesion."}, "3": {"exist": true, "char_ranges": [[32, 184]], "word_ranges": [[5, 28]], "text": "Secondary syphilis usually occurs as a generalized rash or as characteristic lesions on palms and soles (syphilitic nails) with a longer latency period."}, "4": {"exist": true, "char_ranges": [[185, 286]], "word_ranges": [[28, 43]], "text": "Trichomonas balanitis is always associated with urethritis, which is not referred to in the question."}, "5": {"exist": false, "char_ranges": [], "word_ranges": [], "text": ""}}} {"id": 441, "year": 2018, "question_id_specific": 98, "full_question": "A 21-year-old woman comes for consultation with a hormonal study showing FSH 0.29 mUl/mL (normal in follicular phase = 3.50-12.50 mUl/mL) and LH < 0.1 mUl/mL (normal in follicular phase 2.40-12.60 mUl/mL). We should make a differential diagnosis between the following clinical situations EXCEPT:", "full_answer": "This is because in early ovarian failure FSH and LH are elevated. This is confirmed by the SEGO protocol called Early Menoguia. The rest of the diagnoses are consistent with these analytical parameters.", "type": "GYNECOLOGY AND OBSTETRICS", "options": {"1": "Administration of combined oral contraceptives.", "2": "Premature ovarian failure.", "3": "Eating behavior disorder.", "4": "Craniopharyngioma.", "5": NaN}, "correct_option": 2, "explanations": {"1": {"exist": true, "char_ranges": [[128, 202]], "word_ranges": [[22, 33]], "text": "The rest of the diagnoses are consistent with these analytical parameters."}, "2": {"exist": true, "char_ranges": [[0, 127]], "word_ranges": [[0, 22]], "text": "This is because in early ovarian failure FSH and LH are elevated. This is confirmed by the SEGO protocol called Early Menoguia."}, "3": {"exist": true, "char_ranges": [[128, 202]], "word_ranges": [[22, 33]], "text": "The rest of the diagnoses are consistent with these analytical parameters."}, "4": {"exist": true, "char_ranges": [[128, 202]], "word_ranges": [[22, 33]], "text": "The rest of the diagnoses are consistent with these analytical parameters."}, "5": {"exist": false, "char_ranges": [], "word_ranges": [], "text": ""}}} {"id": 302, "year": 2016, "question_id_specific": 111, "full_question": "In a patient with rheumatoid arthritis treated with methotrexate, prednisone and indomethacin who acutely presents edema and increased plasma creatinine with a poorly expressive urine sediment and proteinuria less than 100 mg/24 h the most likely cause is:", "full_answer": "Prerenal acute renal failure due to NSAIDs seems the most plausible option in this case. Tubulointerstitial nephritis due to methotrexate is less likely due to the absence of rash, fever or eosinophilia, which although rare in the MIR usually appear. Although GMN and renal amyloidosis are relatively frequent in RA, the anodyne urine sediment and the absence of proteinuria make these options unlikely.", "type": "NEPHROLOGY", "options": {"1": "Renal amyloidosis.", "2": "Glomerulonephritis secondary to rheumatoid arthritis.", "3": "Renal failure due to non-steroidal anti-inflammatory drugs.", "4": "Interstitial nephritis due to methotrexate.", "5": NaN}, "correct_option": 3, "explanations": {"1": {"exist": true, "char_ranges": [[251, 403]], "word_ranges": [[40, 63]], "text": "Although GMN and renal amyloidosis are relatively frequent in RA, the anodyne urine sediment and the absence of proteinuria make these options unlikely."}, "2": {"exist": false, "char_ranges": [], "word_ranges": [], "text": ""}, "3": {"exist": true, "char_ranges": [[0, 88]], "word_ranges": [[0, 15]], "text": "Prerenal acute renal failure due to NSAIDs seems the most plausible option in this case."}, "4": {"exist": true, "char_ranges": [[89, 250]], "word_ranges": [[15, 40]], "text": "Tubulointerstitial nephritis due to methotrexate is less likely due to the absence of rash, fever or eosinophilia, which although rare in the MIR usually appear."}, "5": {"exist": false, "char_ranges": [], "word_ranges": [], "text": ""}}} {"id": 517, "year": 2021, "question_id_specific": 104, "full_question": "A 34-year-old male is involved in a traffic accident and is attended at the scene of the accident. The medical personnel observe that he is pale, sweaty, has a thready pulse with a blood pressure of 90/50 mmHg, a heart rate of 127 bpm, a respiratory rate of 28 rpm and an oxygen saturation of 92%. He was infused with 20 ml/kg of crystalloids. During his transfer to the hospital his vital signs improve transiently and then deteriorate on arrival at the hospital. Point out the correct answer:", "full_answer": "In a polytraumatized patient, the first cause of shock is blood loss (hemorrhagic shock). The guidelines to follow should always contemplate the replenishment of blood volume (if possible, with blood products as early as possible) and control of the focus. In this case, given the vital compromise, the need for transfusion is very urgent, so the indication is emergency blood transfusion (O Rh negative).", "type": "CRITICAL AND EMERGENCY CARE", "options": {"1": "Type-specific blood transfusion required.", "2": "Requires emergency blood transfusion (O Rh negative).", "3": "A possible blood transfusion with crossmatching tests should be prepared.", "4": "Try again infusion of another 20 ml/kg of crystalloids.", "5": NaN}, "correct_option": 2, "explanations": {"1": {"exist": false, "char_ranges": [], "word_ranges": [], "text": ""}, "2": {"exist": true, "char_ranges": [[257, 405]], "word_ranges": [[40, 64]], "text": "In this case, given the vital compromise, the need for transfusion is very urgent, so the indication is emergency blood transfusion (O Rh negative)."}, "3": {"exist": false, "char_ranges": [], "word_ranges": [], "text": ""}, "4": {"exist": false, "char_ranges": [], "word_ranges": [], "text": ""}, "5": {"exist": false, "char_ranges": [], "word_ranges": [], "text": ""}}} {"id": 520, "year": 2021, "question_id_specific": 38, "full_question": "A 72-year-old patient comes to the emergency department for presenting with severe and significant lingual angioedema. She does not present associated urticaria and has no history of drug or food allergy. His personal history includes type 2 diabetes mellitus, dyslipidemia, hypertension, hypothyroidism and Parkinson's disease. She is on regular treatment with metformin, simvastatin, enalapril, thyroxine and levodopa. Which of the following drugs is most likely the causal drug of the clinical picture described?", "full_answer": "Angioedema is a type of profound urticaria that can be of drug etiology (not IgE-mediated) and in this case, ACEIs (such as enalapril) are considered the most frequent pharmacological cause, so in this case it would be the correct answer (if we speak in probabilistic terms).", "type": "DERMATOLOGY", "options": {"1": "Enalapril.", "2": "Simvastatin.", "3": "Metformin or levodopa equally.", "4": "Levodopa.", "5": NaN}, "correct_option": 1, "explanations": {"1": {"exist": true, "char_ranges": [[0, 275]], "word_ranges": [[0, 46]], "text": "Angioedema is a type of profound urticaria that can be of drug etiology (not IgE-mediated) and in this case, ACEIs (such as enalapril) are considered the most frequent pharmacological cause, so in this case it would be the correct answer (if we speak in probabilistic terms)."}, "2": {"exist": false, "char_ranges": [], "word_ranges": [], "text": ""}, "3": {"exist": false, "char_ranges": [], "word_ranges": [], "text": ""}, "4": {"exist": false, "char_ranges": [], "word_ranges": [], "text": ""}, "5": {"exist": false, "char_ranges": [], "word_ranges": [], "text": ""}}} {"id": 379, "year": 2016, "question_id_specific": 133, "full_question": "A 20-year-old girl comes to the emergency department because she has noticed after getting up, when looking in the mirror, weakness in the right half of the face (including the forehead, closing the eyelid and smiling). She is accompanied by dysgeusia, with a sensation of metallic taste of food as well as hyperacusis and ipsilateral mastoid pain. On examination there is no evidence of strength deficits or sensory deficits in the extremities or speech or language disorders. In this case, which of the following statements is correct?", "full_answer": "We describe a picture of typical peripheral facial paralysis, with dysgeusia, hyperacusis and mastoid pain. In this case the treatment is corticoid. The first one would not be correct because although within the age range of multiple sclerosis debut, the presentation is usually gradual over days and the nuclear involvement of the VII cranial nerve (which could simulate a peripheral facial) would not cause dysgeusia, hyperacusis or mastoid pain. In the third answer they propose us that it is a vascular picture and therefore that the facial paralysis is central, so it is discarded and the last one is not correct because most of the idiopathic peripheral facial paralysis (Bell's palsy) heal without sequelae.", "type": "NEUROLOGY", "options": {"1": "The most probable diagnosis is a demyelinating plaque in the contralateral hemiprotuberance, the most necessary complementary exploration would be a cranial MRI.", "2": "Oral corticosteroids are the treatment of choice for the patient.", "3": "Intravenous fibrinolysis should be considered if the evolution time is less than 3h.", "4": "Most likely the picture is irreversible.", "5": NaN}, "correct_option": 2, "explanations": {"1": {"exist": true, "char_ranges": [[149, 448]], "word_ranges": [[22, 69]], "text": "The first one would not be correct because although within the age range of multiple sclerosis debut, the presentation is usually gradual over days and the nuclear involvement of the VII cranial nerve (which could simulate a peripheral facial) would not cause dysgeusia, hyperacusis or mastoid pain."}, "2": {"exist": true, "char_ranges": [[0, 148]], "word_ranges": [[0, 22]], "text": "We describe a picture of typical peripheral facial paralysis, with dysgeusia, hyperacusis and mastoid pain. In this case the treatment is corticoid."}, "3": {"exist": true, "char_ranges": [[449, 585]], "word_ranges": [[69, 94]], "text": "In the third answer they propose us that it is a vascular picture and therefore that the facial paralysis is central, so it is discarded"}, "4": {"exist": true, "char_ranges": [[590, 714]], "word_ranges": [[95, 114]], "text": "the last one is not correct because most of the idiopathic peripheral facial paralysis (Bell's palsy) heal without sequelae."}, "5": {"exist": false, "char_ranges": [], "word_ranges": [], "text": ""}}} {"id": 600, "year": 2022, "question_id_specific": 184, "full_question": "A 70-year-old woman who has been complaining for 1 month of pain and stiffness in the shoulder girdle and pelvic girdle, which predominates when she gets up and improves with mobilization, without fever, headache or general condition. Physical examination showed stiffness and pain on mobilization of the shoulders and hips. Laboratory tests show elevated ESR and C-reactive protein. What is the most likely diagnosis and what treatment would you prescribe?", "full_answer": "In this case, he is describing the cardinal symptoms of polymyalgia rheumatica. Older patient, over 50 years of age, presenting with pain and stiffness in both girdles, associated with an elevation of acute phase reactants. He presents all the indispensable criteria: age equal to or greater than 50 years, bilateral omalgia and increased acute phase reactants. Of the optional criteria, he presents stiffness, pain or limitation in the hip. Initial treatment is corticosteroids at medium doses.", "type": "RHEUMATOLOGY", "options": {"1": "Rheumatoid arthritis, would initiate treatment with methotrexate.", "2": "Fibromyalgia, would initiate treatment with duloxetine.", "3": "Polymyalgia rheumatica, would initiate treatment with corticosteroids.", "4": "Ankylosing spondylitis, I would initiate treatment with non-steroidal anti-inflammatory drugs.", "5": NaN}, "correct_option": 3, "explanations": {"1": {"exist": false, "char_ranges": [], "word_ranges": [], "text": ""}, "2": {"exist": false, "char_ranges": [], "word_ranges": [], "text": ""}, "3": {"exist": true, "char_ranges": [[0, 361]], "word_ranges": [[0, 56]], "text": "In this case, he is describing the cardinal symptoms of polymyalgia rheumatica. Older patient, over 50 years of age, presenting with pain and stiffness in both girdles, associated with an elevation of acute phase reactants. He presents all the indispensable criteria: age equal to or greater than 50 years, bilateral omalgia and increased acute phase reactants."}, "4": {"exist": false, "char_ranges": [], "word_ranges": [], "text": ""}, "5": {"exist": false, "char_ranges": [], "word_ranges": [], "text": ""}}} {"id": 415, "year": 2018, "question_id_specific": 72, "full_question": "A 45-year-old man with a history of febrile illness and chest pain comes to the hospital with dyspnea and tachypnea. On physical examination his blood pressure is low, jugular venous pressure is elevated with a prominent descending sinus x and he has a paradoxical pulse. What diagnosis would you suspect?", "full_answer": "It is the classic exploration: he had a fever and pain, he had pericarditis with effusion, and now this is causing tamponade. Hence the hypotension (his heart cannot fill well with blood and pump), jugular engorgement and dependence on respiration to achieve preload and cardiac output (i.e. paradoxical pulse).", "type": "CARDIOLOGY AND CARDIOVASCULAR SURGERY", "options": {"1": "Dilated cardiomyopathy.", "2": "Pericardial effusion with cardiac tamponade.", "3": "Constrictive pericarditis.", "4": "Ischemic heart disease.", "5": NaN}, "correct_option": 2, "explanations": {"1": {"exist": false, "char_ranges": [], "word_ranges": [], "text": ""}, "2": {"exist": true, "char_ranges": [[31, 311]], "word_ranges": [[5, 49]], "text": "he had a fever and pain, he had pericarditis with effusion, and now this is causing tamponade. Hence the hypotension (his heart cannot fill well with blood and pump), jugular engorgement and dependence on respiration to achieve preload and cardiac output (i.e. paradoxical pulse)."}, "3": {"exist": false, "char_ranges": [], "word_ranges": [], "text": ""}, "4": {"exist": false, "char_ranges": [], "word_ranges": [], "text": ""}, "5": {"exist": false, "char_ranges": [], "word_ranges": [], "text": ""}}} {"id": 560, "year": 2022, "question_id_specific": 180, "full_question": "A 26-year-old woman diagnosed with systemic lupus erythematosus, on treatment with hydroxychloroquine, who consults for a feeling of generalized weakness that has progressively set in over the last 15 days. The physical examination reveals cutaneous pallor and the CBC shows Hb 7.4 g/dL, Hct 31%, MCV 108. Which of the following tests will be most useful in deciding the course of action?", "full_answer": "They give to know of a patient with SLE in treatment with Hydroxychloroquine. She has a hb of 7.4 and a MCV over 100, adding pallor and weakness, we would be dealing with a macrocytic anemia. In my opinion, the most suitable would be alternative 2, since hydroxyloroquine has been reported to decrease the absorption of folic acid, and since she has a complication of SLE, in this case, we could be dealing with an autoimmune hemolytic anemia with folic acid consumption. And for this it would be first to determine with the direct coombs.", "type": "HEMATOLOGY", "options": {"1": "Haptoglobin.", "2": "Coombs' test.", "3": "Vitamin B12.", "4": "Antinuclear antibodies.", "5": NaN}, "correct_option": 2, "explanations": {"1": {"exist": false, "char_ranges": [], "word_ranges": [], "text": ""}, "2": {"exist": true, "char_ranges": [[192, 539]], "word_ranges": [[36, 95]], "text": "In my opinion, the most suitable would be alternative 2, since hydroxyloroquine has been reported to decrease the absorption of folic acid, and since she has a complication of SLE, in this case, we could be dealing with an autoimmune hemolytic anemia with folic acid consumption. And for this it would be first to determine with the direct coombs."}, "3": {"exist": false, "char_ranges": [], "word_ranges": [], "text": ""}, "4": {"exist": false, "char_ranges": [], "word_ranges": [], "text": ""}, "5": {"exist": false, "char_ranges": [], "word_ranges": [], "text": ""}}} {"id": 358, "year": 2016, "question_id_specific": 232, "full_question": "A 24-year-old woman consults after noticing inguinal lymphadenopathies. The interrogation does not show any local discomfort or data suggestive of sexually transmitted infection. The examination revealed two lymphadenopathies, one in each groin, 1 cm in diameter, soft, mobile, non-painful. No skin lesions are seen on the lower limbs, anus or perineum. Which test do you consider essential?", "full_answer": "The correct answer is 3. In the inguinal region, the lymph nodes can acquire a size of up to 1.5 cm being normal. In addition, the rest of the data of the inguinal exploration (no pain, mobile, soft, etc.) corroborates this. On the other hand, there are no signs or symptoms that make us think of a sexually transmitted infection.", "type": "GYNECOLOGY AND OBSTETRICS", "options": {"1": "A lupus serology since it is most likely a Treponema pallidum infection.", "2": "A gynecological examination to rule out ovarian cancer.", "3": "From the clinical features it seems to be normal lymph nodes and complementary explorations should not be performed.", "4": "A Paul-Bunell test should be performed to rule out infectious mononucleosis.", "5": NaN}, "correct_option": 3, "explanations": {"1": {"exist": true, "char_ranges": [[225, 330]], "word_ranges": [[41, 60]], "text": "On the other hand, there are no signs or symptoms that make us think of a sexually transmitted infection."}, "2": {"exist": false, "char_ranges": [], "word_ranges": [], "text": ""}, "3": {"exist": true, "char_ranges": [[25, 224]], "word_ranges": [[5, 41]], "text": "In the inguinal region, the lymph nodes can acquire a size of up to 1.5 cm being normal. In addition, the rest of the data of the inguinal exploration (no pain, mobile, soft, etc.) corroborates this."}, "4": {"exist": false, "char_ranges": [], "word_ranges": [], "text": ""}, "5": {"exist": false, "char_ranges": [], "word_ranges": [], "text": ""}}} {"id": 414, "year": 2018, "question_id_specific": 71, "full_question": "An 84-year-old man with chronic heart failure in functional grade II secondary to ischemic heart disease with severe non-revascularizable systolic dysfunction, stage 3 chronic kidney disease (glomerular filtration rate 45-50 mL/min) and permanent atrial fibrillation with heart rate >80 beats per minute. Which of the following drugs does NOT provide benefit to the patient according to the currently available evidence?", "full_answer": "Ivabradine, in fact, is contraindicated in patients with atrial fibrillation. Initially (although evidence has recently emerged that qualifies this information) ivabradine was said to be selective over the sodium If current in the sinus node. If the patient is in AF, it makes no sense to act on the sinus node.", "type": "CARDIOLOGY AND CARDIOVASCULAR SURGERY", "options": {"1": "Ivabradine.", "2": "Angiotensin converting enzyme inhibitors.", "3": "Antialdosterone.", "4": "Beta-blockers.", "5": NaN}, "correct_option": 1, "explanations": {"1": {"exist": true, "char_ranges": [[0, 77]], "word_ranges": [[0, 10]], "text": "Ivabradine, in fact, is contraindicated in patients with atrial fibrillation."}, "2": {"exist": false, "char_ranges": [], "word_ranges": [], "text": ""}, "3": {"exist": false, "char_ranges": [], "word_ranges": [], "text": ""}, "4": {"exist": false, "char_ranges": [], "word_ranges": [], "text": ""}, "5": {"exist": false, "char_ranges": [], "word_ranges": [], "text": ""}}} {"id": 131, "year": 2012, "question_id_specific": 221, "full_question": "An individual presents with \"muscle weakness\" (spastic hemiparesis) of both right extremities, with hyperreflexia and Babinski's sign, along with a \"facial flaccid paralysis\" of the left hemiface, with inability to close the left eye or retract the left side of the mouth, in addition to other alterations. From the data described, it is an alteration that affects, among other elements, the motor fascicles: cortico-spinal and cortico-nuclear, but at what level of the neuroaxis would the lesion be located?", "full_answer": "Question always involves the brainstem. We have a right hemiparesis and involvement of the left facial nucleus, since it has affected the upper and lower facial; therefore we have a crossed clinical picture, having to think about brainstem involvement. If we have in a quick diagram where the nuclei of the cranial pairs start from: the first 4 above the pons, the next 4 in the pons, and the last 4 below the pons, we have that the facial nucleus is in the middle 4. That is, left facial nucleus, left pons, so answer 4.", "type": "NEUROLOGY AND NEUROSURGERY", "options": {"1": "At the level of the area 4 of Brodmann of the cerebral cortex of the right side.", "2": "In the internal capsule, posterior arm on the right side.", "3": "In the left cerebral peduncle.", "4": "In the medial portion of the caudal protuberance of the left side.", "5": "In the medulla oblongata before decussation of the right cortico-spinal fascicle."}, "correct_option": 4, "explanations": {"1": {"exist": false, "char_ranges": [], "word_ranges": [], "text": ""}, "2": {"exist": false, "char_ranges": [], "word_ranges": [], "text": ""}, "3": {"exist": false, "char_ranges": [], "word_ranges": [], "text": ""}, "4": {"exist": true, "char_ranges": [[172, 521]], "word_ranges": [[27, 95]], "text": "we have a crossed clinical picture, having to think about brainstem involvement. If we have in a quick diagram where the nuclei of the cranial pairs start from: the first 4 above the pons, the next 4 in the pons, and the last 4 below the pons, we have that the facial nucleus is in the middle 4. That is, left facial nucleus, left pons, so answer 4."}, "5": {"exist": false, "char_ranges": [], "word_ranges": [], "text": ""}}} {"id": 70, "year": 2012, "question_id_specific": 127, "full_question": "A 30-year-old woman with a drug addiction PD in a current detoxification period is brought to the ED with profound somnolence, with no other symptoms. We find out that her treatment includes olanzapine and benzodiazepines. What should we do first?", "full_answer": "Overuse of the drugs he is taking should be suspected. Olanzapine has no antidote, so, in principle, we should antagonize the effect of benzodiazepines. If the patient continues to be drowsy and does not respond to stimuli, we should proceed to intubate to secure the airway.", "type": "ANESTHESIOLOGY, CRITICAL CARE AND EMERGENCY MEDICINE", "options": {"1": "Proceed to IOT.", "2": "Perform a skull X-ray.", "3": "Administer flumazenil.", "4": "Administer naloxone.", "5": "Wait until he wakes up."}, "correct_option": 3, "explanations": {"1": {"exist": false, "char_ranges": [], "word_ranges": [], "text": ""}, "2": {"exist": false, "char_ranges": [], "word_ranges": [], "text": ""}, "3": {"exist": true, "char_ranges": [[0, 152]], "word_ranges": [[0, 24]], "text": "Overuse of the drugs he is taking should be suspected. Olanzapine has no antidote, so, in principle, we should antagonize the effect of benzodiazepines."}, "4": {"exist": false, "char_ranges": [], "word_ranges": [], "text": ""}, "5": {"exist": false, "char_ranges": [], "word_ranges": [], "text": ""}}} {"id": 463, "year": 2019, "question_id_specific": 71, "full_question": "Woman with hypotension and tissue hypoperfusion in whom the following parameters are objectified after insertion of a Swan-Ganz catheter: cardiac index 1.4 L/min/m2, pulmonary capillary wedge pressure 25 mmHg and systemic vascular resistances 2000 dynes/m2. The type of shock presented by the patient is:", "full_answer": "We are not given any further data on the patient except that she is in shock (tissue hypoperfusion) and on introducing a Swan-Ganz catheter we find a low cardiac index (<2), elevated PCP (25mmHg) and elevated systemic resistances (normal: 600-1200 dyne/m2). These data are compatible with cardiogenic origin of shock.", "type": "CRITICAL CARE AND EMERGENCY", "options": {"1": "Cardiogenic.", "2": "Septic.", "3": "Hypovolemic.", "4": "Anaphylactic.", "5": NaN}, "correct_option": 1, "explanations": {"1": {"exist": true, "char_ranges": [[0, 317]], "word_ranges": [[0, 50]], "text": "We are not given any further data on the patient except that she is in shock (tissue hypoperfusion) and on introducing a Swan-Ganz catheter we find a low cardiac index (<2), elevated PCP (25mmHg) and elevated systemic resistances (normal: 600-1200 dyne/m2). These data are compatible with cardiogenic origin of shock."}, "2": {"exist": false, "char_ranges": [], "word_ranges": [], "text": ""}, "3": {"exist": false, "char_ranges": [], "word_ranges": [], "text": ""}, "4": {"exist": false, "char_ranges": [], "word_ranges": [], "text": ""}, "5": {"exist": false, "char_ranges": [], "word_ranges": [], "text": ""}}} {"id": 476, "year": 2020, "question_id_specific": 93, "full_question": "A 58-year-old hypertensive man comes to the emergency department for a 30-second syncope while jogging, with spontaneous recovery and no sequelae. Blood pressure is 135/65 mmHg. Cardiac auscultation shows a loud, intense systolic murmur, which is reduced with Valsalva maneuvers and an attenuated second tone. The ECG shows sinus rhythm at 72 bpm, with criteria of left ventricular hypertrophy and inverted T waves in anterior face. Point out the correct statement:", "full_answer": "The data provided on the characteristics of the murmur (it DECREASES with Valsalva) and the data on left ventricular hypertrophy, lead us to the diagnosis of severe aortic stenosis.", "type": "CRITICAL CARE", "options": {"1": "The clinical picture suggests pulmonary thromboembolism.", "2": "The data provided indicate hypertrophic cardiomyopathy with severe obstruction of the left ventricular outflow tract.", "3": "These data correspond to severe aortic stenosis.", "4": "Aortic dissection should be excluded by contrast-enhanced computed tomography.", "5": NaN}, "correct_option": 3, "explanations": {"1": {"exist": false, "char_ranges": [], "word_ranges": [], "text": ""}, "2": {"exist": false, "char_ranges": [], "word_ranges": [], "text": ""}, "3": {"exist": true, "char_ranges": [[0, 181]], "word_ranges": [[0, 29]], "text": "The data provided on the characteristics of the murmur (it DECREASES with Valsalva) and the data on left ventricular hypertrophy, lead us to the diagnosis of severe aortic stenosis."}, "4": {"exist": false, "char_ranges": [], "word_ranges": [], "text": ""}, "5": {"exist": false, "char_ranges": [], "word_ranges": [], "text": ""}}} {"id": 546, "year": 2022, "question_id_specific": 124, "full_question": "82-year-old woman with chronic atrial fibrillation of 10 years of evolution and preserved biventricular systolic function who presents with episodes of symptomatic complete atrioventricular block, so it is decided to implant a definitive cardiac pacing system. Which of the following is indicated:", "full_answer": "VVI pacemaker (ventricular unicameral).", "type": "CARDIOLOGY", "options": {"1": "DDD pacemaker (bicameral).", "2": "VVI pacemaker (ventricular unicameral).", "3": "AAI pacemaker (single-chamber atrial pacemaker).", "4": "Ventricular resynchronization therapy (CRT).", "5": NaN}, "correct_option": 2, "explanations": {"1": {"exist": false, "char_ranges": [], "word_ranges": [], "text": ""}, "2": {"exist": true, "char_ranges": [[0, 39]], "word_ranges": [[0, 4]], "text": "VVI pacemaker (ventricular unicameral)."}, "3": {"exist": false, "char_ranges": [], "word_ranges": [], "text": ""}, "4": {"exist": false, "char_ranges": [], "word_ranges": [], "text": ""}, "5": {"exist": false, "char_ranges": [], "word_ranges": [], "text": ""}}} {"id": 313, "year": 2016, "question_id_specific": 177, "full_question": "A 57-year-old patient diagnosed with stage T3N1M0 adenocarcinoma of the colon. He was treated with resection followed by postoperative FOLFOX chemotherapy (fluorouracil, leucovorin and oxaliplatin) for 6 months. In a routine control, two years later, carcinoembryonic antigen (CEA) elevation to 30 ng/mL (previously less than 2 ng/mL) was detected. She has no symptoms, physical examination is unremarkable, chest X-ray is normal. CT scan shows a 3 cm mass in the right hepatic lobe that is captured on PET. No other alterations are observed in CT or PET. What is the most correct approach?", "full_answer": "We are faced with a situation of probable liver metastasis from a localized colon adenocarcinoma treated 2 years ago. Probable because we do not have histological confirmation, although as it usually happens, what else could it be? As this is a situation of oligometastasis and the treatments for advanced colon adenocarcinoma allow prolonged survival with good quality of life in these patients, if there is nothing to contraindicate surgery, this should be the first option (correct answer 3). I attach the algorithm proposed by the NCCN in situations of recurrence (http://www.nccn.org/professionals/physician_gls/pdf/colon.pdf). In case of an unresectable situation, chemotherapy treatment would be considered with the intention of converting the single metastatic lesion into a resectable one. Since more than 12 months have passed since FOLFOX treatment, the same scheme could be used, although FOLFIRI+bevacizumab would also be useful. Capecitabine monotherapy, in the patient's current situation, would also not be a good starting option, since the responses are lower compared to FOLFOX or FOLFIRI plus antiangiogenic combinations. Hepatic radiotherapy could be considered, since there are SBRT techniques that achieve very good results, but in this case it is best to start by asking for an assessment by the surgeons and if they rule out surgery we would consider RT.", "type": "ONCOLOGY", "options": {"1": "Chemotherapy with FOLFIRI (fluorouracil, leucovorin and irinotecan) plus bevacizumab.", "2": "Monochemotherapy with capecitabine.", "3": "Assessment for surgical resection of the hepatic lesion.", "4": "Hepatic radiotherapy.", "5": NaN}, "correct_option": 3, "explanations": {"1": {"exist": true, "char_ranges": [[633, 942]], "word_ranges": [[91, 137]], "text": "In case of an unresectable situation, chemotherapy treatment would be considered with the intention of converting the single metastatic lesion into a resectable one. Since more than 12 months have passed since FOLFOX treatment, the same scheme could be used, although FOLFIRI+bevacizumab would also be useful."}, "2": {"exist": true, "char_ranges": [[943, 1140]], "word_ranges": [[137, 165]], "text": "Capecitabine monotherapy, in the patient's current situation, would also not be a good starting option, since the responses are lower compared to FOLFOX or FOLFIRI plus antiangiogenic combinations."}, "3": {"exist": true, "char_ranges": [[232, 495]], "word_ranges": [[37, 78]], "text": "As this is a situation of oligometastasis and the treatments for advanced colon adenocarcinoma allow prolonged survival with good quality of life in these patients, if there is nothing to contraindicate surgery, this should be the first option (correct answer 3)."}, "4": {"exist": true, "char_ranges": [[1141, 1378]], "word_ranges": [[165, 207]], "text": "Hepatic radiotherapy could be considered, since there are SBRT techniques that achieve very good results, but in this case it is best to start by asking for an assessment by the surgeons and if they rule out surgery we would consider RT."}, "5": {"exist": false, "char_ranges": [], "word_ranges": [], "text": ""}}} {"id": 482, "year": 2020, "question_id_specific": 173, "full_question": "A patient is brought to the ED from a fire occurring inside an enclosed building where polyurethane foams were present. He is conscious, but presents with increasing mental dullness, headache, and severe dyspnea. Oxygen saturation by pulse oximetry is 92% and capillary lactic acid is 8 mEq/l. What specific treatment do you consider most appropriate?", "full_answer": "The first thing we need to know is that combustion of polyurethane at high temperature results in the release of cyanide (very intuitive, yes). Knowing this, and that we are dealing with cyanide poisoning, it is easier to answer the question. Among the basic initial treatment measures would be the administration of oxygen therapy with FiO2 1 (not 0.5 as stated in option 1), and an antidote, in this case hydroxycobalamin (this was asked similarly last year). Ventilation with hyperbaric chamber would only be indicated in case of coexistence of carbon monoxide (CO) poisoning. What is stated in option 4 about fluid therapy is not indicated; in case of associated shock, volume would be administered, but in any case not at that rate, but in rapid boluses.", "type": "CRITICAL CARE", "options": {"1": "Administration of oxygen by means of a 50% Venturi type mask.", "2": "Administration of intravenous hydroxycobalamin.", "3": "Ventilation with hyperbaric chamber.", "4": "Fluid therapy with infusion of physiological saline solution at 21 ml/h.", "5": NaN}, "correct_option": 2, "explanations": {"1": {"exist": true, "char_ranges": [[243, 376]], "word_ranges": [[41, 64]], "text": "Among the basic initial treatment measures would be the administration of oxygen therapy with FiO2 1 (not 0.5 as stated in option 1),"}, "2": {"exist": true, "char_ranges": [[243, 423]], "word_ranges": [[41, 71]], "text": "Among the basic initial treatment measures would be the administration of oxygen therapy with FiO2 1 (not 0.5 as stated in option 1), and an antidote, in this case hydroxycobalamin"}, "3": {"exist": true, "char_ranges": [[462, 579]], "word_ranges": [[77, 94]], "text": "Ventilation with hyperbaric chamber would only be indicated in case of coexistence of carbon monoxide (CO) poisoning."}, "4": {"exist": true, "char_ranges": [[580, 759]], "word_ranges": [[94, 127]], "text": "What is stated in option 4 about fluid therapy is not indicated; in case of associated shock, volume would be administered, but in any case not at that rate, but in rapid boluses."}, "5": {"exist": false, "char_ranges": [], "word_ranges": [], "text": ""}}} {"id": 105, "year": 2012, "question_id_specific": 233, "full_question": "In a 35-year-old patient with a depressive syndrome on treatment with serotonin reuptake inhibitors, the use of the following antimicrobials is contraindicated:", "full_answer": "Spontaneous reports of serotonergic syndrome associated with concomitant administration of linezolid and serotonergic agents, including antidepressants such as selective serotonin reuptake inhibitors (SSRIs), have been reported.", "type": "PHARMACOLOGY", "options": {"1": "Doxycycline.", "2": "Amoxicillin-Clavulanic acid.", "3": "Daptomycin.", "4": "Linezolid.", "5": "Vancomycin."}, "correct_option": 4, "explanations": {"1": {"exist": false, "char_ranges": [], "word_ranges": [], "text": ""}, "2": {"exist": false, "char_ranges": [], "word_ranges": [], "text": ""}, "3": {"exist": false, "char_ranges": [], "word_ranges": [], "text": ""}, "4": {"exist": true, "char_ranges": [[0, 228]], "word_ranges": [[0, 26]], "text": "Spontaneous reports of serotonergic syndrome associated with concomitant administration of linezolid and serotonergic agents, including antidepressants such as selective serotonin reuptake inhibitors (SSRIs), have been reported."}, "5": {"exist": false, "char_ranges": [], "word_ranges": [], "text": ""}}} {"id": 73, "year": 2012, "question_id_specific": 62, "full_question": "A 37-year-old man with no past history of interest comes to the emergency department for syncope while walking. He had started two days earlier with dyspnea on exertion. Examination: weight 75 kg, BP 75/50 mm Hg. SatO2 89%. Pulmonary auscultation was normal. Cardiac auscultation: rhythmic tachycardic tones at 130 bpm. Abdomen normal. Extremities: no alterations. Normal hemogram. Troponin 1.2 ng / ml. ECG: Sensual tachycardia at 130 bpm. Inverted T in V1-V4. Thoracic CT angiography: repletion defect in both main pulmonary arteries. One hour after arrival at the emergency room, BP 70/55 mm Hg persists. What would be the most appropriate initial treatment?", "full_answer": "The clinical case is a typical pulmonary thromboembolism. In this case it is associated with persistent arterial hypotension. Option 2 would be correct if the patient were not hypotensive. Option 3 is not correct since diagnostic confirmation is not necessary to initiate anticoagulant therapy in case of PTE without hypotension. Option 4 is not indicated. Option 5 would be correct only if the patient had a contraindication to fibrinolysis. Persistent hypotension in PTE is the most widely accepted indication for fibrinolysis, so the correct option is 1.", "type": "ANESTHESIOLOGY, CRITICAL CARE AND EMERGENCIES", "options": {"1": "Unfractionated heparin, 10,000U iv on clinical suspicion. Fibrinolysis with tPA 100 mg iv once the diagnosis is confirmed.", "2": "Enoxaparin: 80 mg sc every 12 hours, starting upon diagnostic suspicion.", "3": "Enoxaparin; 80 mg sc every 12 hours, starting upon diagnostic confirmation.", "4": "Fondaparinux: 7.5 mg sc daily.", "5": "Emergency pulmonary thromboendarterectomy."}, "correct_option": 1, "explanations": {"1": {"exist": true, "char_ranges": [[443, 557]], "word_ranges": [[69, 87]], "text": "Persistent hypotension in PTE is the most widely accepted indication for fibrinolysis, so the correct option is 1."}, "2": {"exist": true, "char_ranges": [[126, 188]], "word_ranges": [[18, 29]], "text": "Option 2 would be correct if the patient were not hypotensive."}, "3": {"exist": true, "char_ranges": [[189, 329]], "word_ranges": [[29, 50]], "text": "Option 3 is not correct since diagnostic confirmation is not necessary to initiate anticoagulant therapy in case of PTE without hypotension."}, "4": {"exist": true, "char_ranges": [[330, 356]], "word_ranges": [[50, 55]], "text": "Option 4 is not indicated."}, "5": {"exist": true, "char_ranges": [[357, 442]], "word_ranges": [[55, 69]], "text": "Option 5 would be correct only if the patient had a contraindication to fibrinolysis."}}} {"id": 538, "year": 2021, "question_id_specific": 2, "full_question": "A fall with anterior shoulder fracture-dislocation that was reduced in the emergency department. A sling was placed and its removal was recommended after 3 weeks. On removal of the immobilization, an area of dysesthesia circumscribed to the lateral region of the shoulder was observed. The patient can perform abduction, but only reaches 15 degrees. She has a magnetic resonance imaging study in which the rotator cuff is undamaged. What is the most likely diagnosis of suspicion?", "full_answer": "Dysesthesia of the lateral aspect of the deltoid region of the shoulder (area innervated by the axillary nerve) and paralysis of the deltoid (primary muscle for abduction beyond 15°). The rotator cuff is undamaged in the MRI, ruling out option 1 (it would show us tendinopathy of the teres major). Adhesive capsulitis presents with pain and stiffness of the shoulder, especially on active and passive external rotation. The suprascapular nerve innervates the supraspinatus muscle, which contributes to the first 15º of shoulder abduction (preserved in the statement) and does not innervate the skin of the reflected area.", "type": "TRAUMATOLOGY", "options": {"1": "Tendinopathy of the teres major.", "2": "Adhesive capsulitis.", "3": "Suprascapular nerve neuropathy.", "4": "Axillary nerve injury.", "5": NaN}, "correct_option": 4, "explanations": {"1": {"exist": true, "char_ranges": [[184, 297]], "word_ranges": [[29, 50]], "text": "The rotator cuff is undamaged in the MRI, ruling out option 1 (it would show us tendinopathy of the teres major)."}, "2": {"exist": true, "char_ranges": [[298, 419]], "word_ranges": [[50, 67]], "text": "Adhesive capsulitis presents with pain and stiffness of the shoulder, especially on active and passive external rotation."}, "3": {"exist": true, "char_ranges": [[420, 621]], "word_ranges": [[67, 97]], "text": "The suprascapular nerve innervates the supraspinatus muscle, which contributes to the first 15º of shoulder abduction (preserved in the statement) and does not innervate the skin of the reflected area."}, "4": {"exist": true, "char_ranges": [[0, 183]], "word_ranges": [[0, 29]], "text": "Dysesthesia of the lateral aspect of the deltoid region of the shoulder (area innervated by the axillary nerve) and paralysis of the deltoid (primary muscle for abduction beyond 15°)."}, "5": {"exist": false, "char_ranges": [], "word_ranges": [], "text": ""}}} {"id": 139, "year": 2012, "question_id_specific": 166, "full_question": "A 64-year-old smoker and heavy drinker with no other history of interest. The picture he explains is odynodysphagia of 3 months of evolution and left otalgia. He provides a dentist's report that rules out a dental cause. He also refers to having been visited by different specialists and provides a head and neck CAT scan (without iodine contrast) which is reported as normal. On examination there was a decrease in the propulsive capacity of the tongue; palpation of the base of the left tongue showed an induration of stony consistency of approximately 3cm and with indirect laryngoscopy no ulceration of the mucosa was observed. Cervical palpation is negative for lymphadenopathy. What is your presumptive diagnosis and course of action?", "full_answer": "We are presented with a patient with risk factors for oropharyngeal cancer, but in whom the examination and tests are normal. What is going on here? This question is reverse psychology: what did the questioner want me to know? This question was asked by an otolaryngologist. He already tells you that the dentist \"didn't see anything\", so he rules out 1. It is not an abscess of the base of the tongue either, because in three months it would have killed him: neither is 2. So we are left with ankyloglossia, tongue base cancer and non-specific inflammation. When there is doubt that it could be cancer, we should not fool around and order a new CT scan and biopsy. Tongue base cancers are very treacherous and it is one of the fears of any otolaryngologist when he suspects a neoplasm. It is not uncommon for them to arise without exophytic masses or ulcerations of the tongue mucosa and they are difficult to detect on CT when contrast is not used. The correct answer is 4 and I do not think it can be challenged.", "type": "OTORHINOLARYNGOLOGY AND MAXILLOFACIAL SURGERY", "options": {"1": "Dental cause / reevaluation by the dentist.", "2": "Abscess of the base of the tongue / debridement.", "3": "Ankyloglossia / surgical release of the same.", "4": "Malignant process of oropharynx / repeat CT scan and biopsy.", "5": "Nonspecific inflammation / corticosteroid treatment."}, "correct_option": 4, "explanations": {"1": {"exist": true, "char_ranges": [[301, 354]], "word_ranges": [[51, 61]], "text": "the dentist \"didn't see anything\", so he rules out 1."}, "2": {"exist": true, "char_ranges": [[355, 473]], "word_ranges": [[61, 85]], "text": "It is not an abscess of the base of the tongue either, because in three months it would have killed him: neither is 2."}, "3": {"exist": false, "char_ranges": [], "word_ranges": [], "text": ""}, "4": {"exist": true, "char_ranges": [[559, 665]], "word_ranges": [[97, 119]], "text": "When there is doubt that it could be cancer, we should not fool around and order a new CT scan and biopsy."}, "5": {"exist": false, "char_ranges": [], "word_ranges": [], "text": ""}}} {"id": 446, "year": 2018, "question_id_specific": 130, "full_question": "A 28-year-old man comes to the hospital emergency room with gross hematuria. Which of the following urinalysis abnormalities would support a diagnosis of glomerulonephritis?", "full_answer": "Glomerulonephritis is typically associated with dysmorphic red blood cells and hematic casts in the sediment (option 1 correct). Proteinuria can also appear in variable range, although it is not usual in the form of isolated microalbuminuria (incorrect option 2), and hematuria with pyuria although it is not characteristic (incorrect option 3). Clots usually appear in the context of alterations of the lower urinary tract (option 4 incorrect).", "type": "NEPHROLOGY", "options": {"1": "Dysmorphic red blood cells and/or hematic casts.", "2": "Proteinuria of 1 g/day, with negative test strip result and with microalbuminuria greater than 300 mg/24 hours.", "3": "Coexistence of hematuria with pyuria without bacteriuria.", "4": "Clots in the urine with the naked eye.", "5": NaN}, "correct_option": 1, "explanations": {"1": {"exist": true, "char_ranges": [[0, 128]], "word_ranges": [[0, 18]], "text": "Glomerulonephritis is typically associated with dysmorphic red blood cells and hematic casts in the sediment (option 1 correct)."}, "2": {"exist": true, "char_ranges": [[129, 263]], "word_ranges": [[18, 39]], "text": "Proteinuria can also appear in variable range, although it is not usual in the form of isolated microalbuminuria (incorrect option 2),"}, "3": {"exist": true, "char_ranges": [[268, 345]], "word_ranges": [[40, 51]], "text": "hematuria with pyuria although it is not characteristic (incorrect option 3)."}, "4": {"exist": true, "char_ranges": [[346, 445]], "word_ranges": [[51, 67]], "text": "Clots usually appear in the context of alterations of the lower urinary tract (option 4 incorrect)."}, "5": {"exist": false, "char_ranges": [], "word_ranges": [], "text": ""}}} {"id": 551, "year": 2022, "question_id_specific": 130, "full_question": "An 85-year-old man in permanent atrial fibrillation consults for abrupt pain and coldness in the right foot for 6 hours. On examination he presents plantar cyanosis with partial sensory and mobility deficit. The femoral pulse is palpable, while the popliteal and distal pulses of this extremity are absent. Which of the following is the best therapeutic approach?", "full_answer": "Urgent surgical treatment by thromboembolectomy.", "type": "CARDIOLOGY", "options": {"1": "Discourage revascularization surgery because of prolonged ischemia time.", "2": "Apply heat and notify the on-call vascular surgeon.", "3": "Urgent surgical treatment by thromboembolectomy.", "4": "Urgent medical treatment with intravenous prostaglandins.", "5": NaN}, "correct_option": 3, "explanations": {"1": {"exist": false, "char_ranges": [], "word_ranges": [], "text": ""}, "2": {"exist": false, "char_ranges": [], "word_ranges": [], "text": ""}, "3": {"exist": true, "char_ranges": [[0, 48]], "word_ranges": [[0, 5]], "text": "Urgent surgical treatment by thromboembolectomy."}, "4": {"exist": false, "char_ranges": [], "word_ranges": [], "text": ""}, "5": {"exist": false, "char_ranges": [], "word_ranges": [], "text": ""}}} {"id": 383, "year": 2016, "question_id_specific": 150, "full_question": "A 12-month-old boy, who in the health examinations performed since birth presents right testicle in the inguinal canal that is not possible to descend to the scrotum. Mark the correct answer:", "full_answer": "The retractile testicle is characterized by the testicle being permanently outside the scrotal sac, being possible to descend it manually (although when it is released it automatically returns to its initial location). It is usually due to a short pedicle. In this case we are being told that the teste is always in the inguinal canal (cryptorchidism) so we can discard option 1. Hormonal treatment with HCG has already fallen into disuse, due to its low efficacy, possibility of reascension and because apoptotic and inflammatory phenomena have been described in the teste associated with its use (we discard option 3). Orchidopexy is the treatment of choice when we are faced with cryptorchidism, being desirable to perform it between 6 months and one year of age, and should not be deferred beyond two years of age.", "type": "PEDIATRICS", "options": {"1": "The most probable diagnosis is retractile testicle.", "2": "Wait until two years of age for its spontaneous decrease.", "3": "Human chorionic gonadotropin is the treatment of first choice.", "4": "The indication for orchidopexy should not be deferred.", "5": NaN}, "correct_option": 4, "explanations": {"1": {"exist": true, "char_ranges": [[257, 379]], "word_ranges": [[40, 63]], "text": "In this case we are being told that the teste is always in the inguinal canal (cryptorchidism) so we can discard option 1."}, "2": {"exist": false, "char_ranges": [], "word_ranges": [], "text": ""}, "3": {"exist": true, "char_ranges": [[380, 620]], "word_ranges": [[63, 100]], "text": "Hormonal treatment with HCG has already fallen into disuse, due to its low efficacy, possibility of reascension and because apoptotic and inflammatory phenomena have been described in the teste associated with its use (we discard option 3)."}, "4": {"exist": true, "char_ranges": [[621, 766]], "word_ranges": [[100, 125]], "text": "Orchidopexy is the treatment of choice when we are faced with cryptorchidism, being desirable to perform it between 6 months and one year of age,"}, "5": {"exist": false, "char_ranges": [], "word_ranges": [], "text": ""}}} {"id": 614, "year": 2022, "question_id_specific": 118, "full_question": "A 41-year-old male consults for gonalgia of several days' evolution. During the examination, the Thessaly test (pain with internal and external rotation movements with the knee flexed) is positive. Which of the following lesions is most likely to occur?", "full_answer": "The Thessaly maneuver consists of the patient, using monopodal support and knee flexion of 5º and then 20º, performing internal and external rotations of the leg. In case this reproduces the pain that the patient refers, the test will be considered as positive. It is a test used for the diagnosis of meniscal pathology (sensitivity greater than 90% and specificity greater than 95%) (Answer 1 correct).", "type": "TRAUMATOLOGY", "options": {"1": "Meniscal injury.", "2": "Injury by rupture of the anterior cruciate ligament.", "3": "Injury by rupture of the posterior cruciate ligament.", "4": "Degenerative arthropathy injury.", "5": NaN}, "correct_option": 1, "explanations": {"1": {"exist": true, "char_ranges": [[0, 403]], "word_ranges": [[0, 66]], "text": "The Thessaly maneuver consists of the patient, using monopodal support and knee flexion of 5º and then 20º, performing internal and external rotations of the leg. In case this reproduces the pain that the patient refers, the test will be considered as positive. It is a test used for the diagnosis of meniscal pathology (sensitivity greater than 90% and specificity greater than 95%) (Answer 1 correct)."}, "2": {"exist": false, "char_ranges": [], "word_ranges": [], "text": ""}, "3": {"exist": false, "char_ranges": [], "word_ranges": [], "text": ""}, "4": {"exist": false, "char_ranges": [], "word_ranges": [], "text": ""}, "5": {"exist": false, "char_ranges": [], "word_ranges": [], "text": ""}}} {"id": 499, "year": 2020, "question_id_specific": 182, "full_question": "A 60-year-old man with abdominal pain and upper gastrointestinal bleeding was found to have an abdominal tumor related to the gastric wall. Histology shows a spindle cell tumor with few mitoses, positive for CD117. The most likely diagnosis is:", "full_answer": "The GI stromal tumor is better known as GIST, positive CD117, ckit, PDGF, manual.", "type": "MEDICAL ONCOLOGY", "options": {"1": "Neurofibroma of the gastric wall.", "2": "Gastrointestinal stromal tumor.", "3": "Leiomyoma.", "4": "Granulocytic sarcoma.", "5": NaN}, "correct_option": 2, "explanations": {"1": {"exist": false, "char_ranges": [], "word_ranges": [], "text": ""}, "2": {"exist": true, "char_ranges": [[0, 81]], "word_ranges": [[0, 14]], "text": "The GI stromal tumor is better known as GIST, positive CD117, ckit, PDGF, manual."}, "3": {"exist": false, "char_ranges": [], "word_ranges": [], "text": ""}, "4": {"exist": false, "char_ranges": [], "word_ranges": [], "text": ""}, "5": {"exist": false, "char_ranges": [], "word_ranges": [], "text": ""}}} {"id": 613, "year": 2022, "question_id_specific": 118, "full_question": "A 41-year-old male consults for gonalgia of several days' evolution. On examination, the Thessaly test (pain with internal and external rotation movements with the knee flexed) is positive. Which of the following injuries is more likely?", "full_answer": "Thessaly test. Provocation test performed in suspected meniscal injury.", "type": "TRAUMATOLOGY", "options": {"1": "Meniscal injury.", "2": "Injury due to rupture of the anterior cruciate ligament.", "3": "Injury by rupture of the posterior cruciate ligament.", "4": "Degenerative arthropathy injury.", "5": NaN}, "correct_option": 1, "explanations": {"1": {"exist": true, "char_ranges": [[0, 71]], "word_ranges": [[0, 9]], "text": "Thessaly test. Provocation test performed in suspected meniscal injury."}, "2": {"exist": false, "char_ranges": [], "word_ranges": [], "text": ""}, "3": {"exist": false, "char_ranges": [], "word_ranges": [], "text": ""}, "4": {"exist": false, "char_ranges": [], "word_ranges": [], "text": ""}, "5": {"exist": false, "char_ranges": [], "word_ranges": [], "text": ""}}} {"id": 247, "year": 2014, "question_id_specific": 117, "full_question": "A 54-year-old man diagnosed with type 2 diabetes mellitus comes to our office. He reported a fever of 40ºC with chills and shivering, arthromyalgia and headache, for which he started treatment with paracetamol. The symptoms started 24 hours before his return to Spain. Three days later she presented a generalized maculopapular rash that evolved into more intense petechiae on the lower limbs. She presented blood tests with leukopenia with 3,200/mm3 and platelets 91,000/mm3 and severe elevation of aminotransferases. The thick blood drop, peripheral blood smear, PCR and malaria antigen are negative. What is the most likely diagnosis?", "full_answer": "It is striking in this question that we are told that our patient has returned to Spain, but not from where (which could help us to make digressions about the possible cause). However, the description of the picture is very exhaustive as well as typical, so the question is easy. It is a disease that produces fever, arthralgias and a generalized exanthema with petechiae on the lower limbs. The CBC shows leukopenia accompanied by plateletopenia and elevated transaminases. Malaria has been ruled out as a possible cause. All the data are highly suggestive of Dengue, especially the plateletopenia and petechiae (suggestive of capillary fragility), answer 3 correct.", "type": "INFECTIOUS DISEASES", "options": {"1": "Coronavirus infection.", "2": "Lymphocytic choriomeningitis.", "3": "Dengue.", "4": "Chikungunya virus infection.", "5": "Saint Louis encephalitis."}, "correct_option": 3, "explanations": {"1": {"exist": false, "char_ranges": [], "word_ranges": [], "text": ""}, "2": {"exist": false, "char_ranges": [], "word_ranges": [], "text": ""}, "3": {"exist": true, "char_ranges": [[523, 649]], "word_ranges": [[87, 104]], "text": "All the data are highly suggestive of Dengue, especially the plateletopenia and petechiae (suggestive of capillary fragility),"}, "4": {"exist": false, "char_ranges": [], "word_ranges": [], "text": ""}, "5": {"exist": false, "char_ranges": [], "word_ranges": [], "text": ""}}} {"id": 88, "year": 2012, "question_id_specific": 56, "full_question": "A 78-year-old woman with a history of hypertension treated with amlodipine was admitted 3 months ago for heart failure and atrial fibrillation. She came to your office for a cardiological check-up. The discharge report shows the result of the echocardiogram showing moderate left ventricular dysfunction and the discharge treatment: amlodipine was discontinued and furosemide and enalapril were started. The patient reports feeling better, without fatigue and with less leg edema, but she is still not completely well. Physical examination showed no signs of heart failure, but her blood pressure and heart rate were 150/90 mmHg and 120 bpm, respectively. The ECG shows atrial fibrillation at 110-120 bpm. Which treatment is most likely to improve symptoms and reduce mortality from cardiovascular events?", "full_answer": "Typical very long and rambling question but not really too complicated... The patient is in heart failure with moderate ventricular dysfunction and also presents atrial fibrillation; she has criteria for anticoagulation, and between the two options that include acenocoumarol we are inclined to choose option 4, since it includes carvedilol, a drug that has been shown to increase survival in heart failure. Verapamil (and calcium antagonists in general) are contraindicated in systolic heart failure: they do not increase survival (they even increase morbidity and mortality) and specifically the non-dihydropyridines (verapamil and diltiazem) should not be associated with beta-blockers due to the increased risk of conduction system blockade.", "type": "CARDIOLOGY AND VASCULAR SURGERY", "options": {"1": "Digoxin and acetylsalicylic acid.", "2": "Atenolol and acetylsalicylic acid.", "3": "Verapamil and acenocoumarol.", "4": "Carvedilol and acenocoumarol.", "5": "Carvedilol and clopidogrel."}, "correct_option": 4, "explanations": {"1": {"exist": false, "char_ranges": [], "word_ranges": [], "text": ""}, "2": {"exist": false, "char_ranges": [], "word_ranges": [], "text": ""}, "3": {"exist": true, "char_ranges": [[408, 501]], "word_ranges": [[62, 74]], "text": "Verapamil (and calcium antagonists in general) are contraindicated in systolic heart failure:"}, "4": {"exist": true, "char_ranges": [[182, 407]], "word_ranges": [[25, 62]], "text": "fibrillation; she has criteria for anticoagulation, and between the two options that include acenocoumarol we are inclined to choose option 4, since it includes carvedilol, a drug that has been shown to increase survival in heart failure."}, "5": {"exist": false, "char_ranges": [], "word_ranges": [], "text": ""}}} {"id": 536, "year": 2021, "question_id_specific": 122, "full_question": "75-year-old male, hypertensive on treatment with enalapril (20 mg/day), who consults for deterioration of general condition, bitemporal headache and mandibular claudication of 3 weeks of evolution. In addition, in the last hours, he refers two episodes of amaurosis fugax of the left eye. Physical examination revealed thickening and absence of pulse in the left temporal artery. Analysis: CRP 6 mg/dl (VN <1); hemoglobin 10.5 g/dl; ESR 92 mm. Chest X-ray normal. Considering the most likely diagnosis, check the correct option regarding treatment:", "full_answer": "Clinical case of giant cell arteritis. Corticosteroid therapy remains the first choice (answer 1 incorrect), to be administered as pulses (doses differ according to the literature, but at least 125 mg/d) in case of ocular involvement (answer 3 incorrect). The current treatment of choice for corticosteroid sparing is Tocilizumab (answer 4 incorrect), which has also been shown to reduce recurrences (answer 2 correct).", "type": "RHEUMATOLOGY", "options": {"1": "Rituximab should be administered, since glucocorticoids have been relegated to second-line treatment due to their side effects.", "2": "Tocilizumab has been shown to be effective in reducing recurrences and cumulative prednisone dose in more than 50% of patients.", "3": "In the presence of ischemic symptoms, and in order not to worsen the vascular risk of the patient, the use of glucocorticoids at doses higher than 30 mg/day should be avoided.", "4": "In conjunction with glucocorticoid boluses of 1 g/day, infliximab should be used as a first-line drug for remission induction.", "5": NaN}, "correct_option": 2, "explanations": {"1": {"exist": true, "char_ranges": [[0, 108]], "word_ranges": [[0, 15]], "text": "Clinical case of giant cell arteritis. Corticosteroid therapy remains the first choice (answer 1 incorrect),"}, "2": {"exist": true, "char_ranges": [[352, 419]], "word_ranges": [[52, 63]], "text": "which has also been shown to reduce recurrences (answer 2 correct)."}, "3": {"exist": true, "char_ranges": [[109, 255]], "word_ranges": [[15, 39]], "text": "to be administered as pulses (doses differ according to the literature, but at least 125 mg/d) in case of ocular involvement (answer 3 incorrect)."}, "4": {"exist": true, "char_ranges": [[256, 351]], "word_ranges": [[39, 52]], "text": "The current treatment of choice for corticosteroid sparing is Tocilizumab (answer 4 incorrect),"}, "5": {"exist": false, "char_ranges": [], "word_ranges": [], "text": ""}}} {"id": 492, "year": 2020, "question_id_specific": 107, "full_question": "An 18-year-old patient who comes to the emergency department with epistaxis of several days' evolution, with no personal or family history of interest. On examination he is afebrile, multiple ecchymoses are observed, no splenomegaly is palpable. Laboratory tests: leukocytes 7.2 x103/μL, Hb 12.3 g/dL, platelets 6.0 x103/μL. Thrombocytopenia is confirmed in the smear, where platelets of increased size are observed. Coagulation and biochemistry studies are normal. What do you consider the most likely diagnosis?", "full_answer": "This is an easy clinical case only presenting thrombopenia with clinic, petechiae and epistaxis. 1 is false, there would be fever, hemolysis data, neurological symptoms, in the smear there would be schistocytes. In 2 the coagulation must be altered and in this case it is not mentioned.", "type": "HEMATOLOGY", "options": {"1": "Thrombotic thrombocytopenic purpura.", "2": "Disseminated intravascular coagulation.", "3": "Thrombocytopenia induced by infection.", "4": "Primary immune thrombocytopenia.", "5": NaN}, "correct_option": 4, "explanations": {"1": {"exist": true, "char_ranges": [[97, 211]], "word_ranges": [[14, 32]], "text": "1 is false, there would be fever, hemolysis data, neurological symptoms, in the smear there would be schistocytes."}, "2": {"exist": true, "char_ranges": [[212, 286]], "word_ranges": [[32, 47]], "text": "In 2 the coagulation must be altered and in this case it is not mentioned."}, "3": {"exist": false, "char_ranges": [], "word_ranges": [], "text": ""}, "4": {"exist": false, "char_ranges": [], "word_ranges": [], "text": ""}, "5": {"exist": false, "char_ranges": [], "word_ranges": [], "text": ""}}} {"id": 599, "year": 2022, "question_id_specific": 183, "full_question": "A 45-year-old woman with no history of interest consults for dyspneic sensation of about 4 days of evolution. Examination shows rhythmic tachycardia, without murmurs and pulmonary auscultation is normal. Arterial blood gases showed a pO2 of 70 mmHg and a pCO2 of 32 mmHg. Hemogram, renal and hepatic function are normal. Prothrombin time 90%, activated partial thromboplastin time (APTT) ratio of 2 to control (N <1.2). Which of the following is the most likely diagnosis?", "full_answer": "In this clinical case, we are presented with a woman who rapidly presents dyspnea with hypoxemia, a picture compatible with pulmonary thromboembolism. The prothrombin time of 90% and the aPTT with a ratio of 2, is translating a coagulation problem; and antiphospholipid antibodies can be detected through the prolongation of phospholipid-dependent coagulation tests, correct answer 1. On the other hand, although factor V Leiden has been identified as a common cause of familial thrombosis, it would not alter the clotting time. In the case of hemophilia there would be a problem in blood coagulation but it is not a frequent cause of thromboembolism. Finally, although acute pericarditis could manifest itself in 4 days and with tachycardia, the rest of the symptoms or analytical parameters are not characteristic.", "type": "RHEUMATOLOGY", "options": {"1": "Antiphospholipid syndrome.", "2": "Factor V of Leiden.", "3": "Hemophilia.", "4": "Acute pericarditis.", "5": NaN}, "correct_option": 1, "explanations": {"1": {"exist": true, "char_ranges": [[0, 384]], "word_ranges": [[0, 56]], "text": "In this clinical case, we are presented with a woman who rapidly presents dyspnea with hypoxemia, a picture compatible with pulmonary thromboembolism. The prothrombin time of 90% and the aPTT with a ratio of 2, is translating a coagulation problem; and antiphospholipid antibodies can be detected through the prolongation of phospholipid-dependent coagulation tests, correct answer 1."}, "2": {"exist": true, "char_ranges": [[385, 528]], "word_ranges": [[56, 81]], "text": "On the other hand, although factor V Leiden has been identified as a common cause of familial thrombosis, it would not alter the clotting time."}, "3": {"exist": true, "char_ranges": [[529, 651]], "word_ranges": [[81, 103]], "text": "In the case of hemophilia there would be a problem in blood coagulation but it is not a frequent cause of thromboembolism."}, "4": {"exist": true, "char_ranges": [[652, 816]], "word_ranges": [[103, 127]], "text": "Finally, although acute pericarditis could manifest itself in 4 days and with tachycardia, the rest of the symptoms or analytical parameters are not characteristic."}, "5": {"exist": false, "char_ranges": [], "word_ranges": [], "text": ""}}} {"id": 134, "year": 2012, "question_id_specific": 169, "full_question": "A 35-year-old man comes to the emergency department with pain in his right eye of 3 days' evolution. Biomicroscopic examination of the anterior segment shows, after fluorescein staining, a central corneal ulcer in the form of a dendrite. What is the diagnosis?", "full_answer": "Dendritic corneal ulcer is typical of herpes simplex keratitis. It presents with pain, not too striking, and scarce or absent red eye. There are ulcers resembling dendrites that may confuse us in the diagnosis. These pseudodendrites are found in herpes Zoster and in corneal abrasions in resolution. Although in practice we should pay attention to the shape of the dendrite (the true dendrite is deeper and has terminal bulbs at the end of each branch), and pay attention to the evolution of the pain (in the pseudodendrite after abrasion the pain should improve), when answering a MIR question it is simpler: if it is a dendrite, the first thing we have to think is herpes. Fungal and bacterial keratitis have corneal infiltrates. And glaucomatocyclitic crisis (Posner Schlossman syndrome) is a type of idiopathic hypertensive anterior uveitis, which does not present with ulceration.", "type": "OPHTHALMOLOGY", "options": {"1": "Herpetic keratitis.", "2": "Corneal abrasion.", "3": "Fungal keratitis.", "4": "Bacterial keratitis.", "5": "Glaucomatocyclic crisis."}, "correct_option": 1, "explanations": {"1": {"exist": true, "char_ranges": [[0, 134]], "word_ranges": [[0, 22]], "text": "Dendritic corneal ulcer is typical of herpes simplex keratitis. It presents with pain, not too striking, and scarce or absent red eye."}, "2": {"exist": true, "char_ranges": [[610, 674]], "word_ranges": [[101, 115]], "text": "if it is a dendrite, the first thing we have to think is herpes."}, "3": {"exist": true, "char_ranges": [[675, 731]], "word_ranges": [[115, 122]], "text": "Fungal and bacterial keratitis have corneal infiltrates."}, "4": {"exist": true, "char_ranges": [[675, 731]], "word_ranges": [[115, 122]], "text": "Fungal and bacterial keratitis have corneal infiltrates."}, "5": {"exist": true, "char_ranges": [[736, 885]], "word_ranges": [[123, 142]], "text": "glaucomatocyclitic crisis (Posner Schlossman syndrome) is a type of idiopathic hypertensive anterior uveitis, which does not present with ulceration."}}} {"id": 245, "year": 2014, "question_id_specific": 115, "full_question": "A patient undergoing chemotherapy for leukemia is admitted for pneumonia for which treatment with cefepime has been prescribed. A chest X-ray/CT scan shows an infiltrate with halo sign and crescentic meniscus. The lesion is peripheral and a transthoracic puncture is indicated for sampling. Pending definitive histologic and microbiologic results, what antimicrobial would you add to the treatment?", "full_answer": "Direct response question. This is a patient undergoing chemotherapy with respiratory symptoms and the \"halo sign\" on X-ray, very suggestive of invasive pulmonary aspergillosis. Treatment is with voriconazole. Aspergillus is resistant to fluconazole, so it would not be a valid option.", "type": "INFECTIOUS DISEASES", "options": {"1": "Ganciclovir.", "2": "Caspofungin.", "3": "Fluconazole.", "4": "Piperacillin-tazobactam.", "5": "Voriconazole."}, "correct_option": 5, "explanations": {"1": {"exist": false, "char_ranges": [], "word_ranges": [], "text": ""}, "2": {"exist": false, "char_ranges": [], "word_ranges": [], "text": ""}, "3": {"exist": true, "char_ranges": [[209, 284]], "word_ranges": [[28, 41]], "text": "Aspergillus is resistant to fluconazole, so it would not be a valid option."}, "4": {"exist": false, "char_ranges": [], "word_ranges": [], "text": ""}, "5": {"exist": true, "char_ranges": [[26, 208]], "word_ranges": [[3, 28]], "text": "This is a patient undergoing chemotherapy with respiratory symptoms and the \"halo sign\" on X-ray, very suggestive of invasive pulmonary aspergillosis. Treatment is with voriconazole."}}} {"id": 387, "year": 2016, "question_id_specific": 156, "full_question": "A 5-month-old infant exclusively breastfed until the present, who due to maternal work was introduced a bottle with formula and a few days ago was added cereals with and without gluten. Family history: 28-year-old mother with asthma, 32-year-old healthy father and 5-year-old brother with celiac disease and atopic dermatitis. As personal history, pregnancy without incidents and delivery by cesarean section, having been offered a bottle with starter formula on the first day of life in the maternity ward. Since a few days ago she starts with abdominal distension, diarrhea stools, refusal to feed, peribuccal erythema with longer duration after bottle feeding and lately vomiting after feeding, tolerating well the breast. What is the most probable diagnosis?", "full_answer": "Child with multiple family history of allergy (asthma, atopy), who has received artificial milk in the first hours of life (sensitization to cow's milk protein) and who, coinciding with the intake of artificial milk, starts digestive symptoms and exanthema = APLV.", "type": "PEDIATRICS", "options": {"1": "Celiac disease.", "2": "Acute gastroenteritis.", "3": "Cow's milk protein allergy.", "4": "Bottle allergy.", "5": NaN}, "correct_option": 3, "explanations": {"1": {"exist": false, "char_ranges": [], "word_ranges": [], "text": ""}, "2": {"exist": false, "char_ranges": [], "word_ranges": [], "text": ""}, "3": {"exist": true, "char_ranges": [[0, 264]], "word_ranges": [[0, 41]], "text": "Child with multiple family history of allergy (asthma, atopy), who has received artificial milk in the first hours of life (sensitization to cow's milk protein) and who, coinciding with the intake of artificial milk, starts digestive symptoms and exanthema = APLV."}, "4": {"exist": false, "char_ranges": [], "word_ranges": [], "text": ""}, "5": {"exist": false, "char_ranges": [], "word_ranges": [], "text": ""}}} {"id": 41, "year": 2011, "question_id_specific": 139, "full_question": "Given a patient with poor general condition, fever (axillary temperature 39.5º), bilateral submandibular swelling of six days' evolution, mouth pain and trismus, which of the following statements is correct?", "full_answer": "This patient is in serious condition. The trismus and fever suggest a deep cervical space infection and that is not a game because the airway can become obstructed. Knowing this, options 1, 2 and 5 are out of place and I am left with 3 and 4. There may already be mediastinitis; after all, that is the natural history of cervical deep space infections, but answer 4 is more correct and besides, we are in the ENT block, the airway specialty. I like this question because it requires knowledge that is important in clinical practice.", "type": "OTORHINOLARYNGOLOGY AND MAXILLOFACIAL SURGERY", "options": {"1": "The first diagnostic probability is carcinoma of the floor of the mouth with bilateral cervical metastases.", "2": "The therapeutic priority is to ensure the patient's nutrition.", "3": "We are almost certainly facing a mediastinitis.", "4": "We must consider as a priority the risk of airway obstruction.", "5": "We must perform as a first diagnostic measure a cytology by fine needle aspiration puncture."}, "correct_option": 4, "explanations": {"1": {"exist": true, "char_ranges": [[0, 214]], "word_ranges": [[0, 39]], "text": "This patient is in serious condition. The trismus and fever suggest a deep cervical space infection and that is not a game because the airway can become obstructed. Knowing this, options 1, 2 and 5 are out of place"}, "2": {"exist": true, "char_ranges": [[0, 214]], "word_ranges": [[0, 39]], "text": "This patient is in serious condition. The trismus and fever suggest a deep cervical space infection and that is not a game because the airway can become obstructed. Knowing this, options 1, 2 and 5 are out of place"}, "3": {"exist": false, "char_ranges": [], "word_ranges": [], "text": ""}, "4": {"exist": false, "char_ranges": [], "word_ranges": [], "text": ""}, "5": {"exist": true, "char_ranges": [[0, 214]], "word_ranges": [[0, 39]], "text": "This patient is in serious condition. The trismus and fever suggest a deep cervical space infection and that is not a game because the airway can become obstructed. Knowing this, options 1, 2 and 5 are out of place"}}} {"id": 385, "year": 2016, "question_id_specific": 153, "full_question": "What is the most likely diagnosis of a newborn with microcephaly, intrauterine growth retardation, congenital heart disease, vertical talus foot and a peculiar facies (microphthalmia, small palpebral fissures, micrognathia and dysplastic ears), hands with the index and little finger above the middle and ring fingers?", "full_answer": "You are faithfully describing an Edwards' syndrome. Although the cardiopathies and the alteration of the foot that he describes are common to several chromosomopathies, the alteration of the fingers and toes is very characteristic of Edwards' syndrome.", "type": "PEDIATRICS", "options": {"1": "Trisomy 18 (Edwards' syndrome).", "2": "Trisomy 13 (Patau's syndrome).", "3": "Trisomy 21 (Down syndrome).", "4": "Trisomy 9.", "5": NaN}, "correct_option": 1, "explanations": {"1": {"exist": true, "char_ranges": [[52, 252]], "word_ranges": [[7, 37]], "text": "Although the cardiopathies and the alteration of the foot that he describes are common to several chromosomopathies, the alteration of the fingers and toes is very characteristic of Edwards' syndrome."}, "2": {"exist": true, "char_ranges": [[52, 252]], "word_ranges": [[7, 37]], "text": "Although the cardiopathies and the alteration of the foot that he describes are common to several chromosomopathies, the alteration of the fingers and toes is very characteristic of Edwards' syndrome."}, "3": {"exist": false, "char_ranges": [], "word_ranges": [], "text": ""}, "4": {"exist": true, "char_ranges": [[52, 252]], "word_ranges": [[7, 37]], "text": "Although the cardiopathies and the alteration of the foot that he describes are common to several chromosomopathies, the alteration of the fingers and toes is very characteristic of Edwards' syndrome."}, "5": {"exist": false, "char_ranges": [], "word_ranges": [], "text": ""}}} {"id": 194, "year": 2013, "question_id_specific": 162, "full_question": "A 3-year-old girl from Bangladesh consults for intermittent febrile episodes of 3 weeks' duration, associated with weakness and loss of appetite. Physical examination showed marked splenomegaly and mucocutaneous pallor. Complementary tests include: hemoglobin 8.5 mg/dL, hematocrit 26%, MCV 86 fL, MCH 29 pg, leukocytes 2800/mL with 300 neutrophils /ml, platelets 54000/ml, GOT 85 U/l, GPT92 U/l and polyclonal hypergammaglobulinemia in serum proteinogram. Indicate the most probable diagnosis with the data available at this time:", "full_answer": "Data such as pancytopenia and hypergammaglobulinemia are very characteristic of kala-azar. The origin of the girl is also indicative of this diagnosis, although it is a disease that also occurs in the Mediterranean area. In the differential diagnosis, it is important to keep in mind the possibility of leukosis.", "type": "PEDIATRICS", "options": {"1": "Acute lymphoblastic leukemia.", "2": "Burkitt's lymphoma.", "3": "Visceral leishmaniasis.", "4": "Miliary tuberculosis.", "5": "Chronic malaria."}, "correct_option": 3, "explanations": {"1": {"exist": false, "char_ranges": [], "word_ranges": [], "text": ""}, "2": {"exist": false, "char_ranges": [], "word_ranges": [], "text": ""}, "3": {"exist": true, "char_ranges": [[0, 151]], "word_ranges": [[0, 22]], "text": "Data such as pancytopenia and hypergammaglobulinemia are very characteristic of kala-azar. The origin of the girl is also indicative of this diagnosis,"}, "4": {"exist": false, "char_ranges": [], "word_ranges": [], "text": ""}, "5": {"exist": false, "char_ranges": [], "word_ranges": [], "text": ""}}} {"id": 371, "year": 2016, "question_id_specific": 126, "full_question": "A 59-year-old woman with a personal history of breast cancer operated on 8 months ago and osteoporosis, suffers a fall and has a fracture of the neck of the right femur. She undergoes surgery and remains in the hospital resting in a cast for 10 days. A week after her discharge, she suddenly feels short of breath, which leads her to consult her family doctor who notices that her right calf has edema and pain on palpation. Which clinical elements would NOT be important in determining that she is at high clinical risk for pulmonary thromboembolism?", "full_answer": "The other antecedents and findings are very directly related to the increased risk of thrombotic phenomena. Osteoporosis is not a risk factor for thrombosis.", "type": "PULMONOLOGY AND THORACIC SURGERY", "options": {"1": "History of breast cancer.", "2": "History of osteoporosis.", "3": "History of having been at rest for more than 3 days.", "4": "Presence of unilateral edema of the right calf.", "5": NaN}, "correct_option": 2, "explanations": {"1": {"exist": true, "char_ranges": [[0, 107]], "word_ranges": [[0, 16]], "text": "The other antecedents and findings are very directly related to the increased risk of thrombotic phenomena."}, "2": {"exist": true, "char_ranges": [[108, 157]], "word_ranges": [[16, 24]], "text": "Osteoporosis is not a risk factor for thrombosis."}, "3": {"exist": true, "char_ranges": [[0, 107]], "word_ranges": [[0, 16]], "text": "The other antecedents and findings are very directly related to the increased risk of thrombotic phenomena."}, "4": {"exist": true, "char_ranges": [[0, 107]], "word_ranges": [[0, 16]], "text": "The other antecedents and findings are very directly related to the increased risk of thrombotic phenomena."}, "5": {"exist": false, "char_ranges": [], "word_ranges": [], "text": ""}}} {"id": 283, "year": 2016, "question_id_specific": 56, "full_question": "A 54-year-old patient is admitted for thermometric fever of 38°C in the previous five days and dyspnea at rest (LV) that appeared 6 hours before coming to the hospital. Examination in the emergency room was compatible with heart failure and the ECG showed complete atrioventricular block with an escape ventricular rate of 45 bpm. Signs of heart failure are refractory to medical treatment and transesophageal echocardiography shows an aortic valve with an effective regurgitant orifice of 0.5 cm2_ Serial cultures are positive for Streptococcus gallolyticus. Indicate the best course of action:", "full_answer": "Cardiac surgery for aortic valve replacement by mechanical prosthesis with antibiotic therapy according to antibiogram.", "type": "CARDIOLOGY AND VASCULAR SURGERY", "options": {"1": "Cardiac surgery for aortic valve replacement by mechanical prosthesis with antibiotic therapy according to antibiogram.", "2": "Antibiotic therapy according to antibiogram and implantation of intra-aortic balloon counterpulsation and transient pacemaker for up to 3 weeks, after which a permanent pacemaker will be implanted.", "3": "Implantation of transient pacemaker, antibiotic therapy according to antibiogram and percutaneous implantation of aortic valve prosthesis.", "4": "Urgent implantation of definitive pacemaker with antibiotic therapy according to antibiogram for 6 weeks.", "5": NaN}, "correct_option": 1, "explanations": {"1": {"exist": true, "char_ranges": [[0, 119]], "word_ranges": [[0, 15]], "text": "Cardiac surgery for aortic valve replacement by mechanical prosthesis with antibiotic therapy according to antibiogram."}, "2": {"exist": false, "char_ranges": [], "word_ranges": [], "text": ""}, "3": {"exist": false, "char_ranges": [], "word_ranges": [], "text": ""}, "4": {"exist": false, "char_ranges": [], "word_ranges": [], "text": ""}, "5": {"exist": false, "char_ranges": [], "word_ranges": [], "text": ""}}} {"id": 109, "year": 2012, "question_id_specific": 149, "full_question": "A 32-year-old asymptomatic woman goes for a gynecological health check-up because she wishes to become pregnant. During this check-up, a 4cm uterine myoma is detected, partly intramural and partly subserosal, located on the anterior aspect of the uterus and which does not deform the endometrial cavity. What approach would you recommend?", "full_answer": "The correct answer is 5. An asymptomatic myoma of this size that does not deform the endometrial cavity is not a problem for pregnancy.", "type": "GYNECOLOGY AND OBSTETRICS", "options": {"1": "Laparoscopic myomectomy.", "2": "Laparotomic myomectomy.", "3": "Myoma embolization by arterial catheterization.", "4": "Treatment with Gn-Rh analogues for three months before attempting pregnancy.", "5": "Attempting pregnancy without any previous treatment."}, "correct_option": 5, "explanations": {"1": {"exist": false, "char_ranges": [], "word_ranges": [], "text": ""}, "2": {"exist": false, "char_ranges": [], "word_ranges": [], "text": ""}, "3": {"exist": false, "char_ranges": [], "word_ranges": [], "text": ""}, "4": {"exist": false, "char_ranges": [], "word_ranges": [], "text": ""}, "5": {"exist": true, "char_ranges": [[25, 135]], "word_ranges": [[5, 24]], "text": "An asymptomatic myoma of this size that does not deform the endometrial cavity is not a problem for pregnancy."}}} {"id": 92, "year": 2012, "question_id_specific": 235, "full_question": "A 60-year-old patient diagnosed with breast neoplasia 10 years ago. She underwent treatment with radiochemotherapy and then hormonal therapy for 5 years. A bone scan study performed for polytopic bone pain showed the presence of bone metastases. She is currently under treatment with minor opioids and NSAIDs with good pain control. She consulted for headache that did not subside with the current analgesia and a CT scan of the brain showed images compatible with brain metastases. In relation to the treatment of pain, indicate the CORRECT one:", "full_answer": "Brain metastases produce pain due to cranial hypertension. The treatment of choice is corticosteroids. Dexamethasone in oral doses of 4 to 16 mg/day is the treatment of choice.", "type": "PALLIATIVE CARE", "options": {"1": "Switch to major opioids.", "2": "Extra doses of opioids should be administered as necessary.", "3": "Corticoids should be added.", "4": "Switch to a major opioid and maintain NSAIDs.", "5": "The patient should be admitted for intravenous treatment with major opioid."}, "correct_option": 3, "explanations": {"1": {"exist": false, "char_ranges": [], "word_ranges": [], "text": ""}, "2": {"exist": false, "char_ranges": [], "word_ranges": [], "text": ""}, "3": {"exist": true, "char_ranges": [[0, 102]], "word_ranges": [[0, 14]], "text": "Brain metastases produce pain due to cranial hypertension. The treatment of choice is corticosteroids."}, "4": {"exist": false, "char_ranges": [], "word_ranges": [], "text": ""}, "5": {"exist": false, "char_ranges": [], "word_ranges": [], "text": ""}}} {"id": 342, "year": 2016, "question_id_specific": 158, "full_question": "A 24-year-old woman, primigestation, suffers a spontaneous abortion at 7 weeks of gestation. The anatomopathological study of the abortive remains indicates molar disease. We should inform you that:", "full_answer": "Gestational trophoblastic disease should be followed up (regardless of whether the evacuation has been complete or not) and the patient should be advised not to become pregnant again until at least 6 months with negative BHCG titers. The risk of recurrence in another pregnancy is low (1/55) and 90% of cases progress satisfactorily without developing neoplasia.", "type": "GYNECOLOGY AND OBSTETRICS", "options": {"1": "The risk of a new molar gestation in a future pregnancy is 50%.", "2": "She should not become pregnant until she has undergone periodic controls and has spent one year with negative BHCG levels.", "3": "Subsequent controls are not necessary if the evacuation of the trophoblastic tissue was complete.", "4": "It is necessary to carry out periodic controls since 40% of the cases will develop a gestational trophoblastic neoplasia.", "5": NaN}, "correct_option": 2, "explanations": {"1": {"exist": false, "char_ranges": [], "word_ranges": [], "text": ""}, "2": {"exist": true, "char_ranges": [[0, 233]], "word_ranges": [[0, 37]], "text": "Gestational trophoblastic disease should be followed up (regardless of whether the evacuation has been complete or not) and the patient should be advised not to become pregnant again until at least 6 months with negative BHCG titers."}, "3": {"exist": false, "char_ranges": [], "word_ranges": [], "text": ""}, "4": {"exist": false, "char_ranges": [], "word_ranges": [], "text": ""}, "5": {"exist": false, "char_ranges": [], "word_ranges": [], "text": ""}}} {"id": 357, "year": 2016, "question_id_specific": 232, "full_question": "A 24-year-old woman consults after noticing inguinal lymphadenopathy. The interrogation does not reveal any local discomfort or data suggestive of sexually transmitted infection. The examination revealed two lymphadenopathies, one in each groin, 1 cm in diameter, soft, mobile, non-painful. No skin lesions are seen on the lower limbs, anus or perineum. Which test do you consider essential?", "full_answer": "The presence of inguinal nodes up to 1-1.5cm may be normal in the absence of symptoms. In this case, they are very nonspecific nodes, without signs of malignancy (mobile, soft, not painful) that do not lead us to suspect neoplastic infiltration and the lack of other clinical signs also leads us to rule out the presence of STD.", "type": "GYNECOLOGY AND OBSTETRICS", "options": {"1": "A lues serology since it is most likely a Treponema pallidum infection.", "2": "A gynecological examination to rule out ovarian cancer.", "3": "By the clinical characteristics it seems to be normal lymph nodes and complementary explorations should not be done.", "4": "A Paul-Bunell test should be performed to rule out infectious mononucleosis.", "5": NaN}, "correct_option": 3, "explanations": {"1": {"exist": false, "char_ranges": [], "word_ranges": [], "text": ""}, "2": {"exist": false, "char_ranges": [], "word_ranges": [], "text": ""}, "3": {"exist": true, "char_ranges": [[0, 328]], "word_ranges": [[0, 58]], "text": "The presence of inguinal nodes up to 1-1.5cm may be normal in the absence of symptoms. In this case, they are very nonspecific nodes, without signs of malignancy (mobile, soft, not painful) that do not lead us to suspect neoplastic infiltration and the lack of other clinical signs also leads us to rule out the presence of STD."}, "4": {"exist": false, "char_ranges": [], "word_ranges": [], "text": ""}, "5": {"exist": false, "char_ranges": [], "word_ranges": [], "text": ""}}} {"id": 596, "year": 2022, "question_id_specific": 121, "full_question": "79-year-old woman admitted for an osteoporotic hip fracture. Regarding secondary prevention of fragility fractures, point out the WRONG answer:", "full_answer": "This question corresponds to the section on traumatology; however, it could be answered with knowledge of rheumatology and with the osteoporosis recommendations of the Spanish Society of Rheumatology. In osteoporosis, one of the main risks associated with increased fracture risk is low adherence to treatment, so answer 1 is correct. Answer 2 is found in the SER guidelines, which confirm that some studies conclude that bone remodeling markers can be useful for early monitoring of adherence and response to treatment. Answer 4 is correct because again in the SER 2019 guidelines they quote: \"The current scientific evidence allows us to affirm that neither increasing dietary calcium nor taking calcium supplements alone protects against the appearance of fractures\". Therefore, the correct answer to this question is option 3. Patients on pharmacological treatment for OP should use calcium and vitamin D supplements because practically all clinical trials that have demonstrated efficacy of antiosteoporotic drugs routinely include calcium supplements and cholecalciferol (vitamin D3), but not in monotherapy.", "type": "RHEUMATOLOGY", "options": {"1": "Low adherence to treatment is associated with an increased risk of fracture.", "2": "Bone remodeling markers may be useful for early monitoring of response to treatment.", "3": "Vitamin D monotherapy is effective in reducing such fractures in non-institutionalized elderly.", "4": "Increasing dietary calcium or taking calcium supplements alone does not protect against the occurrence of fractures.", "5": NaN}, "correct_option": 3, "explanations": {"1": {"exist": true, "char_ranges": [[201, 334]], "word_ranges": [[28, 50]], "text": "In osteoporosis, one of the main risks associated with increased fracture risk is low adherence to treatment, so answer 1 is correct."}, "2": {"exist": true, "char_ranges": [[335, 520]], "word_ranges": [[50, 80]], "text": "Answer 2 is found in the SER guidelines, which confirm that some studies conclude that bone remodeling markers can be useful for early monitoring of adherence and response to treatment."}, "3": {"exist": true, "char_ranges": [[831, 1114]], "word_ranges": [[127, 164]], "text": "Patients on pharmacological treatment for OP should use calcium and vitamin D supplements because practically all clinical trials that have demonstrated efficacy of antiosteoporotic drugs routinely include calcium supplements and cholecalciferol (vitamin D3), but not in monotherapy."}, "4": {"exist": true, "char_ranges": [[521, 770]], "word_ranges": [[80, 117]], "text": "Answer 4 is correct because again in the SER 2019 guidelines they quote: \"The current scientific evidence allows us to affirm that neither increasing dietary calcium nor taking calcium supplements alone protects against the appearance of fractures\"."}, "5": {"exist": false, "char_ranges": [], "word_ranges": [], "text": ""}}} {"id": 334, "year": 2016, "question_id_specific": 82, "full_question": "A woman diagnosed with type 1 diabetes mellitus for 24 years. She comes to the office with 3 months of clinical evolution of tingling in both feet, with distribution in sock, with paresthetic pain and sensation of hot feet predominantly at night, which significantly interferes with sleep. Which of the following drugs would you use in the first line for the treatment of her pathology?", "full_answer": "Duloxetine; the clinical picture described is of diabetic neuropathy. Of the drugs proposed, the first choice would be a dual antidepressant such as duloxetine.", "type": "PHARMACOLOGY", "options": {"1": "Ibuprofen.", "2": "Oxycodone.", "3": "Duloxetine.", "4": "Paracetamol.", "5": NaN}, "correct_option": 3, "explanations": {"1": {"exist": false, "char_ranges": [], "word_ranges": [], "text": ""}, "2": {"exist": false, "char_ranges": [], "word_ranges": [], "text": ""}, "3": {"exist": true, "char_ranges": [[12, 160]], "word_ranges": [[1, 24]], "text": "the clinical picture described is of diabetic neuropathy. Of the drugs proposed, the first choice would be a dual antidepressant such as duloxetine."}, "4": {"exist": false, "char_ranges": [], "word_ranges": [], "text": ""}, "5": {"exist": false, "char_ranges": [], "word_ranges": [], "text": ""}}} {"id": 81, "year": 2012, "question_id_specific": 44, "full_question": "A 78-year-old patient diagnosed with idiopathic dilated cardiomyopathy with mild left ventricular dysfunction (ejection fraction 48%) and chronic atrial fibrillation. Which of the following drugs should be avoided in his treatment?", "full_answer": "Digoxin is useful as a symptomatic treatment (it is a positive inotropic) in heart failure, although it does not improve survival. Carvedilol is a very well tolerated aß-blocker (due to its anti-a1 effect) that is associated with symptomatic and survival improvement. Acenocoumarol (Sintrom) would be indicated in this patient with chronic AF to avoid thromboembolic events. Enalapril and ACE inhibitors in general, by inhibiting the renin-angiotensin-aldosterone axis, suppress part of the neurohormonal effects that occur in heart failure; they improve survival in patients with systolic dysfunction. Ibuprofen and NSAIDs in general are contraindicated in patients with heart failure because they inhibit prostaglandin synthesis at the renal level, causing an increase in systemic vascular resistance, reduced renal perfusion, and inhibition of sodium and water excretion, which can precipitate decompensation of heart failure.", "type": "CARDIOLOGY AND VASCULAR SURGERY", "options": {"1": "Digoxin.", "2": "Carvedilol.", "3": "Acenocoumarol.", "4": "Enalapril.", "5": "Ibuprofen."}, "correct_option": 5, "explanations": {"1": {"exist": true, "char_ranges": [[0, 130]], "word_ranges": [[0, 21]], "text": "Digoxin is useful as a symptomatic treatment (it is a positive inotropic) in heart failure, although it does not improve survival."}, "2": {"exist": true, "char_ranges": [[131, 267]], "word_ranges": [[21, 41]], "text": "Carvedilol is a very well tolerated aß-blocker (due to its anti-a1 effect) that is associated with symptomatic and survival improvement."}, "3": {"exist": true, "char_ranges": [[268, 374]], "word_ranges": [[41, 56]], "text": "Acenocoumarol (Sintrom) would be indicated in this patient with chronic AF to avoid thromboembolic events."}, "4": {"exist": true, "char_ranges": [[375, 602]], "word_ranges": [[56, 86]], "text": "Enalapril and ACE inhibitors in general, by inhibiting the renin-angiotensin-aldosterone axis, suppress part of the neurohormonal effects that occur in heart failure; they improve survival in patients with systolic dysfunction."}, "5": {"exist": true, "char_ranges": [[603, 750]], "word_ranges": [[86, 107]], "text": "Ibuprofen and NSAIDs in general are contraindicated in patients with heart failure because they inhibit prostaglandin synthesis at the renal level,"}}} {"id": 386, "year": 2016, "question_id_specific": 154, "full_question": "The appearance of a high fever of 39°C in a 10-month-old infant that abruptly subsides after 3-5 days, followed by a cephalocaudal morbilliform exanthema with enanthema consisting of reddish papules on the palate, and resolving within a week, associated with good general condition, is usually caused by:", "full_answer": "The question of exanthematous diseases that pediatricians and MIR exam takers like so much. By the picture of high fever of several days that suddenly subsides with the appearance of a rash, we have a sudden exanthem, caused by Herpes virus type 6 (remember the mnemonic rule of \"sudden Hexanthem\"). Parvovirus B19 is the cause of erythema infectiosum or \"fifth disease\", whose characteristic rash is on the cheeks, giving the appearance of a slapped face. Coxackie A16 causes herpangina and also boca-mano-pie disease, which presents an initially macular rash, which does not respect palms or soles and evolves into bursting vesicles. Finally, EBV infection produces a mononucleosis syndrome with high fever lasting several days, with pharyngeal erythema and grayish exudates on the tonsils. It can present if penicillins are given to treat it (when confused with a streptococcal pharyngotonsillitis) a generalized exanthema. In addition, it also presents with fatigue, hepatic inflammation with increased transaminases and splenomegaly.", "type": "PEDIATRICS", "options": {"1": "Parvovirus B19.", "2": "Herpes virus type 6.", "3": "Coxackie A16 virus.", "4": "Primoinfection by Epstein-Barr virus.", "5": NaN}, "correct_option": 2, "explanations": {"1": {"exist": true, "char_ranges": [[300, 456]], "word_ranges": [[50, 75]], "text": "Parvovirus B19 is the cause of erythema infectiosum or \"fifth disease\", whose characteristic rash is on the cheeks, giving the appearance of a slapped face."}, "2": {"exist": true, "char_ranges": [[92, 299]], "word_ranges": [[14, 50]], "text": "By the picture of high fever of several days that suddenly subsides with the appearance of a rash, we have a sudden exanthem, caused by Herpes virus type 6 (remember the mnemonic rule of \"sudden Hexanthem\")."}, "3": {"exist": true, "char_ranges": [[457, 635]], "word_ranges": [[75, 101]], "text": "Coxackie A16 causes herpangina and also boca-mano-pie disease, which presents an initially macular rash, which does not respect palms or soles and evolves into bursting vesicles."}, "4": {"exist": true, "char_ranges": [[636, 792]], "word_ranges": [[101, 123]], "text": "Finally, EBV infection produces a mononucleosis syndrome with high fever lasting several days, with pharyngeal erythema and grayish exudates on the tonsils."}, "5": {"exist": false, "char_ranges": [], "word_ranges": [], "text": ""}}} {"id": 239, "year": 2014, "question_id_specific": 146, "full_question": "An 80-year-old man is admitted for a sudden onset of aphasia and right hemiparesis. His history includes hypertension, well controlled with diet and cognitive deterioration in the last year under study by his neurologist. The emergency cranial CT scan shows a left frontal lobar hematoma without contrast uptake. What is the most probable cause of the hematoma?", "full_answer": "Amyloid angiopathy is the most common cause of spontaneous non-hypertensive hemorrhage in elderly patients, and is usually lobar in location (as in the case). They often appear in association with Alzheimer's disease (CTO Manual). The response that may cause the most doubts is 2, hypertension, but the most frequent localizations are: putamen, thalamus, pons and cerebellum. Therefore, most of the time these are deep hemorrhages (CTO Manual). In addition, as a clue it tells us that the patient has well-controlled hypertension without drugs, although this data would not serve to rule out the response. Answer 1, a malformation is typical of young patients, rare in older people. It also decreases the chance of not finding a malformation with contrast-enhanced CT. Answer 4, a brain tumor, does not seem likely since the mass is not seen on CT, and as in the previous case, it is more difficult not to find such a lesion after contrast administration. Answer 3 does not seem correct since vasculitis is a rare entity and more so in older people.", "type": "NEUROLOGY", "options": {"1": "Arteriovenous malformation masked by acute hematoma.", "2": "Chronic arterial hypertension.", "3": "Isolated vasculitis of the nervous system.", "4": "Brain tumor.", "5": "Cerebral amyloid angiopathy (congophilic angiopathy)."}, "correct_option": 5, "explanations": {"1": {"exist": true, "char_ranges": [[606, 682]], "word_ranges": [[94, 107]], "text": "Answer 1, a malformation is typical of young patients, rare in older people."}, "2": {"exist": true, "char_ranges": [[299, 444]], "word_ranges": [[46, 67]], "text": "the most frequent localizations are: putamen, thalamus, pons and cerebellum. Therefore, most of the time these are deep hemorrhages (CTO Manual)."}, "3": {"exist": true, "char_ranges": [[956, 1049]], "word_ranges": [[156, 174]], "text": "Answer 3 does not seem correct since vasculitis is a rare entity and more so in older people."}, "4": {"exist": true, "char_ranges": [[769, 955]], "word_ranges": [[120, 156]], "text": "Answer 4, a brain tumor, does not seem likely since the mass is not seen on CT, and as in the previous case, it is more difficult not to find such a lesion after contrast administration."}, "5": {"exist": true, "char_ranges": [[0, 230]], "word_ranges": [[0, 34]], "text": "Amyloid angiopathy is the most common cause of spontaneous non-hypertensive hemorrhage in elderly patients, and is usually lobar in location (as in the case). They often appear in association with Alzheimer's disease (CTO Manual)."}}} {"id": 525, "year": 2021, "question_id_specific": 53, "full_question": "A 42-year-old woman visits her family physician for gonalgia. Opportunistic use is made of the consultation to assess lifestyles, especially smoking. If you want to follow the health education strategy based on the five \"a's\" model, which of the following is NOT included in this strategy:", "full_answer": "The 5A summarizes the activities that should be performed by healthcare personnel during the brief intervention with the patient to stop smoking and are: find out, advise, analyze, help, accompany or agree. Therefore, the one that is not included in the strategy would be option 2: increase.", "type": "PREVENTIVE MEDICINE", "options": {"1": "Inquire: ask about risk factors and risk behaviors (ask the patient if she smokes).", "2": "Increase: increase risk perception to facilitate change (explain the consequences of smoking).", "3": "Advise: give clear, specific and personalized advice (advise smoking cessation).", "4": "Agree: collaboratively agree on goals for change (assess readiness to make a quit attempt).", "5": NaN}, "correct_option": 2, "explanations": {"1": {"exist": false, "char_ranges": [], "word_ranges": [], "text": ""}, "2": {"exist": true, "char_ranges": [[0, 291]], "word_ranges": [[0, 47]], "text": "The 5A summarizes the activities that should be performed by healthcare personnel during the brief intervention with the patient to stop smoking and are: find out, advise, analyze, help, accompany or agree. Therefore, the one that is not included in the strategy would be option 2: increase."}, "3": {"exist": false, "char_ranges": [], "word_ranges": [], "text": ""}, "4": {"exist": false, "char_ranges": [], "word_ranges": [], "text": ""}, "5": {"exist": false, "char_ranges": [], "word_ranges": [], "text": ""}}} {"id": 287, "year": 2016, "question_id_specific": 64, "full_question": "65-year-old patient presenting progressive dyspnea of 5 days of evolution until becoming resting, three-pillow orthopnea and episodes of paroxysmal nocturnal dyspnea. Auscultation highlights bilateral crackles, holosystolic murmur radiating to the axilla and gallop rhythm for the third and fourth sounds. Mark the CORRECT statement:", "full_answer": "The fourth noise usually appears when there is a certain degree of valvular stenosis.", "type": "CARDIOLOGY AND VASCULAR SURGERY", "options": {"1": "The third noise coincides with the rapid filling phase of the ventricular diastole of the cardiac cycle.", "2": "The most likely diagnosis is heart failure.", "3": "The holosystolic murmur may correspond to mitral insufficiency.", "4": "The fourth noise usually appears when there is a certain degree of valvular stenosis.", "5": NaN}, "correct_option": 4, "explanations": {"1": {"exist": false, "char_ranges": [], "word_ranges": [], "text": ""}, "2": {"exist": false, "char_ranges": [], "word_ranges": [], "text": ""}, "3": {"exist": false, "char_ranges": [], "word_ranges": [], "text": ""}, "4": {"exist": true, "char_ranges": [[0, 85]], "word_ranges": [[0, 14]], "text": "The fourth noise usually appears when there is a certain degree of valvular stenosis."}, "5": {"exist": false, "char_ranges": [], "word_ranges": [], "text": ""}}} {"id": 472, "year": 2020, "question_id_specific": 140, "full_question": "65-year-old obese woman who suffers a fall on her hand with her elbow in extension. She presents with arm pain with swelling and functional impotence of the arm and inability to extend the wrist and fingers. It is most likely to present:", "full_answer": "We must take into account all the clinical data provided: after trauma we have pain in the arm with soft tissue involvement and the impossibility of extending the wrist and fingers. The only option that includes all the data is 4. Option 1 does not explain the involvement of the wrist and fingers. The elbow dislocation does not explain the lack of finger mobility as well as the diaphyseal fracture of the humerus and double forearm. \"Direct mechanism is possible, as is indirect transmission of forces from the elbow and hand...radial nerve involvement is extremely frequent, reaching an incidence of 10-18% of cases.\" Proximal humerus fracture-dislocation would have greater involvement at the shoulder, elbow dislocation at the elbow, and there is no data to suggest double forearm fracture.", "type": "ORTHOPEDIC SURGERY AND TRAUMATOLOGY", "options": {"1": "Dislocation fracture of proximal humerus.", "2": "Elbow dislocation.", "3": "Diaphyseal fracture of the humerus associated with double forearm fracture.", "4": "Diaphyseal fracture of humerus with radial nerve injury.", "5": NaN}, "correct_option": 4, "explanations": {"1": {"exist": true, "char_ranges": [[231, 298]], "word_ranges": [[41, 53]], "text": "Option 1 does not explain the involvement of the wrist and fingers."}, "2": {"exist": true, "char_ranges": [[299, 435]], "word_ranges": [[53, 76]], "text": "The elbow dislocation does not explain the lack of finger mobility as well as the diaphyseal fracture of the humerus and double forearm."}, "3": {"exist": true, "char_ranges": [[622, 796]], "word_ranges": [[103, 128]], "text": "Proximal humerus fracture-dislocation would have greater involvement at the shoulder, elbow dislocation at the elbow, and there is no data to suggest double forearm fracture."}, "4": {"exist": true, "char_ranges": [[58, 230]], "word_ranges": [[10, 41]], "text": "after trauma we have pain in the arm with soft tissue involvement and the impossibility of extending the wrist and fingers. The only option that includes all the data is 4."}, "5": {"exist": false, "char_ranges": [], "word_ranges": [], "text": ""}}} {"id": 173, "year": 2013, "question_id_specific": 51, "full_question": "A 53-year-old woman with bronchial asthma comes to the allergy clinic for a checkup. She reports repeated exacerbations with nocturnal symptoms, frequent use of rescue medication and dyspnea on exertion when walking on level ground. Spirometry shows a pre-bronchodilator FEV1/FVC ratio of 60% and FEV1 of 55%. Skin tests are positive for mites and total IgE of 150 IU/ml. The patient is being treated with a combination of high-dose salmeterol/budesonide (50/500 mg: 2 puffs twice daily), maintenance oral prednisone (10 mg/day) and theophylline. Of the following options, which is the most advisable therapeutic approach?", "full_answer": "The indication of omalizumab in a patient such as the one referred to is a correct indication, although in severe asthma that is difficult to control, there are other prior measures that are not contemplated and that must always be taken into account before resorting to this treatment. It is mandatory to ensure firstly that it is truly asthma and not another disease that simulates asthma, secondly that there are no complications that worsen the evolution as could be the presence of gastroesophageal reflux, bronchiectasis, presence of opportunistic germs given the chronic treatment with steroids, etc; and thirdly, it is necessary to confirm that the patient complies with the treatments correctly, especially the inhaled treatment, etc. Once all of the above has been checked, it would be time to start a trial treatment with omalizumab.", "type": "PNEUMOLOGY", "options": {"1": "Add magnesium sulfate.", "2": "Increase the dose of prednisone to 30 mg/day.", "3": "Add omalizumab.", "4": "Nebulized treatment at home.", "5": "Switch to a combination with high doses of budesonide and formoterol."}, "correct_option": 3, "explanations": {"1": {"exist": false, "char_ranges": [], "word_ranges": [], "text": ""}, "2": {"exist": false, "char_ranges": [], "word_ranges": [], "text": ""}, "3": {"exist": true, "char_ranges": [[0, 94]], "word_ranges": [[0, 17]], "text": "The indication of omalizumab in a patient such as the one referred to is a correct indication,"}, "4": {"exist": false, "char_ranges": [], "word_ranges": [], "text": ""}, "5": {"exist": false, "char_ranges": [], "word_ranges": [], "text": ""}}} {"id": 480, "year": 2020, "question_id_specific": 171, "full_question": "A 67-year-old woman with a history of dyslipidemia, who comes to the ED with dysuria and pollakiuria followed by fever, chills, and deterioration of general condition. On arrival she appears severe and is tachycardic, tachypneic, with blood pressure 60/40 mmHg and temperature of 39°C. Which of the following measures would NOT be included in the INITIAL management?", "full_answer": "Among the options presented, the only measure not included in the Surviving Sepsis Campaign guidelines on the management of sepsis and septic shock (like the patient we are dealing with) is the administration of IV dobutamine. The appropriate procedure would consist of fluid administration, blood culture extraction, serum lactate measurement and early administration of broad-spectrum antibiotic therapy directed to the suspected septic focus. If we do not obtain mean arterial pressure for adequate perfusion of the organs, we would initiate noradrenaline perfusion, as discussed in question 51.", "type": "CRITICAL CARE", "options": {"1": "Intravenous perfusion of dobutamine.", "2": "Serum lactate measurement.", "3": "Extraction of blood cultures.", "4": "Fluid administration.", "5": NaN}, "correct_option": 1, "explanations": {"1": {"exist": true, "char_ranges": [[0, 226]], "word_ranges": [[0, 36]], "text": "Among the options presented, the only measure not included in the Surviving Sepsis Campaign guidelines on the management of sepsis and septic shock (like the patient we are dealing with) is the administration of IV dobutamine."}, "2": {"exist": false, "char_ranges": [], "word_ranges": [], "text": ""}, "3": {"exist": false, "char_ranges": [], "word_ranges": [], "text": ""}, "4": {"exist": false, "char_ranges": [], "word_ranges": [], "text": ""}, "5": {"exist": false, "char_ranges": [], "word_ranges": [], "text": ""}}} {"id": 58, "year": 2011, "question_id_specific": 199, "full_question": "An asymptomatic 31-year-old woman (Consultant) is asymptomatic and has a 10-week gestation according to ultrasound study (primiparous). Her younger brother (index case) aged 26 years is affected by ataxia and genetically diagnosed as a heterozygous carrier of an expansive mutation of 70 CAG repeats in the SCA3 gene (chromosome 14). The patient is referred to the genetic counseling office where a possible chorionic villus biopsy to study the fetal genotype is evaluated. Is this invasive procedure indicated as a pre-natal diagnosis (PND) in this case?", "full_answer": "SCA3 is Spinocerebellar Ataxia type 3 also called Machado-Joseph disease. It is caused by mutations in the ATXN3 gene by the CAG triplet repeat expansion mechanism. Affected individuals have between 52 and 86 CAG triplet repeats in heterozygosis with complete penetrance. The heterozygous mutation carrier has the disease as its inheritance pattern is autosomal dominant. In this clinical case, the one who has the disease and has the mutated gene in heterozygosis is the brother (26 years old) of the pregnant woman. I think that before doing the chorion biopsy, the pregnant woman should be tested to know her ATXN3 gene status. She has a 50% chance of carrying the mutation and having the disease, but since she is 31 years old, I think she should have already had symptoms. Nevertheless, I would do the genetic test on the pregnant woman to be on the safe side. If she came out without the mutation, there would be nothing more to do because the disease would no longer be transmitted. If she came out with the mutation, then I would do a chorionic villus biopsy. Answers 1 and 2 are both true, but I think answer 1 is more true than answer 2.", "type": "GENETICS", "options": {"1": "It is indicated after studying the Consultant's genotype and only if it is heterozygous.", "2": "This is not indicated, since SCA3 ataxia is full penetrance and the Consultant is asymptomatic and therefore has not inherited the mutation.", "3": "It may be indicated in the next pregnancy, after studying the genotype of the first child and detecting the mutation in it.", "4": "SCA3 ataxia is recessively inherited, so there is no appreciable risk of disease transmission and PND is not indicated.", "5": "This is indicated regardless of the Consultant's genotype, since SCA3 ataxia is maternally inherited (transmitted by women)."}, "correct_option": 1, "explanations": {"1": {"exist": true, "char_ranges": [[372, 777]], "word_ranges": [[55, 130]], "text": "In this clinical case, the one who has the disease and has the mutated gene in heterozygosis is the brother (26 years old) of the pregnant woman. I think that before doing the chorion biopsy, the pregnant woman should be tested to know her ATXN3 gene status. She has a 50% chance of carrying the mutation and having the disease, but since she is 31 years old, I think she should have already had symptoms."}, "2": {"exist": false, "char_ranges": [], "word_ranges": [], "text": ""}, "3": {"exist": false, "char_ranges": [], "word_ranges": [], "text": ""}, "4": {"exist": false, "char_ranges": [], "word_ranges": [], "text": ""}, "5": {"exist": false, "char_ranges": [], "word_ranges": [], "text": ""}}} {"id": 51, "year": 2011, "question_id_specific": 230, "full_question": "37-week pregnant woman with positive hepatitis B virus surface antigen. She consults about the guidelines to be followed with the newborn and whether she can breastfeed. What advice do you think is most appropriate?", "full_answer": "The correct answer is 4. Children born to HBV carrier mothers should be administered hepatitis B vaccine and immunoglobulin at different puncture sites within the first 24 hours of life. Although the virus is excreted in breast milk, the risk of infection by this route is very low and therefore it is not currently a contraindication for breastfeeding.", "type": "PEDIATRICS", "options": {"1": "Administer hepatitis B vaccine at birth. Artificial feeding.", "2": "Vaccination and immunoglobulins should be administered at birth. Breastfeeding from one month onwards.", "3": "Immunoglobulins at birth and artificial breastfeeding.", "4": "Vaccination and immunoglobulins at birth. Maternal feeding.", "5": "Artificial feeding and isolation for 4 weeks."}, "correct_option": 4, "explanations": {"1": {"exist": false, "char_ranges": [], "word_ranges": [], "text": ""}, "2": {"exist": false, "char_ranges": [], "word_ranges": [], "text": ""}, "3": {"exist": false, "char_ranges": [], "word_ranges": [], "text": ""}, "4": {"exist": true, "char_ranges": [[25, 186]], "word_ranges": [[5, 30]], "text": "Children born to HBV carrier mothers should be administered hepatitis B vaccine and immunoglobulin at different puncture sites within the first 24 hours of life."}, "5": {"exist": false, "char_ranges": [], "word_ranges": [], "text": ""}}} {"id": 184, "year": 2013, "question_id_specific": 65, "full_question": "A patient with obesity (body mass index 38) is referred to your office. In the clinical history the patient refers that he/she has been obese since the age of 17 (currently 36 years old), having been on multiple diets with weight loss ranging from 5 to 10 kg but has subsequently regained weight. The patient is currently on a 1500 Kcal diet (self-administered and not restricted in fat) with a good follow-up of this diet, performing one hour of aerobic exercise 4 days a week. She has lost 3 kg but needs an additional loss of 7 kg more. Given the possibility of adding a drug against obesity, which one would you use to reduce fat absorption?", "full_answer": "Orlistat acts by decreasing fat absorption; it is currently the only drug approved for use in obesity.", "type": "ENDOCRINOLOGY", "options": {"1": "Orlistat.", "2": "Topiramate.", "3": "Sibutramine.", "4": "Liraglutide.", "5": "Metformin."}, "correct_option": 1, "explanations": {"1": {"exist": true, "char_ranges": [[0, 102]], "word_ranges": [[0, 17]], "text": "Orlistat acts by decreasing fat absorption; it is currently the only drug approved for use in obesity."}, "2": {"exist": false, "char_ranges": [], "word_ranges": [], "text": ""}, "3": {"exist": false, "char_ranges": [], "word_ranges": [], "text": ""}, "4": {"exist": false, "char_ranges": [], "word_ranges": [], "text": ""}, "5": {"exist": false, "char_ranges": [], "word_ranges": [], "text": ""}}} {"id": 295, "year": 2016, "question_id_specific": 75, "full_question": "Indicate the clinical situation that, in relation to hepatitis B virus infection, presents a 5-year-old patient from Nigeria, with normal physical examination and with the following serology against hepatitis B: HBsAg + / ANTI- HBs - / HBeAg - / ANTI- Hbe + / ANTI- Hbc IgM - / ANTI- Hbc IgG +/ DNA HBV +:", "full_answer": "Question in which having a table in the head comes out alone discarding options: Option 1: The acute infection would be HBeAg + and ANTI-HBe - . In addition the ANTI-HBec would be IgM type (only for this last data, we would have to discard it in a direct way). Option 2: the correct one. It complies with all the characteristics. Option 3: The vaccinated patient cannot have DNA of the virus (the only thing that positivizes in a vaccinated patient is the AntiBs. IgG type). Option 4: The option that could offer more doubts. In the asymptomatic carrier (chronic carrier / seroconverted) the difference is that there is no viral DNA.", "type": "GENETICS AND IMMUNOLOGY", "options": {"1": "Acute infection.", "2": "Chronic infection.", "3": "Vaccinated patient.", "4": "Asymptomatic carrier.", "5": NaN}, "correct_option": 2, "explanations": {"1": {"exist": true, "char_ranges": [[81, 260]], "word_ranges": [[14, 50]], "text": "Option 1: The acute infection would be HBeAg + and ANTI-HBe - . In addition the ANTI-HBec would be IgM type (only for this last data, we would have to discard it in a direct way)."}, "2": {"exist": true, "char_ranges": [[261, 329]], "word_ranges": [[50, 61]], "text": "Option 2: the correct one. It complies with all the characteristics."}, "3": {"exist": true, "char_ranges": [[330, 391]], "word_ranges": [[61, 72]], "text": "Option 3: The vaccinated patient cannot have DNA of the virus"}, "4": {"exist": true, "char_ranges": [[475, 633]], "word_ranges": [[86, 112]], "text": "Option 4: The option that could offer more doubts. In the asymptomatic carrier (chronic carrier / seroconverted) the difference is that there is no viral DNA."}, "5": {"exist": false, "char_ranges": [], "word_ranges": [], "text": ""}}} {"id": 559, "year": 2022, "question_id_specific": 160, "full_question": "A 35-year-old woman, taking contraceptives, comes to the emergency department with febrile syndrome and paresthesias in the left hemibody. Blood tests show Hb 7.5 g/dL, platelets 7,000/microl, normal leukocytes with normal differential count, LDH 1,200 IU/l, reticulocytes 10% (normal 0.5-2%), undetectable serum haptoglobin, negative direct Coombs' test and peripheral blood smear with schistocytes. Coagulation (prothrombin time and APTT) normal. Which of the following is the most likely diagnosis:", "full_answer": "The fear of all on-call hematologists... that you will get a TTP. Because, although infrequent, it is the greatest hematologic emergency. The clinical case is fairly typical. A young person with fever and neurological symptoms that make him go to the emergency department (he is on the run). Laboratory tests show anemia with evidence of hemolysis (increased LDH and retis, presence of schistocytes in peripheral blood) and severe thrombopenia. In addition, coagulation is normal. To distinguish it from the rest of things (although you should not hesitate...) it tells you that the coombs is negative. As a reminder: URGENT TREATMENT: PLASMATIC REPLACEMENTS. The diagnostic confirmation is the ADAMTS 13 deficit, but treatment is started before having that value, which is not usually done urgently. And, as we all know, these things usually come at night and/or weekends, to make things more difficult....", "type": "HEMATOLOGY", "options": {"1": "Autoimmune hemolytic anemia.", "2": "Thrombotic thrombocytopenic purpura.", "3": "Evans syndrome (hemolytic anemia and immune thrombopenia).", "4": "Idiopathic immune thrombocytopenia.", "5": NaN}, "correct_option": 2, "explanations": {"1": {"exist": false, "char_ranges": [], "word_ranges": [], "text": ""}, "2": {"exist": true, "char_ranges": [[139, 603]], "word_ranges": [[21, 95]], "text": "The clinical case is fairly typical. A young person with fever and neurological symptoms that make him go to the emergency department (he is on the run). Laboratory tests show anemia with evidence of hemolysis (increased LDH and retis, presence of schistocytes in peripheral blood) and severe thrombopenia. In addition, coagulation is normal. To distinguish it from the rest of things (although you should not hesitate...) it tells you that the coombs is negative."}, "3": {"exist": false, "char_ranges": [], "word_ranges": [], "text": ""}, "4": {"exist": false, "char_ranges": [], "word_ranges": [], "text": ""}, "5": {"exist": false, "char_ranges": [], "word_ranges": [], "text": ""}}} {"id": 558, "year": 2022, "question_id_specific": 160, "full_question": "A 35-year-old woman, taking contraceptives, comes to the emergency department with febrile syndrome and paresthesias in the left hemibody. Blood tests showed Hb 7.5 g/dL, platelets 7,000/microl, normal leukocytes with normal differential count, LDH 1,200 IU/l, reticulocytes 10% (normal 0.5-2%), undetectable serum haptoglobin, negative direct Coombs' test and peripheral blood smear with schistocytes. Coagulation (prothrombin time and APTT) normal. Which of the following is the most likely diagnosis:", "full_answer": "The patient presents criteria for Thrombotic Thrombocytopenic Purpura (TTP): 1. Neurological alteration. 2. Febrile syndrome (fever). 3. Coombs negative migroangiopathic hemolytic anemia (presence of schistocytes). 4. Thrombopenia.", "type": "HEMATOLOGY", "options": {"1": "Autoimmune hemolytic anemia.", "2": "Thrombotic thrombocytopenic purpura.", "3": "Evans syndrome (hemolytic anemia and immune thrombopenia).", "4": "Idiopathic immune thrombocytopenia.", "5": NaN}, "correct_option": 2, "explanations": {"1": {"exist": false, "char_ranges": [], "word_ranges": [], "text": ""}, "2": {"exist": true, "char_ranges": [[0, 231]], "word_ranges": [[0, 27]], "text": "The patient presents criteria for Thrombotic Thrombocytopenic Purpura (TTP): 1. Neurological alteration. 2. Febrile syndrome (fever). 3. Coombs negative migroangiopathic hemolytic anemia (presence of schistocytes). 4. Thrombopenia."}, "3": {"exist": false, "char_ranges": [], "word_ranges": [], "text": ""}, "4": {"exist": false, "char_ranges": [], "word_ranges": [], "text": ""}, "5": {"exist": false, "char_ranges": [], "word_ranges": [], "text": ""}}} {"id": 227, "year": 2014, "question_id_specific": 72, "full_question": "A 45-year-old woman with a history of heart murmur detected in childhood. She is admitted to the emergency department for palpitations, easy fatigue and malleolar edema. Physical examination revealed absence of cyanosis. Oxygen saturation by pulse oximeter 97% BP 120/80 mmHg. Irregular heart rhythm at 100 bpm. Ejective systolic murmur (2/6) in pulmonary focus. 2nd unfolded noise, wide and fixed. No rales. Slight hepatomeglaia (2-3 cm). Slight malleolar edema. ECG: complete arrhythmia due to atrial fibrillation at 100 bpm. QRS +120º. Conduction disorder of the right bundle branch of the His bundle. What is its diagnostic orientation?", "full_answer": "If he has a pediatric murmur, I'm thinking congenital. Hepatomegaly, malleolar edema and BRD, something that overloads the right heart. So we rule out aortic and mitral stenosis. A patent ductus would give cyanosis: ruled out. VSD or ASD? If I am told that he has AF, it is because the atria are of size XL, and a VSD does not produce atrial dilatation.", "type": "CARDIOLOGY", "options": {"1": "Ventricular septal defect.", "2": "Aortic stenosis.", "3": "Mitral stenosis.", "4": "Atrial septal defect.", "5": "Patent ductus arteriosus."}, "correct_option": 4, "explanations": {"1": {"exist": true, "char_ranges": [[239, 353]], "word_ranges": [[39, 64]], "text": "If I am told that he has AF, it is because the atria are of size XL, and a VSD does not produce atrial dilatation."}, "2": {"exist": true, "char_ranges": [[0, 178]], "word_ranges": [[0, 28]], "text": "If he has a pediatric murmur, I'm thinking congenital. Hepatomegaly, malleolar edema and BRD, something that overloads the right heart. So we rule out aortic and mitral stenosis."}, "3": {"exist": true, "char_ranges": [[0, 178]], "word_ranges": [[0, 28]], "text": "If he has a pediatric murmur, I'm thinking congenital. Hepatomegaly, malleolar edema and BRD, something that overloads the right heart. So we rule out aortic and mitral stenosis."}, "4": {"exist": true, "char_ranges": [[239, 353]], "word_ranges": [[39, 64]], "text": "If I am told that he has AF, it is because the atria are of size XL, and a VSD does not produce atrial dilatation."}, "5": {"exist": true, "char_ranges": [[179, 226]], "word_ranges": [[28, 36]], "text": "A patent ductus would give cyanosis: ruled out."}}} {"id": 168, "year": 2013, "question_id_specific": 86, "full_question": "What examination would you advise in the first place for an 82-year-old patient with exertional angina and a systolic ejection murmur of intensity 3/6 on auscultation?", "full_answer": "Ejective murmur, our first suspicion is aortic stenosis. Angina may be due to the stenosis itself or to underlying coronary artery disease (probable, given the age). And with this information, we cannot know if the stenosis is severe. Therefore, the first thing we will have to do will be to study the valve disease: we do not want to put the patient to run or overload him with dobuta and have a syncope, nor do we want to do a catheterization without having first performed another noninvasive test. And cardioTC, in someone 80 years old, is going to tell us what we already know: that he has calcium up to his eyebrows.", "type": "CARDIOLOGY AND CARDIOVASCULAR SURGERY", "options": {"1": "A stress test.", "2": "Coronary angiography.", "3": "Exercise echocardiogram.", "4": "Coronary CT scan.", "5": "Doppler echocardiogram."}, "correct_option": 5, "explanations": {"1": {"exist": true, "char_ranges": [[317, 357]], "word_ranges": [[54, 64]], "text": "we do not want to put the patient to run"}, "2": {"exist": true, "char_ranges": [[406, 501]], "word_ranges": [[73, 88]], "text": "nor do we want to do a catheterization without having first performed another noninvasive test."}, "3": {"exist": true, "char_ranges": [[361, 405]], "word_ranges": [[65, 73]], "text": "overload him with dobuta and have a syncope,"}, "4": {"exist": true, "char_ranges": [[506, 622]], "word_ranges": [[89, 112]], "text": "cardioTC, in someone 80 years old, is going to tell us what we already know: that he has calcium up to his eyebrows."}, "5": {"exist": false, "char_ranges": [], "word_ranges": [], "text": ""}}} {"id": 329, "year": 2016, "question_id_specific": 84, "full_question": "In a 70-year-old diabetic man with a history of ischemic heart disease, what is the therapeutic target for LDL cholesterol and glycosylated hemoglobin (Hb A1c)?", "full_answer": "Diabetes with ecv ldlc<70.", "type": "ENDOCRINOLOGY", "options": {"1": "LDLc<15 mg/dL and Hb A1c <6.5%.", "2": "LDLc<100 mg/dL and Hb A1c <7%.", "3": "LDLc<70 mg/dL and Hb A1c <7%.", "4": "LDLc< 115mg/dL and Hb A1c <7%.", "5": NaN}, "correct_option": 3, "explanations": {"1": {"exist": false, "char_ranges": [], "word_ranges": [], "text": ""}, "2": {"exist": false, "char_ranges": [], "word_ranges": [], "text": ""}, "3": {"exist": true, "char_ranges": [[0, 26]], "word_ranges": [[0, 4]], "text": "Diabetes with ecv ldlc<70."}, "4": {"exist": false, "char_ranges": [], "word_ranges": [], "text": ""}, "5": {"exist": false, "char_ranges": [], "word_ranges": [], "text": ""}}} {"id": 447, "year": 2018, "question_id_specific": 131, "full_question": "A 25-year-old man with no known history of interest. He has been presenting with foamy and loaded urine for 2 months, with no other symptoms. Physical examination without findings. Elemental urine: pH 5; glucose negative; hemoglobin ++; protein ++; leukocyte esterase negative; proteinuria 1.5 g/24 hours; urinary Na 60 mEq/L; urinary K 30 mEq/L; urinary Cl 100 mEq/L. Blood biochemistry: creatinine 1.6 mg/dL; urea 80 mg/dL; Na 140 mEq/L; K 3.8 mEq/L. Immunological study: antiDNA negative; ANCA negative; antiMBG negative, rheumatoid factor negative; C3 20 mg/dL (normal 60-120); C4 10 mg/dL (normal 20-40). A renal biopsy is performed. Which of these diagnoses do you think is the most likely to be found in the biopsy?", "full_answer": "We are presented with a lot of analytical data but we only have to look at two: the levels of complement fractions C3 and C4 are low. Of the options given, only membranoproliferative glomerulonephritis presents with hypocomplementemia (option 4 correct).", "type": "NEPHROLOGY", "options": {"1": "Membranous glomerulonephritis.", "2": "Glomerulonephritis with minimal changes.", "3": "Mesangial IgA glomerulonephritis.", "4": "Membranoproliferative glomerulonephritis.", "5": NaN}, "correct_option": 4, "explanations": {"1": {"exist": false, "char_ranges": [], "word_ranges": [], "text": ""}, "2": {"exist": false, "char_ranges": [], "word_ranges": [], "text": ""}, "3": {"exist": false, "char_ranges": [], "word_ranges": [], "text": ""}, "4": {"exist": true, "char_ranges": [[80, 254]], "word_ranges": [[17, 40]], "text": "the levels of complement fractions C3 and C4 are low. Of the options given, only membranoproliferative glomerulonephritis presents with hypocomplementemia (option 4 correct)."}, "5": {"exist": false, "char_ranges": [], "word_ranges": [], "text": ""}}} {"id": 72, "year": 2012, "question_id_specific": 52, "full_question": "A 72-year-old man, ex-smoker, hypertensive and diabetic. History of stroke with residual paresis in the left arm 4 months ago. He comes to the emergency department of a regional hospital with intense precordial pain of 2 hours of evolution; in the physical examination he presents cold and sweaty skin, blood pressure of 80 mm Hg; electrocardiogram with marked ST segment elevation in leads V1-V6, I and aVL. Which of the following treatments of coronary repercussion is the most appropriate?", "full_answer": "This question combines several important facts. To begin with, this is a patient with an extensive anterior and lateral AMI, with cardiogenic shock. On the other hand, in the history, we are told that the patient had a stroke 4 months ago. It is evident from the patient's clinical picture that an urgent reperfusion strategy is indicated, which invalidates options 4 and 5. The basic question is whether to choose fibrinolysis (which can be done in a regional hospital) or PCI (for which the patient should be referred to a tertiary center). Option 2 does not make much sense, since in the event that fibrinolysis is indicated, it would only be a waste of time to make the transfer. A history of stroke (presumably ischemic) 4 months ago is a relative contraindication for fibrinolysis (if the time is less than 3 months it is an absolute contraindication), but the definitive factor that tips the balance towards PCI is the extent of AMI and the presence of shock, which are indications for urgent PCI. Therefore, the correct answer is 3.", "type": "ANESTHESIOLOGY, CRITICAL CARE AND EMERGENCIES", "options": {"1": "Immediate systemic fibrinolysis at the regional hospital.", "2": "Immediate transfer to a tertiary center for treatment with fibrinolysis.", "3": "Immediate transfer to a tertiary center for percutaneous coronary revascularization procedure.", "4": "Hemodynamic stabilization and deferred revascularization procedure.", "5": "Immediate treatment with fondaparinux and abciximab."}, "correct_option": 3, "explanations": {"1": {"exist": true, "char_ranges": [[684, 858]], "word_ranges": [[119, 147]], "text": "A history of stroke (presumably ischemic) 4 months ago is a relative contraindication for fibrinolysis (if the time is less than 3 months it is an absolute contraindication),"}, "2": {"exist": true, "char_ranges": [[543, 683]], "word_ranges": [[92, 119]], "text": "Option 2 does not make much sense, since in the event that fibrinolysis is indicated, it would only be a waste of time to make the transfer."}, "3": {"exist": true, "char_ranges": [[863, 1004]], "word_ranges": [[148, 173]], "text": "the definitive factor that tips the balance towards PCI is the extent of AMI and the presence of shock, which are indications for urgent PCI."}, "4": {"exist": true, "char_ranges": [[240, 374]], "word_ranges": [[42, 63]], "text": "It is evident from the patient's clinical picture that an urgent reperfusion strategy is indicated, which invalidates options 4 and 5."}, "5": {"exist": true, "char_ranges": [[240, 374]], "word_ranges": [[42, 63]], "text": "It is evident from the patient's clinical picture that an urgent reperfusion strategy is indicated, which invalidates options 4 and 5."}}} {"id": 505, "year": 2020, "question_id_specific": 85, "full_question": "A 79-year-old patient is brought to the ED at 7 pm from a nursing home, but we do not have the referral report and therefore do not know his history. He is disoriented in time and space. He has trouble remembering simple information. Sometimes he becomes very nervous and agitated, because he sees threatening people, even when he is alone. At other times he appears drowsy and inattentive. The main data supporting the diagnosis of delirium as opposed to dementia is:", "full_answer": "Little to comment.... is the definition of delirium. Altered state of consciousness.", "type": "PSYCHIATRY", "options": {"1": "The presence of visual hallucinations.", "2": "Cognitive symptoms.", "3": "Psychomotor agitation.", "4": "Fluctuation of the alert level.", "5": NaN}, "correct_option": 4, "explanations": {"1": {"exist": false, "char_ranges": [], "word_ranges": [], "text": ""}, "2": {"exist": false, "char_ranges": [], "word_ranges": [], "text": ""}, "3": {"exist": false, "char_ranges": [], "word_ranges": [], "text": ""}, "4": {"exist": true, "char_ranges": [[22, 84]], "word_ranges": [[3, 12]], "text": "is the definition of delirium. Altered state of consciousness."}, "5": {"exist": false, "char_ranges": [], "word_ranges": [], "text": ""}}} {"id": 360, "year": 2016, "question_id_specific": 93, "full_question": "46-year-old woman with a history of rheumatic valve disease, requiring mitral valve replacement with a mechanical prosthesis. The initial postoperative evolution is favorable. However, after starting treatment with acenocoumarol, she develops a picture of skin necrosis affecting the abdominal region and extremities. Which of the following alterations would justify this picture?", "full_answer": "Warfarin-induced skin necrosis is a rare and serious adverse effect of oral anticoagulant therapy, occurring in only 0.01 to 0.1% of patients and is especially related to coagulation protein C and S deficiency.", "type": "HEMATOLOGY", "options": {"1": "Antithrombin deficiency.", "2": "Factor V Leiden.", "3": "Hyperhomocysteinemia.", "4": "Protein C deficiency.", "5": NaN}, "correct_option": 4, "explanations": {"1": {"exist": false, "char_ranges": [], "word_ranges": [], "text": ""}, "2": {"exist": false, "char_ranges": [], "word_ranges": [], "text": ""}, "3": {"exist": false, "char_ranges": [], "word_ranges": [], "text": ""}, "4": {"exist": true, "char_ranges": [[0, 210]], "word_ranges": [[0, 33]], "text": "Warfarin-induced skin necrosis is a rare and serious adverse effect of oral anticoagulant therapy, occurring in only 0.01 to 0.1% of patients and is especially related to coagulation protein C and S deficiency."}, "5": {"exist": false, "char_ranges": [], "word_ranges": [], "text": ""}}} {"id": 459, "year": 2018, "question_id_specific": 173, "full_question": "A 36-year-old man with no past history of interest who, after a high-energy trauma following a fall from a motorcycle, presents with a Gustilo grade II open midshaft fracture of the tibia. Which of the following maneuvers or therapies is most effective for preventing infection?", "full_answer": "I believe that the most correct is 2, since Gustilo II fractures are mainly covered with Gram (+) by means of a 1st generation Cepha, and it is not a question of prevention but of treatment (it is considered to be contaminated).", "type": "TRAUMATOLOGY AND ORTHOPEDICS", "options": {"1": "Immediate intravenous broad-spectrum antibiotherapy.", "2": "Exhaustive debridement-wound lavage in the operating room with at least 10 liters of saline.", "3": "Urgent stabilization of the fracture by external fixation and early wound closure.", "4": "Early wound closure with antiseptics and oxygen therapy at two liters per minute.", "5": NaN}, "correct_option": 2, "explanations": {"1": {"exist": false, "char_ranges": [], "word_ranges": [], "text": ""}, "2": {"exist": true, "char_ranges": [[0, 133]], "word_ranges": [[0, 25]], "text": "I believe that the most correct is 2, since Gustilo II fractures are mainly covered with Gram (+) by means of a 1st generation Cepha,"}, "3": {"exist": false, "char_ranges": [], "word_ranges": [], "text": ""}, "4": {"exist": false, "char_ranges": [], "word_ranges": [], "text": ""}, "5": {"exist": false, "char_ranges": [], "word_ranges": [], "text": ""}}} {"id": 149, "year": 2012, "question_id_specific": 162, "full_question": "A 52-year-old woman comes to the emergency room with bilateral paralysis of the upper extremities that had started abruptly two hours earlier. In the interview she is smiling and says she does not understand why her family is so worried. The family refers that the day before her son had been arrested by the police for drug trafficking and they had not yet been able to contact him. Which of the following is the most likely diagnosis?", "full_answer": "The nice word to describe this is: \"Belle indeference\", this woman shows herself with an elegant indifference to a paralysis. The diagnosis.... can be excluded 2,3,4 because the clinical does not match: no delirium, no worry, no sadness. 1 would be if he was making a profit, if he was knowingly simulating and to make a profit he was faking his illness. This is not the case. Conversive would be if the symptom wanted to transmit something to us, that is to say, it was a metaphor of its internal world... The most exact in this case: 5.", "type": "PSYCHIATRY", "options": {"1": "Factitious disorder.", "2": "Delusional disorder.", "3": "Hypochondriasis.", "4": "Major depressive disorder.", "5": "Conversive disorder."}, "correct_option": 5, "explanations": {"1": {"exist": true, "char_ranges": [[238, 376]], "word_ranges": [[38, 67]], "text": "1 would be if he was making a profit, if he was knowingly simulating and to make a profit he was faking his illness. This is not the case."}, "2": {"exist": true, "char_ranges": [[144, 237]], "word_ranges": [[22, 38]], "text": "can be excluded 2,3,4 because the clinical does not match: no delirium, no worry, no sadness."}, "3": {"exist": true, "char_ranges": [[144, 237]], "word_ranges": [[22, 38]], "text": "can be excluded 2,3,4 because the clinical does not match: no delirium, no worry, no sadness."}, "4": {"exist": true, "char_ranges": [[144, 237]], "word_ranges": [[22, 38]], "text": "can be excluded 2,3,4 because the clinical does not match: no delirium, no worry, no sadness."}, "5": {"exist": true, "char_ranges": [[377, 506]], "word_ranges": [[67, 91]], "text": "Conversive would be if the symptom wanted to transmit something to us, that is to say, it was a metaphor of its internal world..."}}} {"id": 246, "year": 2014, "question_id_specific": 116, "full_question": "A patient with a history of excessive alcohol consumption has been diagnosed with pulmonary tuberculosis due to cough, fever, expectoration, and sputum culture isolation of Mycobacterium tuberculosis. The patient started treatment with isoniazid, rifampicin, ethambutol and pyrazinamide, with adequate tolerance. Twenty days after starting treatment, a report of rifampicin resistance of M. tuberculosis isolated in sputum is received. Which regimen would you select based on this report?", "full_answer": "Another question that may generate discussion. When a diagnosis of active tuberculosis infection is made, treatment is initiated with isoniazid, rifampicin and pyrazinamide together with ethambutol in case of suspicion of strains resistant to any of the drugs. Once the existence of resistance has been ruled out, ethambutol can be dispensed with. In case of resistance to rifampicin, rifampicin is withdrawn and treatment is completed with the 3 remaining drugs for 12 months. The option of adding a quinolone during the first two months of treatment (moxifloxacin or levofloxacin) is considered in cases of extensive pulmonary involvement. In this case, they do not tell us anything about extensive pulmonary involvement but they give us a data \"excessive alcohol consumption\". You know that in the MIR no data is free, and if the person who posted this question wanted to reflect it, probably his intention was that we reflect on it. Alcohol exerts a direct immunosuppressive effect, moreover, alcoholism is related to the affectation of immunity in the respiratory tract. The social context that usually accompanies the alcoholic patient does not help either. All the factors mentioned above make us lean more towards answer number 3, although it is undeniable that number 1 could also be correct.", "type": "INFECTIOUS DISEASES", "options": {"1": "Isoniazid, ethambutol, pyrazinamide for 12 months.", "2": "Isoniazid, ethambutol, pyrazinamide for 12 months and streptomycin for two months.", "3": "Isoniazid, ethambutol, pyrazinamide for 12 months and a quinolone for two months.", "4": "Isoniazid, ethambutol, pyrazinamide and a quinolone for 18 months.", "5": "Isoniazid, ethambutol, pyrazinamide for 18 months and streptomycin and a quinolone for 2 months."}, "correct_option": 3, "explanations": {"1": {"exist": false, "char_ranges": [], "word_ranges": [], "text": ""}, "2": {"exist": false, "char_ranges": [], "word_ranges": [], "text": ""}, "3": {"exist": true, "char_ranges": [[937, 1075]], "word_ranges": [[150, 169]], "text": "Alcohol exerts a direct immunosuppressive effect, moreover, alcoholism is related to the affectation of immunity in the respiratory tract."}, "4": {"exist": false, "char_ranges": [], "word_ranges": [], "text": ""}, "5": {"exist": false, "char_ranges": [], "word_ranges": [], "text": ""}}} {"id": 457, "year": 2018, "question_id_specific": 126, "full_question": "A 47-year-old man, with a history of a right parotid pleomorphic adenoma, treated with surgery (extrafacial parotidectomy) 6 months ago, who comes to our office for presenting pain with sweating and reddening of the skin in the preauricular region during chewing. Which treatment would be the treatment of choice?", "full_answer": "This clinical picture is called Frey's syndrome and consists of an anomalous reinnervation of regional structures after parotidectomy due to lesion of the auriculotemporal nerve, so that during mastication, erythema and preauricular sweating occur. The treatment is intradermal botulinum toxin injection.", "type": "OTORHINOLARYNGOLOGY AND MAXILLOFACIAL SURGERY", "options": {"1": "Extended total parotidectomy on suspicion of tumor recurrence.", "2": "Pregabalin.", "3": "Intradermal botulinum toxin injection.", "4": "Broad-spectrum antibiotherapy.", "5": NaN}, "correct_option": 3, "explanations": {"1": {"exist": false, "char_ranges": [], "word_ranges": [], "text": ""}, "2": {"exist": false, "char_ranges": [], "word_ranges": [], "text": ""}, "3": {"exist": true, "char_ranges": [[0, 304]], "word_ranges": [[0, 41]], "text": "This clinical picture is called Frey's syndrome and consists of an anomalous reinnervation of regional structures after parotidectomy due to lesion of the auriculotemporal nerve, so that during mastication, erythema and preauricular sweating occur. The treatment is intradermal botulinum toxin injection."}, "4": {"exist": false, "char_ranges": [], "word_ranges": [], "text": ""}, "5": {"exist": false, "char_ranges": [], "word_ranges": [], "text": ""}}} {"id": 165, "year": 2013, "question_id_specific": 229, "full_question": "A 58-year-old woman, weight 130 kg, height 155 cm, body mass index > 30 with mild hypertension, blood glucose 108 mg/dL and absence of edema in the lower limbs. Blood analysis showed Cr 2.0 mg/dL, Urea 86 mg/dL, Alb 3.8 g/1, Na 142 mEq/L, K 4 mEq/L. In urinalysis: sediment without alterations and in 24 h urine proteinuria of 6.3 g/24 h. Which of the following entities is more likely to be present?", "full_answer": "The correct answer is: 2. Focal segmental glomerulonephritis. We are in front of a patient with renal insufficiency whose manifestation is a proteinuria. That places us in the picture of nonproliferative GN (membranous, focal and segmental and minimal changes, discarding the other two). If we also consider that the patient was obese, slightly hypertensive, the most probable is that it is a focal segmental glomerulonephritis.", "type": "NEPHROLOGY", "options": {"1": "Secondary membranous glomerulonephritis.", "2": "Focal and segmental glomerulonephritis.", "3": "IgA nephropathy.", "4": "Rapidly progressive glomerulonephritis.", "5": "Minimal change nephropathy."}, "correct_option": 2, "explanations": {"1": {"exist": false, "char_ranges": [], "word_ranges": [], "text": ""}, "2": {"exist": true, "char_ranges": [[62, 287]], "word_ranges": [[8, 43]], "text": "We are in front of a patient with renal insufficiency whose manifestation is a proteinuria. That places us in the picture of nonproliferative GN (membranous, focal and segmental and minimal changes, discarding the other two)."}, "3": {"exist": false, "char_ranges": [], "word_ranges": [], "text": ""}, "4": {"exist": false, "char_ranges": [], "word_ranges": [], "text": ""}, "5": {"exist": false, "char_ranges": [], "word_ranges": [], "text": ""}}} {"id": 157, "year": 2012, "question_id_specific": 73, "full_question": "A 75-year-old patient with significant gonarthrosis with \"Genu Varo\" that causes marked functional impotence, limited mobility and continuous pain that requires continuous antirheumatic, anti-inflammatory and analgesic medication. What treatment should be indicated in this case?", "full_answer": "We are dealing with a patient who has exhausted the pharmacological route given that he takes everything and has two important problems: genu varum and an important gonarthrosis. This means that we can do nothing with an arthroscopy, let alone a synovectomy (1 and 5 false). The genu varum could falsely make us think about an osteotomy, however even if we did it and corrected the varus now the damage is already done and the osteoarthrosis is severe. Perhaps years ago before the osteoarthritis took place it would have been resolutive but not now (4 false). In addition, in order to consider an osteotomy, we should be given more data to know which bone to apply the osteotomy on, the tibia or the femur. Finally, we may think that it is best to first try rehabilitation before considering surgery, but I discard this option because the question emphasizes that we have exhausted pharmacological measures (although it could be considered because they have not specified whether it has been tried or not). In any case with the knee prosthesis we correct the problem of osteoarthritis pain and we can also correct the knee varus.", "type": "TRAUMATOLOGY AND ORTHOPEDICS", "options": {"1": "Arthroscopic lavage of the knee.", "2": "Rehabilitation of the affected knee.", "3": "Arthroplasty of the affected knee.", "4": "Abduction supratuberosity tibial supratuberosity tibial osteotomy.", "5": "Knee synovectomy."}, "correct_option": 3, "explanations": {"1": {"exist": true, "char_ranges": [[0, 274]], "word_ranges": [[0, 46]], "text": "We are dealing with a patient who has exhausted the pharmacological route given that he takes everything and has two important problems: genu varum and an important gonarthrosis. This means that we can do nothing with an arthroscopy, let alone a synovectomy (1 and 5 false)."}, "2": {"exist": true, "char_ranges": [[717, 1007]], "word_ranges": [[125, 171]], "text": "we may think that it is best to first try rehabilitation before considering surgery, but I discard this option because the question emphasizes that we have exhausted pharmacological measures (although it could be considered because they have not specified whether it has been tried or not)."}, "3": {"exist": true, "char_ranges": [[1020, 1130]], "word_ranges": [[174, 193]], "text": "with the knee prosthesis we correct the problem of osteoarthritis pain and we can also correct the knee varus."}, "4": {"exist": true, "char_ranges": [[275, 560]], "word_ranges": [[46, 96]], "text": "The genu varum could falsely make us think about an osteotomy, however even if we did it and corrected the varus now the damage is already done and the osteoarthrosis is severe. Perhaps years ago before the osteoarthritis took place it would have been resolutive but not now (4 false)."}, "5": {"exist": true, "char_ranges": [[0, 274]], "word_ranges": [[0, 46]], "text": "We are dealing with a patient who has exhausted the pharmacological route given that he takes everything and has two important problems: genu varum and an important gonarthrosis. This means that we can do nothing with an arthroscopy, let alone a synovectomy (1 and 5 false)."}}} {"id": 382, "year": 2016, "question_id_specific": 231, "full_question": "A 33-year-old man, independent in all activities of daily living, with a history of well-controlled hypertension, antiplatelet atrial fibrillation and an adenocarcinoma of the prostate at 73 years of age, currently free of disease. He is brought to the emergency department for clinical aphasia and right hemiparesis of sudden onset, 45 minutes earlier. What is the most correct approach?", "full_answer": "The patient suffers an abrupt neurological deficit probably related to a vascular event, so first we will have to perform a cranial CT scan to rule out a hemorrhagic stroke. If the CT scan does not show bleeding, the patient has an acute ischemic stroke, probably cardioembolic due to his cardiac pathology, not anticoagulated. Given that his tumor process is under control, there are no contraindications for the administration of fibrinolysis in this case, since the patient is in a good baseline condition. Although the patient is 88 years old, age is no longer presented as an absolute exclusion criterion, but the general situation of the patient must be taken into account and in this case it is specified that he presents a good baseline situation.", "type": "NEUROLOGY", "options": {"1": "Perform a cranial CT scan and if there are no hemorrhagic lesions or other contraindications in the analysis, start immediate intravenous thrombolysis.", "2": "Perform urgent cranial CT scan and if there is no bleeding, start anticoagulation.", "3": "Perform cranial CT scan and admission to rehabilitation center, since their attitude will not differ regardless of whether the etiology is ischemic or hemorrhagic.", "4": "Cranial CT scan is not necessary, although I would withdraw the antiaggregation.", "5": NaN}, "correct_option": 1, "explanations": {"1": {"exist": true, "char_ranges": [[98, 458]], "word_ranges": [[15, 74]], "text": "we will have to perform a cranial CT scan to rule out a hemorrhagic stroke. If the CT scan does not show bleeding, the patient has an acute ischemic stroke, probably cardioembolic due to his cardiac pathology, not anticoagulated. Given that his tumor process is under control, there are no contraindications for the administration of fibrinolysis in this case,"}, "2": {"exist": false, "char_ranges": [], "word_ranges": [], "text": ""}, "3": {"exist": false, "char_ranges": [], "word_ranges": [], "text": ""}, "4": {"exist": false, "char_ranges": [], "word_ranges": [], "text": ""}, "5": {"exist": false, "char_ranges": [], "word_ranges": [], "text": ""}}} {"id": 496, "year": 2020, "question_id_specific": 71, "full_question": "40-year-old patient, with normal physical examination, in whom a mediastinal mass and a hepatic nodule have been evidenced in the CT scan, and in the CBC a serum elevation of lactic dehydrogenase (LDH), human chorionic betagonadotropin (beta-HCG) and alpha-fetoprotein.What is your diagnostic suspicion. What is your diagnostic suspicion:", "full_answer": "Very typical too. In the ED it can be diagnosed with a pregnancy test (in males), because it detects that beta HCG in urine. Hepatocarcinoma can elevate (alone) alpha-fetus, seminoma LDH, lymphoma also LDH.", "type": "MEDICAL ONCOLOGY", "options": {"1": "Metastatic hepatocarcinoma.", "2": "Extragonadal metastatic seminoma.", "3": "Mediastinal lymphoma.", "4": "Metastatic extragonadal nonseminomatous germ cell cancer.", "5": NaN}, "correct_option": 4, "explanations": {"1": {"exist": true, "char_ranges": [[125, 206]], "word_ranges": [[24, 34]], "text": "Hepatocarcinoma can elevate (alone) alpha-fetus, seminoma LDH, lymphoma also LDH."}, "2": {"exist": true, "char_ranges": [[125, 206]], "word_ranges": [[24, 34]], "text": "Hepatocarcinoma can elevate (alone) alpha-fetus, seminoma LDH, lymphoma also LDH."}, "3": {"exist": true, "char_ranges": [[125, 206]], "word_ranges": [[24, 34]], "text": "Hepatocarcinoma can elevate (alone) alpha-fetus, seminoma LDH, lymphoma also LDH."}, "4": {"exist": true, "char_ranges": [[18, 124]], "word_ranges": [[3, 24]], "text": "In the ED it can be diagnosed with a pregnancy test (in males), because it detects that beta HCG in urine."}, "5": {"exist": false, "char_ranges": [], "word_ranges": [], "text": ""}}} {"id": 14, "year": 2011, "question_id_specific": 213, "full_question": "A 41-year-old man comes to the clinic with a nodule in the right thyroid, a fine needle aspiration is performed. Cytological analysis will allow us:", "full_answer": "I think the correct answer is 5; FNA does not allow a histological study but a cytological one, therefore questions 1, 2 are eliminated. Question 3 seems to have been put as a filler because it is not relevant. Question 4 could raise doubts if you are not a pathologist but the most logical answer is 5 since papillary carcinoma is the most frequent.", "type": "SURGERY", "options": {"1": "Distinguish between follicular adenoma or minimally invasive follicular carcinoma.", "2": "Identify the areas of capsular or vascular invasion of the follicular carcinoma.", "3": "Recognize G-cell hyperplasia associated with familial spinal cancer.", "4": "Distinguish between a Hurthle cell tumor and an oxyphilic follicular adenoma.", "5": "Identify the typical cytologic features of papillary carcinoma."}, "correct_option": 5, "explanations": {"1": {"exist": true, "char_ranges": [[33, 135]], "word_ranges": [[7, 24]], "text": "FNA does not allow a histological study but a cytological one, therefore questions 1, 2 are eliminated."}, "2": {"exist": true, "char_ranges": [[33, 135]], "word_ranges": [[7, 24]], "text": "FNA does not allow a histological study but a cytological one, therefore questions 1, 2 are eliminated."}, "3": {"exist": true, "char_ranges": [[137, 209]], "word_ranges": [[24, 39]], "text": "Question 3 seems to have been put as a filler because it is not relevant."}, "4": {"exist": true, "char_ranges": [[211, 269]], "word_ranges": [[39, 50]], "text": "Question 4 could raise doubts if you are not a pathologist"}, "5": {"exist": true, "char_ranges": [[274, 350]], "word_ranges": [[51, 64]], "text": "the most logical answer is 5 since papillary carcinoma is the most frequent."}}} {"id": 330, "year": 2016, "question_id_specific": 85, "full_question": "A 22-year-old boy with hyposmia presents with lack of development of secondary sexual characteristics and infertility. Bilateral testicular volume of 4 mL. Analytically, FSH 1.2 U/L (vn 5-15); LH 0.6 U/L (vn 3-15); testosterone 100 ng/diu (vn 300-1200), prolactin normal. Indicate the treatment you will propose to achieve fertility:", "full_answer": "Kallman's syndrome or Morsier's syndrome is a congenital insufficiency associated with hypogonadotropic hypogonadism due to a deficit in GnRH hormone production. Repeated injections of GnRH are performed to restore fertility.", "type": "ENDOCRINOLOGY", "options": {"1": "Gn_RH infusion pump.", "2": "Monthly intramuscular administration of triptorelin.", "3": "Intramuscular administration of FSH and LH once a week.", "4": "Treatment with bromocriptine.", "5": NaN}, "correct_option": 1, "explanations": {"1": {"exist": true, "char_ranges": [[0, 225]], "word_ranges": [[0, 30]], "text": "Kallman's syndrome or Morsier's syndrome is a congenital insufficiency associated with hypogonadotropic hypogonadism due to a deficit in GnRH hormone production. Repeated injections of GnRH are performed to restore fertility."}, "2": {"exist": false, "char_ranges": [], "word_ranges": [], "text": ""}, "3": {"exist": false, "char_ranges": [], "word_ranges": [], "text": ""}, "4": {"exist": false, "char_ranges": [], "word_ranges": [], "text": ""}, "5": {"exist": false, "char_ranges": [], "word_ranges": [], "text": ""}}} {"id": 181, "year": 2013, "question_id_specific": 59, "full_question": "A type 1 diabetic who comes to the emergency room for dyspnea and general malaise. The blood test showed blood glucose 450 mg/dl, Na 142 mEq/l, K 4 mEq/l, pH 7.15, bicarbonate 12 mmol/l. Which treatment would NOT be indicated?", "full_answer": "Easy question since the treatment of diabetic ketoacidosis is a very repeated topic in the MIR. Bicarbonate is indicated at pH < 7.", "type": "ENDOCRINOLOGY", "options": {"1": "Fast IV saline.", "2": "Serum glucose 5% iv when blood glucose is less than 250 mg/dl.", "3": "Rapid human insulin iv.", "4": "Potassium chloride 100 mEq / day diluted in sera.", "5": "Sodium bicarbonate 1M 100 cc iv in 30 minutes."}, "correct_option": 5, "explanations": {"1": {"exist": false, "char_ranges": [], "word_ranges": [], "text": ""}, "2": {"exist": false, "char_ranges": [], "word_ranges": [], "text": ""}, "3": {"exist": false, "char_ranges": [], "word_ranges": [], "text": ""}, "4": {"exist": false, "char_ranges": [], "word_ranges": [], "text": ""}, "5": {"exist": true, "char_ranges": [[96, 131]], "word_ranges": [[16, 23]], "text": "Bicarbonate is indicated at pH < 7."}}} {"id": 182, "year": 2013, "question_id_specific": 61, "full_question": "A 76-year-old woman consults us because she is concerned about her risk of stroke since her mother died of stroke a year ago. She is diagnosed with arterial hypertension and type 2 diabetes mellitus and is being treated with glipizide, aspirin, enalapril and atorvastatin. She smokes 20 cigarettes a day and leads a sedentary life. The examination revealed a blood pressure of 150/80 mm Hg. The blood test shows a hemoglobin A1C of 8% and LDL cholesterol of 110 mg/dl. Which of the following actions is associated with a greater reduction in the risk of stroke?", "full_answer": "The most influential cardiovascular factor for stroke in women over 70 is blood pressure.", "type": "ENDOCRINOLOGY", "options": {"1": "Achieve optimal blood pressure control.", "2": "Achieve optimal hemoglobin A1C levels.", "3": "Add an antioxidant to the treatment.", "4": "Smoking cessation.", "5": "Achieve LDL levels below 100 mg/dl."}, "correct_option": 1, "explanations": {"1": {"exist": true, "char_ranges": [[0, 89]], "word_ranges": [[0, 14]], "text": "The most influential cardiovascular factor for stroke in women over 70 is blood pressure."}, "2": {"exist": false, "char_ranges": [], "word_ranges": [], "text": ""}, "3": {"exist": false, "char_ranges": [], "word_ranges": [], "text": ""}, "4": {"exist": false, "char_ranges": [], "word_ranges": [], "text": ""}, "5": {"exist": false, "char_ranges": [], "word_ranges": [], "text": ""}}} {"id": 21, "year": 2011, "question_id_specific": 111, "full_question": "A 32-year-old man, ADVP and recently released from prison, has a protein derivative skin test (PPD) with 10 mm of induration. One year ago, the PPD was negative, he is currently asymptomatic, his HIV serology is negative, and his Chest X-ray is normal. Which of the following approaches do you think is most appropriate?", "full_answer": "In the face of a PPD conversion within 2 years, and ruling out current TB disease, INH chemoprophylaxis is indicated, although I thought it was for 6 months and not 9. The rest is not questionable.", "type": "INFECTIOUS", "options": {"1": "Do not perform treatment and annual Rx control.", "2": "Sputum culture for mycobacteria and treat only if positive.", "3": "Isoniazid for 9 months.", "4": "Treatment with 4 drugs 4 months.", "5": "Treatment with RIF and PRZ for 2 months."}, "correct_option": 3, "explanations": {"1": {"exist": false, "char_ranges": [], "word_ranges": [], "text": ""}, "2": {"exist": false, "char_ranges": [], "word_ranges": [], "text": ""}, "3": {"exist": true, "char_ranges": [[0, 116]], "word_ranges": [[0, 20]], "text": "In the face of a PPD conversion within 2 years, and ruling out current TB disease, INH chemoprophylaxis is indicated,"}, "4": {"exist": false, "char_ranges": [], "word_ranges": [], "text": ""}, "5": {"exist": false, "char_ranges": [], "word_ranges": [], "text": ""}}} {"id": 60, "year": 2011, "question_id_specific": 59, "full_question": "A 40-year-old male, foreign tourist, is referred to the ED for low level of consciousness, temperature of 37.7°C and dyspnea. He has no trauma. Several tests were performed. The most immediate is an ABE without oxygen, showing PH 7.33, PCO2 50, PO2 65, HCO3 27:", "full_answer": "Actually, it can be all of them, but the most frequent cause of acute uncompensated respiratory acidosis in a young person is bronchopneumonia.", "type": "ANESTHESIOLOGY AND CRITICAL CARE", "options": {"1": "PET SCAN.", "2": "Lobar pneumonia.", "3": "EAP.", "4": "Alveolar hypoventilation.", "5": "Bronchopneumonia."}, "correct_option": 5, "explanations": {"1": {"exist": false, "char_ranges": [], "word_ranges": [], "text": ""}, "2": {"exist": false, "char_ranges": [], "word_ranges": [], "text": ""}, "3": {"exist": false, "char_ranges": [], "word_ranges": [], "text": ""}, "4": {"exist": false, "char_ranges": [], "word_ranges": [], "text": ""}, "5": {"exist": true, "char_ranges": [[37, 143]], "word_ranges": [[8, 23]], "text": "the most frequent cause of acute uncompensated respiratory acidosis in a young person is bronchopneumonia."}}} {"id": 455, "year": 2018, "question_id_specific": 155, "full_question": "In a 70-year-old man, hypertensive, with a right hemiplegia of sudden onset with mild dysarthria, without alterations in evocation or language comprehension, what type of stroke do you think is most likely to have suffered?", "full_answer": "The closest answer is 1. If we are strict we would be talking about a pure motor lacunar infarction (with mild dysarthria). Since the other 3 options do not give us a clinical picture like the one in the statement. Both options 2 and 3 give us a much more florid and extensive clinical picture.", "type": "NEUROLOGY", "options": {"1": "Left capsular lacunar lacunar infarction.", "2": "A cortico-subcortical left frontotemporal sylvian infarction.", "3": "Brain stem infarction.", "4": "A left lenticular hemorrhage.", "5": NaN}, "correct_option": 1, "explanations": {"1": {"exist": true, "char_ranges": [[0, 214]], "word_ranges": [[0, 40]], "text": "The closest answer is 1. If we are strict we would be talking about a pure motor lacunar infarction (with mild dysarthria). Since the other 3 options do not give us a clinical picture like the one in the statement."}, "2": {"exist": true, "char_ranges": [[215, 294]], "word_ranges": [[40, 55]], "text": "Both options 2 and 3 give us a much more florid and extensive clinical picture."}, "3": {"exist": true, "char_ranges": [[215, 294]], "word_ranges": [[40, 55]], "text": "Both options 2 and 3 give us a much more florid and extensive clinical picture."}, "4": {"exist": false, "char_ranges": [], "word_ranges": [], "text": ""}, "5": {"exist": false, "char_ranges": [], "word_ranges": [], "text": ""}}} {"id": 1, "year": 2011, "question_id_specific": 35, "full_question": "A 43-year-old man with no diseases of interest comes to the emergency department for 4 melaenic stools in the last 12 hours. He refers taking NSAIDs in previous days. Physical examination revealed pale skin and mucous membranes. BP 95/65 and HR 110 bpm. The abdomen is not painful and rectal examination shows melenic stools. The CBC shows a Hb of 8.1 gr/dl. After infusion of saline and the start of red blood cell concentrate transfusion, the BP was 120/85 and HR 90 bpm. An endoscopy was performed in which clots were found in the stomach and a 2 cm ulcer in the gastric antrum with fibrin background and a small grayish-white protrusion of 3 mm in the center of the ulcer (visible platelet thrombus vessel). What do you think is the most correct approach?", "full_answer": "Question about non-varicose ADH. Young patient, with no previous history, who after taking NSAIDs presents bleeding with hemodynamic instability requiring transfusion. Answer 1 is not in accordance with what has been said, this is an unstable patient who has bled a lot, who has a Forrest IIa ulcer with a high risk of rebleeding. Answer 2 speaks of somatostatin, which is used for bleeding due to portal hypertension. 4 will only be considered if bleeding is not controlled endoscopically. Between 3 and 5, since there is a high risk of bleeding, endoscopic therapy is recommended, preferably with 2 techniques, so the correct one would be 5.", "type": "DIGESTIVE", "options": {"1": "Since the patient is young, with no associated diseases and the ulcer has stopped bleeding, he can be discharged early with treatment with a PPI and H. Pylori eradication.", "2": "Remove the endoscope, place nasogastric tube for continuous aspiration and initiate treatment with a PPI and somatostatin.", "3": "Take biopsy for H. Pylori, remove the endoscope taking care not to dislodge the platelet thrombus and continue pharmacological treatment with i.v. PPI.", "4": "Consultation to the surgical service to consider urgent surgical treatment.", "5": "Endoscopic treatment by injection of adrenaline into the ulcer and placement of clips (metal staples) followed by i.v. administration of PPI."}, "correct_option": 5, "explanations": {"1": {"exist": true, "char_ranges": [[33, 330]], "word_ranges": [[4, 54]], "text": "Young patient, with no previous history, who after taking NSAIDs presents bleeding with hemodynamic instability requiring transfusion. Answer 1 is not in accordance with what has been said, this is an unstable patient who has bled a lot, who has a Forrest IIa ulcer with a high risk of rebleeding."}, "2": {"exist": true, "char_ranges": [[332, 418]], "word_ranges": [[54, 68]], "text": "Answer 2 speaks of somatostatin, which is used for bleeding due to portal hypertension."}, "3": {"exist": true, "char_ranges": [[492, 644]], "word_ranges": [[79, 106]], "text": "Between 3 and 5, since there is a high risk of bleeding, endoscopic therapy is recommended, preferably with 2 techniques, so the correct one would be 5."}, "4": {"exist": true, "char_ranges": [[420, 490]], "word_ranges": [[68, 79]], "text": "4 will only be considered if bleeding is not controlled endoscopically."}, "5": {"exist": true, "char_ranges": [[492, 644]], "word_ranges": [[79, 106]], "text": "Between 3 and 5, since there is a high risk of bleeding, endoscopic therapy is recommended, preferably with 2 techniques, so the correct one would be 5."}}} {"id": 66, "year": 2012, "question_id_specific": 106, "full_question": "Elderly woman brought to the ER by her neighbors because they see her somewhat groggy and with stool stains on her clothes. BP 100/60 mmHg, HR 100 bpm, sitting 70/30, HR 105 bpm. PVY normal. Kussmaul's respiration. No neurological focality. Weight 50 kg. Lab: PH 7.25, PCO2 14, Bicarbonate 5, Na 133, K 2.5, Cl 118, Creat 3.4, NUS 60, Prot 8. Which of the following answers is correct?", "full_answer": "This is a case of hyperchloremic metabolic acidosis with normal anion gap. Severe metabolic acidosis causes myocardial contractile depression, reduced effectiveness of catecholamines, increased incidence of arrhythmias, vasodilatation and increased capillary permeability, with the consequent reduction of tissue flow, at the cerebral level (leading to states of confusion and even coma), at the renal level, at the hepatic level... These patients frequently present intense tachypnea (Kussmaul's breathing) in an attempt to compensate for MCA by hyperventilation. We must take into account that, in this case, potassium is decreased, so the most frequent cause (supported by the stains on the dress) would be diarrhea, so it is most likely that their renal failure (although they do not give us urine ion data to confirm it) is of prerenal type. The treatment would be water and electrolyte replacement and, in principle, unless the patient worsens and the pH is less than 7.10, we should not administer bicarbonate.", "type": "ANESTHESIOLOGY, CRITICAL CARE AND EMERGENCY MEDICINE", "options": {"1": "The presenting acid-base disorder is a respiratory acidosis.", "2": "Compensation to correct the acidosis is inadequate.", "3": "With this exploration, we rule out that she is dehydrated.", "4": "In any case, we should not put bicarbonate.", "5": "She has an acute renal failure of prerenal cause."}, "correct_option": 5, "explanations": {"1": {"exist": false, "char_ranges": [], "word_ranges": [], "text": ""}, "2": {"exist": false, "char_ranges": [], "word_ranges": [], "text": ""}, "3": {"exist": false, "char_ranges": [], "word_ranges": [], "text": ""}, "4": {"exist": false, "char_ranges": [], "word_ranges": [], "text": ""}, "5": {"exist": true, "char_ranges": [[565, 846]], "word_ranges": [[76, 128]], "text": "We must take into account that, in this case, potassium is decreased, so the most frequent cause (supported by the stains on the dress) would be diarrhea, so it is most likely that their renal failure (although they do not give us urine ion data to confirm it) is of prerenal type."}}} {"id": 521, "year": 2021, "question_id_specific": 76, "full_question": "A 7-year-old girl who comes for consultation because her parents notice her red face since the previous day and in the last few hours they have noticed red spots on her body. On examination she is afebrile, with tonsils and ears without alterations, normal cardiopulmonary auscultation. He presents a confluent maculo papular rash on both cheeks, respecting the upper lip, resembling a \"slapped\" face. In the thorax she presents a reticular, lacy, non-pruritic exanthem. What is the most probable clinical diagnosis?", "full_answer": "Although this question straddles the line between derma and pediatrics, we would like to do our bit. We are being told about erythema súdenita or fifth disease (also called megaloerythema), caused by parvovirus B19. It is typical that after an episode of high fever an erythema appears on both cheeks in the form of a \"slap\" that usually has a cranial-caudal extension in the following days.", "type": "DERMATOLOGY", "options": {"1": "Chickenpox.", "2": "Rubella.", "3": "Erythema infectiosum or 5th disease.", "4": "Sudden exanthema or 6th disease.", "5": NaN}, "correct_option": 3, "explanations": {"1": {"exist": false, "char_ranges": [], "word_ranges": [], "text": ""}, "2": {"exist": false, "char_ranges": [], "word_ranges": [], "text": ""}, "3": {"exist": true, "char_ranges": [[101, 391]], "word_ranges": [[17, 66]], "text": "We are being told about erythema súdenita or fifth disease (also called megaloerythema), caused by parvovirus B19. It is typical that after an episode of high fever an erythema appears on both cheeks in the form of a \"slap\" that usually has a cranial-caudal extension in the following days."}, "4": {"exist": false, "char_ranges": [], "word_ranges": [], "text": ""}, "5": {"exist": false, "char_ranges": [], "word_ranges": [], "text": ""}}} {"id": 20, "year": 2011, "question_id_specific": 110, "full_question": "A 40-year-old male patient, smoker since he was 18 years old, with no history of interest. He consults for symptoms of 24 hours of evolution of chills, thermometric fever of 39ºC and cough with expectoration. Chest X-ray shows an increased density with alveolar pattern in the LSD and the patient is diagnosed with CAP. What is the most frequent etiological agent and therefore of obligatory coverage when choosing empirical atb treatment?", "full_answer": "This is a fairly easy epidemiology question, in adults without other data, Pneumococcus is the 1st.", "type": "INFECTIOUS", "options": {"1": "Mycoplasma pneumoniae.", "2": "Chlamydia pneumoniae.", "3": "Streptococcus pneumoniae.", "4": "Legionella pneumophila serogroup 1.", "5": "Haemophilus influenzae."}, "correct_option": 3, "explanations": {"1": {"exist": false, "char_ranges": [], "word_ranges": [], "text": ""}, "2": {"exist": false, "char_ranges": [], "word_ranges": [], "text": ""}, "3": {"exist": true, "char_ranges": [[45, 98]], "word_ranges": [[7, 16]], "text": "in adults without other data, Pneumococcus is the 1st."}, "4": {"exist": false, "char_ranges": [], "word_ranges": [], "text": ""}, "5": {"exist": false, "char_ranges": [], "word_ranges": [], "text": ""}}} {"id": 422, "year": 2018, "question_id_specific": 88, "full_question": "What is the treatment of choice for a 38-year-old man diagnosed with Cushing's disease in relation to a pituitary macroadenoma of 22 mm in diameter?", "full_answer": "Although the majority of ACTH-producing adenomas recur, surgery is always the first therapeutic option. The available medications (ketoconazole, somatostatin analogues) are aimed at decreasing the effects of excess cortisol or decreasing ACTH release, but they are not curative.", "type": "ENDOCRINOLOGY", "options": {"1": "Initially try medical treatment and, if not effective, resort to surgery.", "2": "Adrenalectomy.", "3": "Selective surgical resection of the adenoma.", "4": "Radiotherapy to try to reduce tumor size.", "5": NaN}, "correct_option": 3, "explanations": {"1": {"exist": false, "char_ranges": [], "word_ranges": [], "text": ""}, "2": {"exist": false, "char_ranges": [], "word_ranges": [], "text": ""}, "3": {"exist": true, "char_ranges": [[0, 103]], "word_ranges": [[0, 14]], "text": "Although the majority of ACTH-producing adenomas recur, surgery is always the first therapeutic option."}, "4": {"exist": false, "char_ranges": [], "word_ranges": [], "text": ""}, "5": {"exist": false, "char_ranges": [], "word_ranges": [], "text": ""}}} {"id": 348, "year": 2016, "question_id_specific": 160, "full_question": "45-year-old woman, mother of 3 children, attends an early diagnosis consultation. The cervicovaginal cytology is compatible with a high-grade squamous intraepithelial lesion. Which of the following options would you choose?", "full_answer": "Colposcopy is indicated for all patients with altered cytology (intraepithelial lesion or suspicion of invasive cancer or HPV infection), as a second level of screening. If lesions are observed in this test, a biopsy of the lesion will be performed. There is no point in repeating the cytology (since this is a screening test, not a diagnostic test), performing a hysterectomy for an intraepithelial lesion that has not been confirmed by biopsy is not indicated either, and performing curettage of the endometrial cavity does not provide information since the lesion is from the cervix.", "type": "GYNECOLOGY AND OBSTETRICS", "options": {"1": "Repeat cytology in 1 month.", "2": "Colposcopy with possible biopsy.", "3": "Hysterectomy with bilateral salpinguectomy and ovarian conservation.", "4": "Fractionated uterine curettage.", "5": NaN}, "correct_option": 2, "explanations": {"1": {"exist": true, "char_ranges": [[250, 350]], "word_ranges": [[40, 58]], "text": "There is no point in repeating the cytology (since this is a screening test, not a diagnostic test),"}, "2": {"exist": true, "char_ranges": [[0, 249]], "word_ranges": [[0, 40]], "text": "Colposcopy is indicated for all patients with altered cytology (intraepithelial lesion or suspicion of invasive cancer or HPV infection), as a second level of screening. If lesions are observed in this test, a biopsy of the lesion will be performed."}, "3": {"exist": true, "char_ranges": [[351, 469]], "word_ranges": [[58, 76]], "text": "performing a hysterectomy for an intraepithelial lesion that has not been confirmed by biopsy is not indicated either,"}, "4": {"exist": true, "char_ranges": [[474, 586]], "word_ranges": [[77, 94]], "text": "performing curettage of the endometrial cavity does not provide information since the lesion is from the cervix."}, "5": {"exist": false, "char_ranges": [], "word_ranges": [], "text": ""}}} {"id": 411, "year": 2018, "question_id_specific": 52, "full_question": "A couple with a first male child (index case) with cystic fibrosis (mucoviscidosis) come for genetic counseling. Molecular diagnosis indicates that the index case is homozygous for the F5O8del mutation and his parents are heterozygous carriers. The clinical geneticist will inform them about the mode of transmission of the disease. Which statement is correct?", "full_answer": "Both parents are carriers of a mutation (because they are not sick and have already had a homozygous child), and the next offspring has a 50% chance of receiving the mutation from each parent, so the probability of receiving it from both is 50%x50%=25%.", "type": "BIOSTATISTICS", "options": {"1": "The probability that the next offspring will be affected is 25%.", "2": "The probability that the next offspring will be affected is 50%.", "3": "Having had an affected child, the next offspring will also be affected.", "4": "Having had a homozygous F5O8del child, the next offspring will be heterozygous carriers (67%) or homozygous genotype without the mutation (33%).", "5": NaN}, "correct_option": 1, "explanations": {"1": {"exist": true, "char_ranges": [[0, 253]], "word_ranges": [[0, 44]], "text": "Both parents are carriers of a mutation (because they are not sick and have already had a homozygous child), and the next offspring has a 50% chance of receiving the mutation from each parent, so the probability of receiving it from both is 50%x50%=25%."}, "2": {"exist": false, "char_ranges": [], "word_ranges": [], "text": ""}, "3": {"exist": false, "char_ranges": [], "word_ranges": [], "text": ""}, "4": {"exist": false, "char_ranges": [], "word_ranges": [], "text": ""}, "5": {"exist": false, "char_ranges": [], "word_ranges": [], "text": ""}}} {"id": 590, "year": 2022, "question_id_specific": 64, "full_question": "A 68-year-old woman who uses +4.00 diopter glasses for distance vision in both eyes. She consults because she has been having eye pain with headache and blurred vision at night when she is watching television in dim light for a few weeks. Which of the following pathologies is she most likely to have?", "full_answer": "We are presented with a 68-year-old female patient. At that age the crystalline lens, although it does not normally have a cataract that needs surgery, it usually has increased in thickness. This implies that it projects the iris forward and reduces the depth of the anterior chamber. In an eye with a normal anterior chamber depth this is usually not a problem. But farsighted people already have a narrow anterior chamber. This is the usual combination we find in primary angular closure: farsightedness and age over 50-60 years. Younger hyperopes usually do not have this problem yet. And neither do older people who are emmetropic, myopic or have had cataract surgery. With this narrow angle, what triggers angular closure is mydriasis. That is, when you are in the penumbra, the pupil dilates and the iris thickens in the peripheral part, where it is already very close to the corneal endothelium in the case of a narrow chamber. Mydriasis then triggers angular closure, which is accompanied by ocular pain that may radiate backwards (headache). And increased intraocular pressure produces corneal edema that causes blurred vision. Therefore, option 1 is correct. Migraine does not have to be related to penumbra, and ocular pain and blurred vision is not particularly typical. Actinic keratitis is caused by exposure to ultraviolet radiation (such as solar radiation), and is not related to hyperopia or to being in penumbra. In addition, the history of exposure is usually clear. And finally, dry eye does not produce these symptoms either: it is more a question of burning, foreign body sensation, heaviness, etc. And it is also unrelated to the other elements: penumbra, hyperopia, etc. As a minor detail, I would have worded the question a little better. It is understood that we are asked which pathology is the most probable in relation to the symptoms and the anamnesis. But as it is worded, we are asked about which of the pathologies she is most likely to present, just like that. Actually, a 68-year-old woman is very likely to have dry eye, even if it does not cause the symptoms for which she is presenting.", "type": "OPHTHALMOLOGY", "options": {"1": "Primary angular closure glaucoma.", "2": "Migraine.", "3": "Actinic keratitis.", "4": "Dry eye syndrome.", "5": NaN}, "correct_option": 1, "explanations": {"1": {"exist": true, "char_ranges": [[741, 1168]], "word_ranges": [[121, 189]], "text": "That is, when you are in the penumbra, the pupil dilates and the iris thickens in the peripheral part, where it is already very close to the corneal endothelium in the case of a narrow chamber. Mydriasis then triggers angular closure, which is accompanied by ocular pain that may radiate backwards (headache). And increased intraocular pressure produces corneal edema that causes blurred vision. Therefore, option 1 is correct."}, "2": {"exist": true, "char_ranges": [[1169, 1282]], "word_ranges": [[189, 208]], "text": "Migraine does not have to be related to penumbra, and ocular pain and blurred vision is not particularly typical."}, "3": {"exist": true, "char_ranges": [[1283, 1431]], "word_ranges": [[208, 232]], "text": "Actinic keratitis is caused by exposure to ultraviolet radiation (such as solar radiation), and is not related to hyperopia or to being in penumbra."}, "4": {"exist": true, "char_ranges": [[1487, 1695]], "word_ranges": [[241, 275]], "text": "And finally, dry eye does not produce these symptoms either: it is more a question of burning, foreign body sensation, heaviness, etc. And it is also unrelated to the other elements: penumbra, hyperopia, etc."}, "5": {"exist": false, "char_ranges": [], "word_ranges": [], "text": ""}}} {"id": 86, "year": 2012, "question_id_specific": 51, "full_question": "A 26-year-old man with no history of interest, asymptomatic, presents on examination with a mesosystolic murmur in the aortic focus and left parasternal border that increases with the Valsalva maneuver. BP and peripheral pulses are normal and the ECG shows signs of marked left ventricular enlargement and very deep negative T waves in V3, V4, V5 and V6. What is the most likely diagnosis?", "full_answer": "A young male patient with an aortic systolic murmur that increases with the Valsalva maneuver and signs of left ventricular enlargement is practically a \"portrait\" of the typical findings that can be found in hypertrophic cardiomyopathy. Very easy question.", "type": "CARDIOLOGY AND VASCULAR SURGERY", "options": {"1": "Functional murmur.", "2": "Aortic stenosis.", "3": "Aortic insufficiency.", "4": "Hypertrophic cardiomyopathy.", "5": "Ventricular septal defect."}, "correct_option": 4, "explanations": {"1": {"exist": false, "char_ranges": [], "word_ranges": [], "text": ""}, "2": {"exist": false, "char_ranges": [], "word_ranges": [], "text": ""}, "3": {"exist": false, "char_ranges": [], "word_ranges": [], "text": ""}, "4": {"exist": true, "char_ranges": [[0, 237]], "word_ranges": [[0, 36]], "text": "A young male patient with an aortic systolic murmur that increases with the Valsalva maneuver and signs of left ventricular enlargement is practically a \"portrait\" of the typical findings that can be found in hypertrophic cardiomyopathy."}, "5": {"exist": false, "char_ranges": [], "word_ranges": [], "text": ""}}} {"id": 498, "year": 2020, "question_id_specific": 129, "full_question": "65-year-old man who consults for dysphagia to solids since two months ago. Esophagoscopy shows a tumor 30 cm from the dental arch, partially stenosing, with pathological anatomy of squamous cell carcinoma. Endoscopic ultrasound and PET-CT scan were requested where no pathological lymphadenopathies were observed. Which of the following options would be the most correct?", "full_answer": "Preoperative chemoradio in all the guidelines.", "type": "MEDICAL ONCOLOGY", "options": {"1": "Transhiatal esophagectomy.", "2": "Esophagectomy according to the Ivor-Lewis technique.", "3": "Chemotherapy plus preoperative radiotherapy.", "4": "Neoadjuvant chemotherapy.", "5": NaN}, "correct_option": 3, "explanations": {"1": {"exist": false, "char_ranges": [], "word_ranges": [], "text": ""}, "2": {"exist": false, "char_ranges": [], "word_ranges": [], "text": ""}, "3": {"exist": true, "char_ranges": [[0, 46]], "word_ranges": [[0, 6]], "text": "Preoperative chemoradio in all the guidelines."}, "4": {"exist": false, "char_ranges": [], "word_ranges": [], "text": ""}, "5": {"exist": false, "char_ranges": [], "word_ranges": [], "text": ""}}} {"id": 31, "year": 2011, "question_id_specific": 233, "full_question": "A patient diagnosed with bronchial asthma who is being treated with low-dose inhaled steroids (200 micrograms of budesonide every 12 hours) and rescue medication (salbutamol on demand) comes for periodic clinical monitoring. The patient reports that in the last 3 months he has had no exacerbations and that he uses salbutamol once a day. He only has dyspnea when climbing stairs and some nights he sleeps irregularly due to coughing. Which of the following therapeutic options should be considered as the treatment of choice?", "full_answer": "Asthma treatment algorithm. We rule out continuing with the same treatment since the patient is symptomatic.", "type": "NEUROLOGY AND THORACIC SURGERY", "options": {"1": "Increase the dose of inhaled corticosteroids to 800 micrograms/day.", "2": "Add tiotropium.", "3": "Maintain the same pharmacological regimen.", "4": "Switch to ocmbined therapy with low-dose inhaled corticosteroids and long-acting beta2-adrenergics.", "5": "Add leukotrienes."}, "correct_option": 4, "explanations": {"1": {"exist": false, "char_ranges": [], "word_ranges": [], "text": ""}, "2": {"exist": false, "char_ranges": [], "word_ranges": [], "text": ""}, "3": {"exist": false, "char_ranges": [], "word_ranges": [], "text": ""}, "4": {"exist": true, "char_ranges": [[0, 108]], "word_ranges": [[0, 16]], "text": "Asthma treatment algorithm. We rule out continuing with the same treatment since the patient is symptomatic."}, "5": {"exist": false, "char_ranges": [], "word_ranges": [], "text": ""}}} {"id": 618, "year": 2022, "question_id_specific": 119, "full_question": "13-year-old female, with no relevant history, with menarche 3 months ago, followed since the age of 10 years for idiopathic scoliosis that has worsened. In the physical examination she presents a hump of 7 degrees in the Adams test and in the scoliogram a T4-L1 thoracolumbar curve of 35 degrees of Cobb and a Risser 0:", "full_answer": "Treatment with a brace in Risser from 0 to 2 can be considered since there is still growth to occur and correction can take place. Its use is considered for curves between 30-40º.", "type": "TRAUMATOLOGY", "options": {"1": "Recommend swimming and review in three months.", "2": "Prescribe a brace-type orthosis.", "3": "Refer to physiotherapy for spine elastification.", "4": "Check in 6 months with a new X-ray.", "5": NaN}, "correct_option": 2, "explanations": {"1": {"exist": false, "char_ranges": [], "word_ranges": [], "text": ""}, "2": {"exist": true, "char_ranges": [[0, 179]], "word_ranges": [[0, 33]], "text": "Treatment with a brace in Risser from 0 to 2 can be considered since there is still growth to occur and correction can take place. Its use is considered for curves between 30-40º."}, "3": {"exist": false, "char_ranges": [], "word_ranges": [], "text": ""}, "4": {"exist": false, "char_ranges": [], "word_ranges": [], "text": ""}, "5": {"exist": false, "char_ranges": [], "word_ranges": [], "text": ""}}}